Pathology 2014 Flashcards

1
Q

TSH

A

Consistent with thyrotoxicosis.
Thyrotoxicosis:
• Metabolic rate is increased: weight loss
• Cardiovascular: tachycardia, AF, palpitations
• Gastrointestinal: stimulates peristalsis, diarrhoea
• Respiratory: tachypnoea
• Skeletal: direct effect on osteoclasts causing osteopenia/osteoporosis
• Reproductive: dysmenorrhoea, infertility
Management:
Make diagnosis: low TSH, elevated T4 and T3
Technitium scan: high uptake vs low uptake
Thyroid autoantibodies (thyroid microsomal)
Aim of treatment is to keep patient safe! Give beta blocker
Think of other AI conditions (pernicious anaemia, coeliac, addisons)
ECG and DEXA scan

Radioactive Iodine: tag onto iodine, uptake into thyroid gland and slowly destroys thyoid gland, leading to hypothyroid over the following years.
• 131I RCP guidlines
• opthalmopathy/tracheal compression
• stop thionamide
• precipitation of thyroid storm: tachycardia, AF (give beta blocker)
• hypothyroidism: make patient underactive and then replace with thyroxine.

How well did you know this?
1
Not at all
2
3
4
5
Perfectly
2
Q

TSH 8.4, Free T4 11.7, Thyroid peroxidase (thyroid antibodies) positive

Normal ranges TSH 0.33-4.5mU/L Free T3 (3.2-6.5pmol/L) Free T4 (10.2-22.0pmol/L) Thyroglobulin

A

Consistent with sub-clinical hypothyroidism with risk of later clinical hypothyroidism.

Subclinical Hypothyroidism – “compensated hypothyroidism”
– Normal T4 levels, but TSH level is elevated
– TPO may be used to predict later thyroid disease
– Unlikely to be cause of symptoms
– Treating will not change the symptoms, unless they have elevated cholesterol levels. There is evidence of an association between subclinical hypothyroidism and hypercholestrolaemia, and these patients do benefit from treatment.

How well did you know this?
1
Not at all
2
3
4
5
Perfectly
3
Q

TSH 1.4, Free T4 12.1

Normal ranges TSH 0.33-4.5mU/L Free T3 (3.2-6.5pmol/L) Free T4 (10.2-22.0pmol/L) Thyroglobulin

A

Consistent with euthyroid status in a patient complaining of tiredness.

Sick Euthyroid Syndrome - alteration in pituitary thyroid axis in non-thyroidal illness, normal physiology, not ‘thyroid symptomatic’
• any severe illness (e.g. sepsis): thyroid tries to shut down to lower BMR
• low T4 when severe
• high normal TSH, later decreased
• low T3 and reduced T3 action

How well did you know this?
1
Not at all
2
3
4
5
Perfectly
4
Q

TSH 22. 4, Free T4 6.3.

Normal ranges TSH 0.33-4.5mU/L Free T3 (3.2-6.5pmol/L) Free T4 (10.2-22.0pmol/L) Thyroglobulin

A

Consistent with clinical primary hypothyroidism

  • Majority of cases: it is a problem with the thyroid gland itself.
  • Hashimoto’s disease (auto-immune condition affecting thyroid)
  • Atrophic (congential or later in life)
  • Post Graves’ disease (RAI, surgery, natural history or thionamides)
  • Presentation: low BMR, CVS (bradycardic), GIT (slows down, constipation), Resp muscles have receptors and so breathing becomes more laboured, repro (irregular periods, infertility), may develop visual problems in pituitary tumour involved.
  • Weight gain with decreased REE and poor appetite, cole hands and feet
  • Hyponatraemia (thyroxine is involved in sodium transport and reabsorption in the kidneys)
  • Normocytic or macrocytic anaemia (if pernicious anaemia)
  • Myxoedema, goitre

Make diagnosis: low free T4, elevated TSH
Treatment: levothyroxine to a normal range TSH (around 100mcg/day), no EBM for excessive T4 treatment: excess treatment can cause osteopenia and atrial fibrillation. No evidence for using T3 instead of T4
Measure antibodies to thyroid peroxidase (suggestive of AI cause for thyroid disease)
Think of other AI conditions (pernicious anaemia, coeliac, addisions, - adrenal anti-bodies, early morning cortisol)
Must do an ECG before starting thyroid treatment as thyroxine increases cardiac contractility so cardiac patients would be at risk of ischaemia

How well did you know this?
1
Not at all
2
3
4
5
Perfectly
5
Q

Thyroglobulin 254

Normal ranges TSH 0.33-4.5mU/L Free T3 (3.2-6.5pmol/L) Free T4 (10.2-22.0pmol/L) Thyroglobulin

A

To screen for recurrence of differentiated thyroid carcinoma.

Thyroglobulin in the serum can be used to indicate if the thyroid cancer has come back – very sensitive. Measure thyrobulin when TSH is still elevated and later in life. Thyroglobulin indicates functioning thyroid tissue and therefore presence of the tumour cells.

How well did you know this?
1
Not at all
2
3
4
5
Perfectly
6
Q

Commonly associated with bradykinesia and rigidity

Pill rolling tremor
Huntingtons Chorea
Sydenhams chorea
Benign essential tremor
Intention tremor
Spastic paraparesis
A

Pill rolling tremor: Parkinson’s disease is characterised by bradykinesia, rigidity and a pill rolling tremor of 4 – 7 Hz.
Lewy Bodies are found at postmortem, and dopamine levels are reduced in the substantia nigra.
Antiemetics and neuroleptics are dopamine antagonists, and as such cause a pill rolling tremor.

How well did you know this?
1
Not at all
2
3
4
5
Perfectly
7
Q

Lewy Bodies are found in post mortem brains in patients who have this movement disorder

Pill rolling tremor
Huntingtons Chorea
Sydenhams chorea
Benign essential tremor
Intention tremor
Spastic paraparesis
A

Pill rolling tremor: Parkinson’s disease is characterised by bradykinesia, rigidity and a pill rolling tremor of 4 – 7 Hz.
Lewy Bodies are found at postmortem, and dopamine levels are reduced in the substantia nigra.
Antiemetics and neuroleptics are dopamine antagonists, and as such cause a pill rolling tremor.

How well did you know this?
1
Not at all
2
3
4
5
Perfectly
8
Q

Dominantly inherited

Pill rolling tremor
Huntingtons Chorea
Sydenhams chorea
Benign essential tremor
Intention tremor
Spastic paraparesis
A
Huntingdon's Chorea: Huntington’s disease is dominantly inherited.
Huntington disease (HD) is caused by expansion of the cytosine-adenine-guanine (CAG) trinucleotide repeats in the HTT gene (also known as the HD or IT15 gene) located on chromosome 4p16.3 that encodes the protein huntingtin. The disease is transmitted in an autosomal dominant manner.  Individuals with early-onset HD tend to have a large number of CAG repeats, while those developing HD late in life typically have a low repeat number.
The most common presenting symptom of HD in adults is chorea (hence the name Huntington chorea). Other usual findings at presentation include memory deficits, affective disturbances, personality changes, and other manifestations of motor dysfunction such as parkinsonism and dystonia. Patients with juvenile-onset HD have minimal or no chorea, but develop myoclonus and seizures as well as cognitive and behavioral problems. Children also have a more rapidly progressive disease.
How well did you know this?
1
Not at all
2
3
4
5
Perfectly
9
Q

Occurs following an infection

Pill rolling tremor
Huntingtons Chorea
Sydenhams chorea
Benign essential tremor
Intention tremor
Spastic paraparesis
A

Sydenham’s Chorea: Rheumatic fever occurs about a month after a streptococcal sore throat.
Sydenham’s chorea (also known as St Vitus’ dance) is a major criterion of acute rheumatic fever, and is now known to be mediated by the immune system.

How well did you know this?
1
Not at all
2
3
4
5
Perfectly
10
Q

In which of the above is the Substantia Nigra affected?

Pill rolling tremor
Huntingtons Chorea
Sydenhams chorea
Benign essential tremor
Intention tremor
Spastic paraparesis
A

Pill rolling tremor: Parkinson’s disease is characterised by bradykinesia, rigidity and a pill rolling tremor of 4 – 7 Hz.
Lewy Bodies are found at postmortem, and dopamine levels are reduced in the substantia nigra.
Antiemetics and neuroleptics are dopamine antagonists, and as such cause a pill rolling tremor.

How well did you know this?
1
Not at all
2
3
4
5
Perfectly
11
Q

Which of the above movement disorders is caused by antiemetics and neuroleptics?

Pill rolling tremor
Huntingtons Chorea
Sydenhams chorea
Benign essential tremor
Intention tremor
Spastic paraparesis
A

Pill rolling tremor: Parkinson’s disease is characterised by bradykinesia, rigidity and a pill rolling tremor of 4 – 7 Hz.
Lewy Bodies are found at postmortem, and dopamine levels are reduced in the substantia nigra.
Antiemetics and neuroleptics are dopamine antagonists, and as such cause a pill rolling tremor.

How well did you know this?
1
Not at all
2
3
4
5
Perfectly
12
Q

The movement disorder that occurs in Rheumatic fever?

Pill rolling tremor
Huntingtons Chorea
Sydenhams chorea
Benign essential tremor
Intention tremor
Spastic paraparesis
A

Sydenham’s Chorea: Rheumatic fever occurs about a month after a streptococcal sore throat.
Sydenham’s chorea (also known as St Vitus’ dance) is a major criterion of acute rheumatic fever, and is now known to be mediated by the immune system.

How well did you know this?
1
Not at all
2
3
4
5
Perfectly
13
Q

The movement disorder that occurs in Parkinson’s disease?

Pill rolling tremor
Huntingtons Chorea
Sydenhams chorea
Benign essential tremor
Intention tremor
Spastic paraparesis
A

Pill rolling tremor: Parkinson’s disease is characterised by bradykinesia, rigidity and a pill rolling tremor of 4 – 7 Hz.
Lewy Bodies are found at postmortem, and dopamine levels are reduced in the substantia nigra.
Antiemetics and neuroleptics are dopamine antagonists, and as such cause a pill rolling tremor.

How well did you know this?
1
Not at all
2
3
4
5
Perfectly
14
Q

The movement disorder that is improved by alcohol?

Pill rolling tremor
Huntingtons Chorea
Sydenhams chorea
Benign essential tremor
Intention tremor
Spastic paraparesis
A

A benign essential tremor is improved by alcohol.

How well did you know this?
1
Not at all
2
3
4
5
Perfectly
15
Q

The movement disorder that is caused by long term alcohol abuse?

Pill rolling tremor
Huntingtons Chorea
Sydenhams chorea
Benign essential tremor
Intention tremor
Spastic paraparesis
A

Cerebellar atrophy occurs in long term alcohol abuse, resulting in an intention tremor.

How well did you know this?
1
Not at all
2
3
4
5
Perfectly
16
Q

The movement disorder mediated by the immune system?

Pill rolling tremor
Huntingtons Chorea
Sydenhams chorea
Benign essential tremor
Intention tremor
Spastic paraparesis
A

Sydenham’s Chorea: Rheumatic fever occurs about a month after a streptococcal sore throat.
Sydenham’s chorea (also known as St Vitus’ dance) is a major criterion of acute rheumatic fever, and is now known to be mediated by the immune system.

How well did you know this?
1
Not at all
2
3
4
5
Perfectly
17
Q

Phagocytes:
Mediated by Toll like receptors which recognise PAMP

Oxidative killing
Pathogen recognition
Opsonsation
Non-oxidative killing

A

Pathogen recognition

How well did you know this?
1
Not at all
2
3
4
5
Perfectly
18
Q

Phagocytes:
May be mediated by anti-bodies, complement components or acute phase proteins and facilitates phagocytosis

Oxidative killing
Pathogen recognition
Opsonsation
Non-oxidative killing

A

Opsonisation

How well did you know this?
1
Not at all
2
3
4
5
Perfectly
19
Q

Phagocytes:
Describes killing mediated by ROS generated by the action of NADPH oxidase complex

Oxidative killing
Pathogen recognition
Opsonsation
Non-oxidative killing

A

Oxidative killing

How well did you know this?
1
Not at all
2
3
4
5
Perfectly
20
Q

Phagocytes:
May be mediated by bactericidal enzymes such as lysozyme

Oxidative killing
Pathogen recognition
Opsonsation
Non-oxidative killing

A

Non-oxidative killing

How well did you know this?
1
Not at all
2
3
4
5
Perfectly
21
Q

Innate Immune System:
Derived from monocytes and resident in peripheral tissues

Neutrophils
NK cells
Dendritic Cells
Macrophages

A

Macrophages

How well did you know this?
1
Not at all
2
3
4
5
Perfectly
22
Q

Innate Immune System:
Polymorphonuclear cells capable of phagocytosing pathogens and killing by oxidative and non-oxidative mechanisms

Neutrophils
NK cells
Dendritic Cells
Macrophages

A

Neutrophils

How well did you know this?
1
Not at all
2
3
4
5
Perfectly
23
Q
Innate Immune System:
Lymphocytes that express inhibitory receptors, capable of recognising HLA class I molecules and have cytotoxic capacity

Neutrophils
NK cells
Dendritic Cells
Macrophages

A

Natural Killer Cells

How well did you know this?
1
Not at all
2
3
4
5
Perfectly
24
Q

Innate Immune System:
Immature cells are adapted for pathogen recognition and uptake whilst mature cells are adapted for antigen presentation to prime T cells

Neutrophils
NK cells
Dendritic Cells
Macrophages

A

Dendritic Cells

How well did you know this?
1
Not at all
2
3
4
5
Perfectly
25
Q
Adaptive Immune System:
Express receptors that recognise peptides usually derived from intracellular proteins and expressed on HLA class I molecules

Th1 cells
CD8 T cells
T follicular helper (Tfh) cells
T regulatory cells

A

CD8 T cells

How well did you know this?
1
Not at all
2
3
4
5
Perfectly
26
Q

Adaptive Immune System:
Subset of lymphocytes that express Foxp3 and CD25

Th1 cells
CD8 T cells
T follicular helper (Tfh) cells
T regulatory cells

A

T regulatory cells

How well did you know this?
1
Not at all
2
3
4
5
Perfectly
27
Q

Adaptive Immune System:
Subset of cells that express CD4 and secrete IFN gamma and IL-2

Th1 cells
CD8 T cells
T follicular helper (Tfh) cells
T regulatory cells

A

Th1 cells

How well did you know this?
1
Not at all
2
3
4
5
Perfectly
28
Q

Adaptive Immune System:
Play an important role in promoting germinal centre reactions and differentiation of B cells into IgG and IgA secreting plasma cells

Th1 cells
CD8 T cells
T follicular helper (Tfh) cells
T regulatory cells

A

T follicular helper (Tfh) cells

How well did you know this?
1
Not at all
2
3
4
5
Perfectly
29
Q

Adaptive Immune System:
Exist with the bone marrow and develop from haematopoietic stem cells

Pre-B cells
IgA
IgG secreting plasma cells
IgM secreting plasma cells

A

Pre-B cells

How well did you know this?
1
Not at all
2
3
4
5
Perfectly
30
Q

Adaptive Immune System:
Cell dependent on the presence of CD4 T cell help for generation

Pre-B cells
IgA
IgG secreting plasma cells
IgM secreting plasma cells

A

IgG secreting plasma cells

How well did you know this?
1
Not at all
2
3
4
5
Perfectly
31
Q

Adaptive Immune System:
Are generated rapidly following antigen recognition and are not dependent on CD4 T cell help

Pre-B cells
IgA
IgG secreting plasma cells
IgM secreting plasma cells

A

IgM secreting plasma cells

How well did you know this?
1
Not at all
2
3
4
5
Perfectly
32
Q

Adaptive Immune System:
Divalent anti-body present within a mucous with helps to provide a constitutive barrier to infection

Pre-B cells
IgA
IgG secreting plasma cells
IgM secreting plasma cells

A

IgA

How well did you know this?
1
Not at all
2
3
4
5
Perfectly
33
Q

Area within secondary lymphoid tissue where B cells proliferate and undergo affinity maturation and isotope switching

Primary Lymphoid organs
Thoracic duct
Thymus
Germinal centre

A

Germinal centre

How well did you know this?
1
Not at all
2
3
4
5
Perfectly
34
Q

Include both the bone marrow and thymus; sites of B and T cell development

Primary Lymphoid organs
Thoracic duct
Thymus
Germinal centre

A

Primary Lymphoid organs

How well did you know this?
1
Not at all
2
3
4
5
Perfectly
35
Q

Carries lymphocytes from lymph nodes back to the blood circulation

Primary Lymphoid organs
Thoracic duct
Thymus
Germinal centre

A

Thoracic duct

How well did you know this?
1
Not at all
2
3
4
5
Perfectly
36
Q

Site of deletion of T cells with inappropriately high or low affinity for HLA molecules and of maturation of T cells into CD4+ or CD8+ cells

Primary Lymphoid organs
Thoracic duct
Thymus
Germinal centre

A

Thymus

How well did you know this?
1
Not at all
2
3
4
5
Perfectly
37
Q

What is the commonest form of prion disease?

Kuru
Iatrogenic CJD
Sporadic CJD
Variant CJD
Gerstmann-Straussler Scheinker syndrome
A

Sporadic CJD
Prion diseases are a protein-only infectious agent causing rapid neurodegeneration and are currently untreatable. Prion protein gene is on chr20 (codon 129 polymorphism, MM, MV, VV). Sporadic CJD = 80%
Acquired = <5% (kuru, vCJD, iatrogenic (blood, surgery, GH)
Genetic = 15% PRNP mutations (Gerstmann-Sträussler-Scheinker syndrome (GSS), fatal familial insomnia (FFI).
Sporadic CJD causes rapid dementia with myoclonus, cortical blindness, akinetic mutism & LMN signs. Mean onset 65 years, death with 6/12. Cause uncertain.
Diagnosis: EEG = periodic, triphasic complexes (nonspecific), MRI basal ganglia (increased signal cortical/striatal signal change on DWI MRI), CSF markers 14-3-3 protein (S100) may be elevated, Neurogenetics to rule out genetic cause, Tonsilar biopsy NOT useful

How well did you know this?
1
Not at all
2
3
4
5
Perfectly
38
Q

A 10-year-old boy is brought by his mother to your clinic. He is very thin, but has a distended abdomen. What is it that his diet does not contain enough of to cause this?

A. Folate
B. Lipid
C. Thiamine
D. Carbohydrate
E. Vitamin C
F. Protein
G. Vitamin A
H. Vitamin D
I. Niacin
J. Fluoride
K. Iron
L. Zinc
M. Copper
N. Vitamin K
O. Riboflavin
P. Iodine
A

Protein
Kwashiorkor is characterized by marked muscle atrophy with normal or increased body fat and the presence of peripheral edema (anasarca). Edema is the defining characteristic for diagnosis. Inadequate protein and energy intake may contribute to the clinical features of kwashiorkor, but the pathogenesis is not fully understood. Anorexia is almost universal. Physical examination findings include:
●Normal or nearly normal weight and height for age
●Anasarca (severe generalized edema)
●Pitting edema in the lower extremities, presacral area, genitalia, and periorbitally
●Apathetic, listless affect
●Rounded prominence of the cheeks (“moon-face”)
●Pursed appearance of the mouth
●Dry, atrophic, peeling skin with confluent areas of hyperkeratosis and hyperpigmentation
●Dry, dull, hypopigmented hair that falls out or is easily plucked
●Hepatomegaly (from fatty liver infiltrates)
●Distended abdomen with dilated intestinal loops, but no ascites
●Hypothermia
Intermittent periods of adequate dietary intake restores hair color, resulting in alternating loss of hair color interspersed between bands of normal pigmentation (flag sign)

How well did you know this?
1
Not at all
2
3
4
5
Perfectly
39
Q

A 50-year-old homeless man walks into A and E. He is very thin, smells of alcohol and is vomiting. On neurological examination, you note he has nystagmus and walks with a broad based gait. You give him advice on how to stop drinking, what else would you give?

A. Folate
B. Lipid
C. Thiamine
D. Carbohydrate
E. Vitamin C
F. Protein
G. Vitamin A
H. Vitamin D
I. Niacin
J. Fluoride
K. Iron
L. Zinc
M. Copper
N. Vitamin K
O. Riboflavin
P. Iodine
A

Thiamine (vitamin B1)
Thiamine deficiency has been associated with three disorders:
●Beriberi (infantile and adult)
●Wernicke-Korsakoff syndrome
●Leigh’s syndrome
Wernicke-Korsakoff syndrome is the best known neurologic complication of thiamine deficiency. Wernicke’s encephalopathy (WE) is an acute syndrome requiring emergent treatment to prevent death and neurologic morbidity. Korsakoff’s syndrome (KS) refers to a chronic neurologic condition that usually occurs as a consequence of WE. It is characterized by impaired short-term memory and confabulation with otherwise grossly normal cognition.

WE is a triad of nystagmus, ophthalmoplegia, and ataxia, along with confusion. This combination is almost exclusively described in chronic alcoholics with thiamine deficiency. The two entities are not separate diseases, but a spectrum of signs and symptoms. There may be a genetic predisposition for the development of WE since not all thiamine deficient patients are affected. Impairment in the synthesis of one of the important enzymes of the pentose phosphate pathway (erythrocyte transketolase) may explain such a predisposition. WE is treated with thiamine supplementation.It is common practice to delay giving dextrose to alcoholic patients until thiamine supplementation has been initiated to avoid precipitating Wernicke’s encephalopathy.

How well did you know this?
1
Not at all
2
3
4
5
Perfectly
40
Q

You see a young boy in your clinic. He complains of bone pain and he says he has felt unwell for a few weeks. On examination he is knock-kneed and walks with a waddling gait. What would you be most likely to treat him with?

A. Folate
B. Lipid
C. Thiamine
D. Carbohydrate
E. Vitamin C
F. Protein
G. Vitamin A
H. Vitamin D
I. Niacin
J. Fluoride
K. Iron
L. Zinc
M. Copper
N. Vitamin K
O. Riboflavin
P. Iodine
A

Vitamin D
Rickets refers to deficient mineralization at the growth plate, as well as architectural disruption of this structure. Osteomalacia refers to impaired mineralization of the bone matrix. Rickets and osteomalacia usually occur together as long as the growth plates are open; only osteomalacia occurs after the growth plates have fused.
Calcipenic rickets usually is caused by dietary deficiency of vitamin D and/or calcium; this is the most common cause of rickets worldwide. Rarely, it is caused by genetic defects in vitamin D metabolism or action leading to vitamin D resistance.
Phosphopenic rickets in children and adolescents is almost always caused by renal phosphate wasting, which is usually an isolated phenomenon, but may be part of a generalized tubular disorder such as Fanconi syndrome, or may rarely result from inadequate dietary phosphorus or intestinal malabsorption.
The skeletal findings are similar for calcipenic and phosphopenic rickets, and may include delayed closure of the fontanelles, parietal and frontal bossing, enlargement of the costochondral junction (“rachitic rosary”), widening of the wrist, and lateral bowing of the femur and tibia (bow legs).
The concentration of serum alkaline phosphatase is elevated in both types of rickets, and it is a good marker of disease activity. Other biochemical findings include hypocalcemia and hypophosphatemia, but the pattern varies depending on the type and severity of the rickets (table 1). Serum concentration of parathyroid hormone (PTH) typically is elevated in calcipenic rickets, but not in phosphopenic rickets.

How well did you know this?
1
Not at all
2
3
4
5
Perfectly
41
Q

A 16-year-old girl presents with pain in her joints and her mother says that she has become increasingly forgetful over recent times. Last week she put a pan of water on the stove and forgot about it until it boiled dry and melted the pan. On examination you notice Kayser-Fleischer rings in the eyes. What substance is this girl most likely to handle abnormally?

A. Folate
B. Lipid
C. Thiamine
D. Carbohydrate
E. Vitamin C
F. Protein
G. Vitamin A
H. Vitamin D
I. Niacin
J. Fluoride
K. Iron
L. Zinc
M. Copper
N. Vitamin K
O. Riboflavin
P. Iodine
A

Copper
Wilson disease (hepatolenticular degeneration) is due to a AR genetic abnormality that leads to impairment of cellular copper transport. Impaired biliary copper excretion leads to accumulation in several organs, (liver, brain, & cornea). Over time, the liver is progressively damaged and eventually becomes cirrhotic. The clinical manifestations of Wilson disease are predominantly hepatic, neurologic, and psychiatric, with many patients having a combination of symptoms. The majority of patients with Wilson disease are diagnosed between the ages of 5 and 35 years. Children most often initially present with liver disease, at an average age of 9 to 13 years, with Wilson disease accounting for 8 to 10 percent of chronic active hepatitis in children. Unlike children, who are more likely to present with hepatic manifestations, older patients (mid-teens and older) are more likely to present with neurologic manifestations. The mean age at presentation for patients with neurologic symptoms ranges between 15 and 21 years.
Signs and symptoms of Wilson disease may include:
●Kayser-Fleischer rings, visible in 50 percent of patients with hepatic disease (seen with all forms of liver involvement)
●Asymptomatic (steatosis, chronic hepatitis, compensated cirrhosis)
●Abdominal pain (acute hepatitis, acute liver failure)
●Jaundice (acute hepatitis, acute liver failure, cirrhosis)
●Hepatomegaly (acute and chronic hepatitis, acute liver failure)
●Splenomegaly (cirrhosis)
●Ascites (cirrhosis)
●Upper gastrointestinal bleeding (cirrhosis with varices or portal hypertensive gastropathy)
●Peripheral stigmata of chronic liver disease (cirrhosis)
●Mental status changes due to hepatic encephalopathy (acute liver failure, cirrhosis)
Laboratory test findings may include:
●Low level of serum ceruloplasmin (seen with all forms of liver involvement, though less likely with acute liver failure)
●Elevated aminotransferases (all forms of liver involvement)
●Thrombocytopenia (cirrhosis with splenomegaly)
●Coagulopathy (cirrhosis or acute liver failure)
●Coombs-negative hemolytic anemia (often seen in conjunction with acute liver failure)
Two main categories of cognitive impairment: a frontal syndrome and subcortical dementia, with some patients having features of both.
Cognitive impairment: the findings can be subtle and may only be recognized retrospectively.
Patients with a frontal syndrome may demonstrate impulsivity, promiscuity, impaired judgement, apathy, executive dysfunction (poor planning and decision making), decreased attention, and emotional lability. When severe, patients may have pseudobulbar features (sudden outbursts of inappropriate laughter or tearfulness).
Findings in subcortical dementia include slowed thinking, memory loss, and executive dysfunction. It lacks cortical signs such as aphasia, apraxia, or agnosia.

How well did you know this?
1
Not at all
2
3
4
5
Perfectly
42
Q

A 40-year-old woman is brought in by her husband. He explains that she has started getting up during the night and going for walks and then forgetting her way home. She says she has terrible diarrhoea day and night and she wakes to go to the toilet. On examination she has a tremor and you see red scaly patches on her skin. Which vitamin is she most likely to be deficient in?

A. Folate
B. Lipid
C. Thiamine
D. Carbohydrate
E. Vitamin C
F. Protein
G. Vitamin A
H. Vitamin D
I. Niacin
J. Fluoride
K. Iron
L. Zinc
M. Copper
N. Vitamin K
O. Riboflavin
P. Iodine
A

Niacin
Pellagra is a rare but is still a common manifestation of niacin deficiency in poorer countries where the local diet consists of cereal, corn, or sorghum. In industrialized countries, pellagra tends to occur in alcoholics, and has been reported as a complication of bariatric surgery or anorexia nervosa.
The most characteristic finding is the presence of a symmetric hyperpigmented rash, similar in color to a sunburn, which is present in the exposed areas of skin. Other clinical findings are a red tongue and many non-specific symptoms, such as diarrhea and vomiting. Neurologic symptoms include insomnia, anxiety, disorientation, delusions, dementia, and encephalopathy.
Niacin deficiency can also be seen in three other settings:
●Carcinoid syndrome, in which metabolism of tryptophan is to 5-OH tryptophan and serotonin rather than to nicotinic acid. This leads to the deficiency of active forms of niacin and the development of pellagra.
●Prolonged use of isoniazid,(depletes stores of pyridoxal phosphate, which enhances the production of tryptophan, a precursor of niacin). Several other drugs induce niacin deficiency by inhibiting the conversion of tryptophan to niacin, including 5-fluorouracil, pyrazinamide, 6-mercaptopurine, hydantoin, ethionamide, phenobarbital, azathioprine, and chloramphenicol.
●Hartnup disease (MIM #234500), AR congenital disorder. Hartnup disease is associated with a defect of a membrane transport in the intestinal and renal cells normally responsible for the absorption of tryptophan (one of the precursors of nicotinamide-adenine dinucleotide). Through this pathway, around 50 percent of the daily niacin needs are synthesized. Due to the resulting niacin deficiency, all the symptoms of pellagra can be expected. The diagnosis is made by detecting a number of neutral amino acids in the urine, something that is not seen with dietary pellagra.

How well did you know this?
1
Not at all
2
3
4
5
Perfectly
43
Q

pH 6.9 bicarbonate 7.0

A. Metabolic alkalosis
B. Respiratory alkalosis
C. Compensated metabolic acidosis
D. Compensated respiratory acidosis
E. Respiratory acidosis
F. Metabolic acidosis
A

Metabolic acidosis
Metabolic acidosis if unopposed, results in a reduction of the serum bicarbonate concentration (normal is 24 meq/L, with a normal range of 22 to 28 meq/L) and a low arterial pH (normal 7.4, with a normal range of 7.35 to 7.45). Acidemia (as opposed to acidosis) is defined as a low arterial pH (

How well did you know this?
1
Not at all
2
3
4
5
Perfectly
44
Q

pH 6.9 bicarbonate 26

A. Metabolic alkalosis
B. Respiratory alkalosis
C. Compensated metabolic acidosis
D. Compensated respiratory acidosis
E. Respiratory acidosis
F. Metabolic acidosis
A

Respiratory acidosis - disorder that elevates the arterial PCO2 and reduces the pH.
No alteration to bicarb indicates no compensation has occurred

How well did you know this?
1
Not at all
2
3
4
5
Perfectly
45
Q

pH 7.6 bicarbonate 30

A. Metabolic alkalosis
B. Respiratory alkalosis
C. Compensated metabolic acidosis
D. Compensated respiratory acidosis
E. Respiratory acidosis
F. Metabolic acidosis
A

Metabolic alkalosis: relatively common clinical problem that is most often induced by diuretic therapy or the loss of gastric secretions due to vomiting or nasogastric suction.
The development and subsequent maintenance of metabolic alkalosis requires two separate abnormalities:
1. An elevation in the plasma bicarbonate concentration due to excessive hydrogen loss in the urine or gastrointestinal tract, hydrogen movement into the cells, the administration of bicarbonate, or volume contraction around a relatively constant amount of extracellular bicarbonate (called a contraction alkalosis).
2. A decrease in net renal bicarbonate excretion (due to enhanced reabsorption and reduced secretion or markedly reduced renal function) since rapid excretion of the excess bicarbonate (primarily as sodium bicarbonate) would otherwise correct the alkalosis
Several factors are responsible for increased net renal bicarbonate reabsorption in metabolic alkalosis. In the absence of advanced renal failure, one or more of these factors must be present to sustain the high plasma bicarbonate concentration:
●A reduction in ECF volume or reduced effective arterial blood volume, which develops in many edematous states such as congestive heart failure and cirrhosis.
●Chloride depletion and hypochloremia.
●Hypokalemia.
●Increased distal tubule delivery and reabsorption of sodium in exchange for hydrogen ions and potassium.

How well did you know this?
1
Not at all
2
3
4
5
Perfectly
46
Q

pH 7.6 bicarbonate 14

A. Metabolic alkalosis
B. Respiratory alkalosis
C. Compensated metabolic acidosis
D. Compensated respiratory acidosis
E. Respiratory acidosis
F. Metabolic acidosis
A

Respiratory alkalosis
The initial acute response is generated by a variety of pH buffering molecules present in all of the fluid compartments of the body (ie, total body buffering). Reactions with these molecules cause the serum HCO3 to increase (in respiratory acidosis) or decrease (in respiratory alkalosis) within minutes. The acute response is relatively modest.

How well did you know this?
1
Not at all
2
3
4
5
Perfectly
47
Q

pH7.1 bicarbonate 4.0

A. Metabolic alkalosis
B. Respiratory alkalosis
C. Compensated metabolic acidosis
D. Compensated respiratory acidosis
E. Respiratory acidosis
F. Metabolic acidosis
A

Compensated metabolic acidosis
Compensation means improving the pH (towards 7.4) at the expense of worsening the bicarbonate or CO2.
The development of metabolic acidosis will normally generate a compensatory respiratory response. The reduction in the serum bicarbonate and pH caused by the metabolic acidosis results in hyperventilation and a reduction of the pCO2. The Henderson-Hasselbalch equation shows that the pH is determined by the ratio between the HCO3 concentration and pCO2. Thus, this fall in pCO2 will partially mitigate the fall in pH caused by a reduced HCO3.

How well did you know this?
1
Not at all
2
3
4
5
Perfectly
48
Q

pH 7.1 bicarbonate 45

A. Metabolic alkalosis
B. Respiratory alkalosis
C. Compensated metabolic acidosis
D. Compensated respiratory acidosis
E. Respiratory acidosis
F. Metabolic acidosis
A

Compensated respiratory acidosis

Compensation means improving the pH (towards 7.4) at the expense of worsening the bicarbonate or CO2. By

How well did you know this?
1
Not at all
2
3
4
5
Perfectly
49
Q

Which is secreted by the zona glomerulosa?

A. Oestradiol
B. Adrenaline
C. Cortisol
D. Renin
E. Nor adrenaline
F. Corticotrophin (ACTH)
G. Aldosterone
H. Histamine
I. Testosterone
J. Gastrin
A

Aldosterone: steroid hormone (mineralocorticoid family) produced by the outer section (zona glomerulosa) of the adrenal cortex in the adrenal gland.It plays a central role in the regulation of blood pressure mainly by acting on the distal tubules and collecting ducts of the nephron, increasing reabsorption of ions and water in the kidney, to cause the conservation of sodium, secretion of potassium, increase in water retention, and increase in blood pressure and blood volume. When dysregulated, aldosterone is pathogenic and contributes to the development and progression of cardiovascular and renal disease. Aldosterone has exactly the opposite function of the atrial natriuretic hormone secreted by the heart.

Drugs that interfere with the secretion or action of aldosterone are in use as antihypertensives, like lisinopril, which lowers blood pressure by blocking the angiotensin-converting enzyme (ACE), leading to lower aldosterone secretion. The net effect of these drugs is to reduce sodium and water retention but increase retention of potassium. Aldosterone is part of the renin-angiotensin system. Another example is spironolactone, a potassium-sparing diuretic, which decreases blood pressure by releasing fluid from the body while retaining potassium.

How well did you know this?
1
Not at all
2
3
4
5
Perfectly
50
Q

Which is secreted by the zona fasciculata?

A. Oestradiol
B. Adrenaline
C. Cortisol
D. Renin
E. Nor adrenaline
F. Corticotrophin (ACTH)
G. Aldosterone
H. Histamine
I. Testosterone
J. Gastrin
A

Cortisol: steroid hormone, more specifically a glucocorticoid, which is produced by the zona fasciculata of the adrenal cortex. It is released in response to stress and a low level of blood glucose.

Its functions are to increase blood sugar through gluconeogenesis, to suppress the immune system, and to aid the metabolism of fat, protein, and carbohydrate. It also decreases bone formation.

How well did you know this?
1
Not at all
2
3
4
5
Perfectly
51
Q

High levels of which of the above is associated with hyper pigmentation?

A. Oestradiol
B. Adrenaline
C. Cortisol
D. Renin
E. Nor adrenaline
F. Corticotrophin (ACTH)
G. Aldosterone
H. Histamine
I. Testosterone
J. Gastrin
A

Corticotrophin (ACTH): Adrenocorticotropic hormone is a polypeptide tropic hormone produced and secreted by the anterior pituitary gland. It is an important component of the HPA and is often produced in response to biological stress (along with its precursor CRH from the hypothalamus). Its principal effects are increased production and release of corticosteroids. Primary adrenal insufficiency, also called Addison’s disease, occurs when adrenal gland production of cortisol is chronically deficient, resulting in chronically elevated ACTH levels; when a pituitary tumor is the cause of elevated ACTH (from the anterior pituitary) this is known as Cushing’s Disease and the constellation of signs and symptoms of the excess cortisol (hypercortisolism) is known as Cushing’s syndrome. A deficiency of ACTH is a cause of secondary adrenal insufficiency.
ACTH is synthesized from pre-pro-opiomelanocortin (pre-POMC) and undergoes cleavage, producing ACTH & MSH

How well did you know this?
1
Not at all
2
3
4
5
Perfectly
52
Q

Which binds to alpha, beta 1 and beta 2 adrenoreceptors?

A. Oestradiol
B. Adrenaline
C. Cortisol
D. Renin
E. Nor adrenaline
F. Corticotrophin (ACTH)
G. Aldosterone
H. Histamine
I. Testosterone
J. Gastrin
A

Adrenaline: a hormone and neurotransmitter that acts on nearly all body tissues. Its actions vary by tissue type and tissue expression of adrenergic receptors. For example, high levels of epinephrine causes smooth muscle relaxation in the airways but causes contraction of the smooth muscle that lines most arterioles.
Epinephrine is a nonselective agonist of all adrenergic receptors, including the major subtypes α1, α2, β1, β2, and β3. Epinephrine’s binding to these receptors triggers a number of metabolic changes. Binding to α-adrenergic receptors inhibits insulin secretion by the pancreas, stimulates glycogenolysis in the liver and muscle, and stimulates glycolysis in muscle. β-Adrenergic receptor binding triggers glucagon secretion in the pancreas, increased adrenocorticotropic hormone (ACTH) secretion by the pituitary gland, and increased lipolysis by adipose tissue. Together, these effects lead to increased blood glucose and fatty acids, providing substrates for energy production within cells throughout the body.
The adult dose for refractory anaphylactic shock is usually 1 mg (1:10,000) IV/IO over 5 min and for cardiac arrest 1 mg (1:10,000) as an IV/IO push.

How well did you know this?
1
Not at all
2
3
4
5
Perfectly
53
Q

Reflects genetic abnormality affecting the innate immune system, often in a site-specific manner

Immunopathology
Auto-Immune Disease
Auto-Inflammatory Disease
Familial Mediterranean Fever
IPEX (Immune dysregulation, polyendocrinopathy, enteropathy, X-linked)
A

Auto-Inflammatory Disease: patients with autoinflammatory diseases do not produce autoantibodies or antigen-specific T or B cells. Instead, the autoinflammatory diseases are characterized by errors in the innate immune system.
The syndromes are diverse, but tend to cause episodes of fever, joint pains, skin rashes, abdominal pains and may lead to chronic complications such as amyloidosis.
Most autoinflammatory diseases are genetic and present during childhood. The most common genetic autoinflammatory syndrome is familial Mediterranean fever, which causes short episodes of fever, abdominal pain, serositis, lasting less than 72 hours. It is caused by mutations in the MEFV gene, which codes for the protein pyrin.

How well did you know this?
1
Not at all
2
3
4
5
Perfectly
54
Q

Single gene mutation involving MEFV and affecting the inflammasome complex, resulting in recurrent episodes of serositis

Immunopathology
Auto-Immune Disease
Auto-Inflammatory Disease
Familial Mediterranean Fever
IPEX (Immune dysregulation, polyendocrinopathy, enteropathy, X-linked)
A

Familial Mediterranean Fever: a disorder characterized by sporadic attacks of fever and serosal inflammation.
Most patients with FMF experience their first attack in early childhood; in 65 percent of cases, the initial attack occurs before the age of 10, and in 90 percent before the age of 20. The typical manifestations of the disease are recurrent attacks of severe pain (due to serositis at one or more sites) and fever, lasting one to three days, and then resolving spontaneously. In between attacks, patients feel entirely well. Acute attacks of FMF are accompanied by elevation in many of the serum markers of systemic inflammation (ESR, beta-2 microglobulin, CRP, SAA (serum amyloid protein), and fibrinogen). The diagnosis is usually made on clinical grounds and a response to colchicine. Genetic testing is also available but an appreciable proportion of patients with clinical FMF have only one or no MEFV mutations.

How well did you know this?
1
Not at all
2
3
4
5
Perfectly
55
Q

May describe damage resulting from the immune response to on-going infection

Immunopathology
Auto-Immune Disease
Auto-Inflammatory Disease
Familial Mediterranean Fever
IPEX (Immune dysregulation, polyendocrinopathy, enteropathy, X-linked)
A

Immunopathology

How well did you know this?
1
Not at all
2
3
4
5
Perfectly
56
Q

Reflects genetic abnormality affecting the adaptive immune system, and is often associated with the presence of auto-antibodies

Immunopathology
Auto-Immune Disease
Auto-Inflammatory Disease
Familial Mediterranean Fever
IPEX (Immune dysregulation, polyendocrinopathy, enteropathy, X-linked)
A

Auto-Immune Disease: pathologic condition caused by an adaptive autoimmune response, an immune response directed against an antigen within the body of the host, termed a self-antigen. The response may be induced by a foreign or self antigen. It usually involves a T-cell and B-cell response This may be restricted to certain organs (e.g. in autoimmune thyroiditis) or involve a particular tissue in different places (e.g. Goodpasture’s disease which may affect the basement membrane in both the lung and the kidney).
The treatment of autoimmune diseases is typically with immunosuppression—medication that decreases the immune response.
Diseases in which there is chronic inflammation but no evidence of autoreactive T cells or B cells, termed autoinflammatory diseases, are associated with the innate immune response.

How well did you know this?
1
Not at all
2
3
4
5
Perfectly
57
Q

Single gene mutation involving FOXp3 resulting in abnormality of T reg cells

Immunopathology
Auto-Immune Disease
Auto-Inflammatory Disease
Familial Mediterranean Fever
IPEX (Immune dysregulation, polyendocrinopathy, enteropathy, X-linked)
A

IPEX (Immune dysregulation, polyendocrinopathy, enteropathy, X-linked): is a rare and potentially fatal autoimmune lymphoproliferative disorder in which regulatory T cells (Tregs) are quantitatively or functionally deficient. These defects are caused by various mutations in FOXP3, a transcription factor fundamental to the functional differentiation of Treg cells. IPEX classically presents in male infants with a triad of enteropathy, dermatitis, and autoimmune endocrinopathy (usually type 1 diabetes or thyroiditis). Some patients have severe food allergy and/or immune-mediated cytopenias. IPEX is due to loss of function mutations in FOXP3. These mutations result in quantitative or functional deficiencies of Treg cells and thereby cause autoimmune disease and allergic inflammation. A large number of mutations have been described, many of which are familial, although sporadic cases have been reported

How well did you know this?
1
Not at all
2
3
4
5
Perfectly
58
Q

Polygenic Auto-inflammatory disease. Estimated 30% of patients have a mutation of CARD15 which may affect response of myeloid cells to bacteria

Crohn’s disease
Ankylosing spondylitis
Giant cell arteritis

A

Crohn’s Disease: disorder of uncertain etiology that is characterized by transmural inflammation of the gastrointestinal tract. CD may involve the entire gastrointestinal tract from mouth to the perianal area. Fatigue, prolonged diarrhea with abdominal pain, weight loss, and fever, with or without gross bleeding, are the hallmarks of CD. Patients can present with symptoms secondary to the transmural involvement of the bowel, including fistulas, phlegmon, abscess, perianal disease, and/or malabsorption. Extraintestinal manifestations, such as arthritis, eye and skin disorders, biliary tract involvement, and kidney stones may occur and tend to be more frequent with colonic involvement. The diagnosis of CD is established with endoscopic and imaging studies of the bowel in a patient with a compatible clinical history. Colonoscopy with intubation of the terminal ileum is used to establish the diagnosis of ileocolonic CD. We perform an MR enterography as the initial test to evaluate the small intestine. Wireless capsule endoscopy may also be useful for detecting small bowel involvement.
CARD15 or inflammatory bowel disease protein 1 (IBD1) is a protein that is encoded by the NOD2 gene located on chromosome 16. NOD2 plays an important role in the immune system. It recognizes bacterial molecules (peptidoglycans) and stimulates an immune reaction.

How well did you know this?
1
Not at all
2
3
4
5
Perfectly
59
Q

Mixed pattern auto-inflammatory disease/auto-immune disease with >90% heritability that results in inflammation, typically involving the sacra-iliac joints and responds to TNF alpha antagonists

Crohn’s disease
Ankylosing spondylitis
Giant cell arteritis

A

Ankylosing spondylitis: a form of spondyloarthritis (SpA - a family of disorders characterized by inflammation around the entheses (the sites of ligament insertion into bone) and an association with the human leukocyte antigen (HLA)-B27)), is a chronic inflammatory disease of the axial skeleton manifested by back pain and progressive stiffness of the spine; it can also involve the hips, shoulders, and peripheral joints.
The symptoms of AS include inflammatory back pain, buttock pain, limited spinal mobility, limited chest expansion, hip and shoulder pain, and enthesitis. Patients may have peripheral arthritis, dactylitis, and constitutional features. Inflammatory low back pain is characterized by age of onset

How well did you know this?
1
Not at all
2
3
4
5
Perfectly
60
Q

Polygenic auto-inflammatory disease resulting in a large vessel vasculitis and requiring immediate treatment with high dose corticosteroids

Crohn’s disease
Ankylosing spondylitis
Giant cell arteritis

A

Giant cell arteritis: chronic vasculitis of large- and medium-sized vessels. The mean age at diagnosis is approximately 72 years, and the disease essentially never occurs in individuals younger than 50. GCA is characteristically a systemic illness and although vascular involvement may be widespread, symptomatic blood vessel inflammation most frequently involves the cranial branches of the arteries that originate from the aortic arch. The most feared complication of GCA, visual loss, is one potential result of the cranial arteritis associated with this disease. Consider if: new headache, Abrupt onset of visual disturbances, Symptoms of PMR, Unexplained fever or anemia, Elevated ESR and/or serum CRP. Temporal artery biopsy is the gold standard for the diagnosis of GCA.

How well did you know this?
1
Not at all
2
3
4
5
Perfectly
61
Q

Tyrosine phosphatase expressed in lymphocytes associated with development of auto-immune disease, including rheumatoid arthritis

HLA DR4
PTPN22
HLA DR15
CTLA4

A

PTPN22: The protein tyrosine phosphatase N22 (PTPN22) gene helps regulate both T and B cells. The frequency of a single nucleotide polymorphism (SNP) in the gene was increased in RA patients

How well did you know this?
1
Not at all
2
3
4
5
Perfectly
62
Q

MHC class II molecule that is associated with development of auto-immune disease, including rheumatoid arthritis

HLA DR4
PTPN22
HLA DR15
CTLA4

A

HLA DR4: The DR4 “family” of alleles contains at least 22 members, only some of which are associated with RA.
Two alleles, DRB10401 and DRB10404, primarily account for the originally observed serological association of DR4 with rheumatoid arthritis (RA) in Caucasians. The most strongly associated RA alleles share a region of highly similar amino acid sequence, called the shared epitope, located at amino acids 67 to 74

How well did you know this?
1
Not at all
2
3
4
5
Perfectly
63
Q

Receptor for CD80/CD86, expressed on T cells, that influences T cell activation and is associated with auto-immune disease, including diabetes and thyroid disease

HLA DR4
PTPN22
HLA DR15
CTLA4

A

CTLA4 - also known as CD152, is a protein receptor that downregulates the immune system. CTLA4 is found on the surface of T cells, which lead the cellular immune attack on antigens. The T cell attack can be turned on by stimulating the CD28 receptor on the T cell. The T cell attack can be turned off by stimulating the CTLA4 receptor, which acts as an “off” switch.
The CTLA4 protein is encoded by the CTLA4 gene. CTLA4 is a member of the immunoglobulin superfamily, which is expressed on the surface of Helper T cells and transmits an inhibitory signal to T cells. CTLA4 is similar to the T-cell co-stimulatory protein, CD28, and both molecules bind to CD80 and CD86, also called B7-1 and B7-2 respectively, on antigen-presenting cells. CTLA4 transmits an inhibitory signal to T cells, whereas CD28 transmits a stimulatory signal. Intracellular CTLA4 is also found in regulatory T cells and may be important to their function.

How well did you know this?
1
Not at all
2
3
4
5
Perfectly
64
Q

Antigen presenting molecule that is strongly associated with the development of anti-GBM anti-bodies

HLA DR4
PTPN22
HLA DR15
CTLA4

A

HLA DR15: positively associated Goodpasture syndrome, early age onset multiple sclerosis, pernicious anaemia, sarcoidosis, hypocretin deficiency associated narcolepsy and a predisposition for postmenopausal osteoporosis.

How well did you know this?
1
Not at all
2
3
4
5
Perfectly
65
Q

T cell that expresses FoxP3 and CD25 and secretes cytokines IL-10 and TGF-beta to suppress activation of other T cells

Central tolerance of T cells
T reg cells
Central tolerance of B cells
T cell Anergy

A
T reg cells
The dominant cytokines for Treg induction in humans are TGF-beta and IL-2. Induction of these cytokines leads to the activation of STAT5, which engages the transcription factor Foxp3. Treg secrete IL-10, TGF-beta, and IL-35. TGF-beta acts as the amplifying cytokine for Treg cells. T regulatory (Treg) cells represent a major subset of CD4+ T cells that may be involved in regulating and attenuating the activity of the three T helper subsets.
T cells can be categorized based upon cell surface expression of CD4 or CD8. CD4+ cells recognize antigen presented in the context of class II major histocompatibility complex (MHC), while CD8+ cells recognize antigen presented in the context of class I MHC. CD4+ T helper subsets include T helper type 1 (Th1), T helper type 2 (Th2), and T helper type 17 (Th17) cells
Th1 are pivotal in defense against intracellular microorganisms in general and mycobacteria in particular. Patients with mutations in the interferon-gamma (IFN-gamma) receptor or interleukin-12 (IL-12) receptor present with recurrent infections with mycobacteria and Salmonella. Th2 cells are integral in expelling parasitic infestations. Th17 seem to play a significant role in defense against extracellular bacteria and some fungi.
Th1 and Th17 cells play major roles in autoimmunity, whereas Th2 cells are the hallmark of atopic disease.
How well did you know this?
1
Not at all
2
3
4
5
Perfectly
66
Q

Within the thymus cells that bind with low affinity to HLA molecules die by neglect and those that bind with a high affinity to HLA molecules are deleted

Central tolerance of T cells
T reg cells
Central tolerance of B cells
T cell Anergy

A

Central tolerance of T cells: T cells are selected for survival much more rigorously than B cells. They undergo both positive and negative selection to produce T cells that recognize self- major histocompatibility complex (MHC) molecules but do not recognize self-peptides. T cell tolerance is induced in the thymus.
Positive selection occurs in the thymic cortex. This process is primarily mediated by thymic epithelial cells, which are rich in surface MHC molecules. If a maturing T cell is able to bind to a surface MHC molecule in the thymus, it is saved from programmed cell death; those cells failing to recognize MHC on thymic epithelial cells will die.
T cells may also undergo negative selection in a process analogous to the induction of self-tolerance in B cells, this occurs in the cortex, at the cortico-medullary junction, and the medulla (mediated in the medulla predominately by medullary thymic epithelial cells (mTECs) and dendritic cells). mTEC display “self” antigens to developing T-cells and signal those “self-reactive” T-cells to die via programmed cell death (apoptosis) and thereby deleted from the T cell repertoire.
Regulatory T cells are another group of T cells maturing in the thymus, they are also involved with immune regulation but are not directly involved in central tolerance.
Genetic defects in central tolerance can lead to autoimmunity: Autoimmune Polyendocrinopathy Syndrome Type I (APECED) is caused by mutations in the human gene AIRE (chr21 - AR). This leads to a lack of expression of peripheral antigens in the thymus, and hence a lack of negative selection towards key peripheral proteins such as insulin. It is a mild immune deficiency, leading to persistent mucosal and cutaneous infections with candida yeasts. There is also decreased function of the spleen (hyposplenism).
Autoimmune dysfunction of the parathyroid gland (leading to hypocalcaemia) and the adrenal gland (Addison’s disease: hypoglycemia, hypotension and severe reactions in disease).

How well did you know this?
1
Not at all
2
3
4
5
Perfectly
67
Q

T cells that recognise HLA/peptide complexes on cells that do not express co-stimulatory molecules subsequently fail to respond to stimulation with antigen

Central tolerance of T cells
T reg cells
Central tolerance of B cells
T cell Anergy

A

T cell anergy. T-cell anergy can arise when the T-cell does not receive appropriate co-stimulation in the presence of specific antigen recognition. B-cell anergy can be induced by exposure to soluble circulating antigen, and is often marked by a downregulation of surface IgM expression and partial blockade of intracellular signaling pathways.

How well did you know this?
1
Not at all
2
3
4
5
Perfectly
68
Q

Cells that bind to polyvalent antigens in the bone marrow are deleted

Central tolerance of T cells
T reg cells
Central tolerance of B cells
T cell Anergy

A

Central tolerance of B cells: Tolerance is classified into central tolerance or peripheral tolerance depending on where the state is originally induced—in the thymus and bone marrow (central) or in other tissues and lymph nodes (peripheral). Central tolerance is the main way the immune system learns to discriminate self from non-self. Peripheral tolerance is key to preventing over-reactivity of the immune system to various environmental entities (allergens, gut microbes, etc.). Deficits in central or peripheral tolerance also cause autoimmune disease. Central tolerance refers to the tolerance established by deleting autoreactive lymphocyte clones before they develop into fully immunocompetent cells. It occurs during lymphocyte development in the thymus and bone marrow for T and B lymphocytes, respectively. In these tissues, maturing lymphocytes are exposed to self-antigens presented by medullary thymic epithelial cells and thymic dendritic cells, or bone marrow cells.

How well did you know this?
1
Not at all
2
3
4
5
Perfectly
69
Q

T cell mediated reaction to antigen

Type I hypersensitivity
Type II hypersensitivity
Type III hypersensitivity
Type IV hypersensitivity

Goodpasture Disease
Eczema
SLE
Multiple Sclerosis

A

Type IV hypersensitivity - Multiple sclerosis (oligodendrocyte proteins), diabetes (pancreatic beta cell antigens), allergic contact dermatitis (metals), RA (type II collagen), Hashimoto’s thyroiditis (thyroglobulin antigen)

Delayed hypersensitivity
Involves T cells and cytokines

Type IV reactions are not mediated by antibodies, in contrast to the other three types above. They involve the activation and expansion of T cells, which requires time (normally many hours or days after antigen exposure), hence the name delayed-type hypersensitivity (DTH). In some cases, other cell types (eg, macrophages, eosinophils, or neutrophils) are also involved. Type IV reactions can take many different forms, which vary in significance from inconvenient to life threatening.
CD4+ helper T cells recognize antigen in a complex with Class 2 major histocompatibility complex. The antigen-presenting cells in this case are macrophages that secrete IL-12, which stimulates the proliferation of further CD4+ Th1 cells. CD4+ T cells secrete IL-2 and interferon gamma, further inducing the release of other Th1 cytokines, thus mediating the immune response. Activated CD8+ T cells destroy target cells on contact, whereas activated macrophages produce hydrolytic enzymes and, on presentation with certain intracellular pathogens, transform into multinucleated giant cells.

How well did you know this?
1
Not at all
2
3
4
5
Perfectly
70
Q

Anti-bodies react with antigen form immune complexes that deposit, often causing vasculitic skin rash, glomerulonephritis and arthritis

Type I hypersensitivity
Type II hypersensitivity
Type III hypersensitivity
Type IV hypersensitivity

Goodpasture Disease
Eczema
SLE
Multiple Sclerosis

A

Type III hypersensitivity - SLE (nuclear antigens), serum sickness

Immune complex mediated hypersensitivity
Involves deposition of Ab/Ag complexes in tissue.

Occurs when there is an excess of antigen, leading to small immune complexes being formed that do not fix complement and are not cleared from the circulation. It involves soluble antigens that are not bound to cell surfaces (as opposed to those in type II hypersensitivity). When these antigens bind antibodies, immune complexes of different sizes form. Large complexes can be cleared by macrophages but macrophages have difficulty in the disposal of small immune complexes. These immune complexes insert themselves into small blood vessels, joints, and glomeruli, causing symptoms. Unlike the free variant, a small immune complex bound to sites of deposition (like blood vessel walls) are far more capable of interacting with complement; these medium-sized complexes, formed in the slight excess of antigen, are viewed as being highly pathogenic. Such depositions in tissues often induce an inflammatory response, and can cause damage wherever they precipitate. The cause of damage is as a result of the action of cleaved complement anaphylotoxins C3a and C5a, which, respectively, mediate the induction of granule release from mast cells (from which histamine can cause urticaria), and recruitment of inflammatory cells into the tissue (mainly those with lysosomal action, leading to tissue damage through frustrated phagocytosis by PMNs and macrophages)
Vasculitis, glomerulonephritis and arthritis are commonly associated conditions as a result of type III hypersensitivity responses.

How well did you know this?
1
Not at all
2
3
4
5
Perfectly
71
Q

Antibodies react with tissue antigens resulting in damage to the tissue

Type I hypersensitivity
Type II hypersensitivity
Type III hypersensitivity
Type IV hypersensitivity

Goodpasture Disease
Eczema
SLE
Multiple Sclerosis

A

Type II hypersensitivity - Goodpasture Disease, Graves Disease

Cytotoxic hypersensitivity
Involves anti-bodies binding to cells with complement mediated host cell destruction (or inhibition/activation or receptor signalling)

Often involves auto-immunity - the antibodies produced by the immune response bind to antigens on the patient’s own cell surfaces. The antigens recognized in this way may either be intrinsic (“self” antigen, innately part of the patient’s cells) or extrinsic (adsorbed onto the cells during exposure to some foreign antigen, possibly as part of infection with a pathogen). These cells are recognized by macrophages or dendritic cells, which act as antigen-presenting cells. This causes a B cell response, wherein antibodies are produced against the foreign antigen.
Examples include Red blood cells in Haemolytic Anaemia, Acetylcholine receptors in Myasthenia Gravis, and TSH receptors in Grave’s Disease. Another example of type II hypersensitivity reaction is Goodpasture’s syndrome where the basement membrane(containing collagen type IV) in the lung and kidney is attacked by one’s own antibodies

How well did you know this?
1
Not at all
2
3
4
5
Perfectly
72
Q

IgE mediated activation of mast cells leading to the release of substances including histamine, leukotrienes and prostaglandins. Reaction usually to foreign rather than self antigen

Type I hypersensitivity
Type II hypersensitivity
Type III hypersensitivity
Type IV hypersensitivity

Goodpasture Disease
Eczema
SLE
Multiple Sclerosis

A

Type I hypersensitivity - Eczema

Anaphylactic hypersensitivity - involves IgE and mast cells
Rarely auto-immunity
Anaphylaxis, atopic asthma

An allergic reaction provoked by reexposure to a specific type of antigen referred to as an allergen. In type 1 hypersensitivity, an antigen is presented to CD4+ Th2 cells specific to the antigen that stimulate B-cell production of IgE antibodies also specific to the antigen. The difference between a normal infectious immune response and a type 1 hypersensitivity response is that in type 1 hypersensitivity the antibody is IgE instead of IgA, IgG, or IgM. During sensitisation, the IgE antibodies bind to Fcε receptors on the surface of tissue mast cells and blood basophils. Mast cells and basophils coated by IgE antibodies are “sensitised.” Later exposure to the same allergen cross-links the bound IgE on sensitised cells, resulting in degranulation and the secretion of pharmacologically active mediators such as histamine, leukotriene (LTC4 and LTD4), and prostaglandin that act on the surrounding tissues. The principal effects of these products are vasodilation and smooth-muscle contraction.

Type 1 hypersensitivity can be further classified into an immediate and late-phase reaction. The immediate hypersensitivity reaction occurs minutes after exposure and includes release of vasoactive amines and lipid mediators, whereas the late-phase reaction occurs 2–4 hours after exposure and includes the release of cytokines

How well did you know this?
1
Not at all
2
3
4
5
Perfectly
73
Q

Binding of immune complexes to this protein triggers the classical pathway of complement activation

C3
C1
C9
MBL

A

C1 The classical pathway is triggered by activation of the C1-complex.
The following are the basic functions of complement:
Opsonization - enhancing phagocytosis of antigens
Chemotaxis - attracting macrophages and neutrophils
Cell Lysis - rupturing membranes of foreign cells
Agglutination- clustering and binding of pathogens together (sticking)

How well did you know this?
1
Not at all
2
3
4
5
Perfectly
74
Q

Cleavage of this protein may be triggered via the classical, MBL or alternative pathways

C3
C1
C9
MBL

A

C3 - It plays a central role in the complement system and contributes to innate immunity. In humans it is encoded on chromosome 19 by a gene called C3. Its activation is required for both classical and alternative complement activation pathways. People with C3 deficiency are susceptible to bacterial infection.
In all three pathways, C3-convertase cleaves and activates component C3, creating C3a and C3b, and causing a cascade of further cleavage and activation events. C3b binds to the surface of pathogens, leading to greater internalization by phagocytic cells by opsonization. C5a is an important chemotactic protein, helping recruit inflammatory cells.
The following are the basic functions of complement:
Opsonization - enhancing phagocytosis of antigens
Chemotaxis - attracting macrophages and neutrophils
Cell Lysis - rupturing membranes of foreign cells
Agglutination- clustering and binding of pathogens together (sticking)

How well did you know this?
1
Not at all
2
3
4
5
Perfectly
75
Q

Binds to microbial surface carbohydrates to activate the complement cascade in an immune complex independent manner

C3
C1
C9
MBL

A

MBL: This pathway is activated by binding of MBL to mannose residues on the pathogen surface, which activates the MBL-associated serine proteases, MASP-1, and MASP-2 (very similar to C1r and C1s, respectively), which can then split C4 into C4a and C4b and C2 into C2a and C2b. C4b and C2a then bind together to form the classical C3-convertase, as in the classical pathway.

How well did you know this?
1
Not at all
2
3
4
5
Perfectly
76
Q

Part of the final common pathway resulting in the generation of the Membrane Attack Complex (MAC)

C3
C1
C9
MBL

A

C9 - protein involved in the complement system. It is a member of the Complement membrane attack complex (MAC) and induces pores on membranes.

How well did you know this?
1
Not at all
2
3
4
5
Perfectly
77
Q

Recurrent chest infections with high neutrophil count on FBC, but no abscess formation

IFN gamma receptor deficiency
Lymphocyte adhesion deficiency
Chronic granulomatous disease
Kostmann Syndrome

A

Lymphocyte adhesion deficiency: In each LAD syndromes, leukocytes (particularly neutrophils) cannot leave the vasculature to migrate normally into tissues under conditions of inflammation or infection.
LAD I: beta 2 integrin family (CD18) is deficient or defective, AR. Characterized clinically by recurrent bacterial infections, a persistent neutrophilia that increases markedly during infection, absent pus formation (a hallmark finding), and impaired wound healing (picture 2). A classic presenting infection is omphalitis, with delayed separation of the umbilical cord
LAD II: fucosylated carbohydrate ligands for selectins are absent, AR, results in defective rolling of hematopoietic cells
LAD III: activation of all beta integrins (1, 2, and 3) is defective, AR

How well did you know this?
1
Not at all
2
3
4
5
Perfectly
78
Q

Recurrent infections with hepatosplenomegaly and abnormal dihydrorhodamine test

IFN gamma receptor deficiency
Lymphocyte adhesion deficiency
Chronic granulomatous disease
Kostmann Syndrome

A

Chronic granulomatous disease: genetically heterogeneous group of immunodeficiencies. The core defect is a failure of phagocytic cells to kill organisms that they have engulfed because of defects in a system of enzymes that produce free radicals and other toxic small molecules.
Classically, patients with chronic granulomatous disease will suffer from recurrent bouts of infection due to the decreased capacity of their immune system to fight off disease-causing organisms. The recurrent infections they acquire are specific and are: pneumonia, abscesses of the skin, tissues and organs, suppurative arthritis, osteomyelitis, bacteremia/fungemia, superficial skin infections such as cellulitis or impetigo.
The nitroblue-tetrazolium (NBT) test is the original and most widely known test for chronic granulomatous disease. It is negative in CGD, meaning that it does not turn blue. A similar test uses dihydrorhodamine (DHR) where whole blood is stained with DHR, incubated, and stimulated to produce superoxide radicals which oxidize DHR to rhodamin in cells with normal function.

How well did you know this?
1
Not at all
2
3
4
5
Perfectly
79
Q

Recurrent infections with no neutrophils on FBC

IFN gamma receptor deficiency
Lymphocyte adhesion deficiency
Chronic granulomatous disease
Kostmann Syndrome

A

Kostmann Syndrome: group of diseases that affect myelopoiesis most prominently, causing severe congenital neutropenia (SCN), usually without other prominent overt physical malformations. SCN manifests in infancy with severe infections. Over 90% of SCN responds to treatment with granulocyte colony-stimulating factor (filgrastim), which has significantly improved survival. Postman Disease - Type 3 (SCN3) is a rare autosomal recessive condition in which severe chronic neutropenia is detected soon after birth, but the commonest subtype of Kostmann syndrome, SCN1, is autosomal dominant. Infants with SCN have frequent infections: 50% have a significant infection within 1 month, most others by 6 months

How well did you know this?
1
Not at all
2
3
4
5
Perfectly
80
Q

Infection with atypical mycobacterium. Normal FBC

IFN gamma receptor deficiency
Lymphocyte adhesion deficiency
Chronic granulomatous disease
Kostmann Syndrome

A

IFN gamma receptor deficiency: patients tend to develop severe disseminated mycobacterial disease in infancy or early childhood, requiring continuous antimycobacterial therapy.

How well did you know this?
1
Not at all
2
3
4
5
Perfectly
81
Q

Catheter associated BSI

MRSA
C. difficile
E. coli
MSSA
R Gram negs
Yeasts / Candida
A

MRSA, MSSA, R Gram negs, Yeasts/Candida

BSI = blood stream infection

How well did you know this?
1
Not at all
2
3
4
5
Perfectly
82
Q

Urinary catheter associated UTI

MRSA
C. difficile
E. coli
MSSA
R Gram negs
Yeasts / Candida
A

MRSA, E. coli, R Gram negs, Yeasts/Candida

How well did you know this?
1
Not at all
2
3
4
5
Perfectly
83
Q

Surgical site infection

MRSA
C. difficile
E. coli
MSSA
R Gram negs
Yeasts / Candida
A

MRSA, MSSA, R Gram negs

How well did you know this?
1
Not at all
2
3
4
5
Perfectly
84
Q

Ventilator associated pneumonia

MRSA
C. difficile
E. coli
MSSA
R Gram negs
Yeasts / Candida
A

E. coli

How well did you know this?
1
Not at all
2
3
4
5
Perfectly
85
Q

Antibiotic associated diarrhoea

MRSA
C. difficile
E. coli
MSSA
R Gram negs
Yeasts / Candida
A

C. difficile

How well did you know this?
1
Not at all
2
3
4
5
Perfectly
86
Q

B Cell deficiencies

Adult with bronchiectasis, recurrent sinusitis and development of atypical SLE

IgA deficiency
Common Variable Immunodeficiency
Bruton’s X linked hypogammaglobulinaemia
X linked hyper IgM syndrome due to CD40 ligand mutation

A

Common Variable Immunodeficiency: the most common form of severe antibody deficiency affecting both children and adults. The characteristic immune defect in CVID is impaired B cell differentiation with defective production of immunoglobulin. CVID is defined by low total serum concentrations of immunoglobulin G (IgG), as well as low immunoglobulin A (IgA) and/or immunoglobulin M (IgM), poor or absent response to immunization, and the absence of any other defined immunodeficiency state.
Age of onset is variable. Most patients are diagnosed between the ages of 20 and 40 years. Bacterial infections of the sinopulmonary tract, particularly sinusitis and pneumonia, are experienced by most patients with CVID. In addition to recurrent infections, patients with CVID have evidence of immune dysregulation leading to autoimmunity, a variety of inflammatory disorders, and malignant disease. Patients may suffer from chronic lung disease, gastrointestinal and liver disorders, granulomatous infiltrations, lymphoid hyperplasia, splenomegaly, or malignancy.
The diagnosis of CVID requires a suggestive clinical history, a reduced total serum concentration of IgG, plus low IgA or IgM, and poor responses to both protein- and polysaccharide-based vaccines

How well did you know this?
1
Not at all
2
3
4
5
Perfectly
87
Q

B Cell deficiencies

Recurrent bacterial infections in a child, episode of pneumocystis pneumonia, high IgM, absent IgA and IgG

IgA deficiency
Common Variable Immunodeficiency
Bruton’s X linked hypogammaglobulinaemia
X linked hyper IgM syndrome due to CD40 ligand mutation

A
X linked hyper IgM syndrome due to CD40 ligand mutation: heterogeneous group of congenital and acquired conditions characterized by defective class-switch recombination (CSR), resulting in normal or increased levels of serum IgM associated with deficiency of IgG, IgA, and IgE and poor antibody function. 
CD40 ligand (CD40L) deficiency is the most common form of hyper-IgM syndrome. It is inherited as an X-linked trait. This disease affects the interaction between activated CD4+ T cells and cell types expressing CD40 (B cells, dendritic cells, monocyte/macrophages, platelets, activated endothelial and epithelial cells).
The clinical phenotype of CD40L deficiency is marked not only by recurrent sinopulmonary infections, but also by opportunistic infections and liver disease
How well did you know this?
1
Not at all
2
3
4
5
Perfectly
88
Q

B Cell deficiencies

1 year old boy. Recurrent bacterial infections. CD4 and CD8 T cells present. B cells absent, IgG, IgA, IgM absent

IgA deficiency
Common Variable Immunodeficiency
Bruton’s X linked hypogammaglobulinaemia
X linked hyper IgM syndrome due to CD40 ligand mutation

A

Bruton’s X linked hypogammaglobulinaemia: primary humoral immunodeficiency characterized by severe hypogammaglobulinemia, antibody deficiency, and increased susceptibility to infection. Clinical symptoms (infections) are generally first noted between 3 and 18 months of age.
XLA is due to defects in a signal transduction molecule called Bruton tyrosine kinase (Btk). Patients who present because of clinical symptoms are usually initially identified by significant hypogammaglobulinemia/agammaglobulinemia and the near absence of CD19+ B cells. The diagnosis is then confirmed with molecular studies identifying a mutation in the BTK gene. The cornerstone of treatment for XLA is replacement therapy with immune globulin.
As a consequence of the failure of B cell development, patients affected by a mutation in BTK have significantly reduced levels of B lymphocytes in their blood and tissues, fail to generate plasma cells, and have severely-decreased production of all classes of immunoglobulins with markedly defective antibody responses.
As a result of their deficient humoral immune response, patients with XLA have an increased susceptibility to infection by encapsulated bacteria and certain blood-borne viruses, reflecting the important role of antibody in opsonization of encapsulated bacteria and neutralization of blood-borne enteroviruses.

How well did you know this?
1
Not at all
2
3
4
5
Perfectly
89
Q

B Cell deficiencies

Recurrent respiratory tract infections absent IgA, normal IgM and IgG

IgA deficiency
Common Variable Immunodeficiency
Bruton’s X linked hypogammaglobulinaemia
X linked hyper IgM syndrome due to CD40 ligand mutation

A

IgA deficiency: the selective deficiency of serum IgA (ie, serum levels of IgG and IgM are normal) in a patient older than four years of age, in whom other causes of hypogammaglobulinemia have been excluded.IgA is concentrated in mucosal secretions and is believed to be important in the immune functioning of the mucosal barrier. However, the vast majority of patients with sIgAD do not suffer from increased infections, possibly because there are redundant immune mechanisms that can compensate in most IgA-deficient individuals. Only a minority of IgA-deficient individuals are symptomatic. These patients may develop recurrent sinopulmonary infections, autoimmune disorders, gastrointestinal disorders, allergic diseases, and rare anaphylactic reactions to blood products.

How well did you know this?
1
Not at all
2
3
4
5
Perfectly
90
Q

T cell deficiency

Severe recurrent infections from 3 months, CD4 and CD8 T cells absent, B cells present, IgM present, IgA and IgG absent

Bare lymphocyte syndrome type II
X linked SCID
Di George’s Syndrome
IFN gamma receptor deficiency

A

X linked SCID: is the most common form of SCID. Patients usually present in the newborn period with recurrent severe infections, chronic diarrhea, and failure to thrive. X-linked SCID is due to defects in the common gamma chain (gamma-c, IL2RG). Peripheral T and NK cells are very low to absent and immunoglobulins are also very low to absent despite normal B cell numbers. Hematopoietic cell transplantation is curative.

How well did you know this?
1
Not at all
2
3
4
5
Perfectly
91
Q

T cell deficiency

Young adult with chronic infection with Mycobacterium marinum

Bare lymphocyte syndrome type II
X linked SCID
Di George’s Syndrome
IFN gamma receptor deficiency

A

IFN gamma receptor deficiency: patients tend to develop severe disseminated mycobacterial disease in infancy or early childhood, requiring continuous antimycobacterial therapy.

How well did you know this?
1
Not at all
2
3
4
5
Perfectly
92
Q

T cell deficiency

Recurrent infections in childhood, abnormal facial features, congenital heart disease, normal B cells, low T cells, low IgA and IgG

Bare lymphocyte syndrome type II
X linked SCID
Di George’s Syndrome
IFN gamma receptor deficiency

A

Di George’s Syndrome: classic triad of features on presentation is conotruncal cardiac anomalies, hypoplastic thymus, and hypocalcemia, although the phenotype is variable. Palatal abnormalities and developmental delay are common.
●Immunodeficiency is common in patients with DGS and can range from recurrent sinopulmonary infections (partial DGS) to severe combined immunodeficiency (SCID) (complete DGS). The severity of the immunodeficiency is related to the degree of thymic hypoplasia.
●Other features of DGS that present outside of infancy and into adulthood include recurrent infections in patients with partial DGS, developmental delay, psychiatric abnormalities, and chronic inflammatory diseases

How well did you know this?
1
Not at all
2
3
4
5
Perfectly
93
Q

T cell deficiency

6 month baby with 2 recent severe bacterial infections. T cells present - but only CD8+ population. B cells present, IgM present but IgG absent

Bare lymphocyte syndrome type II
X linked SCID
Di George’s Syndrome
IFN gamma receptor deficiency

A

Bare lymphocyte syndrome type II: rare recessive genetic condition in which a group of genes called major histocompatibility complex class II (MHC class II) are not expressed. The result is that the immune system is severely compromised and cannot effectively fight infection. Clinically, this is known as a severe combined immunodeficiency (SCID).

How well did you know this?
1
Not at all
2
3
4
5
Perfectly
94
Q

Variant CJD True/False:

  1. The disease mainly effects elderly people
  2. vCJD is more rapidly progressive than sporadic CJD
  3. The initial symptoms are always neurological
  4. Tonsillar biopsy is often diagnostic
  5. EEG is usually abnormal
A
  1. The disease mainly effects elderly people - FALSE - disease mainly effects the younger population (mean age 26 years)
  2. vCJD is more rapidly progressive than sporadic CJD - FALSE - median survival time is 14 months (slightly longer than sporadic CJD)
  3. The initial symptoms are always neurological - FALSE - psychiatric onset of symptoms (dysphoria, anxiety, paranoia, hallucinations) followed by neurological (peripheral sensory symptoms, ataxia, myoclonus, chorea, dementia)
  4. Tonsillar biopsy is often diagnostic - TRUE - 100% sensitive and specific for vCJD (unlike sporadic where isn’t usefull) - often eliminates the need for further investigations (e.g. brain biopsy). MRI shows a positive pulvinar sign: a high signal in the thalamus, unlike in sporadic CJD where it is in the basal ganglia
  5. EEG is usually abnormal - FALSE - may show non-specific slow waves and not useful for diagnosis
How well did you know this?
1
Not at all
2
3
4
5
Perfectly
95
Q

Sporadic CJD True/False:

  1. Median survival time is less than 6 months
  2. Tonsillar biopsy is diagnostic
  3. EEG usually shows periodic complexes
  4. Median age of onset is 65 years old
  5. CSF marker (S100, 14-3-3) of neuronal damage may be elevated
A
  1. Median survival time is less than 6 months - TRUE (shorter than vCJD)
  2. Tonsillar biopsy is diagnostic - FALSE - not useful. Brain biopsy shows spongiform vacuolation (rarely carried out except confirming diagnosis at post mortem).
  3. EEG usually shows periodic complexes - TRUE - triphasic complexes, however these are non-specific, also seen in hepatic encephalopathy and lithium toxicity
  4. Median age of onset is 65 years old - TRUE - commonest form of CJD, cause unknown, thought to be a common cause of Alzheimers in elderly
  5. CSF marker (S100, 14-3-3) of neuronal damage may be elevated - TRUE - although not useful in vCJD
How well did you know this?
1
Not at all
2
3
4
5
Perfectly
96
Q

Complement Deficiency

Membranoproliferative nephritis and bacterial infections

C9 deficiency
C3 deficiency with the presence of nephritic factor
MBL deficiency
C1q deficiency

A

C3 deficiency with the presence of nephritic factor

How well did you know this?
1
Not at all
2
3
4
5
Perfectly
97
Q

Complement Deficiency

Meningococcus meningitis with a family history of sibling dying of the same condition aged 6

C9 deficiency
C3 deficiency with the presence of nephritic factor
MBL deficiency
C1q deficiency

A

C9 deficiency

How well did you know this?
1
Not at all
2
3
4
5
Perfectly
98
Q

Complement Deficiency

Severe childhood onset SLE with normal levels of C3 and C4

C9 deficiency
C3 deficiency with the presence of nephritic factor
MBL deficiency
C1q deficiency

A

C1q deficiency

How well did you know this?
1
Not at all
2
3
4
5
Perfectly
99
Q

Complement Deficiency

Recurrent infections when receiving chemotherapy but previously well

C9 deficiency
C3 deficiency with the presence of nephritic factor
MBL deficiency
C1q deficiency

A

MBL deficiency

How well did you know this?
1
Not at all
2
3
4
5
Perfectly
100
Q

Prion Genetics - True/False

  1. The vast majority of cases of vCJD have been found to be methionine homozygous (MM) at codon 129 of PRNP
  2. Familial prion disease does not cause ataxia
  3. In familial prion disease mutations are usually inherited recessively
  4. Familial CJD is more rapidly progressive than sporadic CJD
A
  1. The vast majority of cases of vCJD have been found to be methionine homozygous (MM) at codon 129 of PRNP - TRUE
  2. Familial prion disease does not cause ataxia - FALSE
  3. In familial prion disease mutations are usually inherited recessively - FALSE - – Specific PRNP mutations (~30 so far) – all dominant mutations
  4. Familial CJD is more rapidly progressive than sporadic CJD - FALSE
How well did you know this?
1
Not at all
2
3
4
5
Perfectly
101
Q

19 year old male presents to A&E with severe respiratory difficulty, light-headedness and a red itchy rash. On examination he has laryngeal oedema, bilateral wheezing across the lung fields and is hypotensive. He has recently been taking antibiotics for a chest infection.

A. Allergic bronchopulmonary Aspergillosis
B. Food allergy
C. Allergic Rhinitis
D. Allergic asthma
E. Acute Urticaria
F. Angioedema
G. Drug allergy
H. Chronic Urticaria
I. Anaphylaxis
J. Contact dermatitis
A

Anaphylaxis

Strictly speaking you can only have a true anaphylactic reaction on 2nd exposure to an allergen (as the first time,
you get the IgE formation, without the cross-linking and mast cell degranulation), whereas anaphylactoid
reactions can occur on first exposure

How well did you know this?
1
Not at all
2
3
4
5
Perfectly
102
Q

A 3 year old girl is brought into A&E by her parents. She has had vomiting and diarrhoea since early yesterday evening when she was at a birthday party. On examination she has urticaria.

A. Allergic bronchopulmonary Aspergillosis
B. Food allergy
C. Allergic Rhinitis
D. Allergic asthma
E. Acute Urticaria
F. Angioedema
G. Drug allergy
H. Chronic Urticaria
I. Anaphylaxis
J. Contact dermatitis
A

Food Allergy

How well did you know this?
1
Not at all
2
3
4
5
Perfectly
103
Q

A 40 year old man presents to his GP complaining of loss smell and nasal itching and discharge. On examination his nasal mucosa are swollen and have a bluish tinge. His symptoms improve with a corticosteroid spray

A. Allergic bronchopulmonary Aspergillosis
B. Food allergy
C. Allergic Rhinitis
D. Allergic asthma
E. Acute Urticaria
F. Angioedema
G. Drug allergy
H. Chronic Urticaria
I. Anaphylaxis
J. Contact dermatitis
A

Allergic Rhinitis

How well did you know this?
1
Not at all
2
3
4
5
Perfectly
104
Q

A 25 year old woman presents to her GP complaining of itchy, red wheals on her torso which have been present for 7 weeks. She can not remember how they started but has noticed they are worse in the heat and when she exercises.

A. Allergic bronchopulmonary Aspergillosis
B. Food allergy
C. Allergic Rhinitis
D. Allergic asthma
E. Acute Urticaria
F. Angioedema
G. Drug allergy
H. Chronic Urticaria
I. Anaphylaxis
J. Contact dermatitis
A

Chronic Urticaria

Acute urticaria is defined being present for less than 6 weeks, whereas chronic urticaria persists for more than 6 weeks. In both cases the urticarial rash is intermittent, comes and goes and normally persists in a single site for less than 24 hours.

Urticarial rashes that last more than 24 hours in a single site, resolve with bruising or skin depigmentation may raise the possibility of an underlying vasculitis. In this instance a skin biopsy of the urticarial lesion is useful to confirm/repute presence of a vasculitis.

How well did you know this?
1
Not at all
2
3
4
5
Perfectly
105
Q

A 30 year old women presents to her GP with a red, itchy, oozing rash around her neck and fingers

A. Allergic bronchopulmonary Aspergillosis
B. Food allergy
C. Allergic Rhinitis
D. Allergic asthma
E. Acute Urticaria
F. Angioedema
G. Drug allergy
H. Chronic Urticaria
I. Anaphylaxis
J. Contact dermatitis
A

Contact Dermatitis

How well did you know this?
1
Not at all
2
3
4
5
Perfectly
106
Q

A 55 year old man with history of angina was advised to take a tablet before a long flight. After taking the pill, he suddenly finds that he has difficulty breathing, feels nauseous and is itching.

A. Acute urticaria
B. Urticarial vasculitis
C. Allergic asthma
D. Mast cell degranulation
E. IgE mediated anaphylaxis
F. Extrinsic allergic alveolitis
G. Idiopathic angioedema
H. Chronic urticaria
I. C1 inhibitor deficiency
J. Coeliac disease
K. Panic attack
A

Mast cell degranulation (not IgE mediated)

Opioids, NSAIDs etc directly trigger mast cell degranulation.

How well did you know this?
1
Not at all
2
3
4
5
Perfectly
107
Q

A 24 year old medical student develops worsening swelling of the hands and feet and abdominal pain before her final year medical exams. She says that similar milder episodes have occurred preciously.

A. Acute urticaria
B. Urticarial vasculitis
C. Allergic asthma
D. Mast cell degranulation
E. IgE mediated anaphylaxis
F. Extrinsic allergic alveolitis
G. Idiopathic angioedema
H. Chronic urticaria
I. C1 inhibitor deficiency
J. Coeliac disease
K. Panic attack
A

C1 inhibitor deficiency

C1 inhibitor has a number of functions, in addition to inhibiting the activation of C1. For example, it also inhibits activation of the kinin pathway, the clotting pathway and the fibrinolytic pathway. It does this through inhibition of factor XII, activated factor XI, and kallikrein as well as C1.

It’s likely that activation of the kinin pathway and production of bradykinin mediates the angioedema associated with this condition.

How well did you know this?
1
Not at all
2
3
4
5
Perfectly
108
Q

A 50 year old Irish woman presents to her GP with episodes of diarrhoea, which is difficult to flush, abdominal pain, weight loss and fatigue. She also describes a blistering itchy rash on her knees.

A. Acute urticaria
B. Urticarial vasculitis
C. Allergic asthma
D. Mast cell degranulation
E. IgE mediated anaphylaxis
F. Extrinsic allergic alveolitis
G. Idiopathic angioedema
H. Chronic urticaria
I. C1 inhibitor deficiency
J. Coeliac disease
K. Panic attack
A

Coeliac disease

Coeliac disease is associated with a superficial, blistering skin rash ‘dermatitis herpetiformis’, which is intensely itchy!

How well did you know this?
1
Not at all
2
3
4
5
Perfectly
109
Q

A 26 year old male who has been suffering from ‘flu-like’ symptoms with fever presents to the GP after developing skin rash in the last few days.

A. Acute urticaria
B. Urticarial vasculitis
C. Allergic asthma
D. Mast cell degranulation
E. IgE mediated anaphylaxis
F. Extrinsic allergic alveolitis
G. Idiopathic angioedema
H. Chronic urticaria
I. C1 inhibitor deficiency
J. Coeliac disease
K. Panic attack
A

Acute Urticaria

Acute urticaria is defined being present for less than 6 weeks, whereas chronic urticaria persists for more than 6 weeks. In both cases the urticarial rash is intermittent, comes and goes and normally persists in a single site for less than 24 hours.

Urticarial rashes that last more than 24 hours in a single site, resolve with bruising or skin depigmentation may raise the possibility of an underlying vasculitis. In this instance a skin biopsy of the urticarial lesion is useful to confirm/repute presence of a vasculitis.

How well did you know this?
1
Not at all
2
3
4
5
Perfectly
110
Q

A 35 year old woman presents with persistent itchy wheels for the last 2 months. She noticed that when this is at its worst, she also has a fever and feels generally unwell. After an acute attack, she has bruising and post-inflammatory residual pigmentation at the site of the itching.

A. Acute urticaria
B. Urticarial vasculitis
C. Allergic asthma
D. Mast cell degranulation
E. IgE mediated anaphylaxis
F. Extrinsic allergic alveolitis
G. Idiopathic angioedema
H. Chronic urticaria
I. C1 inhibitor deficiency
J. Coeliac disease
K. Panic attack
A

Urticarial vasculitis (UV) is a clinicopathologic entity consisting of urticaria and evidence of leukocytoclastic vasculitis on skin biopsy. UV predominantly involves the skin but may affect other organs, particularly the lungs, kidney, and gastrointestinal tract. Hypocomplementemia, when present, may be associated with extensive vasculitis and systemic features.

Differentiating features:
Common urticaria is pruritic and not painful. By comparison, up to one-third of patients with UV report burning and tenderness as well as pruritus.
Individual lesions in UV last longer than those in chronic urticaria, persisting for more than 24 hours in two-thirds of patients, and sometimes for up to 72 hours. Resolution of these wheals may be associated with purpura and hyperpigmentation in up to 35 percent.
UV can present as angioedema when the vasculitis involves the capillary or postcapillary venules of the deeper layers of the dermis and submucosa

How well did you know this?
1
Not at all
2
3
4
5
Perfectly
111
Q

A 19 year old male presents to A&E with increasing breathlessness. On examination his blood pressure is 90/55 mmHg and his respiratory rate is 28/min. He shows you a generalised red itchy skin rash, and examination of his chest reveals bilateral inspiratory and expiratory wheezes throughout.

A. Allergic asthma
B. Allergic rhinitis
C. Acute angioedema
D. Allergic bronchopulmonary aspergillosis
E. Chronic urticaria
F. Allergic conjunctivitis
G. Contact hypersensitivity
H. Anaphylaxis
I. Acute urticaria
J. Hereditary angioedema
A

Anaphylaxis

The combination of hypotension, respiratory distress, urticaria and bronchoconstriction is very suggestive of anaphylaxis

How well did you know this?
1
Not at all
2
3
4
5
Perfectly
112
Q

A 35 year old woman presents with a two day history of a red itchy skin rash which started soon after her first scuba-diving lesson. She is otherwise well.

A. Allergic asthma
B. Allergic rhinitis
C. Acute angioedema
D. Allergic bronchopulmonary aspergillosis
E. Chronic urticaria
F. Allergic conjunctivitis
G. Contact hypersensitivity
H. Anaphylaxis
I. Acute urticaria
J. Hereditary angioedema
A

Acute Urticaria
This rash is very suggestive of acute urticaria. The temporal association with scuba diving may indicate an allergy to latex (in wet suits)
Acute urticaria is defined being present for less than 6 weeks, whereas chronic urticaria persists for more than 6 weeks. In both cases the urticarial rash is intermittent, comes and goes and normally persists in a single site for less than 24 hours.

Urticarial rashes that last more than 24 hours in a single site, resolve with bruising or skin depigmentation may raise the possibility of an underlying vasculitis. In this instance a skin biopsy of the urticarial lesion is useful to confirm/repute presence of a vasculitis. A

How well did you know this?
1
Not at all
2
3
4
5
Perfectly
113
Q

A 22 year old woman presents with an intermittently itchy and desquamating skin rash on her abdomen which is unresponsive to antihistamines

A. Allergic asthma
B. Allergic rhinitis
C. Acute angioedema
D. Allergic bronchopulmonary aspergillosis
E. Chronic urticaria
F. Allergic conjunctivitis
G. Contact hypersensitivity
H. Anaphylaxis
I. Acute urticaria
J. Hereditary angioedema
A

Contact Hypersensitivity
This rash is typical of contact hypersensitivity. The distribution of the rash suggests that the specific agent is nickel, which used to be a component of the studs of jeans and is commonly found in the metal used in belts.

How well did you know this?
1
Not at all
2
3
4
5
Perfectly
114
Q

A 40 year old man complains of loss of smell with nasal itching and discharge over 4 weeks. He also describes morning sneezing. He is otherwise in good health. On examination his nasal mucosa are swollen and hyperaemic.

A. Allergic asthma
B. Allergic rhinitis
C. Acute angioedema
D. Allergic bronchopulmonary aspergillosis
E. Chronic urticaria
F. Allergic conjunctivitis
G. Contact hypersensitivity
H. Anaphylaxis
I. Acute urticaria
J. Hereditary angioedema
A

Allergic Rhinitis

The combination of sneezing, rhinorrhea and loss of smell is very suggestive of allergic rhinitis

How well did you know this?
1
Not at all
2
3
4
5
Perfectly
115
Q

This 45 year old woman presents to A&E with tongue swelling and acute respiratory tract obstruction. She has longstanding hypertension and received a renal transplant two years previously. She has no history of allergic disease. On examination her blood pressure is stable, and examination of her lung fields reveal normal breath sounds. Her current medication includes cyclosporine, azathioprine, captopril and nifedipine.

A. Allergic asthma
B. Allergic rhinitis
C. Acute angioedema
D. Allergic bronchopulmonary aspergillosis
E. Chronic urticaria
F. Allergic conjunctivitis
G. Contact hypersensitivity
H. Anaphylaxis
I. Acute urticaria
J. Hereditary angioedema
A

Acute Angioedema
This woman has angioedema of the tongue, without symptoms suggestive of a generalised allergic reaction. Isolated angioedema may be allergic in origin, but 94% of cases angioedema presenting to A&E are drug induced and the majority of these are associated with ACE inhibitors (eg captopril).

How well did you know this?
1
Not at all
2
3
4
5
Perfectly
116
Q

A 19 year old male presents to A&E with increasing breathlessness. On examination his blood pressure is 90/55 mmHg and his respiratory rate is 28/min. He shows you a generalised red itchy skin rash, and examination of his chest reveals bilateral inspiratory and expiratory wheezes throughout.

A. IM adrenaline 0.5 mL of 1:1000
B. IM adrenaline 1mL of 1:10000
C. Intraarticular corticosteroids
D. IM adrenaline 1mL of 1:1000
E. Intranasal antihistamines
F. PO antihistamines
G. Intracardiac adrenaline
H. IV antihistamines
I. None of the above
J. IV adrenaline 0.3mL of 1:1000
K. Inhaled corticosteroids
L. Inhaled antihistamines
M. Venom immunotherapy
A

IM adrenaline 1mL of 1:1000

The most important treatment of anaphylaxis is adrenaline, which should be given intramuscularly. (Note for final year pharm: 1:1000 means 1mg/mL; 1:10000 means 0.1mg/mL ; 1% means 1g/dL)

How well did you know this?
1
Not at all
2
3
4
5
Perfectly
117
Q

A 35 year old woman presents with a two day history of a red itchy skin rash which started soon after her first scuba-diving lesson. She is otherwise well.

A. IM adrenaline 0.5 mL of 1:1000
B. IM adrenaline 1mL of 1:10000
C. Intraarticular corticosteroids
D. IM adrenaline 1mL of 1:1000
E. Intranasal antihistamines
F. PO antihistamines
G. Intracardiac adrenaline
H. IV antihistamines
I. None of the above
J. IV adrenaline 0.3mL of 1:1000
K. Inhaled corticosteroids
L. Inhaled antihistamines
M. Venom immunotherapy
A

PO anti-histamines

Severe acute urticaria is effectively treated with a short course of oral anti-histamines

How well did you know this?
1
Not at all
2
3
4
5
Perfectly
118
Q

A 22 year old woman is presents with this intermittently itchy and desquamating skin rash which is unresponsive to antihistamines

A. IM adrenaline 0.5 mL of 1:1000
B. IM adrenaline 1mL of 1:10000
C. Intraarticular corticosteroids
D. IM adrenaline 1mL of 1:1000
E. Intranasal antihistamines
F. PO antihistamines
G. Intracardiac adrenaline
H. IV antihistamines
I. None of the above
J. IV adrenaline 0.3mL of 1:1000
K. Inhaled corticosteroids
L. Inhaled antihistamines
M. Venom immunotherapy
A

None of the above

Contact hypersensitivity should be treated by avoidance of the sensitising agent, in this case nickel

How well did you know this?
1
Not at all
2
3
4
5
Perfectly
119
Q

A 40 year old man complains of loss of smell with nasal itching and discharge over 4 weeks. He also describes morning sneezing. He is otherwise in good health. On examination his nasal mucosa are swollen and hyperaemic.

A. IM adrenaline 0.5 mL of 1:1000
B. IM adrenaline 1mL of 1:10000
C. Intraarticular corticosteroids
D. IM adrenaline 1mL of 1:1000
E. Intranasal antihistamines
F. PO antihistamines
G. Intracardiac adrenaline
H. IV antihistamines
I. None of the above
J. IV adrenaline 0.3mL of 1:1000
K. Inhaled corticosteroids
L. Inhaled antihistamines
M. Venom immunotherapy
A

PO anti-histamines
Oral antihistamines and intranasal corticosteroids are the mainstay of treatment of mild allergic rhinitis. (As intranasal corticosteroid is not an option available, the “single best” answer here is oral antihistamines.)

How well did you know this?
1
Not at all
2
3
4
5
Perfectly
120
Q

This 45 year old woman presents to A&E with tongue swelling and acute respiratory tract obstruction. She has longstanding hypertension and received a renal transplant two years previously. She has no history of allergic disease. On examination her blood pressure is stable, and examination of her lung fields reveal normal breath sounds. Her current medication includes cyclosporine, azathioprine, captopril and nifedipine.

A. IM adrenaline 0.5 mL of 1:1000
B. IM adrenaline 1mL of 1:10000
C. Intraarticular corticosteroids
D. IM adrenaline 1mL of 1:1000
E. Intranasal antihistamines
F. PO antihistamines
G. Intracardiac adrenaline
H. IV antihistamines
I. None of the above
J. IV adrenaline 0.3mL of 1:1000
K. Inhaled corticosteroids
L. Inhaled antihistamines
M. Venom immunotherapy
A

IM adrenaline 0.5 mL of 1:1000

Intramuscular adrenalin should be used in patients with severe local angioedema with secondary acute respiratory tract obstruction. However this is not always effective in ACE inhibitor-induced angioedema, and some patients will require intubation. Always stop the causative agent!

How well did you know this?
1
Not at all
2
3
4
5
Perfectly
121
Q

Cytokines exerting an anti-viral effect

A. Alternative complement pathway
B. IgE
C. IgG
D. Major histocompatability complex class 2
E. Innate immune system
F. CD8+
G. Major histocompatability complex class 1
H. IL6
I. Interferons
J. IgA
K. Natural Killer cells
L. Classical complement pathway
M. IgM
A

Interferons

How well did you know this?
1
Not at all
2
3
4
5
Perfectly
122
Q

Immunoglobulin dimer

A. Alternative complement pathway
B. IgE
C. IgG
D. Major histocompatability complex class 2
E. Innate immune system
F. CD8+
G. Major histocompatability complex class 1
H. IL6
I. Interferons
J. IgA
K. Natural Killer cells
L. Classical complement pathway
M. IgM
A

IgA

How well did you know this?
1
Not at all
2
3
4
5
Perfectly
123
Q

MHC associated with Th1 cells

A. Alternative complement pathway
B. IgE
C. IgG
D. Major histocompatability complex class 2
E. Innate immune system
F. CD8+
G. Major histocompatability complex class 1
H. IL6
I. Interferons
J. IgA
K. Natural Killer cells
L. Classical complement pathway
M. IgM
A

Major Histocompatibility complex class 2

How well did you know this?
1
Not at all
2
3
4
5
Perfectly
124
Q

Acts on hepatocytes to induce synthesis of acute phase proteins in response to bacterial infection

A. Alternative complement pathway
B. IgE
C. IgG
D. Major histocompatability complex class 2
E. Innate immune system
F. CD8+
G. Major histocompatability complex class 1
H. IL6
I. Interferons
J. IgA
K. Natural Killer cells
L. Classical complement pathway
M. IgM
A

IL-6

How well did you know this?
1
Not at all
2
3
4
5
Perfectly
125
Q

Arise in the first few days after infection and are important in defence against viruses and tumours

A. Alternative complement pathway
B. IgE
C. IgG
D. Major histocompatability complex class 2
E. Innate immune system
F. CD8+
G. Major histocompatability complex class 1
H. IL6
I. Interferons
J. IgA
K. Natural Killer cells
L. Classical complement pathway
M. IgM
A

Natural Killer cells

How well did you know this?
1
Not at all
2
3
4
5
Perfectly
126
Q

MHC associated with Th2 cells

A. Alternative complement pathway
B. IgE
C. IgG
D. Major histocompatability complex class 2
E. Innate immune system
F. CD8+
G. Major histocompatability complex class 1
H. IL6
I. Interferons
J. IgA
K. Natural Killer cells
L. Classical complement pathway
M. IgM
A

Major histocompatability complex class 2

How well did you know this?
1
Not at all
2
3
4
5
Perfectly
127
Q

MHC associated with cytotoxic T cells

A. Alternative complement pathway
B. IgE
C. IgG
D. Major histocompatability complex class 2
E. Innate immune system
F. CD8+
G. Major histocompatability complex class 1
H. IL6
I. Interferons
J. IgA
K. Natural Killer cells
L. Classical complement pathway
M. IgM
A

Major histocompatability complex class 1

How well did you know this?
1
Not at all
2
3
4
5
Perfectly
128
Q

Along with IgD, is one of the first immunoglobulins expressed on B cells before they undergo antibody class switching

A. Alternative complement pathway
B. IgE
C. IgG
D. Major histocompatability complex class 2
E. Innate immune system
F. CD8+
G. Major histocompatability complex class 1
H. IL6
I. Interferons
J. IgA
K. Natural Killer cells
L. Classical complement pathway
M. IgM
A

IgM

How well did you know this?
1
Not at all
2
3
4
5
Perfectly
129
Q

The most abundant (in terms of g/L) immunoglobulin in normal plasma

A. Alternative complement pathway
B. IgE
C. IgG
D. Major histocompatability complex class 2
E. Innate immune system
F. CD8+
G. Major histocompatability complex class 1
H. IL6
I. Interferons
J. IgA
K. Natural Killer cells
L. Classical complement pathway
M. IgM
A

IgG

How well did you know this?
1
Not at all
2
3
4
5
Perfectly
130
Q

Deficiencies in this predispose to SLE

A. Alternative complement pathway
B. IgE
C. IgG
D. Major histocompatability complex class 2
E. Innate immune system
F. CD8+
G. Major histocompatability complex class 1
H. IL6
I. Interferons
J. IgA
K. Natural Killer cells
L. Classical complement pathway
M. IgM
A

Classical complement pathway

How well did you know this?
1
Not at all
2
3
4
5
Perfectly
131
Q

Kostmanns syndrome is a congenital deficiency of which component of the immune system?

A. Bacterial
B. B lymphocyte
C. T lymphocyte
D. Neutrophil
E. Complement
F. Mast cell
G. Parasitic
H. MHC Class I
I. Fungal
J. MHC Class II
K. Viral
A

Neutrophil

How well did you know this?
1
Not at all
2
3
4
5
Perfectly
132
Q

Which component of the innate immune system is usually one of the first to respond to infection through a cut?

A. Bacterial
B. B lymphocyte
C. T lymphocyte
D. Neutrophil
E. Complement
F. Mast cell
G. Parasitic
H. MHC Class I
I. Fungal
J. MHC Class II
K. Viral
A

Neutrophil

How well did you know this?
1
Not at all
2
3
4
5
Perfectly
133
Q

Which infection is most common as a consequence of B cell deficiency?

A. Bacterial
B. B lymphocyte
C. T lymphocyte
D. Neutrophil
E. Complement
F. Mast cell
G. Parasitic
H. MHC Class I
I. Fungal
J. MHC Class II
K. Viral
A

Bacterial

How well did you know this?
1
Not at all
2
3
4
5
Perfectly
134
Q

Meningococcal infections are quite common as a result of which deficiency of the component of the immune system?

A. Bacterial
B. B lymphocyte
C. T lymphocyte
D. Neutrophil
E. Complement
F. Mast cell
G. Parasitic
H. MHC Class I
I. Fungal
J. MHC Class II
K. Viral
A

Complement

How well did you know this?
1
Not at all
2
3
4
5
Perfectly
135
Q

Produced by the liver, when triggered, enzymatically activate other proteins in a biological cascade and are important in innate and antibody mediated immune response?

A. Bacterial
B. B lymphocyte
C. T lymphocyte
D. Neutrophil
E. Complement
F. Mast cell
G. Parasitic
H. MHC Class I
I. Fungal
J. MHC Class II
K. Viral
A

Complement

How well did you know this?
1
Not at all
2
3
4
5
Perfectly
136
Q

A complete deficiency in this molecule is associated with recurrent respiratory and gastrointestinal infections.

A. Macrophages
B. IgM
C. C3b
D. MAC
E. AP50
F. IgG
G. IgA
H. CH50
I. Neutrophils
J. C1
K. Myeloperoxidase
L. C3a
M. NADPH oxidoase
A

IgA

How well did you know this?
1
Not at all
2
3
4
5
Perfectly
137
Q

Leukocyte Adhesion Deficiency is characterised by a very high count in which of the above?

A. Macrophages
B. IgM
C. C3b
D. MAC
E. AP50
F. IgG
G. IgA
H. CH50
I. Neutrophils
J. C1
K. Myeloperoxidase
L. C3a
M. NADPH oxidoase
A

Neutrophils

How well did you know this?
1
Not at all
2
3
4
5
Perfectly
138
Q

Which crucial enzyme is vital for the oxidative killing of intracellular micro-organisms?

A. Macrophages
B. IgM
C. C3b
D. MAC
E. AP50
F. IgG
G. IgA
H. CH50
I. Neutrophils
J. C1
K. Myeloperoxidase
L. C3a
M. NADPH oxidoase
A

NADPH oxidase

How well did you know this?
1
Not at all
2
3
4
5
Perfectly
139
Q

Which complement factor is an important chemotaxic agent?

A. Macrophages
B. IgM
C. C3b
D. MAC
E. AP50
F. IgG
G. IgA
H. CH50
I. Neutrophils
J. C1
K. Myeloperoxidase
L. C3a
M. NADPH oxidoase
A

C3a

How well did you know this?
1
Not at all
2
3
4
5
Perfectly
140
Q

What is the functional complement test used to investigate the classical pathway?

A. Macrophages
B. IgM
C. C3b
D. MAC
E. AP50
F. IgG
G. IgA
H. CH50
I. Neutrophils
J. C1
K. Myeloperoxidase
L. C3a
M. NADPH oxidoase
A

CH50

How well did you know this?
1
Not at all
2
3
4
5
Perfectly
141
Q

Grave’s Disease

A. Type II – Antigen mediated
B. Type II – Antibody mediated
C. Type III – complement mediated
D. Not an autoimmune disease
E. Type III – Immune complex mediated
F. Type III – T-cell mediated
G. Type IV – T-cell mediated
H. Type IV – Complement mediated
A

Type II – Antibody mediated

How well did you know this?
1
Not at all
2
3
4
5
Perfectly
142
Q

SLE

A. Type II – Antigen mediated
B. Type II – Antibody mediated
C. Type III – complement mediated
D. Not an autoimmune disease
E. Type III – Immune complex mediated
F. Type III – T-cell mediated
G. Type IV – T-cell mediated
H. Type IV – Complement mediated
A

Type III – Immune complex mediated

How well did you know this?
1
Not at all
2
3
4
5
Perfectly
143
Q

Rheumatoid Arthritis

A. Type II – Antigen mediated
B. Type II – Antibody mediated
C. Type III – complement mediated
D. Not an autoimmune disease
E. Type III – Immune complex mediated
F. Type III – T-cell mediated
G. Type IV – T-cell mediated
H. Type IV – Complement mediated
A

Type IV – T-cell mediated

How well did you know this?
1
Not at all
2
3
4
5
Perfectly
144
Q

Asthma

A. Type II – Antigen mediated
B. Type II – Antibody mediated
C. Type III – complement mediated
D. Not an autoimmune disease
E. Type III – Immune complex mediated
F. Type III – T-cell mediated
G. Type IV – T-cell mediated
H. Type IV – Complement mediated
A

Not an autoimmune disease

How well did you know this?
1
Not at all
2
3
4
5
Perfectly
145
Q

Type 1 diabetes

A. Type II – Antigen mediated
B. Type II – Antibody mediated
C. Type III – complement mediated
D. Not an autoimmune disease
E. Type III – Immune complex mediated
F. Type III – T-cell mediated
G. Type IV – T-cell mediated
H. Type IV – Complement mediated
A

Type IV – T-cell mediated

How well did you know this?
1
Not at all
2
3
4
5
Perfectly
146
Q

Immune thrombocytopaenic purpura

A. Type II – Antigen mediated
B. Type II – Antibody mediated
C. Type III – complement mediated
D. Not an autoimmune disease
E. Type III – Immune complex mediated
F. Type III – T-cell mediated
G. Type IV – T-cell mediated
H. Type IV – Complement mediated
A

Type II – Antibody mediated

How well did you know this?
1
Not at all
2
3
4
5
Perfectly
147
Q

ABO hemolytic transfusion reaction

A. Type II – Antigen mediated
B. Type II – Antibody mediated
C. Type III – complement mediated
D. Not an autoimmune disease
E. Type III – Immune complex mediated
F. Type III – T-cell mediated
G. Type IV – T-cell mediated
H. Type IV – Complement mediated
A

Type II – Antibody mediated

How well did you know this?
1
Not at all
2
3
4
5
Perfectly
148
Q

Hepatitis C associated membranoproliferative glomerulonephritis type I

A. Type II – Antigen mediated
B. Type II – Antibody mediated
C. Type III – complement mediated
D. Not an autoimmune disease
E. Type III – Immune complex mediated
F. Type III – T-cell mediated
G. Type IV – T-cell mediated
H. Type IV – Complement mediated
A

Type III – Immune complex mediated

How well did you know this?
1
Not at all
2
3
4
5
Perfectly
149
Q

Goodpasture’s syndrome

A. Type II – Antigen mediated
B. Type II – Antibody mediated
C. Type III – complement mediated
D. Not an autoimmune disease
E. Type III – Immune complex mediated
F. Type III – T-cell mediated
G. Type IV – T-cell mediated
H. Type IV – Complement mediated
A

Type II – Antibody mediated

How well did you know this?
1
Not at all
2
3
4
5
Perfectly
150
Q

Myaesthenia gravis

A. Type II – Antigen mediated
B. Type II – Antibody mediated
C. Type III – complement mediated
D. Not an autoimmune disease
E. Type III – Immune complex mediated
F. Type III – T-cell mediated
G. Type IV – T-cell mediated
H. Type IV – Complement mediated
A

Type II – Antibody mediated

How well did you know this?
1
Not at all
2
3
4
5
Perfectly
151
Q

Systemic lupus erythematous

A. Anti-mitochondrial antibody
B. Anti-centromere antibody
C. Anti-GAD antibody
D. Anti-cardiolipin antibody
E. c-ANCA
F. p-ANCA
G. Anti-nuclear antibody
H. Anti-CCP antibody
I. Anti-DNA antibody
J. Rheumatoid factor
K. Coomb's test
A

Anti-DNA antibody
The rationale for the answer for SLE being anti-DNA antibody rather than ANA is that whilst ANA is very sensitive for SLE, it is not specific. Anti-DNA, in contrast, is highly specific to SLE (~95%).

How well did you know this?
1
Not at all
2
3
4
5
Perfectly
152
Q

Wegener’s granulomatosis

A. Anti-mitochondrial antibody
B. Anti-centromere antibody
C. Anti-GAD antibody
D. Anti-cardiolipin antibody
E. c-ANCA
F. p-ANCA
G. Anti-nuclear antibody
H. Anti-CCP antibody
I. Anti-DNA antibody
J. Rheumatoid factor
K. Coomb's test
A

c-ANCA

Remember that c-ANCA matches with Wegener’s Granulomatosis, whilst p-ANCA would match with polyarteritis nodosa

How well did you know this?
1
Not at all
2
3
4
5
Perfectly
153
Q

Rheumatoid arthritis

A. Anti-mitochondrial antibody
B. Anti-centromere antibody
C. Anti-GAD antibody
D. Anti-cardiolipin antibody
E. c-ANCA
F. p-ANCA
G. Anti-nuclear antibody
H. Anti-CCP antibody
I. Anti-DNA antibody
J. Rheumatoid factor
K. Coomb's test
A

Anti-CCP antibody

Rheumatoid factor is not specific or sensitive to rheumatoid arthritis and is common in the elderly. Anti-CCP is a more specific test for rheumatoid arthritis and a better predictor of an aggressive course.

How well did you know this?
1
Not at all
2
3
4
5
Perfectly
154
Q

Auto-immune haemolytic anaemia

A. Anti-mitochondrial antibody
B. Anti-centromere antibody
C. Anti-GAD antibody
D. Anti-cardiolipin antibody
E. c-ANCA
F. p-ANCA
G. Anti-nuclear antibody
H. Anti-CCP antibody
I. Anti-DNA antibody
J. Rheumatoid factor
K. Coomb's test
A

Coomb’s Test

How well did you know this?
1
Not at all
2
3
4
5
Perfectly
155
Q

Primary biliary cirrhosis

A. Anti-mitochondrial antibody
B. Anti-centromere antibody
C. Anti-GAD antibody
D. Anti-cardiolipin antibody
E. c-ANCA
F. p-ANCA
G. Anti-nuclear antibody
H. Anti-CCP antibody
I. Anti-DNA antibody
J. Rheumatoid factor
K. Coomb's test
A

Anti-mitochondrial antibody

How well did you know this?
1
Not at all
2
3
4
5
Perfectly
156
Q

Goodpastures: What is the specific auto-antigen that is the target of the immune system?

A. Skin
B. Smooth linear pattern
C. Type IV collagen
D. Plasmapheresis
E. Anti-neutrophil cytoplasmic antibodies
F. Lung
G. Blood vessels
H. Lumpy-bumpy pattern
I. Ciclosporin
J. Prednisolone
K. Glomerular basement membrane
L. Mesangium
M. Type II Hypersentivity
N. Type II collagen
A

Type IV collagen

How well did you know this?
1
Not at all
2
3
4
5
Perfectly
157
Q

Goodpastures: The pattern of the antibody deposition in the glomerular basement membrane is typically described as what?

A. Skin
B. Smooth linear pattern
C. Type IV collagen
D. Plasmapheresis
E. Anti-neutrophil cytoplasmic antibodies
F. Lung
G. Blood vessels
H. Lumpy-bumpy pattern
I. Ciclosporin
J. Prednisolone
K. Glomerular basement membrane
L. Mesangium
M. Type II Hypersentivity
N. Type II collagen
A

Smooth linear pattern

How well did you know this?
1
Not at all
2
3
4
5
Perfectly
158
Q

Goodpastures: Name the drug most likely to be used in the treatment of this disease.

A. Skin
B. Smooth linear pattern
C. Type IV collagen
D. Plasmapheresis
E. Anti-neutrophil cytoplasmic antibodies
F. Lung
G. Blood vessels
H. Lumpy-bumpy pattern
I. Ciclosporin
J. Prednisolone
K. Glomerular basement membrane
L. Mesangium
M. Type II Hypersentivity
N. Type II collagen
A

Prednisolone

How well did you know this?
1
Not at all
2
3
4
5
Perfectly
159
Q

Goodpastures: Immune damage may be associated with the kidney and commonly which other tissue?

A. Skin
B. Smooth linear pattern
C. Type IV collagen
D. Plasmapheresis
E. Anti-neutrophil cytoplasmic antibodies
F. Lung
G. Blood vessels
H. Lumpy-bumpy pattern
I. Ciclosporin
J. Prednisolone
K. Glomerular basement membrane
L. Mesangium
M. Type II Hypersentivity
N. Type II collagen
A

Lung

How well did you know this?
1
Not at all
2
3
4
5
Perfectly
160
Q

Which enzyme is defective in Von Gierkes disease?

A. Uricase
B. Glucose-6-phosphatase
C. Phospho Ribosyl Pyro Phosphate (PRPP) Synthase
D. Xanthine oxidase
E. Adenosine deaminase
F. Myophosphorylase
G. Hypoxanthine Guanosine Phospo Ribosyl Transferase (HGPRT)

A

Glucose-6-phosphatase

How well did you know this?
1
Not at all
2
3
4
5
Perfectly
161
Q

Severe combined immunodeficinecy (SCID) is due to deficiency of what enzyme?

A. Uricase
B. Glucose-6-phosphatase
C. Phospho Ribosyl Pyro Phosphate (PRPP) Synthase
D. Xanthine oxidase
E. Adenosine deaminase
F. Myophosphorylase
G. Hypoxanthine Guanosine Phospo Ribosyl Transferase (HGPRT)

A

Adenosine deaminase

How well did you know this?
1
Not at all
2
3
4
5
Perfectly
162
Q

Allopurinol inhibits what?

A. Uricase
B. Glucose-6-phosphatase
C. Phospho Ribosyl Pyro Phosphate (PRPP) Synthase
D. Xanthine oxidase
E. Adenosine deaminase
F. Myophosphorylase
G. Hypoxanthine Guanosine Phospo Ribosyl Transferase (HGPRT)

A

Xanthine oxidase

How well did you know this?
1
Not at all
2
3
4
5
Perfectly
163
Q

Lesch-Nyhan disease is due to the deficiency of which enzyme?

A. Uricase
B. Glucose-6-phosphatase
C. Phospho Ribosyl Pyro Phosphate (PRPP) Synthase
D. Xanthine oxidase
E. Adenosine deaminase
F. Myophosphorylase
G. Hypoxanthine Guanosine Phospo Ribosyl Transferase (HGPRT)

A

Hypoxanthine Guanosine Phospo Ribosyl Transferase (HGPRT)

How well did you know this?
1
Not at all
2
3
4
5
Perfectly
164
Q

Which enzyme is absent in mammals?

A. Uricase
B. Glucose-6-phosphatase
C. Phospho Ribosyl Pyro Phosphate (PRPP) Synthase
D. Xanthine oxidase
E. Adenosine deaminase
F. Myophosphorylase
G. Hypoxanthine Guanosine Phospo Ribosyl Transferase (HGPRT)

A

Uricase

How well did you know this?
1
Not at all
2
3
4
5
Perfectly
165
Q

Over-activity of which enzyme leads to gout?

A. Uricase
B. Glucose-6-phosphatase
C. Phospho Ribosyl Pyro Phosphate (PRPP) Synthase
D. Xanthine oxidase
E. Adenosine deaminase
F. Myophosphorylase
G. Hypoxanthine Guanosine Phospo Ribosyl Transferase (HGPRT)

A

Phospho Ribosyl Pyro Phosphate (PRPP) Synthase

How well did you know this?
1
Not at all
2
3
4
5
Perfectly
166
Q

A 16-year-old male has a past psychiatric history of self mutilation. He complains of pain in his first metatarso phalangeal joint, and polymorphs and needle shaped crystals are seen from an aspirate. When under polarised light, they appear yellow when parallel to the red compensator and blue when perpendicular to it. Name the syndrome that he has?

A. Klinefelters syndrome
B. Pseudo-Gout
C. Lesch-Nyhan syndrome
D. Rheumatoid arthritis
E. Gout
F. Turners syndrome
G. Osteoarthritis
H. Septic arthritis
A

Lesch-Nyhan syndrome

How well did you know this?
1
Not at all
2
3
4
5
Perfectly
167
Q

A 28-year-old female has a painful inflamed swollen knee, and aspirate reveals polymorphs but no crystals?

A. Klinefelters syndrome
B. Pseudo-Gout
C. Lesch-Nyhan syndrome
D. Rheumatoid arthritis
E. Gout
F. Turners syndrome
G. Osteoarthritis
H. Septic arthritis
A

Septic Arthritis

How well did you know this?
1
Not at all
2
3
4
5
Perfectly
168
Q

A 78-year-old female has a painful inflamed swollen knee, and aspirate reveals polymorphs and brick shaped crystals. When under polarised light, they appear blue when parallel to the red compensator and yellow when perpendicular to it?

A. Klinefelters syndrome
B. Pseudo-Gout
C. Lesch-Nyhan syndrome
D. Rheumatoid arthritis
E. Gout
F. Turners syndrome
G. Osteoarthritis
H. Septic arthritis
A

Pseudo-Gout

How well did you know this?
1
Not at all
2
3
4
5
Perfectly
169
Q

A 52-year-old alcoholic male complains of pain in his proximal interphalangeal joint. Polymorphs and needle shaped crystals are seen from an aspirate. When under polarised light, they appear yellow when parallel to the red compensator and blue when perpendicular to it?

A. Klinefelters syndrome
B. Pseudo-Gout
C. Lesch-Nyhan syndrome
D. Rheumatoid arthritis
E. Gout
F. Turners syndrome
G. Osteoarthritis
H. Septic arthritis
A

Gout

How well did you know this?
1
Not at all
2
3
4
5
Perfectly
170
Q

A 24-year-old female with renal failure complains of pain in her first metatarso phalangeal joint, and polymorphs and needle shaped crystals are seen from an aspirate. When under polarised light, they appear yellow when parallel to the red compensator and blue when perpendicular to it?

A. Klinefelters syndrome
B. Pseudo-Gout
C. Lesch-Nyhan syndrome
D. Rheumatoid arthritis
E. Gout
F. Turners syndrome
G. Osteoarthritis
H. Septic arthritis
A

Gout

How well did you know this?
1
Not at all
2
3
4
5
Perfectly
171
Q

Which is the only pituitary hormone to be suppressed by a hypothalamic hormone?

A. Thyroid Stimulating Hormone (TSH or thyrotrophin)
B. Oxytocin
C. Insulin
D. Corticotrophin
E. Adrenaline
F. Gonadotrophin Releasing Hormone (GnRH)
G. Luteinising Hormone
H. Thyrotrophin Releasing Hormone
I. IGF-1
J. Prolactin
K. Growth Hormone (somatotrophin)
L. Follicle Stimulating Hormone
M. Growth Hormone Releasing Hormone (GHRH)
N. Leptin
O. Dopamine
P. Corticotrophin Releasing Hormone (CRH)
A

Prolactin
Prolactin is the only hormone that has an inhibiting factor. All the other anterior pituitary hormones are stimulated by releasing hormones.

How well did you know this?
1
Not at all
2
3
4
5
Perfectly
172
Q

Which hormone is most important in stimulating spermatogenesis?

A. Thyroid Stimulating Hormone (TSH or thyrotrophin)
B. Oxytocin
C. Insulin
D. Corticotrophin
E. Adrenaline
F. Gonadotrophin Releasing Hormone (GnRH)
G. Luteinising Hormone
H. Thyrotrophin Releasing Hormone
I. IGF-1
J. Prolactin
K. Growth Hormone (somatotrophin)
L. Follicle Stimulating Hormone
M. Growth Hormone Releasing Hormone (GHRH)
N. Leptin
O. Dopamine
P. Corticotrophin Releasing Hormone (CRH)
A

FSH
FSH stimulates follicle production in females and sperm production in males. Inhibin feeds back on FSH levels. LH causes ovulation in the female at the “LH surge”. In the male, it is responsible for the stimulation of the secretion of testosterone.

How well did you know this?
1
Not at all
2
3
4
5
Perfectly
173
Q

Which hormone is most important in stimulating the secretion of testosterone?

A. Thyroid Stimulating Hormone (TSH or thyrotrophin)
B. Oxytocin
C. Insulin
D. Corticotrophin
E. Adrenaline
F. Gonadotrophin Releasing Hormone (GnRH)
G. Luteinising Hormone
H. Thyrotrophin Releasing Hormone
I. IGF-1
J. Prolactin
K. Growth Hormone (somatotrophin)
L. Follicle Stimulating Hormone
M. Growth Hormone Releasing Hormone (GHRH)
N. Leptin
O. Dopamine
P. Corticotrophin Releasing Hormone (CRH)
A

LH
FSH stimulates follicle production in females and sperm production in males. Inhibin feeds back on FSH levels. LH causes ovulation in the female at the “LH surge”. In the male, it is responsible for the stimulation of the secretion of testosterone.

How well did you know this?
1
Not at all
2
3
4
5
Perfectly
174
Q

Which hormone is secreted by the liver?

A. Thyroid Stimulating Hormone (TSH or thyrotrophin)
B. Oxytocin
C. Insulin
D. Corticotrophin
E. Adrenaline
F. Gonadotrophin Releasing Hormone (GnRH)
G. Luteinising Hormone
H. Thyrotrophin Releasing Hormone
I. IGF-1
J. Prolactin
K. Growth Hormone (somatotrophin)
L. Follicle Stimulating Hormone
M. Growth Hormone Releasing Hormone (GHRH)
N. Leptin
O. Dopamine
P. Corticotrophin Releasing Hormone (CRH)
A

IGF-1

IGF-1 is produced by the liver in the presence of GH

How well did you know this?
1
Not at all
2
3
4
5
Perfectly
175
Q

Which hormone is secreted from the posterior pituitary?

A. Thyroid Stimulating Hormone (TSH or thyrotrophin)
B. Oxytocin
C. Insulin
D. Corticotrophin
E. Adrenaline
F. Gonadotrophin Releasing Hormone (GnRH)
G. Luteinising Hormone
H. Thyrotrophin Releasing Hormone
I. IGF-1
J. Prolactin
K. Growth Hormone (somatotrophin)
L. Follicle Stimulating Hormone
M. Growth Hormone Releasing Hormone (GHRH)
N. Leptin
O. Dopamine
P. Corticotrophin Releasing Hormone (CRH)
A

Oxytocin

Oxytocin comes from the posterior pituitary as does vasopressin.

How well did you know this?
1
Not at all
2
3
4
5
Perfectly
176
Q

Name the hypothalamic hormone that inhibits prolactin release?

A. Thyroid Stimulating Hormone (TSH or thyrotrophin)
B. Oxytocin
C. Insulin
D. Corticotrophin
E. Adrenaline
F. Gonadotrophin Releasing Hormone (GnRH)
G. Luteinising Hormone
H. Thyrotrophin Releasing Hormone
I. IGF-1
J. Prolactin
K. Growth Hormone (somatotrophin)
L. Follicle Stimulating Hormone
M. Growth Hormone Releasing Hormone (GHRH)
N. Leptin
O. Dopamine
P. Corticotrophin Releasing Hormone (CRH)
A

Dopamine

How well did you know this?
1
Not at all
2
3
4
5
Perfectly
177
Q

Name the hypothalamic hormone that stimulates prolactin release?

A. Thyroid Stimulating Hormone (TSH or thyrotrophin)
B. Oxytocin
C. Insulin
D. Corticotrophin
E. Adrenaline
F. Gonadotrophin Releasing Hormone (GnRH)
G. Luteinising Hormone
H. Thyrotrophin Releasing Hormone
I. IGF-1
J. Prolactin
K. Growth Hormone (somatotrophin)
L. Follicle Stimulating Hormone
M. Growth Hormone Releasing Hormone (GHRH)
N. Leptin
O. Dopamine
P. Corticotrophin Releasing Hormone (CRH)
A

Thyroid Stimulating Hormone (TSH or thyrotrophin)

How well did you know this?
1
Not at all
2
3
4
5
Perfectly
178
Q

An excess of which hormone produces amenorrhoea?

A. Thyroid Stimulating Hormone (TSH or thyrotrophin)
B. Oxytocin
C. Insulin
D. Corticotrophin
E. Adrenaline
F. Gonadotrophin Releasing Hormone (GnRH)
G. Luteinising Hormone
H. Thyrotrophin Releasing Hormone
I. IGF-1
J. Prolactin
K. Growth Hormone (somatotrophin)
L. Follicle Stimulating Hormone
M. Growth Hormone Releasing Hormone (GHRH)
N. Leptin
O. Dopamine
P. Corticotrophin Releasing Hormone (CRH)
A

Prolactin

When leptin levels are low, the hypothalamus stops making GnRH and as a consequence LH and FSH levels fall. Amenorrhoea results. Thus during starvation or anorexia, amenorrhoea results. A high level of prolactin has the same effect, and thus galactorrhoea and amenorrhoea result from high prolactin levels.

How well did you know this?
1
Not at all
2
3
4
5
Perfectly
179
Q

A deficiency of which hormone switches off the secretion of GnRH?

A. Thyroid Stimulating Hormone (TSH or thyrotrophin)
B. Oxytocin
C. Insulin
D. Corticotrophin
E. Adrenaline
F. Gonadotrophin Releasing Hormone (GnRH)
G. Luteinising Hormone
H. Thyrotrophin Releasing Hormone
I. IGF-1
J. Prolactin
K. Growth Hormone (somatotrophin)
L. Follicle Stimulating Hormone
M. Growth Hormone Releasing Hormone (GHRH)
N. Leptin
O. Dopamine
P. Corticotrophin Releasing Hormone (CRH)
A

Leptin
When leptin levels are low, the hypothalamus stops making GnRH and as a consequence LH and FSH levels fall. Amenorrhoea results. Thus during starvation or anorexia, amenorrhoea results. A high level of prolactin has the same effect, and thus galactorrhoea and amenorrhoea result from high prolactin levels.

How well did you know this?
1
Not at all
2
3
4
5
Perfectly
180
Q

Which hormone is suppressed by an oral load of glucose during a glucose tolerance test?

A. Thyroid Stimulating Hormone (TSH or thyrotrophin)
B. Oxytocin
C. Insulin
D. Corticotrophin
E. Adrenaline
F. Gonadotrophin Releasing Hormone (GnRH)
G. Luteinising Hormone
H. Thyrotrophin Releasing Hormone
I. IGF-1
J. Prolactin
K. Growth Hormone (somatotrophin)
L. Follicle Stimulating Hormone
M. Growth Hormone Releasing Hormone (GHRH)
N. Leptin
O. Dopamine
P. Corticotrophin Releasing Hormone (CRH)
A

Growth hormone (somatotrophin)

How well did you know this?
1
Not at all
2
3
4
5
Perfectly
181
Q

X-Linked Agammaglobulinaemia

A. CD3 mutation
B. MHC Class II
C. Bruton’s tyrosine kinase (Btk) gene
D. IFN Receptor 1 gene
E. IL-2 receptor
F. WASP gene
G. CD40 Ligand gene
H. Chromosome 22q11
I. IL12 gene
A

Bruton’s tyrosine kinase (Btk) gene

Bruton’s X linked hypogammaglobulinaemia
• Defective B cell tyrosine kinase gene
• Pre B cells cannot develop to mature B cells -> Absence of mature B cells
• No circulating Ig after ~ 3 months
• Recurrent infections during childhood, bacterial, enterovirus

How well did you know this?
1
Not at all
2
3
4
5
Perfectly
182
Q

DiGeorge’s Syndrome

A. CD3 mutation
B. MHC Class II
C. Bruton’s tyrosine kinase (Btk) gene
D. IFN Receptor 1 gene
E. IL-2 receptor
F. WASP gene
G. CD40 Ligand gene
H. Chromosome 22q11
I. IL12 gene
A

Chromosome 22q11
Facial: High forehead, low set, abnormally folded ears, cleft palate, small mouth and jaw

Hypocalcaemia, oesphageal atresia, T cell lymphopenia

Complex congenital heart disease

Normal B cells numbers, reduced T cell numbers, homeostatic proliferation with age, immune function improves with age

Developmental defect of 3rd/4th pharyngeal pouch, 75% sporadic, probably involves TBX1

How well did you know this?
1
Not at all
2
3
4
5
Perfectly
183
Q

Severe Combined Immunodeficiency

A. CD3 mutation
B. MHC Class II
C. Bruton’s tyrosine kinase (Btk) gene
D. IFN Receptor 1 gene
E. IL-2 receptor
F. WASP gene
G. CD40 Ligand gene
H. Chromosome 22q11
I. IL12 gene
A

IL-2 Receptor

• Clinical phenotype of severe combined immunodeficiency:
o Unwell by 3 months of age
o Infections of all types
o Failure to thrive
o Persistent diarrhoea
o Unusual skin disease
o Colonisation of infant’s empty bone marrow by maternal lymphocytes
o Graft versus host disease
o Family history of early infant death
• Causes of SCID:
o 20 possible pathways identified
o Deficiency of cytokine receptors
o Deficiency of signalling molecules
o Metabolic defects
o Effect on different lymphocyte subsets (T, B, NK) depend on exact mutation
• Commonest form of SCID:
o X-linked SCID:
o 45% of all severe combined immunodeficiency
o Mutation of gamma chain of IL2 receptor on chromosome Xq13.1
o Shared by receptor for IL-2, IL-4, IL-7, IL-9, IL-15 and IL-21
o Inability to respond to cytokines causes early arrest of T cell and NK cell development and production of immature B cells
o Phenotype:
o Very low or absent T cell numbers
o Normal or increased B cell numbers
o Poorly developed lymphoid tissue and thymus

How well did you know this?
1
Not at all
2
3
4
5
Perfectly
184
Q

Wiskott-Aldrich Syndrome

A. CD3 mutation
B. MHC Class II
C. Bruton’s tyrosine kinase (Btk) gene
D. IFN Receptor 1 gene
E. IL-2 receptor
F. WASP gene
G. CD40 Ligand gene
H. Chromosome 22q11
I. IL12 gene
A

WASP gene
Wiskott-Aldrich syndrome is an X-linked recessive disease characterised by: thrombocytopenia lymphopenia and depressed cellular immunity immunosuppression eczema malignant lymphoma

How well did you know this?
1
Not at all
2
3
4
5
Perfectly
185
Q

Bare lymphocyte syndrome

A. CD3 mutation
B. MHC Class II
C. Bruton’s tyrosine kinase (Btk) gene
D. IFN Receptor 1 gene
E. IL-2 receptor
F. WASP gene
G. CD40 Ligand gene
H. Chromosome 22q11
I. IL12 gene
A

MHC Class II

o Defect in one of the regulatory proteins involved in Class II gene expression
o Regulatory factor X
o Class II transactivator
o Absent expression of MHC Class II molecules
o Profound deficiency of CD4+ cells
o Usually have normal number of CD8+ cells
o Normal number of B cells
o Failure to make IgG or IgA antibody
o BLS type 1 also exists due to failure of expression of HLA class I (therefore lack CD8 T cells)
o Clinical Phenotype:
• Unwell by 3 months of age
• Infections of all types
• Failure to thrive
• May be associated with sclerosing cholangitis
• Family history of early infant death

How well did you know this?
1
Not at all
2
3
4
5
Perfectly
186
Q

This region encodes C2, C4 and factor B

A. WASP gene
B. CD40 Ligand gene
C. IL-2 receptor
D. Bruton’s tyrosine kinase (Btk) gene
E. IFN Receptor 1 gene
F. MHC Class III
G. Adenosine Deaminase (ADA) gene
H. CD3 mutation
I. IL12 gene
J. Chromosome 22q11
A

MHC Class III

How well did you know this?
1
Not at all
2
3
4
5
Perfectly
187
Q

Hyper IgM Syndrome

A. WASP gene
B. CD40 Ligand gene
C. IL-2 receptor
D. Bruton’s tyrosine kinase (Btk) gene
E. IFN Receptor 1 gene
F. MHC Class III
G. Adenosine Deaminase (ADA) gene
H. CD3 mutation
I. IL12 gene
J. Chromosome 22q11
A

CD40 Ligand gene

How well did you know this?
1
Not at all
2
3
4
5
Perfectly
188
Q

A 25 year old woman comes to her GP about family planning. She is worried because she had an older brother who died before she was born and her grandmother lost two children which she things were both boys. Her GP thinks there may be a genetic disorder in her family affecting the IL-2 receptor. If correct she has a 50% of inheriting the trait from her mother and being a carrier herself. And there would be a 50% chance of passing it to her children. If inherited, her daughters would be carriers and her sons would require treatment which is usually a bone marrow transplant but gene therapy is sometimes used.

A. Bruton's X linked hypogammaglobulinaemia
B. Severe combined immunodeficiency (SCID)
C. Common variable immune deficiency
D. Reticular dysgenesis
E. TAP deficiency
F. Bare lymphocyte syndrome
G. DiGeorge syndrome
H. Selective IgA deficency
I. Hyper IgM syndrome
A

Severe combined immunodeficiency (SCID)

How well did you know this?
1
Not at all
2
3
4
5
Perfectly
189
Q

A jaundiced 8 month old child presents with failure to thrive, and a history of recurrent infections (viral, bacterial and fungal). On examination there is hepatomegally and blood tests show a raised alk phos and low CD4 count. A defect is found in the proteins that regulate MHC Class II transcription.

A. Bruton's X linked hypogammaglobulinaemia
B. Severe combined immunodeficiency (SCID)
C. Common variable immune deficiency
D. Reticular dysgenesis
E. TAP deficiency
F. Bare lymphocyte syndrome
G. DiGeorge syndrome
H. Selective IgA deficency
I. Hyper IgM syndrome
A

Bare lymphocyte syndrome

How well did you know this?
1
Not at all
2
3
4
5
Perfectly
190
Q

Patient X’s GP writes inquiring about whether to vaccinate. The patient suffers from recurrent respiratory tract infections and has been diagnosed with one of the B-cell maturation defects. For which one is immunisation still effective?

A. Bruton's X linked hypogammaglobulinaemia
B. Severe combined immunodeficiency (SCID)
C. Common variable immune deficiency
D. Reticular dysgenesis
E. TAP deficiency
F. Bare lymphocyte syndrome
G. DiGeorge syndrome
H. Selective IgA deficency
I. Hyper IgM syndrome
A

Selective IgA deficency

How well did you know this?
1
Not at all
2
3
4
5
Perfectly
191
Q

For which disorder would a bone marrow transplant be unhelpful but a thymic transplant may provide a cure?

A. Bruton's X linked hypogammaglobulinaemia
B. Severe combined immunodeficiency (SCID)
C. Common variable immune deficiency
D. Reticular dysgenesis
E. TAP deficiency
F. Bare lymphocyte syndrome
G. DiGeorge syndrome
H. Selective IgA deficency
I. Hyper IgM syndrome
A

Di George syndrome.
The thymus is affected but not the bone marrow, where B cell production and maturation take place. Hence there is no reduction in B cell numbers. B cells require T cell help (CD4+ cells) for activation and production of normal quantities of immunoglobulins.
Without this help, B cell activity is reduced.

How well did you know this?
1
Not at all
2
3
4
5
Perfectly
192
Q

In acute rejection, release of this substance from CD8+ lymphocytes helps kill target cells

A. HLA DR > A > B
B. CD4+ T cells
C. CD8+ T cells
D. Diuretics
E. Antibiotics
F. IV Immunoglobulins and Plasmapheresis
G. Amino acids
H. Diabetes
I. HLA DR > B > A
J. Hypertension
K. Hyperacute
L. Granzyme B
M. CD17+ T cells
N. HLA type
O. Interferon gamma
P. Hypotension
Q. ABO blood type
R. High dose corticosteroids
S. HLA A > B > DR
T. Free radicals
A

Granzyme B

How well did you know this?
1
Not at all
2
3
4
5
Perfectly
193
Q

In acute rejection, these are produced as a result of the activation of neutrophils and macrophages

A. HLA DR > A > B
B. CD4+ T cells
C. CD8+ T cells
D. Diuretics
E. Antibiotics
F. IV Immunoglobulins and Plasmapheresis
G. Amino acids
H. Diabetes
I. HLA DR > B > A
J. Hypertension
K. Hyperacute
L. Granzyme B
M. CD17+ T cells
N. HLA type
O. Interferon gamma
P. Hypotension
Q. ABO blood type
R. High dose corticosteroids
S. HLA A > B > DR
T. Free radicals
A

Free radicals

How well did you know this?
1
Not at all
2
3
4
5
Perfectly
194
Q

A difference in this between host and recipient is the main cause of transplant rejection

A. HLA DR > A > B
B. CD4+ T cells
C. CD8+ T cells
D. Diuretics
E. Antibiotics
F. IV Immunoglobulins and Plasmapheresis
G. Amino acids
H. Diabetes
I. HLA DR > B > A
J. Hypertension
K. Hyperacute
L. Granzyme B
M. CD17+ T cells
N. HLA type
O. Interferon gamma
P. Hypotension
Q. ABO blood type
R. High dose corticosteroids
S. HLA A > B > DR
T. Free radicals
A

HLA type

How well did you know this?
1
Not at all
2
3
4
5
Perfectly
195
Q

Along with anti-HLA antibodies, the most important screen to ensure a match before transplantation

A. HLA DR > A > B
B. CD4+ T cells
C. CD8+ T cells
D. Diuretics
E. Antibiotics
F. IV Immunoglobulins and Plasmapheresis
G. Amino acids
H. Diabetes
I. HLA DR > B > A
J. Hypertension
K. Hyperacute
L. Granzyme B
M. CD17+ T cells
N. HLA type
O. Interferon gamma
P. Hypotension
Q. ABO blood type
R. High dose corticosteroids
S. HLA A > B > DR
T. Free radicals
A

ABO blood type

How well did you know this?
1
Not at all
2
3
4
5
Perfectly
196
Q

Risk factor for chronic allograft rejection

A. HLA DR > A > B
B. CD4+ T cells
C. CD8+ T cells
D. Diuretics
E. Antibiotics
F. IV Immunoglobulins and Plasmapheresis
G. Amino acids
H. Diabetes
I. HLA DR > B > A
J. Hypertension
K. Hyperacute
L. Granzyme B
M. CD17+ T cells
N. HLA type
O. Interferon gamma
P. Hypotension
Q. ABO blood type
R. High dose corticosteroids
S. HLA A > B > DR
T. Free radicals
A

Hypertension

How well did you know this?
1
Not at all
2
3
4
5
Perfectly
197
Q

Transplanting an ABO incompatible kidney will result in ___ rejection

A. HLA DR > A > B
B. CD4+ T cells
C. CD8+ T cells
D. Diuretics
E. Antibiotics
F. IV Immunoglobulins and Plasmapheresis
G. Amino acids
H. Diabetes
I. HLA DR > B > A
J. Hypertension
K. Hyperacute
L. Granzyme B
M. CD17+ T cells
N. HLA type
O. Interferon gamma
P. Hypotension
Q. ABO blood type
R. High dose corticosteroids
S. HLA A > B > DR
T. Free radicals
A

Hyperacute

How well did you know this?
1
Not at all
2
3
4
5
Perfectly
198
Q

Treatment of acute antibody mediated rejection

A. HLA DR > A > B
B. CD4+ T cells
C. CD8+ T cells
D. Diuretics
E. Antibiotics
F. IV Immunoglobulins and Plasmapheresis
G. Amino acids
H. Diabetes
I. HLA DR > B > A
J. Hypertension
K. Hyperacute
L. Granzyme B
M. CD17+ T cells
N. HLA type
O. Interferon gamma
P. Hypotension
Q. ABO blood type
R. High dose corticosteroids
S. HLA A > B > DR
T. Free radicals
A

IV Immunoglobulins and Plasmapheresis

How well did you know this?
1
Not at all
2
3
4
5
Perfectly
199
Q

Treatment of acute cell mediated rejection

A. HLA DR > A > B
B. CD4+ T cells
C. CD8+ T cells
D. Diuretics
E. Antibiotics
F. IV Immunoglobulins and Plasmapheresis
G. Amino acids
H. Diabetes
I. HLA DR > B > A
J. Hypertension
K. Hyperacute
L. Granzyme B
M. CD17+ T cells
N. HLA type
O. Interferon gamma
P. Hypotension
Q. ABO blood type
R. High dose corticosteroids
S. HLA A > B > DR
T. Free radicals
A

High dose corticosteroids

How well did you know this?
1
Not at all
2
3
4
5
Perfectly
200
Q

The 3 most important HLA types to screen for in renal transplantation when matching donor and recipient, in order of importance

A. HLA DR > A > B
B. CD4+ T cells
C. CD8+ T cells
D. Diuretics
E. Antibiotics
F. IV Immunoglobulins and Plasmapheresis
G. Amino acids
H. Diabetes
I. HLA DR > B > A
J. Hypertension
K. Hyperacute
L. Granzyme B
M. CD17+ T cells
N. HLA type
O. Interferon gamma
P. Hypotension
Q. ABO blood type
R. High dose corticosteroids
S. HLA A > B > DR
T. Free radicals
A

HLA DR > B > A

How well did you know this?
1
Not at all
2
3
4
5
Perfectly
201
Q

Lymphocyte that responds to foreign HLA DR types

A. HLA DR > A > B
B. CD4+ T cells
C. CD8+ T cells
D. Diuretics
E. Antibiotics
F. IV Immunoglobulins and Plasmapheresis
G. Amino acids
H. Diabetes
I. HLA DR > B > A
J. Hypertension
K. Hyperacute
L. Granzyme B
M. CD17+ T cells
N. HLA type
O. Interferon gamma
P. Hypotension
Q. ABO blood type
R. High dose corticosteroids
S. HLA A > B > DR
T. Free radicals
A

CD4+ T cells

How well did you know this?
1
Not at all
2
3
4
5
Perfectly
202
Q

Lymphocyte that responds to foreign HLA A types

A. HLA DR > A > B
B. CD4+ T cells
C. CD8+ T cells
D. Diuretics
E. Antibiotics
F. IV Immunoglobulins and Plasmapheresis
G. Amino acids
H. Diabetes
I. HLA DR > B > A
J. Hypertension
K. Hyperacute
L. Granzyme B
M. CD17+ T cells
N. HLA type
O. Interferon gamma
P. Hypotension
Q. ABO blood type
R. High dose corticosteroids
S. HLA A > B > DR
T. Free radicals
A

CD8+ T cells

How well did you know this?
1
Not at all
2
3
4
5
Perfectly
203
Q

Prevents DNA replication especially of T cells

A. Metolazone
B. Cyclophosamide
C. Perindopril
D. Prednisolone
E. Dobutamine
F. Ciclosporin
G. Thyroxine
H. Chloramphenicol
I. Cyproterone acetate
J. Ribavirin
K. Infliximab
L. Gentamicin
M. Mycophenolate mofetil
N. Immunoglobulins
A

Mycophenolate mofetil

Both mycophenolate mofetil and cyclophosphamide prevent lymphocyte proliferation by inhibiting DNA replication. However, mycophenolate mofetil is more selective for T cells, whereas cycophosphamide affects B cells more than T cells. Note that cyclophosphamide at high doses will affect all cells with a high turnover

How well did you know this?
1
Not at all
2
3
4
5
Perfectly
204
Q

Causes a transient increase in neutrophil count

A. Metolazone
B. Cyclophosamide
C. Perindopril
D. Prednisolone
E. Dobutamine
F. Ciclosporin
G. Thyroxine
H. Chloramphenicol
I. Cyproterone acetate
J. Ribavirin
K. Infliximab
L. Gentamicin
M. Mycophenolate mofetil
N. Immunoglobulins
A

Prednisolone

How well did you know this?
1
Not at all
2
3
4
5
Perfectly
205
Q

Monoclonal antibodies inhibiting the actions of cytokines

A. Metolazone
B. Cyclophosamide
C. Perindopril
D. Prednisolone
E. Dobutamine
F. Ciclosporin
G. Thyroxine
H. Chloramphenicol
I. Cyproterone acetate
J. Ribavirin
K. Infliximab
L. Gentamicin
M. Mycophenolate mofetil
N. Immunoglobulins
A

Infliximab

How well did you know this?
1
Not at all
2
3
4
5
Perfectly
206
Q

Can cause gingival hypertrophy as a side effect

A. Metolazone
B. Cyclophosamide
C. Perindopril
D. Prednisolone
E. Dobutamine
F. Ciclosporin
G. Thyroxine
H. Chloramphenicol
I. Cyproterone acetate
J. Ribavirin
K. Infliximab
L. Gentamicin
M. Mycophenolate mofetil
N. Immunoglobulins
A

Ciclosporin

How well did you know this?
1
Not at all
2
3
4
5
Perfectly
207
Q

Administration of this may boost the immune system

A. Metolazone
B. Cyclophosamide
C. Perindopril
D. Prednisolone
E. Dobutamine
F. Ciclosporin
G. Thyroxine
H. Chloramphenicol
I. Cyproterone acetate
J. Ribavirin
K. Infliximab
L. Gentamicin
M. Mycophenolate mofetil
N. Immunoglobulins
A

Immunoglobulins

How well did you know this?
1
Not at all
2
3
4
5
Perfectly
208
Q

Corticosteroids, as well as being directly lymphotoxic in high doses, inhibit T-cell function via which other mechanism?

A. Mycophenolate mofetil
B. Inhibition of DNA synthesis
C. Plasmapheresis
D. Diptheria, Tetanus, Pertussis vaccine
E. Influenza type B vaccine
F. Tacrolimus
G. Blocking cytokine synthesis
H. Polio vaccine
I. Bee/wasp venom allergy
J. Atopic dermatitis
K. Infliximab
L. Bone marrow suppression
M. Goodpasture’s syndrome
A

Blocking cytokine synthesis

How well did you know this?
1
Not at all
2
3
4
5
Perfectly
209
Q

The antiproliferative drug cyclophosphamide inhibits lymphocyte proliferation by which mechanism?

A. Mycophenolate mofetil
B. Inhibition of DNA synthesis
C. Plasmapheresis
D. Diptheria, Tetanus, Pertussis vaccine
E. Influenza type B vaccine
F. Tacrolimus
G. Blocking cytokine synthesis
H. Polio vaccine
I. Bee/wasp venom allergy
J. Atopic dermatitis
K. Infliximab
L. Bone marrow suppression
M. Goodpasture’s syndrome
A

Inhibition of DNA synthesis

How well did you know this?
1
Not at all
2
3
4
5
Perfectly
210
Q

Plasmapheresis may be indicated in which condition?

A. Mycophenolate mofetil
B. Inhibition of DNA synthesis
C. Plasmapheresis
D. Diptheria, Tetanus, Pertussis vaccine
E. Influenza type B vaccine
F. Tacrolimus
G. Blocking cytokine synthesis
H. Polio vaccine
I. Bee/wasp venom allergy
J. Atopic dermatitis
K. Infliximab
L. Bone marrow suppression
M. Goodpasture’s syndrome
A

Goodpasture’s syndrome

How well did you know this?
1
Not at all
2
3
4
5
Perfectly
211
Q

Example of a vaccine that should NOT be given to a severely immunocompromised patient.

A. Mycophenolate mofetil
B. Inhibition of DNA synthesis
C. Plasmapheresis
D. Diptheria, Tetanus, Pertussis vaccine
E. Influenza type B vaccine
F. Tacrolimus
G. Blocking cytokine synthesis
H. Polio vaccine
I. Bee/wasp venom allergy
J. Atopic dermatitis
K. Infliximab
L. Bone marrow suppression
M. Goodpasture’s syndrome
A

Polio vaccine

How well did you know this?
1
Not at all
2
3
4
5
Perfectly
212
Q

A condition where antigen desensitization therapy may be indicated.

A. Mycophenolate mofetil
B. Inhibition of DNA synthesis
C. Plasmapheresis
D. Diptheria, Tetanus, Pertussis vaccine
E. Influenza type B vaccine
F. Tacrolimus
G. Blocking cytokine synthesis
H. Polio vaccine
I. Bee/wasp venom allergy
J. Atopic dermatitis
K. Infliximab
L. Bone marrow suppression
M. Goodpasture’s syndrome
A

Bee/wasp venom allergy

How well did you know this?
1
Not at all
2
3
4
5
Perfectly
213
Q

Prednisolone

A. Pneumonitis, pulmonary fibrosis and cirrhosis
B. Ototoxicity
C. Hair loss
D. Dysrhythmias
E. Bone marrow depression
F. Hypertension and reduced GFR
G. Lethargy
H. Hypertension
I. Anaphylaxis
J. Anorexia
A

Hypertension

How well did you know this?
1
Not at all
2
3
4
5
Perfectly
214
Q

Ciclosporin

A. Pneumonitis, pulmonary fibrosis and cirrhosis
B. Ototoxicity
C. Hair loss
D. Dysrhythmias
E. Bone marrow depression
F. Hypertension and reduced GFR
G. Lethargy
H. Hypertension
I. Anaphylaxis
J. Anorexia
A

Hypertension and reduced GFR

How well did you know this?
1
Not at all
2
3
4
5
Perfectly
215
Q

Azathioprine

A. Pneumonitis, pulmonary fibrosis and cirrhosis
B. Ototoxicity
C. Hair loss
D. Dysrhythmias
E. Bone marrow depression
F. Hypertension and reduced GFR
G. Lethargy
H. Hypertension
I. Anaphylaxis
J. Anorexia
A

Bone marrow suppression

How well did you know this?
1
Not at all
2
3
4
5
Perfectly
216
Q

Methotrexate

A. Pneumonitis, pulmonary fibrosis and cirrhosis
B. Ototoxicity
C. Hair loss
D. Dysrhythmias
E. Bone marrow depression
F. Hypertension and reduced GFR
G. Lethargy
H. Hypertension
I. Anaphylaxis
J. Anorexia
A

Pneumonitis, pulmonary fibrosis and cirrhosis

For methotrexate (MTX) induced cirrhosis monitor serum procollagen III rather than doing liver biopsy. MTX is given once WEEKLY as maintenance therapy in autoimmune disease; more often and you’re looking at anti-tumour regimens. Remember to replace folate.

How well did you know this?
1
Not at all
2
3
4
5
Perfectly
217
Q

Immunoglobulin

A. Pneumonitis, pulmonary fibrosis and cirrhosis
B. Ototoxicity
C. Hair loss
D. Dysrhythmias
E. Bone marrow depression
F. Hypertension and reduced GFR
G. Lethargy
H. Hypertension
I. Anaphylaxis
J. Anorexia
A

Anaphylaxis

How well did you know this?
1
Not at all
2
3
4
5
Perfectly
218
Q

The most important cell in the initiation of normal haemostasis.

A. Thromboxane A2
B. Tissue plasminogen-activator (t-PA)
C. Fibrinogen
D. Fibrin
E. a2 macroglobulin
F. Megakaryocyte
G. Endothelial cell
H. Cycloxygenase
I. Plasmin
J. Platelet
K. Protein C
L. Antithrombin III
M. Erythrocyte
A

Endothelial cell
Damage to the endothelial cells causes the release of substances that INITIATE the process of haemostasis, including platelet activation

How well did you know this?
1
Not at all
2
3
4
5
Perfectly
219
Q

The main component involved in stabilising the primary haemostatic plug.

A. Thromboxane A2
B. Tissue plasminogen-activator (t-PA)
C. Fibrinogen
D. Fibrin
E. a2 macroglobulin
F. Megakaryocyte
G. Endothelial cell
H. Cycloxygenase
I. Plasmin
J. Platelet
K. Protein C
L. Antithrombin III
M. Erythrocyte
A

Fibrin

How well did you know this?
1
Not at all
2
3
4
5
Perfectly
220
Q

A serine protease which assists in the break down of blood clots by binding to the clot and localising agents which break it down.

A. Thromboxane A2
B. Tissue plasminogen-activator (t-PA)
C. Fibrinogen
D. Fibrin
E. a2 macroglobulin
F. Megakaryocyte
G. Endothelial cell
H. Cycloxygenase
I. Plasmin
J. Platelet
K. Protein C
L. Antithrombin III
M. Erythrocyte
A

Tissue plasminogen-activator (t-PA)

How well did you know this?
1
Not at all
2
3
4
5
Perfectly
221
Q

9am Cortisol: 650nM
After 0.5mg dexamethasone suppression test:
9am Cortisol: 500nM

What is the Diagnosis:
A. Pituitary dependent Cushing’s Disease
B. Adrenal tumour causing Cushing’s syndrome
C. Ectopic ACTH causing Cushing’s syndrome
D. Normal Obese person
E. Cushing’s syndrome of indeterminate cause

A

Cushing’s syndrome of indeterminate cause

How well did you know this?
1
Not at all
2
3
4
5
Perfectly
222
Q

9am Cortisol: 500nM
After 2mg dexamethasone suppression test:
9am Cortisol: 170nM

What is the Diagnosis:
A. Pituitary dependent Cushing’s Disease
B. Adrenal tumour causing Cushing’s syndrome
C. Ectopic ACTH causing Cushing’s syndrome
D. Normal Obese person
E. Cushing’s syndrome of indeterminate cause

A

Pituitary dependent Cushing’s Disease

Pituitary receptors still have a little bit of functionality

How well did you know this?
1
Not at all
2
3
4
5
Perfectly
223
Q

A 10 year old girl with a painful knee - Normal WCC, raised ESR and CRP, x-ray shows abnormality of the tibia

Septic Arthritis
Haemophilia A
Osteomyelitis
Thrombocytopenia
Haemophilia B
A

Osteomyelitis
Unlikely to be haemophilia (female and febrile!)
Septic and Osteomyelitis are both possibilities but the x-ray changes point towards osteomyelitis

How well did you know this?
1
Not at all
2
3
4
5
Perfectly
224
Q

Test for Addison’s

Low dose Dexamethosone suppression test
High dose Dexamethosone suppression test
SynACTHen test
Glucose tolerance test

A

SynACTHen test

How well did you know this?
1
Not at all
2
3
4
5
Perfectly
225
Q

A 34 year old obese woman with type 2 diabetes presents with hypertension and bruising.
Na 146, K 2.9, U 4.0, Glucose 14.0
Aldosterone s and suggests another hormone is causing the hypertension

True or false

A

True

How well did you know this?
1
Not at all
2
3
4
5
Perfectly
226
Q

Dynamic Test for Cushing’s?

Insulin Tolerance (Stress) Test
Dexamethasone Suppression Test
SynACTHen Test
Glucose Tolerance Test
TRH Stimulation Test
A

Dexamethasone Suppression Test

How well did you know this?
1
Not at all
2
3
4
5
Perfectly
227
Q

Inhibitor of RANK ligand

Ustekinumab/Etanercept
Infliximab
Denosumab
Adalimumab/Tocilizumab

A

Denosumab - Treatment of osteoporosis

How well did you know this?
1
Not at all
2
3
4
5
Perfectly
228
Q

Inhibits IL-12/23 or TNF alpha

Ustekinumab/Etanercept
Infliximab
Denosumab
Adalimumab/Tocilizumab

A

Ustekinumab/Etanercept - Treatment of psoriasis

How well did you know this?
1
Not at all
2
3
4
5
Perfectly
229
Q

Inhibits TNF alpha

Ustekinumab/Etanercept
Infliximab
Denosumab
Adalimumab/Tocilizumab

A

Infliximab - Treatment of Crohn’s disease

How well did you know this?
1
Not at all
2
3
4
5
Perfectly
230
Q

Inhibits IL-6 or TNF alpha

Ustekinumab/Etanercept
Infliximab
Denosumab
Adalimumab/Tocilizumab

A

Adalimumab/Tocilizumab - treatment of RA

How well did you know this?
1
Not at all
2
3
4
5
Perfectly
231
Q

IFN alpha

Boosting the Immune System:
Post-transplant lymphoproliferative disorder
Part of Treatment for HepC
X linked hyper IgM syndrome
X linked SCID
Chronic Granulomatous Disease
Immunosuppressed seronegative individual after chicken pox exposure

A

Part of treatment for HepC

How well did you know this?
1
Not at all
2
3
4
5
Perfectly
232
Q

Bone Marrow Transplantation

Boosting the Immune System:
Post-transplant lymphoproliferative disorder
Part of Treatment for HepC
X linked hyper IgM syndrome
X linked SCID
Chronic Granulomatous Disease
Immunosuppressed seronegative individual after chicken pox exposure

A

X linked SCID

How well did you know this?
1
Not at all
2
3
4
5
Perfectly
233
Q

IFN gamma

Boosting the Immune System:
Post-transplant lymphoproliferative disorder
Part of Treatment for HepC
X linked hyper IgM syndrome
X linked SCID
Chronic Granulomatous Disease
Immunosuppressed seronegative individual after chicken pox exposure

A

Chronic Granulomatous Disease

How well did you know this?
1
Not at all
2
3
4
5
Perfectly
234
Q

EBV-specific CD8 T cells

Boosting the Immune System:
Post-transplant lymphoproliferative disorder
Part of Treatment for HepC
X linked hyper IgM syndrome
X linked SCID
Chronic Granulomatous Disease
Immunosuppressed seronegative individual after chicken pox exposure

A

Post-transplant lymphoproliferative disorder

How well did you know this?
1
Not at all
2
3
4
5
Perfectly
235
Q

Human Normal Immunoglobulin

Boosting the Immune System:
Post-transplant lymphoproliferative disorder
Part of Treatment for HepC
X linked hyper IgM syndrome
X linked SCID
Chronic Granulomatous Disease
Immunosuppressed seronegative individual after chicken pox exposure

A

X linked hyper IgM syndrome

How well did you know this?
1
Not at all
2
3
4
5
Perfectly
236
Q

Varicella zoster immunoglobulin

Boosting the Immune System:
Post-transplant lymphoproliferative disorder
Part of Treatment for HepC
X linked hyper IgM syndrome
X linked SCID
Chronic Granulomatous Disease
Immunosuppressed seronegative individual after chicken pox exposure

A

Immunosuppressed seronegative individual after chicken pox exposure

How well did you know this?
1
Not at all
2
3
4
5
Perfectly
237
Q

Basiliximab - Anti-IL2 receptor

  • Inhibits T cell migration but may only be used in highly active relapsing/remitting MS
  • Inhibits T cell activation and is effective in RA
  • Depletes B cells and is effective in treatment of B cell lymphoma and RA
  • Inhibits function of lymphoid and myeloid cells and used in management of RA
  • Antibody specific for CD25 which inhibits T cell activation and is used to prevent rejection
A

Antibody specific for CD25 which inhibits T cell activation and is used to prevent rejection

How well did you know this?
1
Not at all
2
3
4
5
Perfectly
238
Q

Abatacept (CTLA4-Ig fusion protein)

  • Inhibits T cell migration but may only be used in highly active relapsing/remitting MS
  • Inhibits T cell activation and is effective in RA
  • Depletes B cells and is effective in treatment of B cell lymphoma and RA
  • Inhibits function of lymphoid and myeloid cells and used in management of RA
  • Antibody specific for CD25 which inhibits T cell activation and is used to prevent rejection
A

Inhibits T cell activation and is effective in RA

How well did you know this?
1
Not at all
2
3
4
5
Perfectly
239
Q

Rituximab (Anti-CD20)

  • Inhibits T cell migration but may only be used in highly active relapsing/remitting MS
  • Inhibits T cell activation and is effective in RA
  • Depletes B cells and is effective in treatment of B cell lymphoma and RA
  • Inhibits function of lymphoid and myeloid cells and used in management of RA
  • Antibody specific for CD25 which inhibits T cell activation and is used to prevent rejection
A

Depletes B cells and is effective in treatment of B cell lymphoma and RA

How well did you know this?
1
Not at all
2
3
4
5
Perfectly
240
Q

Natalizumab (Anti-alpha4 integrin)

  • Inhibits T cell migration but may only be used in highly active relapsing/remitting MS
  • Inhibits T cell activation and is effective in RA
  • Depletes B cells and is effective in treatment of B cell lymphoma and RA
  • Inhibits function of lymphoid and myeloid cells and used in management of RA
  • Antibody specific for CD25 which inhibits T cell activation and is used to prevent rejection
A

Inhibits T cell migration but may only be used in highly active relapsing/remitting MS

How well did you know this?
1
Not at all
2
3
4
5
Perfectly
241
Q

Tocilizumab (Anti-IL6 receptor)

  • Inhibits T cell migration but may only be used in highly active relapsing/remitting MS
  • Inhibits T cell activation and is effective in RA
  • Depletes B cells and is effective in treatment of B cell lymphoma and RA
  • Inhibits function of lymphoid and myeloid cells and used in management of RA
  • Antibody specific for CD25 which inhibits T cell activation and is used to prevent rejection
A

Inhibits function of lymphoid and myeloid cells and used in management of RA

How well did you know this?
1
Not at all
2
3
4
5
Perfectly
242
Q

Cyclophosphamide

Side Effects:
Osteoporosis
Infertility
Progressive Multifocal Leukoencephalopathy
Neutropenia particularly if TPMT low
Hypertension
A

Infertility

How well did you know this?
1
Not at all
2
3
4
5
Perfectly
243
Q

Prednisolone

Side Effects:
Osteoporosis
Infertility
Progressive Multifocal Leukoencephalopathy
Neutropenia particularly if TPMT low
Hypertension
A

Osteoporosis

How well did you know this?
1
Not at all
2
3
4
5
Perfectly
244
Q

Azathioprine

Side Effects:
Osteoporosis
Infertility
Progressive Multifocal Leukoencephalopathy
Neutropenia particularly if TPMT low
Hypertension
A

Neutropenia particularly if TPMT low

How well did you know this?
1
Not at all
2
3
4
5
Perfectly
245
Q

Cyclosporin

Side Effects:
Osteoporosis
Infertility
Progressive Multifocal Leukoencephalopathy
Neutropenia particularly if TPMT low
Hypertension
A

Hypertension

How well did you know this?
1
Not at all
2
3
4
5
Perfectly
246
Q

Mycophenolate mofetil

Side Effects:
Osteoporosis
Infertility
Progressive Multifocal Leukoencephalopathy
Neutropenia particularly if TPMT low
Hypertension
A

Progressive Multifocal Leukoencephalopathy

How well did you know this?
1
Not at all
2
3
4
5
Perfectly
247
Q

FBC of an 83 year old man with no abnormal physical findings shows lymphocytosis and smear cells

Acute Lymphoblastic Leukaemia
Chronic Lymphocytic Leukaemia
HIV Infection
Infectious Mononucleosis
Whooping Cough
A

Chronic Lymphocytic Leukaemia

How well did you know this?
1
Not at all
2
3
4
5
Perfectly
248
Q

FBC of a 67 year old woman with facial plethora shows high WCC, RBCs, Hb and Haematocrit, Neutrophils and Basophils

Chronic Myeloid Leukamia
Polycythaemia vera
Pseudo-polycythaemia
Renal Artery Stenosis
Smoking Induced Hypoxia
A

Polycythaemia vera

How well did you know this?
1
Not at all
2
3
4
5
Perfectly
249
Q

FBC from a 67 year old in ITU, WCC 37.5 - raised Neutrophils, reduced lymphocyte count, immature precursors. Film shows left shift, increased Rouleaux and toxic granulation of neutrophils

Chronic Myeloid Leukaemia
Normal for age and gender
Reactive Neutrophilia
Laboratory Error
Acute Myeloid Leukaemia
A

Reactive Neutrophilia

How well did you know this?
1
Not at all
2
3
4
5
Perfectly
250
Q

FBC of a 64 year old as symptomatic Spanish woman with splenomegaly shows raised WCC. Film shows increased basophils, eosinophils and neutrophils

Chronic Myeloid Leukaemia
Normal for age and gender
Reactive Neutrophilia
Laboratory Error
Acute Myeloid Leukaemia
A

Chronic Myeloid Leukaemia

How well did you know this?
1
Not at all
2
3
4
5
Perfectly
251
Q

FBC of a North African woman with an 18 month old baby shows microcytic anaemia, hypochromia, anisocytosis, some elongated RBCs.

Normal for a North African
Beta thalassaemia major
Lead Poisoning
Beta thalassaemia trait
IDA
A

IDA - commonest causes are diet, menorrhagia, blood loss
Beta thalassaemia major is unlikely if she’s just had a baby
Lead Poisoning - rare
Beta thalassaemia trait - Hb too low for this, Hb will often be normal but there will be significant microcytosis

How well did you know this?
1
Not at all
2
3
4
5
Perfectly
252
Q

A 1 year old boy presents with joint bleeding
Normal Hb, WCC, platelet count
APTT prolonged - corrected by mixing plasma with normal plasma
PT Normal

Haemophilia A
Haemophilia B
Von Willebrand Disease
Thrombotic thrombocytopenic purpura
Has taken mother's warfarin tablets
A
Haemophilia A (Coagulation defect in a baby boy)
Results indicate something at the top of the intrinsic pathway has gone wrong - factors APTT = intrinsic pathway, used to monitor heparin treatment. Normal APTT requires factors I, II, V, VIII, IX, X, XI, &amp; XII. Notably, deficiencies in factors VII or XIII will not be detected with the APTT test.
Prolonged in: heparin use, anti-phospholipid syndrome (especially lupus anticoagulant - which paradoxically increases propensity to thrombosis), coagulation factor deficiency (Haemophilia), Sepsis (coag factor depletion), Presence of anti-bodies against coagulation factors
PT = extrinsic pathway, measures factors I (fibrinogen), II (prothrombin), V, VII and X. It is used to measure the clotting tendency of the blood - warfarin dosage, liver damage and vitamin K status
How well did you know this?
1
Not at all
2
3
4
5
Perfectly
253
Q

FBC of a North African woman with an 18 month old baby shows microcytic anaemia, hypochromia, anisocytosis, some elongated RBCs.

Normal for a North African
Beta thalassaemia major
Lead Poisoning
Beta thalassaemia trait
IDA
A

IDA
Beta thalassaemia major is unlikely if she’s just had a baby
Lead Poisoning - rare
Beta thalassaemia trait - Hb too low for this, Hb will often be normal but there will be significant microcytosis

How well did you know this?
1
Not at all
2
3
4
5
Perfectly
254
Q

6 year old Afro-Caribbean boy presents with chest and abdominal pain.
Hb 63 g/L, MCV 85fl, blood film shows sickle cells.

Most likely diagnosis?
Sickle Cell trait
Sickle Cell Anaemia
Sickle Cell/Beta Thalassaemia

A

Sickle Cell Anaemia

How well did you know this?
1
Not at all
2
3
4
5
Perfectly
255
Q

Afro Caribbean boy with jaundice, anaemia and abnormal blood film - most likely diagnosis?

Hepatitis A
Hepatits B
Hereditary Spherocytosis
G6PD deficiency

A

G6PD deficiency

Why can G6PD levels be normal in G6PD deficiency? In an acute haemolytic crisis we would mount a reticulocyte response (young red cells) which have a high level of G6PD, therefore this may elevate the measured G6PD level. Therefore you would have to measure the G6PD level after the acute haemolytic episode has resolved, to see if the patient is truly G6PD deficient. ps polychromatic macrocytes = reticulocytes (polychromasia due to ribosomal RNA)

How well did you know this?
1
Not at all
2
3
4
5
Perfectly
256
Q

Siblings with sickle cell anaemia present simultaneously with severe anaemia and a low reticulocyte count - likely diagnosis?

Splenic Sequestration
Parvovirus B19 infection
Folic Acid Deficiency
Haemolytic Crisis
Vitamin B12 deficiency
A

Parvovirus B19 infection

How well did you know this?
1
Not at all
2
3
4
5
Perfectly
257
Q

Complications of a sickle cell anaemia that are MORE common in adults than children include

Hand-foot syndrome
Hyposplenism
Red cell aplasia
Splenic Sequestration
Stroke
A

Hyposplenism

How well did you know this?
1
Not at all
2
3
4
5
Perfectly
258
Q

If you suspect an acute porphyria, what is the most useful sample to send?

Blood
CSF
Urine
Muscle Biopsy
Stool
Skin Biopsy
A

Urine to look for porphyrins

How well did you know this?
1
Not at all
2
3
4
5
Perfectly
259
Q

A single chain glycoprotein, synthesised by the liver and endothelium, which has strongly anticoagulant action and is important in the mode of action of heparin.

A. Thromboxane A2
B. Tissue plasminogen-activator (t-PA)
C. Fibrinogen
D. Fibrin
E. a2 macroglobulin
F. Megakaryocyte
G. Endothelial cell
H. Cycloxygenase
I. Plasmin
J. Platelet
K. Protein C
L. Antithrombin III
M. Erythrocyte
A

Antithrombin III - substance in plasma that inactivates thrombin
Its activity is increased manyfold by the anticoagulant drug heparin, which enhances the binding of antithrombin to factor II and factor X.

How well did you know this?
1
Not at all
2
3
4
5
Perfectly
260
Q

A single chain glycoprotein, synthesised by the liver and endothelium, which has strongly anticoagulant action and is important in the mode of action of heparin.

A. Thromboxane A2
B. Tissue plasminogen-activator (t-PA)
C. Fibrinogen
D. Fibrin
E. a2 macroglobulin
F. Megakaryocyte
G. Endothelial cell
H. Cycloxygenase
I. Plasmin
J. Platelet
K. Protein C
L. Antithrombin III
M. Erythrocyte
A

Antithrombin III

How well did you know this?
1
Not at all
2
3
4
5
Perfectly
261
Q

This product of the cyclic endoperoxides induces platelet aggregation

A. Sensitised platelet
B. Haemophilia
C. Christmas disease
D. Thromboxane A2
E. Ehlers-Danlos syndrome
F. Autoimmune thrombocytopenic purpura
G. Vitamin K deficiency
H. Megakaryocyte
I. Factor VIII deficiency
J. Prostacyclin PGI2
K. Factor XII deficiency
L. von Willebrand deficiency
M. Marfan syndrome
A

Thromboxane A2

262
Q

A 62 year old overweight woman presents to the Emergency Department following a Road Traffic Accident. A full set of investigations is carried out – which shows an increased Activated Partial Thromboplastin Time (APTT) and Prothrombin Time (PT)

A. Sensitised platelet
B. Haemophilia
C. Christmas disease
D. Thromboxane A2
E. Ehlers-Danlos syndrome
F. Autoimmune thrombocytopenic purpura
G. Vitamin K deficiency
H. Megakaryocyte
I. Factor VIII deficiency
J. Prostacyclin PGI2
K. Factor XII deficiency
L. von Willebrand deficiency
M. Marfan syndrome
A

Vitamin K deficiency
If thrombin time is up there is a fibrinogen problem.
If APTT or PT is up there is a coagulation factor dysfunction or deficiency.
If bleeding time is up there is a platelet dysfunction or deficiency.
If D-dimers (fibrin split products) are zero you can rule out DVT.
To diagnose DIC you need low platelets, low fibrinogen, raised APTT and PT, raised D-dimers, schistocytes on blood smear,
and a compatible clinical setting.

Common pathway - factors I, II, V, X
Extrinsic pathway - factor VII
Intrinsic pathway - factors VIII, IX, XI, XII

Raised PT - problem in common or extrinsic pathway
Raised APTT - problem in common or intrinisc pathway. A deficiency of any factor except VII and XIII can raise the APTT.

263
Q

A 25 year old man presents to the Emergency Department a day after attending his dentist for a routine check-up. After treatment at the dentists the previous day, his gums had not stopped bleeding. On investigation, his APTT and bleeding time are prolonged but a normal PT.

A. Sensitised platelet
B. Haemophilia
C. Christmas disease
D. Thromboxane A2
E. Ehlers-Danlos syndrome
F. Autoimmune thrombocytopenic purpura
G. Vitamin K deficiency
H. Megakaryocyte
I. Factor VIII deficiency
J. Prostacyclin PGI2
K. Factor XII deficiency
L. von Willebrand deficiency
M. Marfan syndrome
A

von Willebrand deficiency

If thrombin time is up there is a fibrinogen problem.
If APTT or PT is up there is a coagulation factor dysfunction or deficiency.
If bleeding time is up there is a platelet dysfunction or deficiency.
If D-dimers (fibrin split products) are zero you can rule out DVT.
To diagnose DIC you need low platelets, low fibrinogen, raised APTT and PT, raised D-dimers, schistocytes on blood smear,
and a compatible clinical setting.

Common pathway - factors I, II, V, X
Extrinsic pathway - factor VII
Intrinsic pathway - factors VIII, IX, XI, XII

Raised PT - problem in common or extrinsic pathway
Raised APTT - problem in common or intrinisc pathway. A deficiency of any factor except VII and XIII can raise the APTT.

264
Q

A 25 year old man presents to the Emergency Department a day after attending his dentist for a routine check-up. After treatment at the dentists the previous day, his gums had not stopped bleeding. On investigation, his APTT and bleeding time are prolonged but a normal PT.

A. Sensitised platelet
B. Haemophilia
C. Christmas disease
D. Thromboxane A2
E. Ehlers-Danlos syndrome
F. Autoimmune thrombocytopenic purpura
G. Vitamin K deficiency
H. Megakaryocyte
I. Factor VIII deficiency
J. Prostacyclin PGI2
K. Factor XII deficiency
L. von Willebrand deficiency
M. Marfan syndrome
A

von Willebrand deficiency

265
Q

A 16 year old girl presents to the Haematology Outpatients clinic describing a fluctuating history of easy bruising, epistaxis and menorrhagia. On investigation there is a thrombocytopaenia with increased megakaryocytes on BM examination.

A. Sensitised platelet
B. Haemophilia
C. Christmas disease
D. Thromboxane A2
E. Ehlers-Danlos syndrome
F. Autoimmune thrombocytopenic purpura
G. Vitamin K deficiency
H. Megakaryocyte
I. Factor VIII deficiency
J. Prostacyclin PGI2
K. Factor XII deficiency
L. von Willebrand deficiency
M. Marfan syndrome
A

Autoimmune thrombocytopenic purpura

266
Q

Which protein, important in haemostasis, is vitamin K dependent but is not a serine protease?

A. Tissue factor
B. Vascular subendothelium
C. Vascular endothelium
D. Arichidonic acid
E. Protein S
F. Tissue factor pathway inhibitor
G. Factor VII
H. Activated factor X
I. Platelets
J. Thrombin
K. Cyclooxygenase
L. Protein C
A

Protein S

267
Q

Which option is required as a cofactor for protein C activity?

A. Tissue factor
B. Vascular subendothelium
C. Vascular endothelium
D. Arichidonic acid
E. Protein S
F. Tissue factor pathway inhibitor
G. Factor VII
H. Activated factor X
I. Platelets
J. Thrombin
K. Cyclooxygenase
L. Protein C
A

Protein S

268
Q

Which option synthesises tissue factor, vWF, prostacyclin, plasminogen activator, antithrombin III and thrombomodulin?

A. Tissue factor
B. Vascular subendothelium
C. Vascular endothelium
D. Arichidonic acid
E. Protein S
F. Tissue factor pathway inhibitor
G. Factor VII
H. Activated factor X
I. Platelets
J. Thrombin
K. Cyclooxygenase
L. Protein C
A

Vascular endothelium

269
Q

Which enzyme, important for platelet aggregation, is irreversibly inhibited by aspirin?

A. Tissue factor
B. Vascular subendothelium
C. Vascular endothelium
D. Arichidonic acid
E. Protein S
F. Tissue factor pathway inhibitor
G. Factor VII
H. Activated factor X
I. Platelets
J. Thrombin
K. Cyclooxygenase
L. Protein C
A

Cyclooxygenase

270
Q

Which key clotting factor activates both factors V and VIII, and also activates protein C?

A. Tissue factor
B. Vascular subendothelium
C. Vascular endothelium
D. Arichidonic acid
E. Protein S
F. Tissue factor pathway inhibitor
G. Factor VII
H. Activated factor X
I. Platelets
J. Thrombin
K. Cyclooxygenase
L. Protein C
A

Thrombin

271
Q

A 37 year old mother of 4 children, presents to her GP because of recurrent nose bleeds and feeling tired all the time and heavy periods.

A. Bile acid malabsorption
B. Warfarin overdose
C. Antiphospholipid antibody syndrome
D. Sickle cell anaemia
E. Haemophilia A
F. Malignancy
G. Von Willebrand’s Disease
H. Henoch – Schönlein Purpura
I. Osler-Weber-Rendu Syndrome
J. Vitamin K Deficiency
K. Factor V Leiden
L. Disseminated intravascular coagulation
M. Christmas Disease
N. B-Thalassaemia
A

Osler-Weber-Rendu Syndrome
A rare autosomal dominant disorder. Alternative name = hereditary haemorrhagic telangiectasia. There is a structural abnormality of the blood vessels, resulting in telangiectases, which are thin walled so are likely to bleed. This leads to haemorrhage and anaemia. It is more common in females, and may not present until later in life. Epistaxis is the commonest presenting symptom. This patient is feeling tired, not just because of her 4 children, but because she also has iron deficiency anaemia.

272
Q

A 3 year old boy is brought to see his GP by his mother. A fortnight ago he had been brought along because of cold-like symptoms, unsurprising since it was the middle of winter and he attends nursery. He was therefore sent home with some Calpol, and as expected his symptoms soon resolved. However this morning his mother noticed a rash on his bottom, and he said his tummy ached.

A. Bile acid malabsorption
B. Warfarin overdose
C. Antiphospholipid antibody syndrome
D. Sickle cell anaemia
E. Haemophilia A
F. Malignancy
G. Von Willebrand’s Disease
H. Henoch – Schönlein Purpura
I. Osler-Weber-Rendu Syndrome
J. Vitamin K Deficiency
K. Factor V Leiden
L. Disseminated intravascular coagulation
M. Christmas Disease
N. B-Thalassaemia
A

Henoch – Schönlein Purpura
Affects children between 2-8yrs old. More common in winter. Usually presents following an upper respiratory tract infection. Rapid onset, with a palpable purpuric rash over the buttocks and legs, as well as symmetrical urticarial plaques, and haemorrhagic bullae. Arthritis of the knee and ankle. Abdominal pain – perhaps due to mesenteric vasculitis. Can have renal involvement – with haematuria/proteinuria. (not idiopathic thrombocytopenic purpura – because it’s not an option here)

273
Q

22 year old Saharawi refugee presents with anaemia, weight loss, loose stools and blood tests reveal an increased PT and slightly increased APTT, with normal thrombin time and platelet count.

A. Bile acid malabsorption
B. Warfarin overdose
C. Antiphospholipid antibody syndrome
D. Sickle cell anaemia
E. Haemophilia A
F. Malignancy
G. Von Willebrand’s Disease
H. Henoch – Schönlein Purpura
I. Osler-Weber-Rendu Syndrome
J. Vitamin K Deficiency
K. Factor V Leiden
L. Disseminated intravascular coagulation
M. Christmas Disease
N. B-Thalassaemia
A

Vitamin K Deficiency
Prevalence of coeliac disease is highest in Saharawi refugees. This patient has coeliac disease, and as a result of malabsorption is losing weight and has loose stools (steatorrhoea), and vitamin K deficiency. The blood results related to vitamin K deficiency.

274
Q

A 5 year old boy has the following blood results: normal PT, increased APTT, normal platelet count, decreased VIII:C and decreased vWF.

A. Bile acid malabsorption
B. Warfarin overdose
C. Antiphospholipid antibody syndrome
D. Sickle cell anaemia
E. Haemophilia A
F. Malignancy
G. Von Willebrand’s Disease
H. Henoch – Schönlein Purpura
I. Osler-Weber-Rendu Syndrome
J. Vitamin K Deficiency
K. Factor V Leiden
L. Disseminated intravascular coagulation
M. Christmas Disease
N. B-Thalassaemia
A

Von Willebrand’s Disease
The most common hereditary bleeding disorder, affect 1% of the population. vWF is a carrier protein for factor VIII and stabilises it. Mutation is in chromosome 12.

Haemophilia A = low FVIII and Haemophilia B = low FIX - Haemophilia is X-linked (the qu is about a boy), Von Willebrand’s Disease is autosomal dominant. - Remember that Haemophilia causes prolonged APTT, normal PT and normal bleeding time, with normal VWF level whereas von willebrand’s disease (VWD) causes prolonged APTT AND prolonged bleeding time (very impt to remember!), with low VWF level (but normal level in VWD type 2, which is functionally abnormal) - Both Haemophilia and VWD have normal platelet count. - Remember that FVIII levels may also be low in von willebrand’s disease, as von willebrand’s factor (VWF) is the carrier molecule for FVIII, preventing its premature degredation in the circulation, so if VWF is low, FVIII is vulnerable to degredation and is therefore also low.

275
Q

A 32 week pregnant lady who has gestational diabetes and is epileptic has a caesarean section while on holiday in rural China. Her newborn baby is suffering from bleeding from the umbilical stump, as well as nose and gums. What is wrong with the baby?

A. Bile acid malabsorption
B. Warfarin overdose
C. Antiphospholipid antibody syndrome
D. Sickle cell anaemia
E. Haemophilia A
F. Malignancy
G. Von Willebrand’s Disease
H. Henoch – Schönlein Purpura
I. Osler-Weber-Rendu Syndrome
J. Vitamin K Deficiency
K. Factor V Leiden
L. Disseminated intravascular coagulation
M. Christmas Disease
N. B-Thalassaemia
A

Vitamin K Deficiency
Drugs, such as anticonvulsants, which the mother is likely to be taking as she suffers from epilepsy, as well as isoniazid, rifampicin and anticoagulants, are risk factors for haemorrhagic disease of the newborn – which is what this baby has. This is due to vitamin K deficiency – although rare now in the UK as prophylactic vitamin K is given to newborns.

276
Q

A fit 48-year-old investment banker presents to A&E with a painful R arm that was present when he woke up that morning. He is otherwise well and there is no history of trauma or abnormalities of any system. On examination there is marked tenderness and mild erythema along the anterolateral aspect of the forearm and cubital fossa, with no abnormality of the upper arm or axilla.

A. Axillary vein thrombosis
B. Deep vein thrombosis
C. Inferior vena caval obstruction
D. Superficial venous thrombosis
E. Thrombophlebitis
F. Pulmonary embolism
G. DIC
H. Postphlebitic syndrome
I. Varicose veins
J. Superior vena caval obstruction
A

Superficial venous thrombosis - what it says on the tin - thrombosis of a superficial vein.

277
Q

A 45-year-old lady, known heavy smoker with chronic respiratory problems, presents to her GP with increasing dyspnoea and swelling of her R arm and face. On examination of her chest there is no asymmetry or tracheal deviation, but there are added sounds over the R upper lobe and on bending forward her face becomes congested.

A. Axillary vein thrombosis
B. Deep vein thrombosis
C. Inferior vena caval obstruction
D. Superficial venous thrombosis
E. Thrombophlebitis
F. Pulmonary embolism
G. DIC
H. Postphlebitic syndrome
I. Varicose veins
J. Superior vena caval obstruction
A

Superior vena caval obstruction

278
Q

A 56-year-old woman returns to the Vascular Clinic with recurrence of her L leg ulcer after the area has been knocked by a shopping trolley. On examination the ulcer is situated above the medial malleolus, its dimensions being 6cm x 5cm. The base is filled with yellowish slough and the surrounding area is erythematous, with prominent oedema.

A. Axillary vein thrombosis
B. Deep vein thrombosis
C. Inferior vena caval obstruction
D. Superficial venous thrombosis
E. Thrombophlebitis
F. Pulmonary embolism
G. DIC
H. Postphlebitic syndrome
I. Varicose veins
J. Superior vena caval obstruction
A

Postphlebitic syndrome

279
Q

A 48-year-old man develops R-sided pleuritic chest pain and coughs up a trace of bloodstained sputum 8 days after a R hemicolectomy. He has mild dyspnoea but chest examination and chest radiography are normal.

A. Axillary vein thrombosis
B. Deep vein thrombosis
C. Inferior vena caval obstruction
D. Superficial venous thrombosis
E. Thrombophlebitis
F. Pulmonary embolism
G. DIC
H. Postphlebitic syndrome
I. Varicose veins
J. Superior vena caval obstruction
A

Pulmonary embolism

280
Q

A 32-year-old lady develops acute swelling of her L leg 2 days post-partum. She had bilateral leg swelling during the pregnancy but the delivery was normal. On examination there is tense swelling of the leg and thigh and some deep tenderness over the calf and medial aspect of the thigh.

A. Axillary vein thrombosis
B. Deep vein thrombosis
C. Inferior vena caval obstruction
D. Superficial venous thrombosis
E. Thrombophlebitis
F. Pulmonary embolism
G. DIC
H. Postphlebitic syndrome
I. Varicose veins
J. Superior vena caval obstruction
A

Deep vein thrombosis

281
Q

A drug that is administered intravenously and has a rapid effect by potentiating the action of antithrombin. Action can be reversed quickly which is of relevance in myocardial infarction patients who may require early invasive treatment (ie PTCA).

A. Aspirin
B. Prothrombin time (PT)
C. Warfarin
D. Thrombin time (TT)
E. 5mg, 5mg, 5mg, 5mg, measure on 5th day, 8th day and then every 4 days
F. Dipyridamole modified release (MR) and aspirin
G. Clopidogrel and aspirin
H. Streptokinase
I. LMWH and aspirin
J. Dalteparin (LMWH)
K. 10mg, 10mg, 5mg, measure on 4th day then every 2 days
L. Clopidogrel
M. Unfractionated heparin (UFH)
N. APTT
O. Pentapolysaccharide
P. Calciparone
A

Unfractionated heparin (UFH)

282
Q

Potentiates antithrombin III. Usually given subcutaneously. Can cause osteoporosis and hyperkalaemia.

A. Aspirin
B. Prothrombin time (PT)
C. Warfarin
D. Thrombin time (TT)
E. 5mg, 5mg, 5mg, 5mg, measure on 5th day, 8th day and then every 4 days
F. Dipyridamole modified release (MR) and aspirin
G. Clopidogrel and aspirin
H. Streptokinase
I. LMWH and aspirin
J. Dalteparin (LMWH)
K. 10mg, 10mg, 5mg, measure on 4th day then every 2 days
L. Clopidogrel
M. Unfractionated heparin (UFH)
N. APTT
O. Pentapolysaccharide
P. Calciparone
A

Dalteparin (LMWH)

283
Q

Used to monitor patients undergoing warfarin therapy.

A. Aspirin
B. Prothrombin time (PT)
C. Warfarin
D. Thrombin time (TT)
E. 5mg, 5mg, 5mg, 5mg, measure on 5th day, 8th day and then every 4 days
F. Dipyridamole modified release (MR) and aspirin
G. Clopidogrel and aspirin
H. Streptokinase
I. LMWH and aspirin
J. Dalteparin (LMWH)
K. 10mg, 10mg, 5mg, measure on 4th day then every 2 days
L. Clopidogrel
M. Unfractionated heparin (UFH)
N. APTT
O. Pentapolysaccharide
P. Calciparone
A

Prothrombin time (PT)

284
Q

Used to monitor patients undergoing unfractionated heparin therapy.

A. Aspirin
B. Prothrombin time (PT)
C. Warfarin
D. Thrombin time (TT)
E. 5mg, 5mg, 5mg, 5mg, measure on 5th day, 8th day and then every 4 days
F. Dipyridamole modified release (MR) and aspirin
G. Clopidogrel and aspirin
H. Streptokinase
I. LMWH and aspirin
J. Dalteparin (LMWH)
K. 10mg, 10mg, 5mg, measure on 4th day then every 2 days
L. Clopidogrel
M. Unfractionated heparin (UFH)
N. APTT
O. Pentapolysaccharide
P. Calciparone
A

APTT

285
Q

This anticoagulant drug is directly contraindicated in pregnancy, especially the first 16 and last 4 weeks of a 40 week gestation.

A. Aspirin
B. Prothrombin time (PT)
C. Warfarin
D. Thrombin time (TT)
E. 5mg, 5mg, 5mg, 5mg, measure on 5th day, 8th day and then every 4 days
F. Dipyridamole modified release (MR) and aspirin
G. Clopidogrel and aspirin
H. Streptokinase
I. LMWH and aspirin
J. Dalteparin (LMWH)
K. 10mg, 10mg, 5mg, measure on 4th day then every 2 days
L. Clopidogrel
M. Unfractionated heparin (UFH)
N. APTT
O. Pentapolysaccharide
P. Calciparone
A

Warfarin

286
Q

Reflects the amount and activity of fibrinogen.

A. Aspirin
B. Prothrombin time (PT)
C. Warfarin
D. Thrombin time (TT)
E. 5mg, 5mg, 5mg, 5mg, measure on 5th day, 8th day and then every 4 days
F. Dipyridamole modified release (MR) and aspirin
G. Clopidogrel and aspirin
H. Streptokinase
I. LMWH and aspirin
J. Dalteparin (LMWH)
K. 10mg, 10mg, 5mg, measure on 4th day then every 2 days
L. Clopidogrel
M. Unfractionated heparin (UFH)
N. APTT
O. Pentapolysaccharide
P. Calciparone
A

Thrombin time (TT)

287
Q

Antiplatelet action. Indicated for primary prophylaxis of stroke in a patient experiencing recurrent retinal TIAs (amaurosis fugax). Ineffective for DVT prophylaxis.

A. Aspirin
B. Prothrombin time (PT)
C. Warfarin
D. Thrombin time (TT)
E. 5mg, 5mg, 5mg, 5mg, measure on 5th day, 8th day and then every 4 days
F. Dipyridamole modified release (MR) and aspirin
G. Clopidogrel and aspirin
H. Streptokinase
I. LMWH and aspirin
J. Dalteparin (LMWH)
K. 10mg, 10mg, 5mg, measure on 4th day then every 2 days
L. Clopidogrel
M. Unfractionated heparin (UFH)
N. APTT
O. Pentapolysaccharide
P. Calciparone
A

Aspirin

288
Q

Antiplatelet action. Licensed for secondary prophylaxis of stroke. More effective than aspirin alone. Cheap.

A. Aspirin
B. Prothrombin time (PT)
C. Warfarin
D. Thrombin time (TT)
E. 5mg, 5mg, 5mg, 5mg, measure on 5th day, 8th day and then every 4 days
F. Dipyridamole modified release (MR) and aspirin
G. Clopidogrel and aspirin
H. Streptokinase
I. LMWH and aspirin
J. Dalteparin (LMWH)
K. 10mg, 10mg, 5mg, measure on 4th day then every 2 days
L. Clopidogrel
M. Unfractionated heparin (UFH)
N. APTT
O. Pentapolysaccharide
P. Calciparone
A

Dipyridamole modified release (MR) and aspirin

289
Q

Antiplatelet action. Licensed for primary prevention of stroke in aspirin allergic patients, secondary prevention of stroke (but expensive) and in acute myocardial infarction in addition to aspirin.

A. Aspirin
B. Prothrombin time (PT)
C. Warfarin
D. Thrombin time (TT)
E. 5mg, 5mg, 5mg, 5mg, measure on 5th day, 8th day and then every 4 days
F. Dipyridamole modified release (MR) and aspirin
G. Clopidogrel and aspirin
H. Streptokinase
I. LMWH and aspirin
J. Dalteparin (LMWH)
K. 10mg, 10mg, 5mg, measure on 4th day then every 2 days
L. Clopidogrel
M. Unfractionated heparin (UFH)
N. APTT
O. Pentapolysaccharide
P. Calciparone
A

Clopidogrel

290
Q

Dangerous combination with no added efficacy and increased GI bleed.

A. Aspirin
B. Prothrombin time (PT)
C. Warfarin
D. Thrombin time (TT)
E. 5mg, 5mg, 5mg, 5mg, measure on 5th day, 8th day and then every 4 days
F. Dipyridamole modified release (MR) and aspirin
G. Clopidogrel and aspirin
H. Streptokinase
I. LMWH and aspirin
J. Dalteparin (LMWH)
K. 10mg, 10mg, 5mg, measure on 4th day then every 2 days
L. Clopidogrel
M. Unfractionated heparin (UFH)
N. APTT
O. Pentapolysaccharide
P. Calciparone
A

Clopidogrel and aspirin

291
Q

New (recommended, Tait) model of starting warfarin

A. Aspirin
B. Prothrombin time (PT)
C. Warfarin
D. Thrombin time (TT)
E. 5mg, 5mg, 5mg, 5mg, measure on 5th day, 8th day and then every 4 days
F. Dipyridamole modified release (MR) and aspirin
G. Clopidogrel and aspirin
H. Streptokinase
I. LMWH and aspirin
J. Dalteparin (LMWH)
K. 10mg, 10mg, 5mg, measure on 4th day then every 2 days
L. Clopidogrel
M. Unfractionated heparin (UFH)
N. APTT
O. Pentapolysaccharide
P. Calciparone
A

5mg, 5mg, 5mg, 5mg, measure on 5th day, 8th day and then every 4 days

292
Q

In patients with metallic heart valves, this drug is the most effective anticoagulant

A. Aspirin
B. Prothrombin time (PT)
C. Warfarin
D. Thrombin time (TT)
E. 5mg, 5mg, 5mg, 5mg, measure on 5th day, 8th day and then every 4 days
F. Dipyridamole modified release (MR) and aspirin
G. Clopidogrel and aspirin
H. Streptokinase
I. LMWH and aspirin
J. Dalteparin (LMWH)
K. 10mg, 10mg, 5mg, measure on 4th day then every 2 days
L. Clopidogrel
M. Unfractionated heparin (UFH)
N. APTT
O. Pentapolysaccharide
P. Calciparone
A

Warfarin

293
Q

In patients with cancer and acute venous thromboembolism, the most effective drug at reducing the risk of recurrent VTE is __?

A. Aspirin
B. Prothrombin time (PT)
C. Warfarin
D. Thrombin time (TT)
E. 5mg, 5mg, 5mg, 5mg, measure on 5th day, 8th day and then every 4 days
F. Dipyridamole modified release (MR) and aspirin
G. Clopidogrel and aspirin
H. Streptokinase
I. LMWH and aspirin
J. Dalteparin (LMWH)
K. 10mg, 10mg, 5mg, measure on 4th day then every 2 days
L. Clopidogrel
M. Unfractionated heparin (UFH)
N. APTT
O. Pentapolysaccharide
P. Calciparone
A

Dalteparin (LMWH)

294
Q

This drug when given alone initially increases the clotting risk

A. Aspirin
B. Prothrombin time (PT)
C. Warfarin
D. Thrombin time (TT)
E. 5mg, 5mg, 5mg, 5mg, measure on 5th day, 8th day and then every 4 days
F. Dipyridamole modified release (MR) and aspirin
G. Clopidogrel and aspirin
H. Streptokinase
I. LMWH and aspirin
J. Dalteparin (LMWH)
K. 10mg, 10mg, 5mg, measure on 4th day then every 2 days
L. Clopidogrel
M. Unfractionated heparin (UFH)
N. APTT
O. Pentapolysaccharide
P. Calciparone
A

Warfarin alone initially increases the clotting risk because proteins C and S (the anticoagulant proteins) have shorter half lives and disappear from the blood faster than factors II, VII, IX, X (the procoagulant proteins). Since this is dangerous, warfarin is always started under heparin cover.

295
Q

Side effects include cutaneous necrosis

A. Aspirin
B. Prothrombin time (PT)
C. Warfarin
D. Thrombin time (TT)
E. 5mg, 5mg, 5mg, 5mg, measure on 5th day, 8th day and then every 4 days
F. Dipyridamole modified release (MR) and aspirin
G. Clopidogrel and aspirin
H. Streptokinase
I. LMWH and aspirin
J. Dalteparin (LMWH)
K. 10mg, 10mg, 5mg, measure on 4th day then every 2 days
L. Clopidogrel
M. Unfractionated heparin (UFH)
N. APTT
O. Pentapolysaccharide
P. Calciparone
A

Warfarin

296
Q

The drug most likely to cause thrombocytopaenia with paradoxical thrombosis

A. Aspirin
B. Prothrombin time (PT)
C. Warfarin
D. Thrombin time (TT)
E. 5mg, 5mg, 5mg, 5mg, measure on 5th day, 8th day and then every 4 days
F. Dipyridamole modified release (MR) and aspirin
G. Clopidogrel and aspirin
H. Streptokinase
I. LMWH and aspirin
J. Dalteparin (LMWH)
K. 10mg, 10mg, 5mg, measure on 4th day then every 2 days
L. Clopidogrel
M. Unfractionated heparin (UFH)
N. APTT
O. Pentapolysaccharide
P. Calciparone
A

Unfractionated heparin (UFH)
Heparin induced thrombocytopaenia (HIT).
Type I is a predictable dose related effect mediated by platelet aggregation.
Type II is an unpredictable immune response with antibodies directed against the heparin-platelet factor 4 complex.
The large aggregates cause thrombosis.
Treatment is to stop the heparin and start a direct thrombin inhibitor, e.g. hirudin.
HIT more commonly follows treatment with UFH than with LMWH.

297
Q

Indicated as thrombotic prophylaxis in DIC

A. Aspirin
B. Prothrombin time (PT)
C. Warfarin
D. Thrombin time (TT)
E. 5mg, 5mg, 5mg, 5mg, measure on 5th day, 8th day and then every 4 days
F. Dipyridamole modified release (MR) and aspirin
G. Clopidogrel and aspirin
H. Streptokinase
I. LMWH and aspirin
J. Dalteparin (LMWH)
K. 10mg, 10mg, 5mg, measure on 4th day then every 2 days
L. Clopidogrel
M. Unfractionated heparin (UFH)
N. APTT
O. Pentapolysaccharide
P. Calciparone
A

Dalteparin (LMWH)
DIC is a systemic process with the potential for causing thrombosis and hemorrhage. The processes of coagulation and fibrinolysis become abnormally (and often massively) activated within the vasculature, leading to ongoing coagulation and fibrinolysis.
Procoagulant exposure – Blood is exposed to one or more procoagulants, such as tissue factor (TF), from which it is normally protected.
Coagulation – Activation of the coagulation cascade leads to the production of thrombi consisting of fibrin and platelets. These can occur in the microvasculature and/or larger vessels. Formation of thrombi in turn leads to consumption of endogenous coagulation factors, platelets, and anticoagulant factors (eg, protein S, protein C, antithrombin).
Fibrinolysis – Fibrinolysis is activated at sites of thrombus formation, with generation of fibrin degradation products (FDP) that, when present in significant amounts, interfere with both fibrin clot formation and platelet aggregation.
End organ damage – Tissue or organ damage may result from reduced perfusion, thrombosis, and/or bleeding. Often, contributions from DIC itself and the condition that precipitated it are intertwined. Organ failure may result in significant morbidity and mortality.
Common causes of DIC include the following:
●Sepsis from a variety of organisms (bacterial, fungal, viral, and parasitic)
●Malignancy, especially acute promyelocytic leukemia, mucinous tumors (eg, pancreatic, gastric, ovarian), and brain tumors
●Trauma, especially to the central nervous system
●Obstetrical complications, including preeclampsia, retained dead fetus, acute fatty liver of pregnancy
●Intravascular hemolysis, often due to acute hemolytic transfusion reaction (AHTR) in the setting of ABO incompatible transfusion, but also in other forms of hemolysis such as in severe malaria

298
Q

Contra-indicated if recent sore throat, if ever used before, or in the presence of proliferative retinopathy.

A. Aspirin
B. Prothrombin time (PT)
C. Warfarin
D. Thrombin time (TT)
E. 5mg, 5mg, 5mg, 5mg, measure on 5th day, 8th day and then every 4 days
F. Dipyridamole modified release (MR) and aspirin
G. Clopidogrel and aspirin
H. Streptokinase
I. LMWH and aspirin
J. Dalteparin (LMWH)
K. 10mg, 10mg, 5mg, measure on 4th day then every 2 days
L. Clopidogrel
M. Unfractionated heparin (UFH)
N. APTT
O. Pentapolysaccharide
P. Calciparone
A

Streptokinase: single chain polypeptide derived from beta-hemolytic streptococcus cultures. It binds to plasminogen, forming a complex that becomes an active enzyme that cleaves peptide bonds on other plasminogen molecules, leading to plasmin activation

299
Q

A 65 year old patient presents with hepatosplenomegaly. He is mildly anaemic and thrombocytompenic. A blood monocyte count of 1.2 x 109/l is observed. Bone marrow aspirate reveals ring sideroblasts at 15% of total blasts. Auer rods are observed.

A. Myelodysplastic syndrome,unclassifiable
B. Refractory anaemia
C. Refractory Anaemia with excess of Blasts II
D. Aplastic Anaemia
E. Refractory Cytopaenia with Multilineage Dysplasia
F. Chronic Myelomonocytic Anaemia
G. Myelodysplastic syndrome associated with isolated del(5q) chromosome abnormality (5q syndrome)
H. Acute Myeloid Leukaemia
I. Secondary Sideroblastic Anaemia
J. Refractory Anaemia with Ring Sideroblasts
K. Refractory Anaemia with excess of Blasts I

A

Chronic Myelomonocytic Anaemia
By definition, this has to be CMML, because the patient has:

a) 1/0 x 10^9/L

The diagnosis of AML requires >20% blasts, so clearly this patient doesn’t meet the criteria.

Chronic myelomonocytic leukemia (CMML)
- not to be confused with chronic myelogenous leukemia or CML
- characterized by less than 20% myeloblasts in the bone marrow
and greater than 1000 * 109/uL monocytes circulating in the peripheral blood.
Also something that often comes up in exams:
- syndrome, typically seen in older women with normal or high platelet counts
and isolated deletions of the long arm of chromosome 5 in bone marrow cells.

300
Q

An alcoholic presents to your clinic with anaemia. Sideroblasts are observed on morphological examination.

A. Myelodysplastic syndrome,unclassifiable
B. Refractory anaemia
C. Refractory Anaemia with excess of Blasts II
D. Aplastic Anaemia
E. Refractory Cytopaenia with Multilineage Dysplasia
F. Chronic Myelomonocytic Anaemia
G. Myelodysplastic syndrome associated with isolated del(5q) chromosome abnormality (5q syndrome)
H. Acute Myeloid Leukaemia
I. Secondary Sideroblastic Anaemia
J. Refractory Anaemia with Ring Sideroblasts
K. Refractory Anaemia with excess of Blasts I

A

Secondary Sideroblastic Anaemia

(Refractory anemia with ringed sideroblasts (RARS)
- also characterized by less than 5% myeloblasts in the bone marrow,
but distinguished by the presence of 15% or greater red cell precursors in the marrow
being abnormal iron-stuffed cells called ‘ringed sideroblasts’)

301
Q

A 58 year old lady complains of lethargy and “easy bruising”. She presents with purpura. Her FBC reveals Hb 10.5g/dl; WBCs 2.3x109/l and platelets 8x109/l. Blood film reveals

A

Refractory Cytopaenia with Multilineage Dysplasia: To count as significant, dysplasia must involve at least 10% of cells in a lineage. To count as RCMD, at least 2 MYELOID lineages must be dysplastic, and there must be bi or pancytopenia in the peripheral blood. (This can include anaemia!)

Refractory cytopenia with multilineage dysplasia:
Diagnosis requires at least 2 of the following:
• low red blood cell count
• low white blood cell count
• low platelet count
With monocytes less than 1 x 109/L, no/few blast cells and no Auer rods.
Bone marrow shows dysplasia in 2 or more cell types, affecting at least 10% of the cells, and less than 5% blasts
It is a form of Myelodysplastic Syndromes (MDS) - a group of related disorders in which stem cells in the bone marrow malfunction. Stem cells develop into red blood cells, white blood cells, and platelets. In MDS, defective stem cells produce too many defective blood cells and too few normal blood cells.

302
Q

A 78 year old male patient with recurring infections of the face and maxillary sinuses associated with neutropenia. His bloods are: Hb 9.8 g/dl; WBC 1.3x109/l; Neutrophils 0.3x109/l; platelets 38x109/l.The lab informs you that there are Blasts approximately compromise 17% of bone marrow aspirate.

A. Myelodysplastic syndrome,unclassifiable
B. Refractory anaemia
C. Refractory Anaemia with excess of Blasts II
D. Aplastic Anaemia
E. Refractory Cytopaenia with Multilineage Dysplasia
F. Chronic Myelomonocytic Anaemia
G. Myelodysplastic syndrome associated with isolated del(5q) chromosome abnormality (5q syndrome)
H. Acute Myeloid Leukaemia
I. Secondary Sideroblastic Anaemia
J. Refractory Anaemia with Ring Sideroblasts
K. Refractory Anaemia with excess of Blasts I

A

Refractory Anaemia with excess of Blasts II
- characterized by 5-19% myeloblasts in the marrow
Refractory anaemia with an excess of blasts can be categorised into 1 and 2.
Both involve have monocytes less than 1 x 109/L, bone marrow showing dysplasia in 1 or more cell types and at least 1 of the following:
• low red blood cell count
• low white blood cell count
• low platelet count
But in type 1: there are no Auer rods in the peripheral film or bone marrow, less than 5% blasts in peripheral film and 5-19% blasts in the bone marrow
Whereas type 2: shows Auer rods in the peripheral film and bone marrow, 5-19% blasts in peripheral film, 10-19% blasts in the bone marrow
Refractory anemia with excess blasts in transformation (RAEB-T) - characterized by 20-29% myeloblasts in the marrow

303
Q

You are called to A&E to see a 65 year old man. He is complaining of fever, shortness of breath, and has lost 5Kg in the last few months. His notes say he was previously diagnosed with “Refractory Anaemia with excess Blasts in Transformation” (RAEB-t). His blast cell count is approximately 30% of all nucleated cells.

A. Myelodysplastic syndrome,unclassifiable
B. Refractory anaemia
C. Refractory Anaemia with excess of Blasts II
D. Aplastic Anaemia
E. Refractory Cytopaenia with Multilineage Dysplasia
F. Chronic Myelomonocytic Anaemia
G. Myelodysplastic syndrome associated with isolated del(5q) chromosome abnormality (5q syndrome)
H. Acute Myeloid Leukaemia
I. Secondary Sideroblastic Anaemia
J. Refractory Anaemia with Ring Sideroblasts
K. Refractory Anaemia with excess of Blasts I

A

Acute Myeloid Leukaemia
Refractory anaemia with an excess of blasts only takes you upto 20% in the bone marrow, anything greater than that is AML…remember AML is an associated condition and can develop from MDS
Cancer of the myeloid line of blood cells, characterized by the rapid growth of abnormal white blood cells that accumulate in the bone marrow and interfere with the production of normal blood cells. Acute myeloid leukemia (AML) is the most common acute leukemia in adults
The symptoms of AML are caused by replacement of normal bone marrow with leukemic cells, which causes a drop in red blood cells, platelets, and normal white blood cells. These symptoms include fatigue, shortness of breath, easy bruising and bleeding, and increased risk of infection.
Enlargement of the spleen may occur in AML, but it is typically mild and asymptomatic. Lymph node swelling is rare in AML, in contrast to acute lymphoblastic leukemia.
Down syndrome is associated with a 10- to 18-fold increase in the risk of AML.

304
Q

A 34 year old man with peripheral cytopenia suffers from bleeding gums. Peripheral blood shows 5% blast cells and bone marrow 42% blast cells.

A. Refractory cytopenia with multilineage dysplasia
B. Inherited aplastic anaemia
C. Secondary aplastic anaemia
D. Juvenile myelomonocytic leukaemia
E. Acute myeloid leukaemia
F. 5q syndrome
G. Refractory anaemia with an excess of blasts
H. Idiopathic aplastic anaemia
I. Myelofibrosis
J. Refractory anaemia
A

Acute myeloid leukaemia
Refractory anaemia with an excess of blasts only takes you upto 20% in the bone marrow, anything greater than that is AML…remember AML is an associated condition and can develop from MDS
Cancer of the myeloid line of blood cells, characterized by the rapid growth of abnormal white blood cells that accumulate in the bone marrow and interfere with the production of normal blood cells. Acute myeloid leukemia (AML) is the most common acute leukemia in adults
The symptoms of AML are caused by replacement of normal bone marrow with leukemic cells, which causes a drop in red blood cells, platelets, and normal white blood cells. These symptoms include fatigue, shortness of breath, easy bruising and bleeding, and increased risk of infection.
Enlargement of the spleen may occur in AML, but it is typically mild and asymptomatic. Lymph node swelling is rare in AML, in contrast to acute lymphoblastic leukemia.
Down syndrome is associated with a 10- to 18-fold increase in the risk of AML.

305
Q

A 74 year old woman with high-normal platelet count. Bone marrow aspirate shows hyperplasia of hypolobulated micromegakaryocytes. Responds well to lenalidomide.

A. Refractory cytopenia with multilineage dysplasia
B. Inherited aplastic anaemia
C. Secondary aplastic anaemia
D. Juvenile myelomonocytic leukaemia
E. Acute myeloid leukaemia
F. 5q syndrome
G. Refractory anaemia with an excess of blasts
H. Idiopathic aplastic anaemia
I. Myelofibrosis
J. Refractory anaemia
A

5q Syndrome - myelodysplasia with the 5q- syndrome
The Clinical picture is usually:
- Female, 74 years with mild to moderate degrees of anemia and a low white blood cell count.
- Normal or increased platelet counts along with bone marrow hyperplasia of hypo-lobulated micro-megakaryocytes
- Less than 5% blasts and no Auer rods

Approximately 5 percent of patients with MDS present with “5q minus syndrome” characterized by severe anemia, preserved platelet counts, and del(5q) as the sole cytogenetic abnormality. Patients with 5q minus syndrome have high response rates to treatment with low-dose lenalidomide.

306
Q

A 20 year old man with hepatitis C complains of fatigue and breathlessness and bruises very easily.

A. Refractory cytopenia with multilineage dysplasia
B. Inherited aplastic anaemia
C. Secondary aplastic anaemia
D. Juvenile myelomonocytic leukaemia
E. Acute myeloid leukaemia
F. 5q syndrome
G. Refractory anaemia with an excess of blasts
H. Idiopathic aplastic anaemia
I. Myelofibrosis
J. Refractory anaemia
A

Secondary aplastic anaemia - fatigue and breathlessness (anaemia), bruises (thrombocytopenia)
Aplastic anemia (AA) is characterized by diminished or absent hematopoietic precursors in the bone marrow, most often due to injury to the pluripotent stem cell. The diagnosis of AA should be considered in every patient presenting with pancytopenia (simultaneous presence of anemia, thrombocytopenia, and neutropenia).
Examination of the peripheral blood smear reveals normocytic or macrocytic red cells with a marked reduction in polychromatophilia (reticulocytes). Neutrophils and platelets are decreased in number. Abnormal circulating white blood cells are not present. The bone marrow biopsy is profoundly hypocellular with a decrease in all elements; the marrow space is composed mostly of fat cells and marrow stroma. Residual hematopoietic cells are morphologically normal, and infiltration with malignant cells or fibrosis is not present.
Hepatitis viruses and HIV can cause severe aplasia. The mechanism may involve T cell activation with release of cytokines (see below), or activation of a cytotoxic T cell clone which recognizes similar target antigens on both liver and bone marrow cells. Hepatitis-associated disease most often affects boys and young men, with aplasia developing two to three months after an episode of acute hepatitis

307
Q

This patients blood film shows classic Pelger-Huet neutrophils and bone marrow blasts make up 15% of cells.

A. Refractory cytopenia with multilineage dysplasia
B. Inherited aplastic anaemia
C. Secondary aplastic anaemia
D. Juvenile myelomonocytic leukaemia
E. Acute myeloid leukaemia
F. 5q syndrome
G. Refractory anaemia with an excess of blasts
H. Idiopathic aplastic anaemia
I. Myelofibrosis
J. Refractory anaemia
A

Refractory anaemia with an excess of blasts
Pelger-Huet anomaly: can occur as an inherited disorder or can be acquired in patients with myelodysplastic syndromes. There is reduced lobulation of mature neutrophils. Such cells typically have a bilobed nucleus connected by a thin strand, giving a “pince-nez” appearance, often accompanied by reduced or absent granulation
Refractory anaemia with an excess of blasts can be categorised into 1 and 2.
Both involve have monocytes less than 1 x 109/L, bone marrow showing dysplasia in 1 or more cell types and at least 1 of the following:
• low red blood cell count
• low white blood cell count
• low platelet count
But in type 1: there are no Auer rods in the peripheral film or bone marrow, less than 5% blasts in peripheral film and 5-19% blasts in the bone marrow
Whereas type 2: shows Auer rods in the peripheral film and bone marrow, 5-19% blasts in peripheral film, 10-19% blasts in the bone marrow

308
Q

In this case haemoglobin is normal but there is a reduction in platelets and neutrophils

A. Refractory cytopenia with multilineage dysplasia
B. Inherited aplastic anaemia
C. Secondary aplastic anaemia
D. Juvenile myelomonocytic leukaemia
E. Acute myeloid leukaemia
F. 5q syndrome
G. Refractory anaemia with an excess of blasts
H. Idiopathic aplastic anaemia
I. Myelofibrosis
J. Refractory anaemia
A

Refractory cytopenia with multilineage dysplasia:
Diagnosis requires at least 2 of the following:
• low red blood cell count
• low white blood cell count
• low platelet count
With monocytes less than 1 x 109/L, no/few blast cells and no Auer rods.
Bone marrow shows dysplasia in 2 or more cell types, affecting at least 10% of the cells, and less than 5% blasts
It is a form of Myelodysplastic Syndromes (MDS) - a group of related disorders in which stem cells in the bone marrow malfunction. Stem cells develop into red blood cells, white blood cells, and platelets. In MDS, defective stem cells produce too many defective blood cells and too few normal blood cells.

309
Q

A 64-year old woman receiving long-term chemotherapy for lymphoma presents with worsening bone pain, recurrent fever and night sweats. Blood film shows blast cells with Auer rods.

A. Vincristine poisoning
B. DIC
C. Chronic lymphocytic leukaemia
D. Chronic myeloid leukaemia
E. Tumour-lysis syndrome
F. Bronchial carcinoma
G. Septicaemia
H. Hairy cell leukaemia
I. Acute promyelocytic leukaemia
J. Lung fibrosis
K. Acute myeloid leukaemia
L. Hypothyroidism
M. Acute lymphoblastic leukaemia
N. Richter's syndrome
A

Acute Myeloid Leukaemia
Cancer of the myeloid line of blood cells, characterized by the rapid growth of abnormal white blood cells that accumulate in the bone marrow and interfere with the production of normal blood cells. Acute myeloid leukemia (AML) is the most common acute leukemia in adults
The symptoms of AML are caused by replacement of normal bone marrow with leukemic cells, which causes a drop in red blood cells, platelets, and normal white blood cells. These symptoms include fatigue, shortness of breath, easy bruising and bleeding, and increased risk of infection.
Enlargement of the spleen may occur in AML, but it is typically mild and asymptomatic. Lymph node swelling is rare in AML, in contrast to acute lymphoblastic leukemia.
Down syndrome is associated with a 10- to 18-fold increase in the risk of AML.

310
Q

A 61-year-old man with CLL presents with recurrent pneumonia and haemoptysis. On fibreoptic bronchoscopy, the patient is found to have an endobronchial mass. The biopsy shows anaplastic, large cell lymphoma.

A. Vincristine poisoning
B. DIC
C. Chronic lymphocytic leukaemia
D. Chronic myeloid leukaemia
E. Tumour-lysis syndrome
F. Bronchial carcinoma
G. Septicaemia
H. Hairy cell leukaemia
I. Acute promyelocytic leukaemia
J. Lung fibrosis
K. Acute myeloid leukaemia
L. Hypothyroidism
M. Acute lymphoblastic leukaemia
N. Richter's syndrome
A

Richter’s Syndrome - development of an aggressive large-cell lymphoma in the setting of underlying chronic lymphocytic leukaemia.
The onset of RT is heralded by sudden clinical deterioration, characterized by a marked increase in lymphadenopathy at one or more sites (often abdominal), splenomegaly, and worsening “B” symptoms (ie, fever, night sweats, weight loss). The serum level of lactate dehydrogenase (LDH) is elevated in 50 to 80 percent of patients with RT compared with 8 percent of CLL patients.
Biopsy is required to confirm the diagnosis, and usually shows a histologic pattern consistent with diffuse large B cell lymphoma.
The prognosis and outcome are historically poor for RT and the disease is invariably fatal if left untreated

311
Q

A newly diagnosed ALL patient complains of tiredness, polyuria, polydipsia, abdominal pain and vomiting on receiving chemotherapy. On examination, BP: 160/100mmHg, temp: 39ºC, and ECG shows tented T waves. Blood test shows serum K+: 6.9mmol/L and phosphate: 7.1 mmol/L. The patient later dies of cardiac arrest.

A. Vincristine poisoning
B. DIC
C. Chronic lymphocytic leukaemia
D. Chronic myeloid leukaemia
E. Tumour-lysis syndrome
F. Bronchial carcinoma
G. Septicaemia
H. Hairy cell leukaemia
I. Acute promyelocytic leukaemia
J. Lung fibrosis
K. Acute myeloid leukaemia
L. Hypothyroidism
M. Acute lymphoblastic leukaemia
N. Richter's syndrome
A

Tumour Lysis Syndrome (TLS) is an oncologic emergency that is caused by massive tumor cell lysis with the release of large amounts of potassium, phosphate, and nucleic acids into the systemic circulation resulting in hyperkalemia, hyperphosphatemia, secondary hypocalcemia, hyperuricemia, and acute kidney injury. Catabolism of the nucleic acids to uric acid leads to hyperuricemia, and the marked increase in uric acid excretion can result in the precipitation of uric acid in the renal tubules and can also induce renal vasoconstriction, impaired autoregulation, decreased renal blood flow, and inflammation, resulting in acute kidney injury. Hyperphosphatemia with calcium phosphate deposition in the renal tubules can also cause acute kidney injury.
The symptoms associated with TLS largely reflect the associated metabolic abnormalities (hyperkalemia, hyperphosphatemia, and hypocalcemia). They include nausea, vomiting, diarrhea, anorexia, lethargy, hematuria, heart failure, cardiac dysrhythmias, seizures, muscle cramps, tetany, syncope, and possible sudden death
TLS most often occurs after the initiation of cytotoxic therapy in patients with high-grade lymphomas (particularly the Burkitt subtype) and acute lymphoblastic leukemia.

312
Q

A routine medical of 33-year-old footballer reveals: Hb = 9.9g/dl and WCC = 130 x 109/L. His blood film shows whole spectrum of myeloid precursors, including a few blast cells. He admits to having frequent night sweats and blurred vision. There is a presence of Ph chromosome t(9;22) on cytogenetic analysis.

A. Vincristine poisoning
B. DIC
C. Chronic lymphocytic leukaemia
D. Chronic myeloid leukaemia
E. Tumour-lysis syndrome
F. Bronchial carcinoma
G. Septicaemia
H. Hairy cell leukaemia
I. Acute promyelocytic leukaemia
J. Lung fibrosis
K. Acute myeloid leukaemia
L. Hypothyroidism
M. Acute lymphoblastic leukaemia
N. Richter's syndrome
A

Chronic Myeloid Leukaemia: a myeloproliferative neoplasm characterized by the dysregulated production and uncontrolled proliferation of mature and maturing granulocytes with fairly normal differentiation.
CML is associated with the fusion of two genes: BCR (on chromosome 22) and ABL1 (on chromosome 9) resulting in the BCR-ABL1 fusion gene. This abnormal fusion typically results from a reciprocal translocation between chromosomes 9 and 22, t(9;22)(q34;q11), that gives rise to an abnormal chromosome 22 called the Philadelphia (Ph) chromosome. It is this deregulated tyrosine kinase that is implicated in the pathogenesis of CML.
The clinical hallmark of CML is the uncontrolled production of mature and maturing granulocytes, predominantly neutrophils, but also basophils and eosinophils. In the absence of treatment, CML has a triphasic or biphasic clinical course as it progresses from a chronic phase to an accelerated phase and on to a terminal blast crisis.
The peripheral smear typically demonstrates a leukocytosis. Bone marrow aspiration and biopsy demonstrates granulocytic hyperplasia with a maturation pattern that reflects that seen in the peripheral smear
Treatment: tyrosine kinase inhibitor (imatinib)

313
Q

A 5-year-old girl presents with failure to thrive, recurrent fever and bruising. Immunotyping reveals the presence of CD10.

A. Vincristine poisoning
B. DIC
C. Chronic lymphocytic leukaemia
D. Chronic myeloid leukaemia
E. Tumour-lysis syndrome
F. Bronchial carcinoma
G. Septicaemia
H. Hairy cell leukaemia
I. Acute promyelocytic leukaemia
J. Lung fibrosis
K. Acute myeloid leukaemia
L. Hypothyroidism
M. Acute lymphoblastic leukaemia
N. Richter's syndrome
A

Acute Lymphoblastic Leukaemia, the most common form of cancer in children, comprises approximately 30 percent of all childhood malignancies. Not likely to be AML as AML accounts for less than 10 percent of acute leukemias in children less than 10 years of age.
The most common presenting symptoms of ALL are nonspecific (eg, fever, bleeding, bone pain, lymphadenopathy). Bone pain, particularly affecting the long bones, and caused by leukemic involvement of the periosteum, is a presenting symptom in 21 to 38 percent of cases of acute leukaemia.
Lymphadenopathy associated with malignancy usually is non-tender, firm, rubbery, and matted - Approximately 50 percent of children with ALL present with lymphadenopathy.
Most children with ALL have anemia and/or thrombocytopenia with either normal or depressed WBC counts and lymphoblasts on peripheral smear. Approximately one-half of children with ALL present with bleeding (including petechiae and purpura) and three-quarters have a platelet count

314
Q

A 6-year-old boy presents with bone pain. On examination you notice he looks pale and has many bruises. What is his diagnosis?

A. Magnesium exposure
B. Chronic myeloid leukaemia
C. Lymphocytes
D. Ionising radiation
E. Thalassaemia
F. Acute lymphocytic leukaemia
G. Blast cells
H. Chronic lymphocytic leukaemia
I. Acute myeloid leukaemia
J. Neutrophils
K. Sickle cell disease
L. Marfan’s syndrome
M. Down’s syndrome
A

Acute Lymphoblastic Leukaemia, the most common form of cancer in children, comprises approximately 30 percent of all childhood malignancies. Not likely to be AML as AML accounts for less than 10 percent of acute leukemias in children less than 10 years of age.
The most common presenting symptoms of ALL are nonspecific (eg, fever, bleeding, bone pain, lymphadenopathy). Bone pain, particularly affecting the long bones, and caused by leukemic involvement of the periosteum, is a presenting symptom in 21 to 38 percent of cases of acute leukaemia.
Lymphadenopathy associated with malignancy usually is non-tender, firm, rubbery, and matted - Approximately 50 percent of children with ALL present with lymphadenopathy.
Most children with ALL have anemia and/or thrombocytopenia with either normal or depressed WBC counts and lymphoblasts on peripheral smear. Approximately one-half of children with ALL present with bleeding (including petechiae and purpura) and three-quarters have a platelet count

315
Q

A patient has acute lymphoblastic leukaemia. A bone marrow biopsy will show infiltration by which cells?

A. Magnesium exposure
B. Chronic myeloid leukaemia
C. Lymphocytes
D. Ionising radiation
E. Thalassaemia
F. Acute lymphocytic leukaemia
G. Blast cells
H. Chronic lymphocytic leukaemia
I. Acute myeloid leukaemia
J. Neutrophils
K. Sickle cell disease
L. Marfan’s syndrome
M. Down’s syndrome
A

Blast Cells (>20%)
Acute leukemia, the most common form of cancer in children, comprises approximately 30 percent of all childhood malignancies, with acute lymphoblastic leukemia (ALL) being five times more common than acute myeloid leukemia (AML).
The most common presenting symptoms of ALL are nonspecific (eg, fever, bleeding, bone pain, lymphadenopathy). Bone pain, particularly affecting the long bones, and caused by leukemic involvement of the periosteum, is a presenting symptom in 21 to 38 percent of cases of acute leukaemia.
Lymphadenopathy associated with malignancy usually is non-tender, firm, rubbery, and matted - Approximately 50 percent of children with ALL present with lymphadenopathy.
Most children with ALL have anemia and/or thrombocytopenia with either normal or depressed WBC counts and lymphoblasts on peripheral smear. Approximately one-half of children with ALL present with bleeding (including petechiae and purpura) and three-quarters have a platelet count

316
Q

Patients with this inherited disorder have an increased risk of developing acute leukaemia.

A. Magnesium exposure
B. Chronic myeloid leukaemia
C. Lymphocytes
D. Ionising radiation
E. Thalassaemia
F. Acute lymphocytic leukaemia
G. Blast cells
H. Chronic lymphocytic leukaemia
I. Acute myeloid leukaemia
J. Neutrophils
K. Sickle cell disease
L. Marfan’s syndrome
M. Down’s syndrome
A

Downs Syndrome
Acute lymphoblastic leukemia (ALL) is the most common form of cancer in children. The peak incidence occurs between two and five years of age. Children with certain genetic and immunodeficiency syndromes are at increased risk. These include Down syndrome, Neurofibromatosis type 1, Bloom syndrome, and ataxia telangiectasia.
The most common presenting symptoms of ALL are nonspecific: fever, infection, bleeding, bone pain, or lymphadenopathy. Unexplained persistence of any of these common signs or symptoms should prompt consideration of malignancy as a possible cause

317
Q

An environmental factor associated with acute leukaemia.

A. Magnesium exposure
B. Chronic myeloid leukaemia
C. Lymphocytes
D. Ionising radiation
E. Thalassaemia
F. Acute lymphocytic leukaemia
G. Blast cells
H. Chronic lymphocytic leukaemia
I. Acute myeloid leukaemia
J. Neutrophils
K. Sickle cell disease
L. Marfan’s syndrome
M. Down’s syndrome
A

Ionising Radiation

318
Q

The commonest adult leukaemia.

A. Magnesium exposure
B. Chronic myeloid leukaemia
C. Lymphocytes
D. Ionising radiation
E. Thalassaemia
F. Acute lymphocytic leukaemia
G. Blast cells
H. Chronic lymphocytic leukaemia
I. Acute myeloid leukaemia
J. Neutrophils
K. Sickle cell disease
L. Marfan’s syndrome
M. Down’s syndrome
A

Chronic Lymphocytic Leukaemia is a chronic lymphoproliferative disorder (lymphoid neoplasm) characterized by a progressive accumulation of functionally incompetent lymphocytes, which are usually monoclonal in origin.
CLL is the most common leukemia in Western countries, accounting for approximately 30 percent of all leukemias in the United States. It has a male predominance and is more common in Caucasians. The median age at diagnosis is 70 years. There are no clearly discernible occupational or environmental risk factors.
The vast majority of patients are asymptomatic at diagnosis and only come to the physician’s attention based upon abnormalities found on routine blood counts. Alternatively, some patients may present with painless swelling of lymph nodes, often in the cervical area, which spontaneously wax and wane, but do not altogether disappear. Less common presentations include constitutional “B” symptoms of lymphoma, symptoms related to an acquired immunodeficiency, or autoimmune complications.
Most patients with CLL have a prominent lymphocytosis in the peripheral blood and bone marrow at diagnosis. Neutropenia, anemia, and thrombocytopenia may also be observed at the time of initial diagnosis, and are usually of relatively mild degree.
Immunophenotypic analysis reveals a clonal population (kappa or lambda light chain) of cells that express B cell associated antigens (CD19, CD20, and CD23), the T cell associated antigen CD5, and low levels of surface immunoglobulin

319
Q

A 50yr old man presents to his GP complaining of weight loss, tiredness, easy bruising and a painful big toe. On examination his spleen is massively enlarged. Investigation shows a raised serum urate. The peripheral blood film is abnormal, showing proliferation of which type of cell?

A. Pelger-Huet cells
B. Clonal B lymphocytes
C. Chromosome 9;22 translocation
D. Chromosome 11q23 deletion
E. Neutrophils
F. Eosinophils
G. Blast cells
H. Auer rods
I. Reticulocytes
J. Platelets
K. Spherocytes
A

Neutrophils - CML. Increased mass of turning-over cells generates urate.
Chronic myeloid leukemia (CML) is a myeloproliferative disorder associated with the Philadelphia chromosome t(9;22)(q34;q11) and/or the BCR-ABL fusion gene. This genetic abnormality results in the formation of a unique gene product (BCR-ABL), which results in a constitutively active tyrosine kinase. It is this deregulated tyrosine kinase that is implicated in the development of CML and is the target of current therapies - Imatinib.
The clinical hallmark of CML is the uncontrolled production of mature and maturing granulocytes, predominantly neutrophils, but also basophils and eosinophils.
Decreased efficiency of renal uric acid excretion is responsible for about 85 to 90 percent of primary or secondary hyperuricemia. The remaining 10 to 15 percent of patients with hyperuricemia ostensibly overproduce uric acid. These disorders are often due to inherited defects in regulation of purine nucleotide synthesis, disordered adenosine triphosphate (ATP) metabolism, or disorders resulting in increased rates of cell turnover.

320
Q

A 65yr old lady is seen in the haematology clinic where she has been treated for 7 years with Imantinib for chronic myeloid leukaemia. Having been previously well, she is now complaining of shortness of breath and general weakness. Examination reveals splenomegally. Her peripheral blood film has changed from previous appointments and reflects the progression of her disease. Which type of cell is now proliferating?

A. Pelger-Huet cells
B. Clonal B lymphocytes
C. Chromosome 9;22 translocation
D. Chromosome 11q23 deletion
E. Neutrophils
F. Eosinophils
G. Blast cells
H. Auer rods
I. Reticulocytes
J. Platelets
K. Spherocytes
A

Blast Cells - CML, blast phase. Transformation tends to be into AML but in 20% is lymphoblastic (ALL).
Chronic myeloid leukemia (CML) is a myeloproliferative disorder associated with the Philadelphia chromosome t(9;22)(q34;q11) and/or the BCR-ABL fusion gene. This genetic abnormality results in the formation of a unique gene product (BCR-ABL), which results in a constitutively active tyrosine kinase. It is this deregulated tyrosine kinase that is implicated in the development of CML and is the target of current therapies. CML has historically been a triphasic disease. Approximately 85 to 90 percent of patients present in a chronic stable phase. Without treatment, this inevitably progresses to a more aggressive, accelerated phase, and then culminates in a very difficult to treat blast crisis.
In approximately 30 percent of cases, blast crisis in CML is of the lymphoid (ie, acute lymphoblastic leukemia [ALL]), rather than the myeloid (ie, acute myeloid leukemia [AML]) phenotype. Transformation may be suggested clinically by the development of signs and symptoms more typical of acute leukemia (eg, night sweats, weight loss, fever, bone pain, symptoms of anemia).

321
Q

A 70yr old man complains of a year’s history of fatigue, weight loss and recurrent sinusitis. His white cell count is raised with a lymphocytosis of 283x109 /L. Blood film shows features of haemolysis and Coomb’s test is positive. Further investigation show the bone marrow, blood and lymph nodes are infiltrated with which cell population?

A. Pelger-Huet cells
B. Clonal B lymphocytes
C. Chromosome 9;22 translocation
D. Chromosome 11q23 deletion
E. Neutrophils
F. Eosinophils
G. Blast cells
H. Auer rods
I. Reticulocytes
J. Platelets
K. Spherocytes
A

Clonal B lymphocytes: CLL.
On immunophenotyping only two chronic B cell leukaemia/lymphomas are CD5+(an antigen commonly expressed by T cells): CLL (CD5+ CD23+) and Mantle Cell Lymphoma (CD5+ CD23-). CLL may be assoc. with Coombs positive AIHA and ITP. The combination is called Evans syndrome.
CLL is the most common leukemia in adults in Western countries and is considered to be mainly a disease of older adults (median age at diagnosis of 70 years). Most patients feel entirely well with no symptoms when a routine blood count reveals an absolute lymphocytosis, leading to a diagnosis of CLL. The most common abnormal finding on physical examination of the patient with CLL is lymphadenopathy, the spleen is the second most frequently enlarged lymphoid organ. The peripheral blood smear of patients with CLL demonstrates a lymphocytosis. The leukemic cells are typically small, mature appearing lymphocytes with a dense nucleus, partially aggregated (clumped) chromatin, and without discernible nucleoli. Bone marrow aspirate and biopsy are not required for the diagnosis of CLL. If bone marrow biopsy and aspiration are performed at the time of initial diagnosis, they usually demonstrate normal to increased cellularity, with lymphocytes accounting for >30 percent of all nucleated cells
Patients with CLL have an increased incidence of autoimmune hemolytic anemia (AIHA)

322
Q

A routine full blood count on a 62yr old gardener reveals a high white cell count of 154x109 /L, and the differential shows this to be a neutrophilia. The haemoglobin and platelet count are normal. Biopsy shows a hypercellular “packed” bone marrow, and cells show the presence of which chromosomal abnormality?

A. Pelger-Huet cells
B. Clonal B lymphocytes
C. Chromosome 9;22 translocation
D. Chromosome 11q23 deletion
E. Neutrophils
F. Eosinophils
G. Blast cells
H. Auer rods
I. Reticulocytes
J. Platelets
K. Spherocytes
A

Chromosome 9;22 translocation
CML is a myeloproliferative neoplasm characterized by the dysregulated production and uncontrolled proliferation of mature and maturing granulocytes with fairly normal differentiation.
CML is associated with the fusion of two genes: BCR (on chromosome 22) and ABL1 (on chromosome 9) resulting in the BCR-ABL1 fusion gene. This abnormal fusion typically results from a reciprocal translocation between chromosomes 9 and 22, t(9;22)(q34;q11), that gives rise to an abnormal chromosome 22 called the Philadelphia (Ph) chromosome. It is this deregulated tyrosine kinase that is implicated in the pathogenesis of CML.
The clinical hallmark of CML is the uncontrolled production of mature and maturing granulocytes, predominantly neutrophils, but also basophils and eosinophils. In the absence of treatment, CML has a triphasic or biphasic clinical course as it progresses from a chronic phase to an accelerated phase and on to a terminal blast crisis.
The peripheral smear typically demonstrates a leukocytosis. Bone marrow aspiration and biopsy demonstrates granulocytic hyperplasia with a maturation pattern that reflects that seen in the peripheral smear
Treatment: tyrosine kinase inhibitor (imatinib)

323
Q

The following are contraindications to renal transplantation: True/False

  • Aged above 65 years
  • Active Sepsis
  • HIV positive
  • BMI > 30
A
  • Aged above 65 years: FALSE - must have a predicted survival
  • BMI>30: FALSE - obesity is only a relative contraindication
    Contraindications are
    Active malignancy is a contraindication for transplantation because immunosuppressive therapy may aggravate underlying malignancy
    Active HIV disease is a contraindication for transplantation
324
Q

A 40 year old female presents with a rash and AKI is diagnosed. What is the most likely cause of her renal failure from the following list?

NSAIDS
Systemic Vasculitis
Amyloidosis
Tumour Lysis Syndrome following chemotherapy for lymphoma
Myeloma
A

Systemic Vasculitis
AKI - abrupt or rapid decline in renal filtration function. This condition is usually marked by a rise in serum creatinine concentration
Can be divided into 3 categories:
- Prerenal - As an adaptive response to severe volume depletion and hypotension, with structurally intact nephrons
- Intrinsic - In response to cytotoxic, ischemic, or inflammatory insults to the kidney, with structural and functional damage
- Post-renal - From obstruction to the passage of urine
Systemic vasculitis includes all the large (Takayasu, giant cell), medium (Polyarteritis nodosa, Kawasaki disease) and small (Wegener’s granulomatosis, Churg-Strauss, microscopic polyarteritis, Henoch Schonlein purpura) vessel vasculitides

325
Q

Regarding Hyperkalaemia, which of the following is true?

A. ECG changes are peaked p waves and flattened t waves
B. In those with CKD, dietary intake is a major cause and high potassium levels are found in foods such as milk, chocolate, dried fruits and tomatoes.
C. NSAIDs can lower potassium levels
D. Hyperaldosteronism is a common cause
E. All of the above

A

TRUE - In those with CKD, dietary intake is a major cause and high potassium levels are found in foods such as milk, chocolate, dried fruits and tomatoes.

ECG changes are tall tented t waves, loss of p waves and a broad QRS

Hypoaldosteronism is a cause

326
Q

You patient with CKD has been started on an ESA but does not respond. What could be the cause?

Iron deficiency
TB
Malignancy
B12 and folate deficiency
Hyper-parathyroidism
Any of the above
A

Any of the above
Anemia has been implicated as a contributing factor in many of the symptoms associated with reduced kidney function. The partial correction of anemia in chronic kidney disease (CKD) patients with end-stage renal disease (ESRD) and CKD improves physiologic and clinical parameters and quality of life, compared with the severely low hemoglobin (Hgb) levels that were common prior to the availability of erythropoiesis-stimulating agents (ESAs)

327
Q

A 68 year old man with previously normal renal function is found to have a creatinine of 624umol/L. Renal ultrasound shows hydro-nephrosis in both kidneys. What is the likely cause of his AKI?

Right sided kidney stone
Left ureteric transitional cell carcinoma
Membranous glomerulonephropathy
Benign prostatic hypertrophy
Amyloid
A

Benign prostatic hypertrophy: something that is causing compression of both sides!

328
Q

Why does each drug predispose to patients to developing pre-renal AKI?

NSAIDs
Calcineurin inhibitors
ACEI/ARBs

A

NSAIDs - decrease afferent arteriolar dilatation
Calcineurin inhibitors - decrease afferent arteriolar dilatation
ACEI/ARBs - decreases efferent arteriolar constriction
Diuretics

329
Q

Which is the following is true regarding urine dipstick testing?

A. If the dipstick is negative for blood it reliably excludes haematuria
B. Haematuria is the only cause of a positive dipstick test for blood
C. You can reliably exclude bacteriuria if the urine dipstick is negative for nitrites
D. The urine dipstick detects Bence Jones proteins
E. Glycosuria detected by the dipstick means the patient has diabetes

A

If the dipstick is negative for blood it reliably excludes haematuria

Not all bacteria will convert nitrates in the urine to nitrites - so even if this test is negative bacteria may still be present

Bence Jones proteins are not detectable on dipstick but will show as a monoclonal band on electrophoresis

330
Q

Which factor limits the use of a serum creatinine as a marker of GFR?

A. It is influenced by intake of fat
B. It is lower in the black population
C. It is reabsorbed by the renal tubules 
D. It is related to muscle mass
E. All of the above
A

It is related to muscle mass

  • It is influenced by the intake of protein, not fat
  • Tends to be higher in the black population (associated with a higher muscle mass in these patients)
331
Q

Which has the lowest calcium?

A. Primary Hyperparathyroidism
B. Secondary Hyperparathyroidism
C. Osteoporosis
D. Paget's disease of the bone
E. Breast Cancer
A

Secondary Hyperparathyroidism - increased PTH production in response to hypocalcaemia

Primary Hyperparathyroidism - excess PTH production by parathyroids - usually an adenoma

Osteoporosis doesn’t alter blood calcium levels.

Paget’s disease of the bone = increased calcium and ALP

332
Q

A 65-year-old chronic alcoholic presents to the A&E Department with a minor head injury. On examination he is found to be pale. Blood tests show a high MCV. What is the likeliest result of MCV (fl) in a normal person?

A. 15  
B. 2.2   
C.130 
D. 30 
E. 4 
F. 290 
G. 90
A

90: The normal range (95 percent confidence) for the MCV in adults is 80 to 96 fL.
By definition, microcytosis is taken to mean the presence of red blood cells (RBCs) with a mean corpuscular volume (MCV) less than normal, while macrocytosis means the presence of RBCs with an MCV greater than normal. Since the MCV is an average value, a sample with an MCV in the normal range may still have a significant population of small and/or large red cells. This situation is best assessed by direct examination of the peripheral smear, although a clue to the presence of cells of varying sizes (ie, anisocytosis) can be obtained from a determination of the RBC distribution width.

CAUSES OF NORMOCYTIC ANEMIA — By definition, the mean red blood cell (RBC) volume is normal (MCV between 80 and 96 fL) in patients with normocytic anemia. This is an extremely large and amorphous category, which can be narrowed somewhat by examination of the blood smear to determine if there is a subpopulation of RBCs with distinctive size or shape abnormalities, which would place the patient in one of the categories listed below (ie, early microcytic or early macrocytic anemia), and by use of a kinetic approach to determine the mechanism(s) underlying the anemia.

CAUSES OF MACROCYTOSIS — Macrocytic anemias are characterized by a mean corpuscular volume (MCV) above 100 fL

Depending upon the population studied, the most common causes of an increased MCV are:
●The presence of an increased number of reticulocytes
●Macrocytosis associated with alcoholism
●Deficiency of folate or vitamin B12
●Antiviral treatment of HIV infection
●Use of chemotherapeutic agents, especially hydroxyurea
●Presence of one of the myelodysplastic disorders

333
Q

A 40-year-old woman presents with a two month history of tiredness, intermittent pyrexia and abdominal pain. On examination she has an enlarged palpable spleen. Blood tests show anaemia with a raised white cell count. What is the likeliest result of a white cell count (x 109 per l) in a normal person?

A. 15  
B. 2.2   
C.130 
D. 30 
E. 4 
F. 290 
G. 90
A

4: The normal white cell count is usually between 4 and 11 x 109/L.

Neutrophils are the most abundant white blood cell, constituting 60-70% of the circulating leukocytes. They defend against bacterial or fungal infection. They are usually first responders to microbial infection; their activity and death in large numbers forms pus. They are commonly referred to as polymorphonuclear (PMN) leukocytes, although, in the technical sense, PMN refers to all granulocytes. They have a multi-lobed nucleus, which consists of three to five lobes connected by slender strands.The cytoplasm may look transparent because of fine granules that are pale lilac when stained. Neutrophils are active in phagocytosing bacteria and are present in large amount in the pus of wounds. These cells are not able to renew their lysosomes (used in digesting microbes) and die after having phagocytosed a few pathogens. Neutrophils are the most common cell type seen in the early stages of acute inflammation.

Eosinophils compose about 2-4% of the WBC total. This count fluctuates throughout the day, seasonally, and during menstruation. It rises in response to allergies, parasitic infections, collagen diseases, and disease of the spleen and central nervous system. They are rare in the blood, but numerous in the mucous membranes of the respiratory, digestive, and lower urinary tracts.

They primarily deal with parasitic infections. Eosinophils are also the predominant inflammatory cells in allergic reactions. The most important causes of eosinophilia include allergies such as asthma, hay fever, and hives; and also parasitic infections. They secrete chemicals that destroy these large parasites, such as hook worms and tapeworms, that are too big for any one WBC to phagocytize. In general, their nucleus is bi-lobed. The lobes are connected by a thin strand. The cytoplasm is full of granules that assume a characteristic pink-orange color with eosin staining.

Basophils are chiefly responsible for allergic and antigen response by releasing the chemical histamine causing the dilation of blood vessels. Because they are the rarest of the white blood cells (less than 0.5% of the total count) and share physicochemical properties with other blood cells, they are difficult to study.[11] They can be recognized by several coarse, dark violet granules, giving them a blue hue. The nucleus is bi- or tri-lobed, but it is hard to see because of the number of coarse granules that hide it.

They excrete two chemicals that aid in the body’s defenses: histamine and heparin. Histamine is responsible for widening blood vessels and increasing the flow of blood to injured tissue. It also makes blood vessels more permeable so neutrophils and clotting proteins can get into connective tissue more easily. Heparin is an anticoagulant that inhibits blood clotting and promotes the movement of white blood cells into an area. Basophils can also release chemical signals that attract eosinophils and neutrophils to an infection site

334
Q

A 5-year-old boy presents with a purpuric rash and petechiae following a recent viral infection. Blood tests showed thrombocytopenia. What is the likeliest result of a platelet count (x109/l) in a normal adult?

A. 15  
B. 2.2   
C.130 
D. 30 
E. 4 
F. 290 
G. 90
A

290

Thrombocytopenia (ie, platelet count

335
Q

A 35-year-old man presents with hypertension. Blood tests show normal sodium, urea and glucose and a raised potassium. What is the likeliest result of potassium (mmol/l) in a normal person?

A. 15  
B. 2.2   
C.130 
D. 30 
E. 4 
F. 290 
G. 90
A

4
Hyperkalemia is most often due to impaired urinary potassium excretion due to acute or chronic kidney disease and/or disorders or drugs that inhibit the renin-angiotensin-aldosterone axis. Less commonly, redistributive hyperkalemia results from the movement of potassium out of the cells, even though the total body potassium may be reduced. The main cause of redistributive hyperkalemia is uncontrolled hyperglycemia.

  • Mild - 5.5-5.9 mmol/L
  • Moderate - 6.0-6.4 mmol/L
  • Severe - >6.5 mmol/L

The most serious manifestations of hyperkalemia are muscle weakness or paralysis, cardiac conduction abnormalities, and cardiac arrhythmias. These manifestations usually occur when the serum potassium concentration is ≥7.0 meq/L with chronic hyperkalemia or possibly at lower levels with an acute rise in serum potassium. Patients with skeletal muscle or cardiac manifestations typically have one or more of the characteristic ECG abnormalities associated with hyperkalemia (tall tented T-waves, shortened QT–> lengthening PR interval & QRS duration, P-wave may disppear & ultimately QRS widens to a sine wave pattern –> flat line).

Stop further potassium accumulation:
•Stop any potassium supplements or drugs that conserve potassium.
•Consider stopping digoxin and beta-blockers, as these may prevent buffering of intracellular potassium and reduce effectiveness of insulin-glucose.
•Decrease high intake of potassium in the diet.

Protect cardiac membrane:•
Give 10 ml 10% calcium gluconate (calcium chloride is an alternative ideally given via central access) which will improve ECG changes within 1-3 minutes, but this effect only has a transient effect of 30-60 minutes.
•If there is no improvement then give 10 ml every 10 minutes until ECG normalises (may need up to 50 ml).
•In patients who are taking digoxin, give calcium gluconate in an infusion (add to 100 ml glucose 5%) and run over 20 minutes (otherwise, can precipitate myocardial digoxin toxicity).

Shift potassium into cells:
•Insulin-glucose IV - usually 10 units of Actrapid® are added to 50 ml of glucose 50% and infused over 30 minutes.
•Capillary blood glucose needs to be checked before, during and after.
•Potassium will decrease (0.6-1.0 mmol/L) in 15 minutes and the reduction lasts for 60 minutes.
•Check potassium 30 minutes afterwards and if there is a good response check U&E 1-2 hours later.
•Also give 10-20 mg nebulised salbutamol - this reduces potassium (0.5-1.0 mmol/L) in 15-30 minutes and lasts for two hours.

Remove potassium from the body:
•Calcium polystyrene sulfonate resin (Calcium Resonium®) with regular lactulose will remove potassium via the gastrointestinal tract.
•Each gram removes 1 mmol/L of potassium but onset is slow, taking over two hours.
•Haemodialysis will also remove potassium from the body

336
Q

A 70-year-old woman presents in a coma with a long history of polyuria and polydipsia. Investigations show that her plasma osmolarity is raised. What is the likeliest result of plasma osmolarity (mmol/l) in a normal person?

A. 15  
B. 2.2   
C.130 
D. 30 
E. 4 
F. 290 
G. 90
A

290
Normal human reference range of osmolality in plasma is about 285-295 milli-osmoles per kilogram

Calculated osmolarity = 2 Na + Glucose + Urea ( all in mmol/L).
Osmolality of blood increases with dehydration and decreases with overhydration. In normal people, increased osmolarity in the blood will stimulate secretion of antidiuretic hormone (ADH). This will result in increased water reabsorption, more concentrated urine, and less concentrated blood plasma. A low serum osmolality will suppress the release of ADH, resulting in decreased water reabsorption and more concentrated plasma.

337
Q

A 14-year-old boy presents with symptoms of chronic liver failure. LFTs display abnormally high levels of transaminases with normal alk phos & bilirubin levels. There’s marked accumulation of copper-associated protein in hepatocytes obtained from a biopsy. His serum copper levels and caeruloplasmin are abnormally low.

A. Chronic hepatitis B 
B. Chronic hepatitis C 
C.Crigler Najjar syndrome 
D. Budd-Chiari syndrome 
E. Hepatitis A 
F. Primary hepatocellular carcinoma 
G. Primary biliary cirrhosis 
H. Wilson's disease
A
Wilson disease (hepatolenticular degeneration) is due to a genetic abnormality inherited in an autosomal recessive manner that leads to impairment of cellular copper transport. It is found worldwide, with a prevalence of approximately 1 case in 30,000 live births in most populations. Impaired biliary copper excretion leads to accumulation of copper in several organs, most notably the liver, brain, and cornea. Over time, the liver is progressively damaged and eventually becomes cirrhotic. A small percent of patients develop acute liver failure, most often in the setting of advanced fibrosis of the liver. In addition, patients may develop neurologic complications, which can be severe.
The clinical manifestations of Wilson disease are predominantly hepatic, neurologic, and psychiatric, with many patients having a combination of symptoms [1]. Hemolysis is also a common finding in patients with acute liver failure due to Wilson disease.

Signs and symptoms of hepatic Wilson disease may include:

●Kayser-Fleischer rings, visible in 50 percent of patients with hepatic disease (seen with all forms of liver involvement)
●Asymptomatic (steatosis, chronic hepatitis, compensated cirrhosis)
●Abdominal pain (acute hepatitis, acute liver failure)
●Jaundice (acute hepatitis, acute liver failure, cirrhosis)
●Hepatomegaly (acute and chronic hepatitis, acute liver failure)
●Splenomegaly (cirrhosis)
●Ascites (cirrhosis)
●Upper gastrointestinal bleeding (cirrhosis with varices or portal hypertensive gastropathy)
●Peripheral stigmata of chronic liver disease (cirrhosis)
●Mental status changes due to hepatic encephalopathy (acute liver failure, cirrhosis)

338
Q

A 30-year-old Thai male presents to a day surgery unit for a cholecystectomy. His LFTs reveal very elevated transaminases with normal bilirubin & alk phos levels. Microscopy of a liver biopsy identifies antigens from a dsDNA virus in the cytosol of hepatocytes.

A. Chronic hepatitis B 
B. Chronic hepatitis C 
C.Crigler Najjar syndrome 
D. Budd-Chiari syndrome 
E. Hepatitis A 
F. Primary hepatocellular carcinoma 
G. Primary biliary cirrhosis 
H. Wilson's disease
A

Chronic hepatitis B
Many patients with chronic hepatitis B are asymptomatic (unless they progress to decompensated cirrhosis or have extrahepatic manifestations), while others have nonspecific symptoms such as fatigue. Some patients experience exacerbations of the infection which may be asymptomatic, mimic acute hepatitis, or manifest as hepatic failure.

Physical examination may be normal, or there may be stigmata of chronic liver disease. Jaundice, splenomegaly, ascites, peripheral edema, and encephalopathy may be present in patients with decompensated cirrhosis. Laboratory tests may be normal, but most patients have a mild to moderate elevation in serum AST and ALT. During exacerbations, the serum ALT concentration may be as high as 50 times the upper limit of normal, and alfa-fetoprotein (AFP) concentrations as high as 1000 ng/mL may be seen [14]. A progression to cirrhosis is suspected when there is evidence of hypersplenism (decreased white blood cell and platelet counts) or impaired hepatic synthetic function (hypoalbuminemia, prolonged prothrombin time, hyperbilirubinemia).
This type of infection dramatically increases the incidence of hepatocellular carcinoma (liver cancer). Across Europe hepatitis B and C cause approximately 50% of hepatocellular carcinomas. A persistent elevation of serum ALT for more than six months indicates a progression to chronic hepatitis.

Chronic carriers are encouraged to avoid consuming alcohol as it increases their risk for cirrhosis and liver cancer. Hepatitis B virus has been linked to the development of membranous glomerulonephritis (MGN).

Hepatitis D (HDV) can occur only with a concomitant hepatitis B infection, because HDV uses the HBV surface antigen to form a capsid. Co-infection with hepatitis D increases the risk of liver cirrhosis and liver cancer.

339
Q

A 58-year-old woman presents with recent onset of Jaundice. LFTs reveal increased bilirubin & markedly elevated alk phos & normal transaminases. Further investigations uncovered raised IgM and serum cholesterol. Anti mitochondrial antibodies are also detected. A liver biopsy shows enlargement of the portal tracts by white blood cells and granulomas. Bile ducts are also less than normal.

A. Chronic hepatitis B 
B. Chronic hepatitis C 
C.Crigler Najjar syndrome 
D. Budd-Chiari syndrome 
E. Hepatitis A 
F. Primary hepatocellular carcinoma 
G. Primary biliary cirrhosis 
H. Wilson's disease
A

Primary biliary cirrhosis
Characterized by a T-lymphocyte-mediated attack on small intralobular bile ducts. A continuous assault on the bile duct epithelial cells leads to their gradual destruction and eventual disappearance. The sustained loss of intralobular bile ducts causes the signs and symptoms of cholestasis and eventually results in cirrhosis and liver failure The diagnosis should be considered in the patient, particularly a woman, who complains of unexplained itching, fatigue, jaundice, or unexplained weight loss with right-upper-quadrant discomfort, and/or whose serum alkaline phosphatase is unaccountably elevated. Such patients should be questioned about symptoms of diseases frequently associated with PBC, such as Sjögren’s syndrome (dry eyes and mouth), arthritis, and Raynaud phenomenon.
Primary biliary cirrhosis is probably present if the serum alkaline phosphatase and IgM concentrations are both elevated and the antimitochondrial antibody test is positive.

340
Q

A 48-year-old male returning from a 6mths round the world trip presents with a recent Hx of nausea, anorexia & distaste for cigarettes. He developed jaundice; his urine became dark and his stools pale. His spleen was palpable. Investigations showed bilirubinuria, increased urinary urobilinogen & a raised serum AST & ALT. Within 4 weeks his symptoms had completely subsided.

A. Chronic hepatitis B 
B. Chronic hepatitis C 
C.Crigler Najjar syndrome 
D. Budd-Chiari syndrome 
E. Hepatitis A 
F. Primary hepatocellular carcinoma 
G. Primary biliary cirrhosis 
H. Wilson's disease
A

Hepatitis A: 27 nm, nonenveloped, icosahedral, positive-stranded RNA virus classified in the Heparnavirus genus of the Picornaviridae family.
HAV is spread via the fecal-oral route and is more prevalent in low socioeconomic areas where a lack of adequate sanitation and poor hygienic practices facilitate spread of the infection.
Hepatitis A virus (HAV) infection usually results in an acute, self-limited illness and only rarely leads to fulminant hepatic failure. Fulminant hepatic failure occurs more commonly in patients with underlying liver disease, particularly chronic hepatitis C virus (HCV) infection.
The incubation period averages 30 days (range 15 to 49 days), after which the illness begins with the abrupt onset of prodromal symptoms including fatigue, malaise, nausea, vomiting, anorexia, fever, and right upper quadrant pain. Within a few days to one week, patients note dark urine, acholic stool (light-colored stools lacking bilirubin pigment), jaundice, and pruritus. The prodromal symptoms usually diminish when jaundice appears; jaundice typically peaks within two weeks.
The diagnosis of acute hepatitis A virus (HAV) infection is made by the detection of anti-HAV antibodies in a patient with the typical clinical presentation. Serum IgM anti-HAV is the gold standard for the detection of acute illness.
Because the disease is usually self-limited, the treatment is supportive.

341
Q

A 55-year-old woman presents with a short Hx of nausea and abdominal pain; tender hepatomegaly and ascities. LFTs show mildly raised transaminases, bilirubin and normal alk phos. The woman also had polycythaemia rubra vera. Liver biopsy suggests venous outflow obstruction.

A. Chronic hepatitis B 
B. Chronic hepatitis C 
C.Crigler Najjar syndrome 
D. Budd-Chiari syndrome 
E. Hepatitis A 
F. Primary hepatocellular carcinoma 
G. Primary biliary cirrhosis 
H. Wilson's disease
A

Budd-Chiari syndrome: hepatic venous outflow tract obstruction, independent of the level or mechanism of obstruction, provided the obstruction is not due to cardiac disease, pericardial disease, or sinusoidal obstruction syndrome (veno-occlusive disease). Primary Budd-Chiari syndrome is present when there is obstruction due to a primarily venous process (thrombosis or phlebitis), whereas secondary Budd-Chiari is present when there is compression or invasion of the hepatic veins and/or the inferior vena cava by a lesion that originates outside of the vein.
Symptoms in patients with Budd-Chiari syndrome may include fever, abdominal pain, abdominal distension (from ascites), lower extremity edema, jaundice, gastrointestinal bleeding, and/or hepatic encephalopathy. Patients with subacute or chronic Budd-Chiari syndrome may be asymptomatic. In such patients, the hepatic venous outflow obstruction is often discovered as part of the evaluation of abnormal liver blood tests or when imaging is obtained for other reasons.

342
Q

A liver enzyme raised after a myocardial infarction

A. Aspartate transaminase
B. Gamma globulin
C. Alanine transaminase
D. Direct bilirubin
E. Activated partial thromboplastin time
F. Gamma glutamyl transpeptidase
G. Alkaline phosphatase
H. Total bilirubin
I. Prothrombin time
J. Albumin
A

Aspartate transaminase
Large increases in mitochondrial AST occur in serum after extensive tissue necrosis and assay of mitochondrial AST has been advocated as an accurate test for the detection of myocardial infarction [10]. However, other serum tests, such as troponins, are considered the standard for the diagnosis of myocardial infarction.

343
Q

A test of the integrity of the extrinsic pathway

A. Aspartate transaminase
B. Gamma globulin
C. Alanine transaminase
D. Direct bilirubin
E. Activated partial thromboplastin time
F. Gamma glutamyl transpeptidase
G. Alkaline phosphatase
H. Total bilirubin
I. Prothrombin time
J. Albumin
A

Prothrombin time

The intrinsic pathway is initiated by the activation of the ‘contact factor’ of plasma and can be measured by the aPTT test (heparin - antithrombin III & factor 10 if unfract). The extrinsic pathway is initiated by the release of tissue factor and can be measured by the PT test (warfarin - vitamin K - F2,7,9,10).

344
Q

An enzyme markedly raised in obstructive jaundice along with direct bilirubin

A. Aspartate transaminase
B. Gamma globulin
C. Alanine transaminase
D. Direct bilirubin
E. Activated partial thromboplastin time
F. Gamma glutamyl transpeptidase
G. Alkaline phosphatase
H. Total bilirubin
I. Prothrombin time
J. Albumin
A

Alkaline phosphatase
Cholestasis may develop in the setting of extrahepatic or intrahepatic biliary obstruction (table 6). In patients with cholestasis, the alkaline phosphatase is typically elevated to at least four times the upper limit of normal. The magnitude of the serum alkaline phosphatase level does not distinguish extrahepatic cholestasis from intrahepatic cholestasis. Lesser degrees of elevation are nonspecific and may be seen in many other types of liver disease, such as viral hepatitis, infiltrative diseases of the liver, and congestive hepatopathy. The gamma-glutamyl transpeptidase (GGT) may also be elevated in the setting of cholestasis.
To confirm an isolated elevation in the alkaline phosphatase is coming from the liver, a GGT level or serum 5’-nucleotidase level should be obtained. These tests are usually elevated in parallel with the alkaline phosphatase in liver disorders, but are not increased in bone disorders. An elevated serum alkaline phosphatase with a normal GGT or 5’-nucleotidase should prompt an evaluation for bone diseases.

345
Q

Raised in alcohol abuse

A. Aspartate transaminase
B. Gamma globulin
C. Alanine transaminase
D. Direct bilirubin
E. Activated partial thromboplastin time
F. Gamma glutamyl transpeptidase
G. Alkaline phosphatase
H. Total bilirubin
I. Prothrombin time
J. Albumin
A

Gamma-glutamyl transpeptidase (GGT) catalyzes the transfer of the gamma-glutamyl group from gamma-glutamyl peptides such as glutathione to other peptides and to L-amino acids. GGT is present in cell membranes in many tissues, including the kidneys, pancreas, liver, spleen, heart, brain, and seminal vesicles [50]. It is thought to play a role in amino acid transport.
The normal range is 0 to 30 IU/L (0 to 0.5 mkat/L)
An isolated elevation in serum GGT or a GGT elevation out of proportion to that of other enzymes (such as the alkaline phosphatase and alanine aminotransferase) may be an indicator of alcohol abuse or alcoholic liver disease

346
Q

Levels can be affected by diet

A. Aspartate transaminase
B. Gamma globulin
C. Alanine transaminase
D. Direct bilirubin
E. Activated partial thromboplastin time
F. Gamma glutamyl transpeptidase
G. Alkaline phosphatase
H. Total bilirubin
I. Prothrombin time
J. Albumin
A

Albumin: is the most abundant protein in human blood plasma. It is produced in the liver. Albumin constitutes about half of the blood serum protein. It is soluble and monomeric.

Albumin transports hormones, fatty acids, and other compounds, buffers pH, and maintains osmotic pressure, among other functions.

Albumin is synthesized in the liver as preproalbumin, which has an N-terminal peptide that is removed before the nascent protein is released from the rough endoplasmic reticulum. The product, proalbumin, is in turn cleaved in the Golgi vesicles to produce the secreted albumin.

The reference range for albumin concentrations in serum is approximately 35 - 50 g/L. It has a serum half-life of approximately 20 days.

Maintains oncotic pressure
Transports thyroid hormones
Transports other hormones, in particular, ones that are fat-soluble
Transports fatty acids (“free” fatty acids) to the liver and to myocytes for utilization of energy
Transports unconjugated bilirubin
Transports many drugs; serum albumin levels can affect the half-life of drugs
Competitively binds calcium ions (Ca2+)
Buffers pH
Serum albumin, as a negative acute-phase protein, is down-regulated in inflammatory states. As such, it is not a valid marker of nutritional status; rather, it is a marker of an inflammatory state
Prevents photodegradation of folic acid

Low blood albumin levels (hypoalbuminemia) can be caused by: This condition is a sign of chronic dehydration:
Liver disease; cirrhosis of the liver is most common
Excess excretion by the kidneys (as in nephrotic syndrome)
Excess loss in bowel (protein-losing enteropathy, e.g., Ménétrier’s disease)
Burns (plasma loss in the absence of skin barrier)
Redistribution (hemodilution [as in pregnancy], increased vascular permeability or decreased lymphatic clearance)
Acute disease states (referred to as a negative acute-phase protein)
Malnutrition and wasting[3]
Mutation causing analbuminemia (very rare)

347
Q

A 26-year-old receptionist presents to her GP with a history steatorrhoea, abdominal pain and weight loss, as well as feeling tired all the time. Initial blood tests reveal a microcytic anaemia.

A. Ham's test
B. p-ANCA
C. ANA
D. Anti-acetylcholine receptor antibody
E. Anti-gastric parietal cell antibodies
F. c-ANCA
G. Anti-DsDNA
H. Anti-endomysial antibodies
I. Anti-GAD
J. Anti-smooth muscle antibody
K. Osmotic fragility test
L. Anti-scl70
M. Anti-mitochondrial antibody
A

Anti-endomysial antibodies or Tissue-transglutaminase antibodies - coeliac disease: close association with the HLA-DQ2 and/or DQ8 gene loci

ELISA for IgA antibodies to gliadin and the immunofluorescence test for IgA antibodies to endomysium, a structure of the smooth muscle connective tissue, is virtually pathognomonic for celiac disease

The classic definition of celiac disease or gluten-sensitive enteropathy includes the following three features: villous atrophy; symptoms of malabsorption such as steatorrhea, weight loss, or other signs of nutrient or vitamin deficiency [7]; and resolution of the mucosal lesions and symptoms upon withdrawal of gluten-containing foods, usually within a few weeks to months. Patients with classic disease present with diarrhea, weight loss, or malabsorption, and possess antibodies against gliadin and especially tissue transglutaminase.

The histologic severity ranges from a mild alteration characterized by increased intraepithelial lymphocytes (type 0 lesion) to a flat mucosa with total mucosal atrophy, complete loss of villi, enhanced epithelial apoptosis and crypt hyperplasia (type 3 lesion)

Patients may present with classic signs, including diarrhea with bulky, foul-smelling, floating stools due to steatorrhea and flatulence. These symptoms are paralleled by the consequences of malabsorption, such as growth failure in children, weight loss, severe anemia, neurologic disorders from deficiencies of B vitamins, and osteopenia from deficiency of vitamin D and calcium.

Celiac disease is frequently associated with dermatitis herpetiformis, Down syndrome, selective IgA deficiency, and other conditions which have autoimmune features such as type 1 diabetes mellitus, thyroid disease, and liver disease. Patients with celiac disease (and their families) may also be more likely to have atopic dermatitis

348
Q

A 60-year-old woman with hypothyroidism presents with progressive dyspnoea and tiredness. FBC reveals macrocytic anaemia.

A. Ham's test
B. p-ANCA
C. ANA
D. Anti-acetylcholine receptor antibody
E. Anti-gastric parietal cell antibodies
F. c-ANCA
G. Anti-DsDNA
H. Anti-endomysial antibodies
I. Anti-GAD
J. Anti-smooth muscle antibody
K. Osmotic fragility test
L. Anti-scl70
M. Anti-mitochondrial antibody
A
Anti-gastric parietal cell antibodies
Pernicious anemia (PA; B12 deficiency caused by lack of intrinsic factor) is a common cause of Cbl deficiency. It is usually under-diagnosed, and is not an uncommon problem in older adult subjects. 
The underlying pathogenesis of PA is thought to be autoimmune and may be part of a more general autoimmune disorder called polyglandular autoimmune syndrome type 2 (PAS 2), which can include such disorders as autoimmune thyroid disease, Addison's disease, type 1 diabetes mellitus, and vitiligo.
Autoantibody formation against intrinsic factor – Cbl deficiency in PA is thought to result directly from an autoimmune attack on gastric intrinsic factor (IF) [1,6]. Anti-intrinsic factor antibodies are detectable in the serum in 50 to 70 percent of patients with PA, providing a highly specific (>95 percent) but relatively insensitive (50 to 84 percent) test [1,11]. There are two types of anti-IF antibodies: one that blocks the attachment of Cbl to IF; and one that blocks attachment of the Cbl-IF complex to ileal receptors [12]. The net result of these anti-IF antibodies is to prevent absorption of dietary Cbl, leading to Cbl deficiency and its various sequelae.
The chronic atrophic gastritis in PA is associated with an increased risk of intestinal-type gastric cancer and of gastric carcinoid tumors. 
Gastrectomy and gastritis are other gastric abnormalities that can produce Cbl deficiency
349
Q

A 40-year-old plumber presents to his GP with a history of wheezing and lethargy, along with recurrent nose bleeds. On examination he has crackles in his upper left lung field. Urine dipstick is positive for blood and protein.

A. Ham's test
B. p-ANCA
C. ANA
D. Anti-acetylcholine receptor antibody
E. Anti-gastric parietal cell antibodies
F. c-ANCA
G. Anti-DsDNA
H. Anti-endomysial antibodies
I. Anti-GAD
J. Anti-smooth muscle antibody
K. Osmotic fragility test
L. Anti-scl70
M. Anti-mitochondrial antibody
A

Wegener’s : Antibody to Proteinase-3 : 3 is the 3rd letter of the alphabet; c-ANCA.
Granulomatosis with polyangiitis - GPA.
Antineutrophil cytoplasmic antibody (ANCA)-associated vasculitides include granulomatosis with polyangiitis (GPA), microscopic polyangiitis (MPA), the Churg-Strauss syndrome (CSS) and renal-limited vasculitis.
MPA is distinguished from GPA and CSS by the absence of granuloma formation and the presence of a necrotizing vasculitis.
systemic disorder that involves both granulomatosis and polyangiitis. It is a form of vasculitis (inflammation of blood vessels) that affects small- and medium-size vessels in many organs. Damage to the lungs and kidneys can be fatal. It requires long-term immunosuppression.
Kidney: rapidly progressive glomerulonephritis (75%), leading to chronic kidney failure
Upper airway, eye and ear disease:
Nose: pain, stuffiness, nosebleeds, rhinitis, crusting, saddle-nose deformity due to a perforated septum
Ears: conductive hearing loss due to auditory tube dysfunction, sensorineural hearing loss (unclear mechanism)
Oral cavity: strawberry gingivitis, underlying bone destruction with loosening of teeth, non-specific ulcerations throughout oral mucosa
Eyes: pseudotumours, scleritis, conjunctivitis, uveitis, episcleritis
Trachea: subglottal stenosis
Lungs: pulmonary nodules (referred to as “coin lesions”), infiltrates (often interpreted as pneumonia), cavitary lesions, pulmonary haemorrhage causing haemoptysis, and rarely bronchial stenosis.
Arthritis: Pain or swelling (60%), often initially diagnosed as rheumatoid arthritis
Skin: nodules on the elbow, purpura, various others

The standard treatment for GPA is cyclophosphamide and high dose corticosteroids for remission induction and less toxic immunosuppressants like azathioprine, leflunomide, methotrexate or mycophenolate mofetil.

350
Q

A 30-year-old market trader presents with tiredness and jaundice, and further history reveals he suffered from a chest infection one week previously. On examination mild splenomegaly is noted, and blood tests show reticulocytosis, hyperbilirubinaemia, and spherocytosis.

A. Ham's test
B. p-ANCA
C. ANA
D. Anti-acetylcholine receptor antibody
E. Anti-gastric parietal cell antibodies
F. c-ANCA
G. Anti-DsDNA
H. Anti-endomysial antibodies
I. Anti-GAD
J. Anti-smooth muscle antibody
K. Osmotic fragility test
L. Anti-scl70
M. Anti-mitochondrial antibody
A

Spherocytosis: Osmotic fragility test.
Causes of hemolysis are subdivided into two groups: those that are intrinsic (intracorpuscular) and those extrinsic to the red blood cell (RBC) (table 2). With few exceptions (eg, paroxysmal nocturnal hemoglobinuria and the rare condition of acquired alpha thalassemia), the causes of intrinsic RBC defects are hereditary. Extracorpuscular causes are almost always acquired conditions leading to increased destruction of otherwise normal RBCs.

351
Q

An 80-year-old retired clerk presents with a 2-month history of skin itching and lethargy. Examination is normal. LFTs are: bilirubin 6umol/l (reference range 0-17umol/l); ALT 24U/l (reference range 0-31U/l); Alk Phos 500U/l (reference range 30-130U/l).

A. Ham's test
B. p-ANCA
C. ANA
D. Anti-acetylcholine receptor antibody
E. Anti-gastric parietal cell antibodies
F. c-ANCA
G. Anti-DsDNA
H. Anti-endomysial antibodies
I. Anti-GAD
J. Anti-smooth muscle antibody
K. Osmotic fragility test
L. Anti-scl70
M. Anti-mitochondrial antibody
A

PBC - AMA E2 subtype of pyruvate dehydrogenase complex
Primary biliary cirrhosis
Characterized by a T-lymphocyte-mediated attack on small intralobular bile ducts. A continuous assault on the bile duct epithelial cells leads to their gradual destruction and eventual disappearance. The sustained loss of intralobular bile ducts causes the signs and symptoms of cholestasis and eventually results in cirrhosis and liver failure The diagnosis should be considered in the patient, particularly a woman, who complains of unexplained itching, fatigue, jaundice, or unexplained weight loss with right-upper-quadrant discomfort, and/or whose serum alkaline phosphatase is unaccountably elevated. Such patients should be questioned about symptoms of diseases frequently associated with PBC, such as Sjögren’s syndrome (dry eyes and mouth), arthritis, and Raynaud phenomenon.
Primary biliary cirrhosis is probably present if the serum alkaline phosphatase and IgM concentrations are both elevated and the antimitochondrial antibody test is positive.

352
Q

A 10-year-old girl presents with weight loss, polyuria, tachypnoea, vomiting. Looks very dehydrated. Beta hydroxybutyrate is raised in the blood.

A. Ham's test
B. p-ANCA
C. ANA
D. Anti-acetylcholine receptor antibody
E. Anti-gastric parietal cell antibodies
F. c-ANCA
G. Anti-DsDNA
H. Anti-endomysial antibodies
I. Anti-GAD
J. Anti-smooth muscle antibody
K. Osmotic fragility test
L. Anti-scl70
M. Anti-mitochondrial antibody
A

Type 1 diabetes: Anti-Glutamic acid decarboxylase antibodies (Anti-GAD).

Type 1A diabetes mellitus results from autoimmune destruction of the insulin-producing beta cells in the islets of Langerhans
The pathogenesis of type 1A diabetes is quite different from that of type 2 diabetes mellitus, in which both decreased insulin release (not on an autoimmune basis) and insulin resistance play an important role.
Polymorphisms of multiple genes are reported to influence the risk of type 1A diabetes (including, HLA-DQalpha, HLA-DQbeta, HLA-DR, preproinsulin, the PTPN22 gene, CTLA-4, interferon-induced helicase, IL2 receptor (CD25)
Antibodies to GAD (a 65-kD protein) are found in about 70 percent of patients with type 1 diabetes at the time of diagnosis.

353
Q

A 55-year-old woman is warned of future risk of AML given her recent diagnosis of PNH following a spontaneous cerebral venous sinus thrombosis.

A. Ham's test
B. p-ANCA
C. ANA
D. Anti-acetylcholine receptor antibody
E. Anti-gastric parietal cell antibodies
F. c-ANCA
G. Anti-DsDNA
H. Anti-endomysial antibodies
I. Anti-GAD
J. Anti-smooth muscle antibody
K. Osmotic fragility test
L. Anti-scl70
M. Anti-mitochondrial antibody
A

Ham’s test
Paroxysmal nocturnal hemoglobinuria (PNH) is a rare disorder. Clinical findings include unexplained hemolytic anemia, thrombosis in an atypical location, or nonspecific symptoms attributable to consequences of hemolysis.
Hemolysis may cause symptoms directly attributable to anemia and/or red blood cell (RBC) lysis, as well as findings caused indirectly by hemoglobin release, such as smooth muscle dystonia, pulmonary hypertension, and renal insufficiency.
Laboratory findings in PNH include typical findings of hemolytic anemia, loss of glycosylphosphatidylinositol (GPI)-anchored proteins, and findings associated with organ damage from hemolysis and/or thrombosis.
Screening for PNH is appropriate in patients with a direct antiglobulin (Coombs) negative hemolytic anemia, aplastic anemia, refractory anemia, or unexplained thrombosis in conjunction with cytopenias or hemolysis.
Some patients with PNH develop acute leukemia [60-68]. The lifetime risk is 5 percent or less. The leukemia may evolve from either the PNH clone or a non-PNH clone. Acute myeloid leukemia (AML) is most common

354
Q

A 40-year-old woman presents with polyuria and polydipsia. She has a fasting glucose 5.1mmol/L and an oral glucose tolerance test value of 5.0mmol/L. She has a corrected calcium of 2.80mmol/L and a PTH of 7.2pmol/L.

A. Diabetes mellitus type 2
B. Psychogenic polydipsia
C. Sarcoidosis
D. Lung cancer
E. Gestational diabetes
F. Impaired fasting glucose
G. Hypocalcaemia
H. Primary hyperparathyroidism
I. Crohn’s disease
J. Tuberculosis
K. Impaired glucose tolerance
L. Vitamin D deficiency
M. Secondary hyperthyroidism
N. Malignancy
O. Diabetes mellitus type 1
A

Primary hyperparathyroidism
If someone is complaining of polyuria, if the cause is glycosuria, the
glucose has to be above 10mM to hit the renal threshold. Thus a glucose of 5.6 or 6.1 cannot cause polyuria!!!

Normal Ca: 2.12 to 2.62 mmol/L

Hypercalcaemia is an uncommon problem. Primary hyperparathyroidism is the most common cause of raised calcium levels

3.5mmol/l: All above plus abdo pain, vomiting, dehydration, lethargy, cardiac arrhythmias, shortened QT interval, coma, pancreatitis

Non-PTH-mediated hypercalcaemia:
Malignancy - the most common cause of non-PTH-mediated hypercalcaemia.
Granulomatous conditions - eg, sarcoidosis and tuberculosis.
Endocrine conditions - eg, thyrotoxicosis, phaeochromocytoma and primary adrenal insufficiency.
Drugs - eg, thiazide diuretics, vitamin D and vitamin A supplements.[7]
Familial - eg, familial hypocalciuric hypercalcaemia.[5]
Other - eg, prolonged immobilisation, calcium-alkali syndrome, AIDS.

A raised albumin level in the presence of a raised urea indicates dehydration.
A raised albumin level in the presence of a normal urea suggests a cuffed specimen.
A normal alkaline phosphatase is indicative of myeloma (raised plasma protein), calcium-alkali syndrome (formerly milk-alkali syndrome),[9] thyrotoxicosis or sarcoidosis.
A raised alkaline phosphatase suggests bony metastases, sarcoidosis or thyrotoxicosis.
A raised calcitonin level is suggestive of B-cell lymphoma.

Raised PTH levels are suggestive of primary, secondary or tertiary hyperparathyroidism, or familial hypocalciuric hypercalcaemia.
Low PTH levels are seen in granulomatous disease, iatrogenic causes (eg, renal dialysis), adrenal insufficiency, thyrotoxicosis,and vitamin D intoxication.
The levels in malignancy may be low, normal or high.

355
Q

A 35-year-old Afro-Caribbean woman presents with polyuria and polydipsia. She also complains of a dry cough. She has a fasting glucose of 5.8mmol/L and an oral glucose tolerance test value of 6.5mmol/L. She has a corrected calcium of 2.7mmol/L and a PTH of

A

Sarcoidosis

Normal Ca: 2.12 to 2.62 mmol/L

Hypercalcaemia, low PTH, afro-carib, dry cough

Sarcoidosis is a multisystem granulomatous disorder of unknown etiology that affects individuals worldwide and is characterized pathologically by the presence of noncaseating granulomas in involved organs. It typically affects young adults, and initially presents with one or more of the following abnormalities:

●Bilateral hilar adenopathy
●Pulmonary reticular opacities
●Skin, joint, and/or eye lesions

Sarcoidosis most frequently involves the lung, but up to 30 percent of patients present with extrathoracic manifestations of sarcoidosis (table 1) [11-13]. Diffuse interstitial lung disease is the classic type of lung involvement; other less common pulmonary manifestations include pneumothorax, pleural thickening, chylothorax, and pulmonary hypertension.
Common presenting respiratory symptoms include cough, dyspnea, and chest pain; these are frequently accompanied by fatigue, malaise, fever, and weight loss
he serum angiotensin converting enzyme (ACE) level is elevated in 75 percent of untreated patients with sarcoidosis

The Kveim test uses a suspension of heat-sterilized splenic cells from patients with sarcoidosis (the Kveim-Siltzbach reagent) in an intradermal skin test (similar to a tuberculin skin test) to evoke a sarcoid granulomatous response over approximately three weeks [32]. The Kveim test is essentially a research tool due to limited availability of the reagent and concerns about disease transmission with testing.
Biopsy should be performed in most cases of suspected sarcoidosis.
The characteristic morphologic feature of sarcoidosis is the noncaseating granuloma.
The sarcoid granuloma is a focal, chronic inflammatory reaction formed by the accumulation of epithelial cells, monocytes, lymphocytes, macrophages, and fibroblasts

356
Q

A 15-year-old girl presents with weight loss, polyuria and polydipsia. Over the last few months she reports feeling increasingly tired and complains of perianal itching. On examination you notice a small perianal abscess. Her fasting glucose is 22.3mmol/L. Her corrected calcium is 2.5mmol/L and his PTH is 7.0pmol/L.

A. Diabetes mellitus type 2
B. Psychogenic polydipsia
C. Sarcoidosis
D. Lung cancer
E. Gestational diabetes
F. Impaired fasting glucose
G. Hypocalcaemia
H. Primary hyperparathyroidism
I. Crohn’s disease
J. Tuberculosis
K. Impaired glucose tolerance
L. Vitamin D deficiency
M. Secondary hyperthyroidism
N. Malignancy
O. Diabetes mellitus type 1
A

Diabetes mellitus type 1
Type 1A diabetes mellitus results from autoimmune destruction of the insulin-producing beta cells in the islets of Langerhans [1]. This process occurs in genetically susceptible subjects, is probably triggered by one or more environmental agents, and usually progresses over many months or years during which the subject is asymptomatic and euglycemic. This long latent period is a reflection of the large number of functioning beta cells that must be lost before hyperglycemia occurs
Polymorphisms of multiple genes are known to influence the risk of type 1A diabetes (HLA-Dqalpha; HLA-Dqbeta; HLA-DR, preproinsulin, the PTPN22 gene, and CTLA-4),
There are a number of autoantigens within the pancreatic beta cells that may play important roles in the initiation or progression of autoimmune islet injury including glutamic acid decarboxylase (GAD), insulin, insulinoma-associated protein 2 (IA-2), and zinc transporter ZnT8.

357
Q

A 56-year-old obese woman presents with polyuria and polydipsia. She complains of tiredness and depression. Her fasting glucose is 4.9mmol/L and her OGTT is 4.5mmol/L. She has a corrected calcium of 2.4mmol/L and a PTH of 7.1mmol/L

A. Diabetes mellitus type 2
B. Psychogenic polydipsia
C. Sarcoidosis
D. Lung cancer
E. Gestational diabetes
F. Impaired fasting glucose
G. Hypocalcaemia
H. Primary hyperparathyroidism
I. Crohn’s disease
J. Tuberculosis
K. Impaired glucose tolerance
L. Vitamin D deficiency
M. Secondary hyperthyroidism
N. Malignancy
O. Diabetes mellitus type 1
A

Psychogenic polydipsia - nothing wrong with her biochem!!!!

358
Q

A 58-year-old Afro-Caribbean gentleman presents with polyuria, polydipsia and weight loss. He has an oral glucose tolerance test of 10.1mmol/L. His corrected calcium is 2.5mmol/L and his PTH is 7.0pmol/L.

A. Diabetes mellitus type 2
B. Psychogenic polydipsia
C. Sarcoidosis
D. Lung cancer
E. Gestational diabetes
F. Impaired fasting glucose
G. Hypocalcaemia
H. Primary hyperparathyroidism
I. Crohn’s disease
J. Tuberculosis
K. Impaired glucose tolerance
L. Vitamin D deficiency
M. Secondary hyperthyroidism
N. Malignancy
O. Diabetes mellitus type 1
A

Impaired glucose tolerance

A normal fasting blood glucose level is less than 6 mmol/L.

Diabetes : a fasting blood glucose > 7 mmol/L , OR
a blood glucose > 11.1 mmol/L after a two-hour oral glucose tolerance test (GTT)

Impaired glucose tolerance: a fasting blood glucose 7.8 mmol/L but

359
Q

Varies with posture when sample is taken.

A. Potassium
B. Creatinine Kinase
C. Triglycerides
D. Urea
E. Albumin
F. ALP
G. Glucose
H. ALT
I. Cortisol
A

Albumin
Plasma renin activity also varies with posture - it rises in the upright position. Some people have so-called benign postural and/or exercise-induced albuminuria.

360
Q

Varies with exercise

A. Potassium
B. Creatinine Kinase
C. Triglycerides
D. Urea
E. Albumin
F. ALP
G. Glucose
H. ALT
I. Cortisol
A

Creatinine Kinase

There are 3 main iso-forms of CK.
CK-MM: present in muscles.
CK-BB: Present in Brain.
CK-MB: Present in cardiac muscle.

361
Q

Increases during pregnancy

A. Potassium
B. Creatinine Kinase
C. Triglycerides
D. Urea
E. Albumin
F. ALP
G. Glucose
H. ALT
I. Cortisol
A

ALP

362
Q

Varies with race

A. Potassium
B. Creatinine Kinase
C. Triglycerides
D. Urea
E. Albumin
F. ALP
G. Glucose
H. ALT
I. Cortisol
A

Creatinine Kinase

There are 3 main iso-forms of CK.
CK-MM: present in muscles.
CK-BB: Present in Brain.
CK-MB: Present in cardiac muscle.

363
Q

Most likely to vary with time of sampling

A. Potassium
B. Creatinine Kinase
C. Triglycerides
D. Urea
E. Albumin
F. ALP
G. Glucose
H. ALT
I. Cortisol
A

Cortisol

364
Q

A 19-year-old woman admitted to hospital with acute asthma suffered a cardiac arrest after treatment. She was already taking several medications for her respiratory condition. What drug excess is likely to have caused this problem?

A. Kidneys
B. Low therapeutic index
C. Liver
D. Warfarin
E. Digoxin
F. Oxidation by cytochrome P450
G. Gentamicin
H. GI system
I. Rosiglitazone
J. Poor compliance
K. Conjugation by sulphate/gluconaride
L. High therapeutic index
M. Theophylline
N. Lungs
A

Theophylline has bronchodilatory, antiinflammatory, immunomodulating, and bronchoprotective effects. Theophylline remains a potentially useful and inexpensive medication for patients with chronic asthma whose symptoms are not controlled with conventional doses of inhaled glucocorticoids, and patients who cannot take or are poorly compliant with inhaled medications.
Theophylline is not routinely indicated for the treatment of acute severe asthma in the intensive care unit. Intravenous theophylline (as theophylline or aminophylline) may be added when patients with severe acute asthma fail to respond to vigorous use of inhaled albuterol and ipratropium with systemic glucocorticoids, although evidence for this is lacking.

365
Q

Failure to respond to drug therapy is commonly caused by what?

A. Kidneys
B. Low therapeutic index
C. Liver
D. Warfarin
E. Digoxin
F. Oxidation by cytochrome P450
G. Gentamicin
H. GI system
I. Rosiglitazone
J. Poor compliance
K. Conjugation by sulphate/gluconaride
L. High therapeutic index
M. Theophylline
N. Lungs
A

Poor compliance

366
Q

Lipid soluble drugs require metabolism by the liver in two phases. What is Phase I?

A. Kidneys
B. Low therapeutic index
C. Liver
D. Warfarin
E. Digoxin
F. Oxidation by cytochrome P450
G. Gentamicin
H. GI system
I. Rosiglitazone
J. Poor compliance
K. Conjugation by sulphate/gluconaride
L. High therapeutic index
M. Theophylline
N. Lungs
A

Oxidation by cytochrome P450

367
Q

Drugs are mainly excreted by which organ?

A. Kidneys
B. Low therapeutic index
C. Liver
D. Warfarin
E. Digoxin
F. Oxidation by cytochrome P450
G. Gentamicin
H. GI system
I. Rosiglitazone
J. Poor compliance
K. Conjugation by sulphate/gluconaride
L. High therapeutic index
M. Theophylline
N. Lungs
A

Kidneys

368
Q

The effect of which drug can be measured by the surrogate marker HbA1C

A. Kidneys
B. Low therapeutic index
C. Liver
D. Warfarin
E. Digoxin
F. Oxidation by cytochrome P450
G. Gentamicin
H. GI system
I. Rosiglitazone
J. Poor compliance
K. Conjugation by sulphate/gluconaride
L. High therapeutic index
M. Theophylline
N. Lungs
A

Rosiglitazone
Rosiglitazone and pioglitazone are used as monotherapy or with a sulfonylurea, metformin, or insulin. However, there are concerns with combined thiazolidinedione and insulin therapy because of an increased incidence of heart failure (HF).
The thiazolidinediones increase insulin sensitivity by acting on adipose, muscle, and liver to increase glucose utilization and decrease glucose production.
hey bind to and activate one or more peroxisome proliferator-activated receptors (PPARs), which regulate gene expression in response to ligand binding

369
Q

A 58-year-old man presents to your A&E complaining of chest pain and palpitations. He says he takes several drugs for his ‘heart problems’ and admits to being diabetic. What drug could be causing his problems?

A. Kidneys
B. Low therapeutic index
C. Liver
D. Warfarin
E. Digoxin
F. Oxidation by cytochrome P450
G. Gentamicin
H. GI system
I. Rosiglitazone
J. Poor compliance
K. Conjugation by sulphate/gluconaride
L. High therapeutic index
M. Theophylline
N. Lungs
A

Possible features of DIGOXIN TOXICITY include:

arrhythmia: the most common arrhythmias are ventricular extrasystoles, ventricular bigeminy / trigeminy and atrial tachycardia with complete heart block

anorexia, nausea and vomiting and occasionally, diarrhoea

confusion especially in the elderly

yellow vision (xanthopsia), blurred vision and photophobia

370
Q

Peak and trough levels of this drug should be taken

A. Lithium
B. Phenytoin
C. Phenobarbitone
D. Theophylline
E. Clonazepam
F. Carbamazepine
G. Heparin - unfractionated
H. Digoxin
I. Warfarin
J. Gentamicin
K. Heparin - Low molecular weight
L. Ethosuximide
M. Ciclosporin
N. Aspirin
A

Gentamicin is a bactericidal antibiotic that works by irreversibly binding the 30S subunit of the bacterial ribosome, interrupting protein synthesis. This mechanism of action is similar to other Aminoglycosides
Active against a wide range of human bacterial infections, mostly Gram-negative bacteria including Pseudomonas, Proteus, Serratia, and the Gram-positive Staphylococcus.[4] Gentamicin is not used for Neisseria gonorrhoeae, Neisseria meningitidis or Legionella pneumophila bacterial infections (because of the risk of the patient going into shock from lipid A endotoxin found in certain Gram-negative organisms).
Aminoglycosides are toxic to the sensory cells of the ear, but they vary greatly in their relative effects on hearing versus balance.
Prevention of nephrotoxicity includes judicouss use of IV fluids to correct and avoid volume depletion, correction of hypokalemia and hypomagnesemia. Once daily dosing has been shown to be less toxic than multiple daily doses. Gentamicin is usually dosed by ideal body weight. Various formulae exist for calculating gentamicin dosage. Trough and peak serum levels of gentamicin are monitored during treatment to individualize therapy and prevent excess exposure.[9]

Gentamicin, like other aminoglycosides, causes nephrotoxicity by inhibiting protein synthesis in renal cells. This mechanism specifically causes necrosis of cells in the proximal tubule, resulting in acute tubular necrosis which can lead to acute renal failure

371
Q

Symptoms of under-treatment and toxicity may be similar

A. Lithium
B. Phenytoin
C. Phenobarbitone
D. Theophylline
E. Clonazepam
F. Carbamazepine
G. Heparin - unfractionated
H. Digoxin
I. Warfarin
J. Gentamicin
K. Heparin - Low molecular weight
L. Ethosuximide
M. Ciclosporin
N. Aspirin
A

Digoxin

Possible features of DIGOXIN TOXICITY include:

arrhythmia: the most common arrhythmias are ventricular extrasystoles, ventricular bigeminy / trigeminy and atrial tachycardia with complete heart block

anorexia, nausea and vomiting and occasionally, diarrhoea

confusion especially in the elderly

yellow vision (xanthopsia), blurred vision and photophobia

372
Q

Decreased excretion, increased plasma concentration and increased risk of toxicity may occur when this taken in conjunction with thiazide diuretics

A. Lithium
B. Phenytoin
C. Phenobarbitone
D. Theophylline
E. Clonazepam
F. Carbamazepine
G. Heparin - unfractionated
H. Digoxin
I. Warfarin
J. Gentamicin
K. Heparin - Low molecular weight
L. Ethosuximide
M. Ciclosporin
N. Aspirin
A

Lithium is used primarily for bipolar disorder. Those who use lithium should receive regular serum level tests and should monitor thyroid and kidney function for abnormalities, as it interferes with the regulation of sodium and water levels in the body, and can cause dehydration. Dehydration, which is compounded by heat, can result in increasing lithium levels. The dehydration is due to lithium inhibition of the action of antidiuretic hormone, which normally enables the kidney to reabsorb water from urine. This causes an inability to concentrate urine, leading to consequent loss of body water and thirst.
Hyponatremia can cause lithium retention and thus increased lithium levels.

Lithium concentrations are known to be increased with concurrent use of diuretics — especially loop diuretics (such as furosemide) and thiazides — and non-steroidal anti-inflammatory drugs (NSAIDs) such as ibuprofen and aspirin.[1] Lithium concentrations can also be increased with concurrent use of ACE inhibitors such as captopril, enalapril, and lisinopril.[43]

Lithium is mainly removed from the body through glomerular function, but some are then reabsorbed together with sodium through the proximal tubule. Its levels are therefore sensitive to water and electrolyte balance.[44] Diuretics act by lowering water and sodium levels. This causes more reabsorption of lithium in the proximal tubules so that the removal of lithium from the body is less, leading to increased levels of lithium.[44][45] ACE inhibitors have also been shown in a retrospective case-control study to increase lithium concentrations. A possible mechanism is that ACE inhibitors can lead to a decrease in sodium and water. This will increase lithium reabsorption and its concentrations in the body.[44]

There are also drugs that can increase the clearance of lithium from the body, which can result in decreased lithium levels in the blood. These drugs include theophylline, caffeine, and acetazolamide. Additionally, increasing dietary sodium intake may also reduce lithium levels by prompting the kidneys to excrete more lithium.[46]

Lithium is also known to be a potential precipitant of serotonin syndrome in people concurrently on serotonergic medications such as antidepressants, buspirone and certain opioids such as pethidine (meperidine), tramadol, oxycodone, fentanyl and others.[1][47] Lithium co-treatment is also a risk factor for neuroleptic malignant syndrome in people on antipsychotics and other antidopaminergic medications.

ENZYME INDUCERS: GPPARCS SSS NN
Griseofulvin
Phenytoin
Phenobarbitone and other barbiturates (Phenobarbital, meprobamate, primidone)
Alcohol (chronic use) - CYP 2E1
Rifampicin, Rifabutin, Rifapentine
Carbamazepine > Oxcarbazepine, Topiramate (induces the Oral Contraceptive Pill)

Sulfonylureas
Smoking
St John’s Wort (Hypericum perforatum)

NNRTIs: Nevirapine (substrate and inducer), Efavirenz (substrate, inducer, inhibitor = the ‘triple’)
and the PI Ritonavir (substrate, inducer, inhibitor = the ‘triple’)

ENZYME NEUTRAL
Lamotrigine, Pregabalin, Levetiracetam
Benzodiazepines (not barbiturates)
Azithromycin (Not other macrolides)
Tetracycline 
Quinolones (Not Ciprofloxacin)

ENZYME INHIBITORS: Depressed GP DAVe to visit SICKFACES.COM ASAP

Depressed: MAO-Is, Duloxetine (SNRI), Fluoxetine, Fluvoxamine, Sertraline, Paroxetine, Not Citalopram

Grapefruit juice
Protease Inhibitors (Ritonavir is the only 'triple', the rest are straight inhibitors)

Diltiazem
Amiodarone
Verapamil
e

to visit

Sodium Valproate
Isoniazid especially in SLOW acetylators
Cimetidine not ranitidine
Ketoconazole (most potent inhibitor) > Itraconazole > Fluconazole (mildest)
(Fluconazole)
Alcohol (binge drinking)
Chloramphenicol
Erythromycin, Clarithromycin, Telithromycin, not Azithromycin
Synercid (Quinupristin-Dalfopristin)
Dot : Disulfiram
Ciprofloxacin (inhibits 1A2.. yay warfarin and theophylline toxicity)
Omeprazole not lansoprazole
Metronidazole (remember metro has a disulfiram like effect)

Allopurinol
Sulfinpyrazone
(Atorvastatin)
Phenylbutazone

373
Q

Is ototoxic and nephrotoxic

A. Lithium
B. Phenytoin
C. Phenobarbitone
D. Theophylline
E. Clonazepam
F. Carbamazepine
G. Heparin - unfractionated
H. Digoxin
I. Warfarin
J. Gentamicin
K. Heparin - Low molecular weight
L. Ethosuximide
M. Ciclosporin
N. Aspirin
A

Gentamicin is a bactericidal antibiotic that works by irreversibly binding the 30S subunit of the bacterial ribosome, interrupting protein synthesis. This mechanism of action is similar to other Aminoglycosides
Active against a wide range of human bacterial infections, mostly Gram-negative bacteria including Pseudomonas, Proteus, Serratia, and the Gram-positive Staphylococcus.[4] Gentamicin is not used for Neisseria gonorrhoeae, Neisseria meningitidis or Legionella pneumophila bacterial infections (because of the risk of the patient going into shock from lipid A endotoxin found in certain Gram-negative organisms).
Aminoglycosides are toxic to the sensory cells of the ear, but they vary greatly in their relative effects on hearing versus balance.
Prevention of nephrotoxicity includes judicouss use of IV fluids to correct and avoid volume depletion, correction of hypokalemia and hypomagnesemia. Once daily dosing has been shown to be less toxic than multiple daily doses. Gentamicin is usually dosed by ideal body weight. Various formulae exist for calculating gentamicin dosage. Trough and peak serum levels of gentamicin are monitored during treatment to individualize therapy and prevent excess exposure.[9]

Gentamicin, like other aminoglycosides, causes nephrotoxicity by inhibiting protein synthesis in renal cells. This mechanism specifically causes necrosis of cells in the proximal tubule, resulting in acute tubular necrosis which can lead to acute renal failure

374
Q

Requires regular monitoring of APTT

A. Lithium
B. Phenytoin
C. Phenobarbitone
D. Theophylline
E. Clonazepam
F. Carbamazepine
G. Heparin - unfractionated
H. Digoxin
I. Warfarin
J. Gentamicin
K. Heparin - Low molecular weight
L. Ethosuximide
M. Ciclosporin
N. Aspirin
A

Heparin - unfractionated

highly sulfated glycosaminoglycan, is widely used as an injectable anticoagulant, and has the highest negative charge density of any known biological molecule

Antidote to heparin overdose[edit]
Protamine sulfate (1 mg per 100 units of heparin that had been given over the past four hours) has been given to counteract the anticoagulant effect of heparin.

Heparin binds to the enzyme inhibitor antithrombin III (AT), causing a conformational change that results in its activation through an increase in the flexibility of its reactive site loop.[13] The activated AT then inactivates thrombin and other proteases involved in blood clotting, most notably factor Xa. The rate of inactivation of these proteases by AT can increase by up to 1000-fold due to the binding of heparin.

The effects of heparin are measured in the lab by the partial thromboplastin time (aPTT)

375
Q

A man was put into custody after driving under the influence of drugs. On arrest he was reported as acting extremely aggressive and paranoid. He also claimed his heart was racing. One hour later he was found dead. There was suspicion of police brutality.

A. Cyanide
B. Cannabis
C. Heroin
D. Paracetamol
E. Ethanol
F. Carbon monoxide
G. Police brutality
H. Aspirin
I. Strychnine
J. Cocaine
K. Amphetamines
L. Ecstasy
M. Methadone
N. Methanol
O. Organophosphate
P. Benzodiazepines
A

Cocaine: EME = ecgonine methyl ester and BE = benzoylecgonine are the two degredation products of cocaine produced by pseudocholinesterases and hydrolysis respectively.

It is a stimulant, an appetite suppressant, and a nonspecific voltage gated sodium channel blocker, which in turn causes it to produce anaesthesia at low doses. Biologically, cocaine acts as a serotonin–norepinephrine–dopamine reuptake inhibitor, also known as a triple reuptake inhibitor (TRI). It is addictive due to its effect on the mesolimbic reward pathway.[6] At high doses, it is markedly more dangerous than other CNS stimulants, including the entire amphetamine drug class,[7] due to its effect on sodium channels, since blockade of Nav1.5 can cause sudden cardiac death.
With excessive or prolonged use, the drug can cause itching, tachycardia, hallucinations, and paranoid delusions.[15] Overdoses cause hyperthermia and a marked elevation of blood pressure, which can be life-threatening,[15] arrhythmias,[8] and death.

376
Q

A 24-year-old woman goes to a party where she has some pills. She subsequently becomes feverish and confused. She was found to be hyperthermic and blood results showed a raised urea and creatinine, her myoglobin was also found to be high.

A. Cyanide
B. Cannabis
C. Heroin
D. Paracetamol
E. Ethanol
F. Carbon monoxide
G. Police brutality
H. Aspirin
I. Strychnine
J. Cocaine
K. Amphetamines
L. Ecstasy
M. Methadone
N. Methanol
O. Organophosphate
P. Benzodiazepines
A

Ecstacy
Peak effects of MDMA toxicity occur within two hours of ingestion and typically last four to six hours. Concentrations of MDMA contained in illicitly produced pills vary widely.
MDMA intoxication can cause a myriad of dangerous effects including severe hypertension, hyperthermia, delirium, psychomotor agitation, and profound hyponatremia. Potential life-threatening complications of these effects include intracranial hemorrhage, myocardial infarction, aortic dissection, disseminated intravascular coagulation, rhabdomyolysis, seizure, and serotonin syndrome.

We recommend severe hypertension and psychomotor agitation be treated with benzodiazepines (eg, lorazepam, 1 to 2 mg IV push; may repeat until hypertension is controlled or patient is sedated) (Grade 1C). Refractory hypertension may be treated with nitroprusside or phentolamine. We avoid beta-blocking agents, including labetalol, in the treatment of sympathomimetic poisoning because they may lead to unopposed alpha-adrenergic stimulation and a paradoxical increase in blood pressure. (See ‘Psychomotor agitation’ above.)
●For a recent ingestion (ie, less than one hour) of MDMA, we suggest a single dose of activated charcoal (1 g/kg; maximum dose 50 g) be administered (Grade 2C). Charcoal should be withheld in patients who are sedated and may not be able to protect their airway, unless endotracheal intubation is performed first. However, endotracheal intubation should not be performed solely for the purpose of giving charcoal.

377
Q

James Pond comes to A&E claiming he’s been poisoned. Minutes later he dies. His skin was brick red and there was a faint odour of almonds.

A. Cyanide
B. Cannabis
C. Heroin
D. Paracetamol
E. Ethanol
F. Carbon monoxide
G. Police brutality
H. Aspirin
I. Strychnine
J. Cocaine
K. Amphetamines
L. Ecstasy
M. Methadone
N. Methanol
O. Organophosphate
P. Benzodiazepines
A

Cyanide: Because of the decreased utilization of oxygen by tissues, venous oxyhemoglobin concentration will be high, making venous blood appear bright red. Therefore, despite hypotension, apnea, and/or bradycardia, the patient does not appear cyanotic in the setting of cyanide poisoning.
Antidotal treatment of cyanide poisoning involves three strategies: binding of cyanide, induction of methemoglobinemia, and use of sulfur donors.

378
Q

Following a death in the family, a young woman is brought into the hospital with confusion. On inspection she appears jaundiced. Her friend reports that she had been vomiting earlier and that she had found an empty medicine bottle in her room.

A. Cyanide
B. Cannabis
C. Heroin
D. Paracetamol
E. Ethanol
F. Carbon monoxide
G. Police brutality
H. Aspirin
I. Strychnine
J. Cocaine
K. Amphetamines
L. Ecstasy
M. Methadone
N. Methanol
O. Organophosphate
P. Benzodiazepines
A

Paracetamol: Appropriate acetaminophen doses produce a small amount of NAPQI which is rapidly conjugated with hepatic glutathione, forming nontoxic cysteine and mercaptate compounds that are excreted in the urine [26,31]. However, with toxic doses of a acetaminophen the sulfation and glucuronidation pathways are saturated, and more acetaminophen is metabolized to NAPQI via the cytochrome P450 enzymes [32]. When hepatic glutathione stores are depleted by approximately 70 to 80 percent, NAPQI begins to react with hepatocytes, and injury ensues. NAPQI arylates and binds covalently to the cysteine groups on hepatic macromolecules, forming NAPQI-protein adducts [35-37]. This process is irreversible and leads to oxidative injury and hepatocellular centrilobular necrosis

In the first 24 hours after overdose, patients often manifest nausea, vomiting, diaphoresis, pallor, lethargy, and malaise. Some patients remain asymptomatic.
From 24 to 72 hours after ingestion, the clinical and laboratory evidence of hepatotoxicity and, occasionally, nephrotoxicity become evident.
Liver function abnormalities peak from 72 to 96 hours after ingestion. The systemic symptoms of stage I reappear in conjunction with jaundice, confusion (hepatic encephalopathy), a marked elevation in hepatic enzymes, hyperammonemia, and a bleeding diathesis
Signs of severe hepatotoxicity include plasma ALT and AST levels that often exceed 10,000 IU/L, prolongation of the PT or INR, hypoglycemia, lactic acidosis, and a total bilirubin concentration above 4.0 mg/dL

379
Q

A man was found collapsed on the floor of his room and his breathing was found to be severely depressed. A urine test was found to be positive for 6-MAM.

A. Cyanide
B. Cannabis
C. Heroin
D. Paracetamol
E. Ethanol
F. Carbon monoxide
G. Police brutality
H. Aspirin
I. Strychnine
J. Cocaine
K. Amphetamines
L. Ecstasy
M. Methadone
N. Methanol
O. Organophosphate
P. Benzodiazepines
A

Heroin
The classic signs of opioid intoxication include: depressed mental status, decreased respiratory rate, decreased tidal volume, decreased bowel sounds, and miotic pupils. The best predictor of opioid poisoning is a respiratory rate

380
Q

A 30-year-old farmer presents to casualty complaining of diarrhoea and painful mouth ulcers. On questioning he admitted accidentally ingesting liquid paraquat

A. N-acetylcysteine 
B. Haemodialysis  
C. Dicobalt edentate
D. Naloxone 
E. Symptomatic and Supportive treatment 
F. Glucagon 
G. Atropine 
H. Hyperbaric oxygen 
I. Activated charcoal 
J. Gastric lavage 
K. Desferrioxamine
A

Activated charcoal
Paraquat is used in agriculture, and when ingested oxidative stress created by the production of free radicals and the depletion of NADPH directly causes cell damage (via lipid peroxidation, mitochondrial dysfunction, necrosis and apoptosis) and triggers a pronounced secondary inflammatory response.
Over a period of hours to days these processes lead to multi-organ failure. The organs most affected are those with high blood flow, oxygen tension, and energy requirements, in particular the lungs, heart, kidneys, and liver. The brain is uncommonly affected as paraquat does not readily cross the blood-brain barrier, although paraquat has been detected in the CSF

Oral and gastrointestinal symptoms are common. Patients usually have a painful mouth and pain with swallowing. Nausea, vomiting, and abdominal pain occur in most patients. 
Activated charcoal (1 g/kg in water; maximum dose 50 g) or Fuller’s Earth (2 g/kg in water; maximum 150 g in water) should be given as soon as possible per oral or via a nasogastric tube to patients who present within approximately two hours of ingestion.
381
Q

A 15-year-old girl presents with sweats and hyperventilation indicative of a severe metabolic acidosis; after taking a large number of salicylate tablets

A. N-acetylcysteine 
B. Haemodialysis  
C. Dicobalt edentate
D. Naloxone 
E. Symptomatic and Supportive treatment 
F. Glucagon 
G. Atropine 
H. Hyperbaric oxygen 
I. Activated charcoal 
J. Gastric lavage 
K. Desferrioxamine
A

Haemodialysis

There are 2 reasons why haemodialysis is a better answer than activated charcoal. One, because
you have no direct information about when she took the overdose - most guidelines only advise giving activated charcoal if the patient presents within 1 hour of taking the tablets. Having said that, the fact that she is severely acidotic with hyperventilation suggests some time has elapsed since the tablets were taken, as a metabolic acidosis and partial respiratory compensation don’t develop instantaneously. This leads into the second reason why haemodialysis is a better answer - this girl
is really sick! Lots of body functions, including much enzyme activity, is pH dependent, so she needs treatment for her acid-base and electrolyte imbalance. Sodium bicarbonate can be used to correct acidosis, but haemodialysis is the treatment of choice for severe poisoning and those with high salicylate levels.

Early symptoms of acute aspirin toxicity include tinnitus, vertigo, nausea, vomiting, and diarrhea; subsequent symptoms portending a more severe intoxication include altered mental status (ranging from agitation to lethargy), hyperpyrexia, noncardiac pulmonary edema, and coma. Early symptoms are typically present within one to two hours after a single acute ingestion. Hyperpnea is often observed in salicylate overdose. Salicylates stimulate the respiratory center directly, resulting in an early fall in the PCO2 and respiratory alkalosis. An anion-gap metabolic acidosis then follows, due primarily to the accumulation of organic acids, including lactic acid and ketoacids.
Treatment of salicylate intoxication is directed toward decreasing the fraction of uncharged (protonated) molecules, which is accomplished by increasing the systemic pH (ie, lowering the H+ ion concentration). This is referred to as “alkalinization” and is most easily accomplished by administering sodium bicarbonate. Increasing the systemic pH reduces the diffusion of salicylate anions into the central nervous system, as charged molecules do not easily diffuse across the blood-brain barrier. Alkalinization also “traps” salicylate anions within the renal tubule, preventing back-diffusion across the renal epithelium into the systemic circulation.

Indications for hemodialysis include the following:

●Altered mental status
●Pulmonary or cerebral edema
●Renal insufficiency that interferes with salicylate excretion
●Fluid overload that prevents the administration of sodium bicarbonate
●A serum salicylate concentration >100 mg/dL (7.2 mmol/L) in acute overdose
●Clinical deterioration despite aggressive and appropriate supportive care

Aspirin intoxication may decrease cerebral glucose concentrations despite a normal serum glucose. Therefore, we suggest that adults with salicylate poisoning who are hypoglycemic or manifest alterations in mental status, regardless of their serum glucose concentration, be treated with supplemental glucose

382
Q

A 26-year-old woman collapses after a massive overdose of atenolol. She remains in cardogenic shock despite initial treatment with IV atropine

A. N-acetylcysteine 
B. Haemodialysis  
C. Dicobalt edentate
D. Naloxone 
E. Symptomatic and Supportive treatment 
F. Glucagon 
G. Atropine 
H. Hyperbaric oxygen 
I. Activated charcoal 
J. Gastric lavage 
K. Desferrioxamine
A

Glucagon in beta-blocker overdose is to counteract hypoglycaemia secondary to beta-
block of glycogenolysis. However, it can also have an effect on bradycardia and subsequent cardiogenic shock because glucagon
receptors are G-protein coupled receptors that lead to an increase in adenyl cyclase activity and increased intra-cellular cAMP.
This bypasses the need for B1-receptor activity to increase cAMP.

Complications following beta blocker overdose are related to excessive beta adrenergic blockade, and occasionally the proarrhythmic (membrane-stabilizing) activity of these agents on cardiac conduction [2]. Ingestion of other cardioactive agents in association with beta blockers increases mortality following overdose [2,3]. Common and potentially dangerous coingestions include calcium channel blockers, cyclic antidepressants, and neuroleptics

●Beta 1, which are found primarily in heart muscle. Activation of these receptors results in an increase in heart rate, contractility, atrioventricular (AV) conduction, and a decrease in AV node refractoriness.
●Beta 2, which are present in heart muscle but are more prominent in bronchial and peripheral vascular smooth muscle. Activation of these receptors results in vasodilation and bronchodilation.
●Beta 3, which are found in adipose tissue and the heart. Activation of these receptors may mediate catecholamine-induced thermogenesis and may reduce cardiac contractility.

Bradycardia and hypotension are the most common effects, and in severe overdoses can result in profound myocardial depression and cardiogenic shock. Ventricular dysrhythmias are seen more frequently following propranolol and acebutolol exposures
Beta blockers decrease conduction velocity across the atrioventricular (AV) node, resulting in PR prolongation; they also slow automaticity within the sinoatrial (SA) node, causing bradycardia

  • Stabilization of the airway as necessary (avoid induction agents that exacerbate hypotension)
  • Additional IV boluses of isotonic crystalloid
  • IV glucagon
  • IV calcium salts
  • Vasopressor (eg, epinephrine)
  • IV high-dose insulin and glucose
  • IV lipid emulsion therapy
383
Q

A pregnant 30-year-old woman is found drowsy in her rented flat. She complains of severe nausea for the last 3 hours. Her carboxyhaemoglobin level is 41%.

A. N-acetylcysteine 
B. Haemodialysis  
C. Dicobalt edentate
D. Naloxone 
E. Symptomatic and Supportive treatment 
F. Glucagon 
G. Atropine 
H. Hyperbaric oxygen 
I. Activated charcoal 
J. Gastric lavage 
K. Desferrioxamine
A

Hyperbaric oxygen
CO poisoning is most common during winter in cold climates, but it may occur in all seasons and environments. Smoke inhalation is responsible for most unintentional cases. Other potential sources of CO include poorly functioning heating systems, improperly vented fuel-burning devices (eg, kerosene heaters, charcoal grills, camping stoves, gasoline-powered electrical generators), and motor vehicles operating in poorly ventilated areas.
CO diffuses rapidly across the pulmonary capillary membrane and binds to the iron moiety of heme with approximately 240 times the affinity of oxygen. The degree of carboxyhemoglobinemia (COHb) is a function of the relative amounts of CO and oxygen in the environment, duration of exposure, and minute ventilation.
The clinical findings of CO poisoning are highly variable and largely nonspecific. Mild to moderate CO-intoxicated patients often present with constitutional symptoms, including headache (most common), malaise, nausea, and dizziness, and may be misdiagnosed with acute viral syndromes. In the absence of concurrent trauma or burns, physical findings in CO poisoning are usually confined to alterations in mental status, ranging from mild confusion to seizures and coma. A careful neurologic examination is crucial.
The diagnosis of CO poisoning is based upon a compatible history and physical examination in conjunction with an elevated carboxyhemoglobin level. Diagnosis requires quantification by cooximetry of a blood gas sample; standard pulse oximetry (SP02) is unable to distinguish between oxyhemoglobin and COHb. Blood PO2 measurements tend to be normal because PO2 reflects O2 dissolved in blood, and this process is not affected by CO

Hyperbaric oxygen therapy (HBO): to decrease the incidence and severity of delayed neurocognitive deficits following CO poisoning

384
Q

A 25-year-old man is delirious and hyperpyrexial after taking a pill in a club. He is hyperreflexic and is hyponatraemic

A. N-acetylcysteine 
B. Haemodialysis  
C. Dicobalt edentate
D. Naloxone 
E. Symptomatic and Supportive treatment 
F. Glucagon 
G. Atropine 
H. Hyperbaric oxygen 
I. Activated charcoal 
J. Gastric lavage 
K. Desferrioxamine
A

Symptomatic and Supportive treatment
MDMA-associated hyponatremia frequently produces obtundation to a degree that necessitates endotracheal intubation. No induction or paralytic agent commonly used for rapid sequence intubation (RSI) is contraindicated in the setting of MDMA intoxication.

We recommend severe hypertension and psychomotor agitation be treated with benzodiazepines (eg, lorazepam, 1 to 2 mg IV push; may repeat until hypertension is controlled or patient is sedated). Refractory hypertension may be treated with nitroprusside or phentolamine. We avoid beta-blocking agents, including labetalol, in the treatment of sympathomimetic poisoning because they may lead to unopposed alpha-adrenergic stimulation and a paradoxical increase in blood pressure.

For a recent ingestion (ie, less than one hour) of MDMA, we suggest a single dose of activated charcoal (1 g/kg; maximum dose 50 g) be administered. Charcoal should be withheld in patients who are sedated and may not be able to protect their airway, unless endotracheal intubation is performed first. However, endotracheal intubation should not be performed solely for the purpose of giving charcoal.

Butyrophenones (such as haloperidol and droperidol) interfere with heat dissipation, may prolong the QTc, and may reduce the seizure threshold. We suggest butyrophenones NOT be used to sedate patients with MDMA toxicity

Seizures, hyperthermia, and serotonin syndrome should be treated in standard fashion

385
Q

An 18 year old female is brought in to A&E from a rave in the early hours of the morning. On initial examination she is agitated with a heart rate of 120 bpm. She is very sweaty and has wide dilated pupils

A. Lithium
B. Tricyclic antidepressants 
C. Carbon Monoxide 
D. Ecstasy 
E. Naloxone 
F. Paracetamol 
G. Desferrioxamine 
H. Methanol 
I. Acetylcysteine 
J. Salicylates 
K. Organophosphates
A

Ecstacy
MDMA-associated hyponatremia frequently produces obtundation to a degree that necessitates endotracheal intubation. No induction or paralytic agent commonly used for rapid sequence intubation (RSI) is contraindicated in the setting of MDMA intoxication.

We recommend severe hypertension and psychomotor agitation be treated with benzodiazepines (eg, lorazepam, 1 to 2 mg IV push; may repeat until hypertension is controlled or patient is sedated). Refractory hypertension may be treated with nitroprusside or phentolamine. We avoid beta-blocking agents, including labetalol, in the treatment of sympathomimetic poisoning because they may lead to unopposed alpha-adrenergic stimulation and a paradoxical increase in blood pressure.

For a recent ingestion (ie, less than one hour) of MDMA, we suggest a single dose of activated charcoal (1 g/kg; maximum dose 50 g) be administered. Charcoal should be withheld in patients who are sedated and may not be able to protect their airway, unless endotracheal intubation is performed first. However, endotracheal intubation should not be performed solely for the purpose of giving charcoal.

Butyrophenones (such as haloperidol and droperidol) interfere with heat dissipation, may prolong the QTc, and may reduce the seizure threshold. We suggest butyrophenones NOT be used to sedate patients with MDMA toxicity

Seizures, hyperthermia, and serotonin syndrome should be treated in standard fashion

386
Q

A 25 year old male is admitted with hyperventilation. He is sweating and appears nauseous. He says that he has ringing in his ears. Blood gases show that he has mixed acid-base disturbance

A. Lithium
B. Tricyclic antidepressants 
C. Carbon Monoxide 
D. Ecstasy 
E. Naloxone 
F. Paracetamol 
G. Desferrioxamine 
H. Methanol 
I. Acetylcysteine 
J. Salicylates 
K. Organophosphates
A

Salicylates
Sodium bicarbonate can be used to correct acidosis, but haemodialysis is the treatment of choice for severe poisoning and those with high salicylate levels.

Early symptoms of acute aspirin toxicity include tinnitus, vertigo, nausea, vomiting, and diarrhea; subsequent symptoms portending a more severe intoxication include altered mental status (ranging from agitation to lethargy), hyperpyrexia, noncardiac pulmonary edema, and coma. Early symptoms are typically present within one to two hours after a single acute ingestion. Hyperpnea is often observed in salicylate overdose. Salicylates stimulate the respiratory center directly, resulting in an early fall in the PCO2 and respiratory alkalosis. An anion-gap metabolic acidosis then follows, due primarily to the accumulation of organic acids, including lactic acid and ketoacids.
Treatment of salicylate intoxication is directed toward decreasing the fraction of uncharged (protonated) molecules, which is accomplished by increasing the systemic pH (ie, lowering the H+ ion concentration). This is referred to as “alkalinization” and is most easily accomplished by administering sodium bicarbonate. Increasing the systemic pH reduces the diffusion of salicylate anions into the central nervous system, as charged molecules do not easily diffuse across the blood-brain barrier. Alkalinization also “traps” salicylate anions within the renal tubule, preventing back-diffusion across the renal epithelium into the systemic circulation.

Indications for hemodialysis include the following:

●Altered mental status
●Pulmonary or cerebral edema
●Renal insufficiency that interferes with salicylate excretion
●Fluid overload that prevents the administration of sodium bicarbonate
●A serum salicylate concentration >100 mg/dL (7.2 mmol/L) in acute overdose
●Clinical deterioration despite aggressive and appropriate supportive care

Aspirin intoxication may decrease cerebral glucose concentrations despite a normal serum glucose. Therefore, we suggest that adults with salicylate poisoning who are hypoglycemic or manifest alterations in mental status, regardless of their serum glucose concentration, be treated with supplemental glucose

387
Q

An 80 year old man and his 79 year old wife were brought in after a neighbour found them collapsed in their home. On questioning the neighbour it was found that the couple had not been feeling well for a few weeks and had been complaining of nausea, headaches and dizziness

A. Lithium
B. Tricyclic antidepressants 
C. Carbon Monoxide 
D. Ecstasy 
E. Naloxone 
F. Paracetamol 
G. Desferrioxamine 
H. Methanol 
I. Acetylcysteine 
J. Salicylates 
K. Organophosphates
A

Carbon Monoxide
CO poisoning is most common during winter in cold climates, but it may occur in all seasons and environments. Smoke inhalation is responsible for most unintentional cases. Other potential sources of CO include poorly functioning heating systems, improperly vented fuel-burning devices (eg, kerosene heaters, charcoal grills, camping stoves, gasoline-powered electrical generators), and motor vehicles operating in poorly ventilated areas.
CO diffuses rapidly across the pulmonary capillary membrane and binds to the iron moiety of heme with approximately 240 times the affinity of oxygen. The degree of carboxyhemoglobinemia (COHb) is a function of the relative amounts of CO and oxygen in the environment, duration of exposure, and minute ventilation.
The clinical findings of CO poisoning are highly variable and largely nonspecific. Mild to moderate CO-intoxicated patients often present with constitutional symptoms, including headache (most common), malaise, nausea, and dizziness, and may be misdiagnosed with acute viral syndromes. In the absence of concurrent trauma or burns, physical findings in CO poisoning are usually confined to alterations in mental status, ranging from mild confusion to seizures and coma. A careful neurologic examination is crucial.
The diagnosis of CO poisoning is based upon a compatible history and physical examination in conjunction with an elevated carboxyhemoglobin level. Diagnosis requires quantification by cooximetry of a blood gas sample; standard pulse oximetry (SP02) is unable to distinguish between oxyhemoglobin and COHb. Blood PO2 measurements tend to be normal because PO2 reflects O2 dissolved in blood, and this process is not affected by CO

Hyperbaric oxygen therapy (HBO): to decrease the incidence and severity of delayed neurocognitive deficits following CO poisoning

388
Q

A depressed 30 year old woman was brought into A&E after being found by a friend. On examination she appears very drowsy with sinus tachycardia and wide dilated pupils. She has marked reflexes and extensor plantar responses. ECG shows a wide QRS interval

A. Lithium
B. Tricyclic antidepressants 
C. Carbon Monoxide 
D. Ecstasy 
E. Naloxone 
F. Paracetamol 
G. Desferrioxamine 
H. Methanol 
I. Acetylcysteine 
J. Salicylates 
K. Organophosphates
A

Tricylic Anti-Depressants
Both TCA od’s and ecstasy od’s can cause wide dilated pupils. Ecstasy is more likely to lead to agitation and TCA drowsiness.This points very clearly to TCA overdose (reflexes and widened QRS complexes). Also….remember that ecstasy may induce vasopressin secretion and an SIADH, with hyponatraemia
Symptoms and signs of TCA intoxication generally consist of vital sign abnormalities, mental status changes, seizures, and anticholinergic toxicity. Sinus tachycardia and hypotension are common. Sedation is the most typical alteration in mental status, but confusion, delirium, or hallucinations may occur. Anticholinergic toxicity commonly manifests as hyperthermia, flushing, dilated pupils that respond poorly to light, delirium, intestinal ileus, and urinary retention.
Cardiac conduction abnormalities are common in patients with TCA poisoning. These abnormalities can degenerate into ventricular tachycardia and ventricular fibrillation (VT and VF), which occur in approximately 4 percent of TCA overdose cases. The electrocardiogram (ECG) is a most valuable tool in determining the extent of TCA poisoning. The following signs suggest cardiotoxicity:
•Prolongation of the QRS >100 msec
•Abnormal morphology of the QRS (eg, deep, slurred S wave in leads I and AVL)

389
Q

A 45 year old farm worker is admitted complaining primarily of nausea and vomiting. On further questioning it is revealed that he also has a headache, hypersalivation and he is finding it hard to breathe. On examination the patient appears sweaty and has flaccid paresis of his limb muscles

A. Lithium
B. Tricyclic antidepressants 
C. Carbon Monoxide 
D. Ecstasy 
E. Naloxone 
F. Paracetamol 
G. Desferrioxamine 
H. Methanol 
I. Acetylcysteine 
J. Salicylates 
K. Organophosphates
A

Organophosphates
Acute toxicity from organophosphorus agents presents with manifestations of cholinergic excess. The dominant clinical features of acute cholinergic toxicity include bradycardia, miosis, lacrimation, salivation, bronchorrhea, bronchospasm, urination, emesis, and diarrhea.
Moderately to severely poisoned patients with markedly depressed mental status require 100 percent oxygen and immediate tracheal intubation. In addition, patients who appear mildly poisoned may rapidly develop respiratory failure. Succinylcholine should be avoided when performing rapid sequence intubation in patients with OP poisoning. Bradycardia and hypotension are usually present in moderate to severe poisonings. Adequate volume resuscitation with isotonic crystalloid should be performed concomitantly with other resuscitative and diagnostic efforts.
We recommend atropine therapy for all patients with moderate to severe cholinergic toxicity from OP or carbamate poisoning. Atropine is started at a dose of 2 to 5 mg IV for adults
We suggest that oxime therapy (eg, pralidoxime) be given to all patients with evidence of cholinergic toxicity, patients with neuromuscular dysfunction, or patients with exposures to organophosphorus agents known to cause delayed neurotoxicity

390
Q

Which of the above techniques can be used to test for all classes of drugs of abuse (DOA)?

A. Thin layer chromotography
B. Barbituates
C. Immunoassay
D. Benzodiazepines
E. Blood sample
F. Liquid chromotography
G. Urine sample
H. Drugs of abuse (DOA)
I. Stool sample
J. Paracetamol
K. Liver sample
A

Immunoassays are by far the most widely used method of initial testing for DOA in the clinical setting. Typically providing a result within minutes after sample application, immunoassays are able to detect low concentrations of a substance with a high degree of specificity. They are technically easy to perform and relatively inexpensive.

Immunoassays use antibodies that recognize a drug or metabolite. There are two general types of immunoassay techniques: noncompetitive and competitive. Noncompetitive assays recognize an analyte that is sandwiched between two antibodies, each of which recognizes a different site (or epitope) of the molecule. In a competitive immunoassay, non-labeled analyte in the patient’s serum or urine competes for a limited number of binding sites with a labeled version of the analyte that is provided with the immunoassay; displacement of the labeled analyte is the signal that suggests the presence of the drug.

The immunoassays most widely used for routine DOA testing are microparticle capture assays. These use a substance, often latex, that can collect in high concentration in a particular location, forming a colored band that can be visually read. The simplest such design uses an antidrug antibody with colored micro-particles and a capture zone consisting of the immobilized drug. Similar technology is used in other widely available point-of-care (POC) testing kits, such as those for urine pregnancy or streptococcal antigen. These are all typically conducted in a cassette containing the strip or matrix to which the biologic sample (and sometimes reagent) is added. After a period of minutes, the presence or absence of a colored band is interpreted as a positive or negative result.

391
Q

What sample is required for use with gas chromatography mass spectroscopy?

A. Thin layer chromotography
B. Barbituates
C. Immunoassay
D. Benzodiazepines
E. Blood sample
F. Liquid chromotography
G. Urine sample
H. Drugs of abuse (DOA)
I. Stool sample
J. Paracetamol
K. Liver sample
A

Blood sample
Chromatography refers to several related techniques whose common approach involves physical separation of substances. The multiple methods (such as liquid chromatography, thin layer chromatography, and gas chromatography) all include a combination of a mobile phase and a stationary phase. The stationary phase usually consists of fine particles, and the mobile phase is usually liquid or gas. The time required to traverse the length of the chromatography column or the distance traveled in a media during a set time (in thin layer chromatography) is consistent and highly reproducible.

392
Q

Colorimetric can be used to test for which drug commonly taken in overdose?

A. Thin layer chromotography
B. Barbituates
C. Immunoassay
D. Benzodiazepines
E. Blood sample
F. Liquid chromotography
G. Urine sample
H. Drugs of abuse (DOA)
I. Stool sample
J. Paracetamol
K. Liver sample
A

Paracetamol

393
Q

Which of the above techniques can be used to test for benzodiazepines and various antipsychotic drugs?

A. Thin layer chromotography
B. Barbituates
C. Immunoassay
D. Benzodiazepines
E. Blood sample
F. Liquid chromotography
G. Urine sample
H. Drugs of abuse (DOA)
I. Stool sample
J. Paracetamol
K. Liver sample
A

Liquid Chromatography

Chromatography offers a highly sensitive and specific technique for detecting drugs or metabolites. However, it requires highly trained laboratory staff, instruments, and commonly takes many hours to provide results, and thus it is usually not a methodology used for initial DOA testing. A notable exception is when an extended, comprehensive toxicology screening, which may detect hundreds of medications and drugs, is performed as an initial investigation.

394
Q

Which of the above techniques can be used to analyse samples of stool, liver and also urine?

A. Thin layer chromotography
B. Barbituates
C. Immunoassay
D. Benzodiazepines
E. Blood sample
F. Liquid chromotography
G. Urine sample
H. Drugs of abuse (DOA)
I. Stool sample
J. Paracetamol
K. Liver sample
A

Thin layer chromotography

395
Q

Which option is the best specimen for assessing long-term drug use?

A. MDMA
B. Saliva
C. Cocaine
D. THC
E. Blood
F. Urine
G. Morphine
H. Hair
I. Paracetamol
J. Forensics
K. Toxicology
A

Hair
Hair drug testing is a method that can detect drug use over a much longer period of time, and is often used for highly safety-critical positions where there is zero tolerance of illegal drug use. Standard hair follicle screen covers a period of 30 to 90 days. The growth of hair is usually at the rate of 0.5 inches per month. The hair sample is cut close to the scalp and 80 to 120 strands of hair are needed for the test. In the absence of hair on the head, body hair can be used as an acceptable substitute.[29] This includes facial hair, the underarms, arms, and legs or even pubic hair. Because body hair grows at a different rate than head hair, the timeframe changes, with scientists estimating that drug use can be detected in body hair for up to 12 months.

396
Q

Which drug is found in the most addict related deaths?

A. MDMA
B. Saliva
C. Cocaine
D. THC
E. Blood
F. Urine
G. Morphine
H. Hair
I. Paracetamol
J. Forensics
K. Toxicology
A

Morphine
The classic signs of opioid intoxication include: depressed mental status, decreased respiratory rate, decreased tidal volume, decreased bowel sounds, and miotic pupils. The best predictor of opioid poisoning is a respiratory rate <12.
Although suppression of respiratory drive is most prominent, opioid intoxication can also be complicated by hypothermia, coma, seizure, head trauma, aspiration pneumonia, and rhabdomyolysis.
In cases of suspected opioid overdose, we recommend the short-acting opioid antagonist naloxone be given (Grade 1B). While the intravenous (IV) route is preferred, naloxone may be given subcutaneously or intramuscularly if IV access is unavailable.
Activated charcoal and gastric emptying are almost never indicated in opioid poisoning. The large volume of distribution of the opioids precludes removal of a significant quantity of drug by hemodialysis.

397
Q

Which option is responsible for the analysis of samples for drugs and poisons?

A. MDMA
B. Saliva
C. Cocaine
D. THC
E. Blood
F. Urine
G. Morphine
H. Hair
I. Paracetamol
J. Forensics
K. Toxicology
A

Toxicology

398
Q

Which option is the best example of a quick, cheap, easy and non-invasive specimen which is likely to be adulterated for forensic drug analysis? Disadvantages include a small window of detection.

A. MDMA
B. Saliva
C. Cocaine
D. THC
E. Blood
F. Urine
G. Morphine
H. Hair
I. Paracetamol
J. Forensics
K. Toxicology
A

Saliva
Saliva / oral fluid-based drug tests can generally detect use during the previous few days. Is better at detecting very recent use of a substance.
Detection in saliva tests begins almost immediately upon use of the following substances, and lasts for approximately the following times:
Alcohol: 6-12 h
Marijuana: 1-24h

399
Q

Auto-Antibody against:

Cardiolipin, Apoplipoprotein H, anti-β2 glycoprotein

A

Anti-Phospholipid Syndrome

400
Q

Auto-Antibody against:

Smooth Muscle, Liver Kidney Microsomal, Soluble liver Antigen, Actin

A

Autoimmune Hepatitis

401
Q

Auto-Antibody against Rhesus

A

AIHA

402
Q

Auto-Antibody against cANCA / proteinase-3 (Cytoplasmic)

A

Wegeners/ Granulomatous Polyangitis

403
Q

Auto-Antibody against pANCA / MPO (Perinuclear)

A

Churg Strauss
Microscopic polyangiitis
Primary sclerosing cholangitis

404
Q

Auto-Antibody against platelet (GPIIb/IIIa)

A

ITP

405
Q

Auto-Antibody against:
Jo-1 (t-RNA synthetase) (Cytoplasmic
speckled)

A

Dermatomyositis

Polymyositis

406
Q

Auto-Antibody against:

SCL-70 (Topoisomerase) (Nucleolar), RNA Pol 1,2,3 (Nucleolar), Fibrillarin (Nucleolar)

A

Diffuse Systemic sclerosis

407
Q

Auto-Antibody against centromere

A

CREST

408
Q

Auto-Antibody against:

GBM (NC α3 COL4)

A

Goodpastures, HLA-DR2

409
Q

Auto-Antibody against:

Desmosomes (cadherins)

A

Pemphigus

410
Q

Auto-Antibody against:

Hemidesmosomes (collagen XVII)

A

Pemphigoid

411
Q

HLA-B8

A

PSC

412
Q

HLA-B27

A

Seronegative Arthritis:

  • Reactive/Reiter’s
  • Ank Spondy
  • Psoriatic
  • Enteropathic
413
Q

HLA-DR2

A

Goodpastures, Auto-Antibody against:

GBM (NC α3 COL4)

414
Q

HLA-DR3

A

Graves
SLE
PSC
Primary Sjögren’s

415
Q

HLA-DR4

A

RA

416
Q

HLA-DR3/4

A

T1DM

417
Q

HLA-DR5

A

Systemic Sclerosis

418
Q

HLA-DRB1

HLA-DQB1

A

Sarcoidosis

419
Q

HLA-DQ2

A

Coeliac

420
Q

HLA-DQ8

A

Coeliac

421
Q

Mutations:

JAK2

A

99% polycythaemia vera, 50% essential thrombocythaemia

422
Q

Mutations:

5q-

A

MDS with 5q deletion

423
Q

Mutations:

∆F508

A

Cystic Fibrosis

424
Q

Mutations:

HFE on Chr6

A

hereditary haemochromatosis

425
Q

Mutations:

Point mutation Glu-Val

A

Sickle Cell disease

426
Q

Mutations:

403 Arg-Gln in MHC

A

HOCM

427
Q
Mutations:
45XO
47XXY 
Trisomy 21
Trisomy 18
Trisomy 13
Monosomy 5p
Del 22q11
Del 7q21
Paternal 15q11-13
Maternal 15q11-13
CAG repeat Chr 4
CCG repeat Chr X
APC
A
45XO: Turner's
47XXY: Klinefelter's
Trisomy 21: Down's
Trisomy 18: Edward's
Trisomy 13: Patau's
Monosomy 5p: Cri du Chat
Del 22q11: Di George
Del 7q21: William's
Paternal 15q11-13: Prader-Willi
Maternal 15q11-13: Angelman's
CAG repeat Chr 4: Huntingdon's
CCG repeat Chr X: Fragile X
APC: Familial Adenomatous Polyposis Coli
428
Q
Translocation:
t(9;22)
t(15;17)
t(8;21)
t(8;14)
t(11;14)
t(11;18)
t(2;5)
t(11;22)
A
t(9;22): Philedelphia Chr, Bcr-ABL, CML
t(15;17): APML
t(8;21): EPO, AML with maturation (M2)
t(8;14): c-myc -> Burkitt's 
t(11;14): cyclin-D1 (Bcl-1) mantle Zone Lymphoma
t(11;18): MALToma/Marginal Zone Lymphoma
t(2;5): Anaplastic large Cell Lymphoma
t(11;22): Ewing's Sarcoma
429
Q
A 50-year-old woman with known diabetes has a routine blood test which demonstrates the following:
Na 130 (135–145mmol/L)
K 4.1 (3.5–5.0mmol/L)
Urea 4.2 (3.0–7.0mmol/L)
Glucose 3.1 (2.2–5.5mmol/L)
Osmolality 283 (275–295mOsm/kg)
A Ethanol
B SIADH
C Frusemide
D Chronic kidney disease
E Conn’s syndrome
F Diarrhoea
G Congestive cardiac failure
H Addison’s disease
I Hyperlipidaemia
A

Hyperlipidaemia: Pseudo-hyponatraemia can occur in patients with hyperlipidaemia
(I) or hyperproteinaemia. In such states, lipids and proteins will occupy a high proportion of the total serum volume. Although the sodium concentration in serum water is in fact normal, a lower sodium concentration will be detected due to dilution by increased lipids and protein molecules. As a consequence, there is an apparent hyponatraemia. A spurious result due to the sample being taken from the drip arm can also cause pseudo-hyponatraemia

430
Q

A 45-year-old man is seen by his specialist. His last blood and urine tests demonstrated the following:

Na 129 (135–145mmol/L)
K 5.5 (3.5–5.0mmol/L)
Urea 8.2 (3.0–7.0mmol/L)
Glucose 4.2 (2.2–5.5mmol/L)
Osmolality 265 (275–295mOsm/kg)
Urine osmolality 26mOsm/kg
A Ethanol
B SIADH
C Frusemide
D Chronic kidney disease
E Conn’s syndrome
F Diarrhoea
G Congestive cardiac failure
H Addison’s disease
I Hyperlipidaemia
A

Chronic kidney disease: A true hyponatraemic state occurs when the osmolality is simultaneously low. Chronic kidney disease (CKD; D) results in urinary protein loss and hence oedema. A reduced circulating volume causes activation of the renin–angiotensin system, thereby raising blood sodium levels.
This in turn causes release of antidiuretic hormone (ADH) from the posterior pituitary leading to water retention and hypervolaemic hyponatraemia. Water reabsorption in the renal tubules increases urine osmolality (>20mmol/L indicates a renal cause of hyponatraemia). CKD is also associated with hyperkalaemia and azotaemia.

431
Q

A 30-year-old woman visits her GP due to pigmentation of her palmar creases. Two weeks later the following blood and urine tests are received:

Na 128 (135–145mmol/L)
K 5.9 (3.5–5.0mmol/L)
Urea 5.2 (3.0–7.0mmol/L)
Glucose 1.8 (2.2–5.5mmol/L)
Osmolality 264 (275–295mOsm/kg)
Urine osmolality 24mOsm/kg
A Ethanol
B SIADH
C Frusemide
D Chronic kidney disease
E Conn’s syndrome
F Diarrhoea
G Congestive cardiac failure
H Addison’s disease
I Hyperlipidaemia
A

Addison’s disease (H) is also known as primary adrenal insufficiency
(reduced aldosterone and cortisol); consequently there is a rise in the production of adrenocorticotropic hormone (ACTH). An impaired synthesis of aldosterone reduces reabsorption of sodium and increases excretion of potassium in the distal convoluted tubule and collecting ducts of the kidney; this leads to a simultaneous hyponatraemia and hyperkalaemia. Reduced cortisol production causes hypoglycaemia due to impaired gluconeogenesis. Clinical features of Addison’s disease include hyperpigmentation, postural hypotension and weight loss.

432
Q
A 30-year old woman is seen by her GP after a 5-day episode of productive cough and lethargy. The GP notes dullness on percussion of the patient’s left lower lung. Blood and urine tests reveal the following:
Na 128 (135–145mmol/L)
K 4.1 (3.5–5.0mmol/L)
Urea 3.5 (3.0–7.0mmol/L)
Glucose 3.2 (2.2–5.5mmol/L)
Osmolality 265 (275–295mOsm/kg)
Urine osmolality 285mOsm/kg
A Ethanol
B SIADH
C Frusemide
D Chronic kidney disease
E Conn’s syndrome
F Diarrhoea
G Congestive cardiac failure
H Addison’s disease
I Hyperlipidaemia
A

The syndrome of inappropriate ADH secretion (B; SIADH) results from the excess release of ADH. In this case the clinical features suggest pneumonia is the cause, but the aetiologies of SIADH are numerous, including malignancy, meningitis and drugs (carbamazepine). Criteria to diagnose SIADH include the following:
•Hyponatraemia 100mmol/L
•High urine sodium >20mmol/L
•Euvolaemia
•No adrenal, renal or thyroid dysfunction

Characteristically the urine osmolality is inappropriately high; in normal circumstances if the plasma osmolality is low, the urine osmolality will stop rising as reduced ADH secretion prevents water retention. As a rule of thumb in SIADH, urine osmolality is greater than plasma osmolality.

433
Q

A 63-year-old man with chronic obstructive pulmonary disease (COPD) sees his GP due to oedematous ankles. His blood and urine tests show the following:

Na 130 (135–145mmol/L)
K 4.4 (3.5–5.0mmol/L)
Urea 4.2 (3.0–7.0mmol/L)
Glucose 3.1 (2.2–5.5mmol/L)
Osmolality 268 (275–295mOsm/kg)
Urine osmolality 16–mmol/LmOsm/kg
A Ethanol
B SIADH
C Frusemide
D Chronic kidney disease
E Conn’s syndrome
F Diarrhoea
G Congestive cardiac failure
H Addison’s disease
I Hyperlipidaemia
A

Congestive cardiac failure (G) may present with shortness of breath, pitting peripheral oedema and/or raised jugular venous pulse (JVP). In this scenario, shortness of breath may be masked by the patient’s COPD.
The clinical picture together with the blood result demonstrating a low sodium and low osmolality suggest a hypervolaemic hyponatraemia.
This scenario can be differentiated from hypervolaemia as a result of CKD (D) by the urine osmolality, which is less than 20mmol/L in this instance, thereby suggesting a non-renal cause for the hyponatraemia.

Ethanol (A) may cause hyponatraemia in the context of a raised plasma osmolality (>295mmol/L). Other low molecular weight solutes that can cause hyponatraemia (when osmolality is raised) include mannitol and glucose.

Frusemide (C) and other diuretics cause a hypovolaemic hyponatraemia. As well as a low plasma sodium and osmolality, the urine osmolality will be greater than 20mmol/L, signifying a renal cause of hyponatraemia.

Conn’s syndrome (E), also known as primary aldosteronism, results from an aldosterone-producing adenoma producing excess aldosterone. Biochemical (and concurrent clinical) features include hypernatraemia
(hypertension) and hypokalaemia (paraesthesia, tetany and weakness).

Diarrhoea (F) leads to a hypovolaemic hyponatraemia (as does vomiting). Plasma sodium and osmolality will be low and urine osmolality will be lower than 20mmol/L indicating an extra-renal cause of
hyponatraemia.

434
Q

A 15-year-old boy presents to accident and emergency with loss of consciousness. His blood sugars are found to be extremely low. Blood tests demonstrate the following:

Na 138 (135–145mmol/L)
K 3.0 (3.5–5.0mmol/L)
Urea 4.2 (3.0–7.0mmol/L)
Creatinine 74 (60–120mmol/L)
pH 7.48 (7.35–7.45)
HCO3 31 (22–28mmol/L)
A Spurious sample
B Anorexia
C Diarrhoea
D Renal tubular acidosis
E Insulin overdose
F Bartter syndrome
G Frusemide
H Renal failure
I ACE inhibitors
A
Insulin overdose
Insulin overdose (E) in a diabetic patient will cause a redistributive hypokalaemia and concurrent metabolic alkalosis. Insulin causes a shift of potassium ions from the extracellular space to the intracellular space, thereby lowering blood potassium levels. Metabolic alkalosis can also cause a redistributive hypokalaemia; a reduced hydrogen ion concentration in the blood causes increased intracellular hydrogen ion loss to increase extracellular levels via Na+/H+ATPase; potassium ions therefore diffuse intracellularly to maintain the electrochemical potential. Adrenaline and re-feeding syndrome also cause redistributive hypokalaemia
435
Q

A 64-year-old man who is an inpatient on the Care of the Elderly ward is found to have the following blood results:

Na 136 (135–145mmol/L)
K 5.5 (3.5–5.0mmol/L)
Urea 14.4 (3.0–7.0mmol/L)
Creatinine 165 (60–120mmol/L)
pH 7.44 (7.35–7.45)
HCO3 27  (22–28mmol/L)
A Spurious sample
B Anorexia
C Diarrhoea
D Renal tubular acidosis
E Insulin overdose
F Bartter syndrome
G Frusemide
H Renal failure
I ACE inhibitors
A

Renal failure (H) can lead to hyperkalaemia secondary to reduced distal renal delivery of sodium ions. As a consequence, there is reduced exchange of potassium ions via the Na/K ATPase pump in the collecting duct, which thereby leads to accumulation of potassium ions in the blood and hence hyperkalaemia. An increase in aldosterone release will initially cause a compensatory loss of potassium ions; as renal failure progresses, this homeostatic mechanism will become decompensated and hyperkalaemia will result. Renal failure will also be reflected in the deranged urea and creatinine levels due to reduced excretion

436
Q

A 16-day-old baby girl is found to have low blood pressure. Urinary calcium levels are found to be elevated. Blood tests demonstrate the following results:

Na 138 (135–145mmol/L)
K 2.8 (3.5–5.0mmol/L)
Urea 3.4 (3.0–7.0mmol/L)
Creatinine 62 (60–120mmol/L)
pH 7.51 (7.35–7.45)
HCO3 33 (22–28mmol/L)
A Spurious sample
B Anorexia
C Diarrhoea
D Renal tubular acidosis
E Insulin overdose
F Bartter syndrome
G Frusemide
H Renal failure
I ACE inhibitors
A

Bartter syndrome (F) is an autosomal recessive condition due to a defect in the thick ascending limb of the loop of Henle. It is characterized by hypokalaemia, alkalosis and hypotension. The condition may also lead to increased calcium loss via the urine (hypercalcuria) and the kidneys (nephrocalcinosis). Various genetic defects have been discovered; neonatal Bartter syndrome is due to mutations in either the NKCC2 or ROMK genes. In the associated milder Gitelman syndrome, the potassium transporting defect is in the distal convoluted tubule of the kidney.

437
Q

A 32-year-old man presents to his GP for a check-up. His serum aldosterone is
found to be low. Blood tests reveal the following:

Na 140 (135–145mmol/L)
K 5.6 (3.5–5.0mmol/L)
Urea 5.3 (3.0–7.0mmol/L)
Creatinine 92 (60–120mmol/L)
pH 7.38 (7.35–7.45)
HCO3 24  (22–28mmol/L)
A Spurious sample
B Anorexia
C Diarrhoea
D Renal tubular acidosis
E Insulin overdose
F Bartter syndrome
G Frusemide
H Renal failure
I ACE inhibitors
A

ACE inhibitors (I) will lead to hyperkalaemia due to reduced potassium excretion. ACE inhibitors antagonize the effect of angiotensin converting enzyme, the enzyme which catalyzes the production of angiotensin II from angiotensin I. A decreased level of angiotensin II reduces the production of aldosterone in the adrenal glands, a key hormone causing the excretion of potassium. Other causes of reduced excretion of potassium include Addison’s disease, renal failure and potassium sparing diuretics.

438
Q

A 68-year-old woman on the Care of the Elderly ward is found to have the
following blood results:

Na 138 (135–145mmol/L)
K 3.0 (3.5–5.0mmol/L)
Urea 4.2 (3.0–7.0mmol/L)
Creatinine 74 (60–120mmol/L)
pH 7.31 (7.35–7.45)
HCO3 28  (22–28mmol/L)
A Spurious sample
B Anorexia
C Diarrhoea
D Renal tubular acidosis
E Insulin overdose
F Bartter syndrome
G Frusemide
H Renal failure
I ACE inhibitors
A

Renal tubular acidosis (D) occurs when there is a defect in hydrogen ion secretion into the renal tubules. Potassium secretion into the renal tubules therefore increases to balance sodium reabsorption. This results in hypokalaemia with acidosis. Renal tubular acidosis is classified according to the location of the defect: type 1 (distal tubule), type 2 (proximal tubule), type 3 (both distal and proximal tubules). Type 4 results from a defect in the adrenal glands and is included in the classification as it results in a metabolic acidosis and hyperkalaemia.

Spurious sampling (A) of blood results in hyperkalaemia. Excessive vacuuming of blood or using too fine a needle can cause haemolysis, leading to a raised potassium.

Anorexia (B) will result in reduced potassium intake and hence hypokalaemia. Other causes of reduced potassium intake include dental problems, alcoholism and total parental nutrition deficient in potassium.

Diarrhoea (C) results in hypokalaemia due to increased gastrointestinal losses of potassium. Other causes of increased gastrointestinal loss of potassium include villous adenoma and VIPoma.

Frusemide (G) intake leads to hypokalaemia secondary to increased renal loss of potassium. This occurs due to increased collecting duct permeability and hence potassium loss.

439
Q

pH 7.31 (7.35–7.45)
pO2 7.6 (10.6–13kPa)
pCO2 8.2 (4.7–6.0kPa)
HCO3 26 (22–28mmol/L)

A Metabolic acidosis
B Metabolic acidosis with respiratory compensation
C Metabolic alkalosis
D Metabolic alkalosis with respiratory compensation
E Respiratory acidosis
F Respiratory acidosis with metabolic compensation
G Respiratory alkalosis
H Respiratory alkalosis with metabolic compensation
I Mixed metabolic and respiratory
acidosis

A
Respiratory acidosis (E) is defined by a low pH (acidosis) together with a high pCO2, due to carbon dioxide retention secondary to a pulmonary, neuromuscular or physical causes. There is no metabolic compensation in this case, suggesting this is an acute pathology; a compensatory metabolic rise in HCO3 from the kidneys can take hours or days. 
This patient is also hypoxic with a low pO2. Causes of an acute respiratory acidosis include an acute exacerbation of asthma, foreign body obstruction and cardiac arrest
440
Q

pH 7.36 (7.35–7.45)
pO2 14.2 (10.6–13kPa)
pCO2 4.1 (4.7–6.0kPa)
HCO3 14 (22–28mmol/L)

A Metabolic acidosis
B Metabolic acidosis with respiratory compensation
C Metabolic alkalosis
D Metabolic alkalosis with respiratory compensation
E Respiratory acidosis
F Respiratory acidosis with metabolic compensation
G Respiratory alkalosis
H Respiratory alkalosis with metabolic compensation
I Mixed metabolic and respiratory
acidosis

A

Metabolic acidosis with respiratory compensation (B) occurs when pH is low (acidosis) and HCO3 is low with concurrent respiratory compensation by decreasing pCO2. The anion gap can differentiate between causes of metabolic acidosis (anion gap =[Na++ K+] – [Cl−+ HCO3]; normal range between 10 and 18mmol/L). Causes of a raised anion gap can be remembered by the mnemonic MUDPILES: methanol/metformin, uraemia, diabetic ketoacidosis,paraldehyde, iron, lactate, ethanol and salicylates. Causes of a normal anion gap include diarrhoea, Addison’s disease and renal tubular acidosis

441
Q

pH 7.45 (7.35–7.45)
pO2 10.2 (10.6–13kPa)
pCO2 7.2 (4.7–6.0kPa)
HCO3 32 (22–28mmol/L

A Metabolic acidosis
B Metabolic acidosis with respiratory compensation
C Metabolic alkalosis
D Metabolic alkalosis with respiratory compensation
E Respiratory acidosis
F Respiratory acidosis with metabolic compensation
G Respiratory alkalosis
H Respiratory alkalosis with metabolic compensation
I Mixed metabolic and respiratory
acidosis

A

Metabolic alkalosis with respiratory compensation (D) occurs when pH is high (alkalosis) and HCO3 is high with a compensatory reduction in respiratory effort that increases pCO2. As respiratory effort is reduced there is the possibility of the patient becoming hypoxic. Causes of metabolic alkalosis include vomiting, potassium depletion secondary to diuretic use, burns and sodium bicarbonate ingestion. Respiratory compensation increase serum CO2 concentration, which reduces pH back towards normal

442
Q

pH 7.30 (7.35–7.45)
pO2 8.2 (10.6–13kPa)
pCO2 7.2 (4.7–6.0kPa)
HCO3 19 (22–28mmol/L)

A Metabolic acidosis
B Metabolic acidosis with respiratory compensation
C Metabolic alkalosis
D Metabolic alkalosis with respiratory compensation
E Respiratory acidosis
F Respiratory acidosis with metabolic compensation
G Respiratory alkalosis
H Respiratory alkalosis with metabolic compensation
I Mixed metabolic and respiratory
acidosis

A

Mixed metabolic and respiratory acidosis (I) occurs when there is a low pH and a simultaneous high pCO2 and low HCO3. In the case of a mixed metabolic and respiratory acidosis, the metabolic acidosis component may be due to conditions such as uraemia, ketones produced as a result of diabetes mellitus or renal tubular acidosis. The respiratory acidosis component may be due to any cause of respiratory failure. Hence, this mixed picture may occur in a COPD patient with concurrent diabetes mellitus.

443
Q

pH 7.49 (7.35–7.45)
pO2 13.6 (10.6–13kPa)
pCO2 4.1 (4.7–6.0kPa)
HCO3 23 (22–28mmol/L)

A Metabolic acidosis
B Metabolic acidosis with respiratory compensation
C Metabolic alkalosis
D Metabolic alkalosis with respiratory compensation
E Respiratory acidosis
F Respiratory acidosis with metabolic compensation
G Respiratory alkalosis
H Respiratory alkalosis with metabolic compensation
I Mixed metabolic and respiratory
acidosis

A

Respiratory alkalosis (G) is biochemically defined by a raised pH (alkalosis) and reduced pCO2. As previously mentioned, metabolic compensation can take hours or days to occur. The primary pathology causing respiratory alkalosis is hyperventilation which causes increased CO2 to be lost via the lungs. Causes of hyperventilation may be due to central nervous system disease, for example stroke. Other causes of hyperventilation include anxiety (panic attack), pulmonary embolism and drugs (salicylates).

444
Q
AST 65 (3–35IU/L)
ALT 72 (3–35IU/L)
GGT 82 (11–51IU/L)
ALP 829 (35–51IU/L)
Total bilirubin 234 (3–17μmol/L)
Conjugated bilirubin 63 (1.0–5.1μmol/L)
A Alcohol abuse
B Gilbert’s syndrome
C Gallstones
D Dublin–Johnson syndrome
E Non-alcoholic fatty liver disease
F Crigler–Najjar syndrome
G Alcoholic liver disease
H Paracetamol poisoning
I Hepatocellular carcinoma
A

Gallstones (C) may be composed of cholesterol, bilirubin or mixed in nature. The major complication of gallstones is cholestasis, whereby the flow of bile is blocked from the liver to the duodenum. This results in right upper quadrant abdominal pain, nausea and vomiting. Other causes of cholestasis include primary biliary cirrhosis, primary sclerosing cholangitis and abdominal masses compressing the biliary tree. Biochemically, cholestasis is defined by rises in GGT and ALP (obstructive picture) that are greater than the rises in AST and ALT

445
Q
AST 32 (3–35IU/L)
ALT 29 (3–35IU/L)
GGT 34 (11–51IU/L)
ALP 53 (35–51IU/L)
Total bilirubin 36 (3–17μmol/L)
Conjugated bilirubin 3.4 (1.0–5.1μmol/L)
A Alcohol abuse
B Gilbert’s syndrome
C Gallstones
D Dublin–Johnson syndrome
E Non-alcoholic fatty liver disease
F Crigler–Najjar syndrome
G Alcoholic liver disease
H Paracetamol poisoning
I Hepatocellular carcinoma
A

Gilbert’s syndrome (B) is an autosomal dominant condition in which there is a mutation in the enzyme UDP glucuronosyl transferase which reduces conjugation of bilirubin in the liver. As a consequence patients experience mild, intermittent jaundice. Jaundice in patients with Gilbert’s syndrome may be precipitated by infection or starved states.
Biochemistry will reveal that all liver function tests are normal apart from an isolated raised unconjugated bilirubin level, while conjugated bilirubin is within the normal range.

446
Q
AST 1259 (3–35IU/L)
ALT 1563 (3–35IU/L)
GGT 73 (11–51IU/L)
ALP 46 (35–51IU/L)
Total bilirubin 15.2 (3–17μmol/L)
Conjugated bilirubin 4.2 (1.0–5.1μmol/L)
A Alcohol abuse
B Gilbert’s syndrome
C Gallstones
D Dublin–Johnson syndrome
E Non-alcoholic fatty liver disease
F Crigler–Najjar syndrome
G Alcoholic liver disease
H Paracetamol poisoning
I Hepatocellular carcinoma
A

Non-alcoholic fatty liver disease (NAFLD; E) is due to fatty deposits in the liver (steatosis), but where the underlying cause is not due to alcohol. In such circumstances, aetiological factors include obesity, diabetes, parenteral feeding and inherited metabolic disorders (glycogen storage disease type 1). NAFLD may present with right upper quadrant pain or may be asymptomatic. Liver function tests will reveal raised AST and ALT levels (AST:ALT ratio

447
Q
AST 2321 (3–35IU/L)
ALT 2562 (3–35IU/L)
GGT 62 (11–51IU/L)
ALP 182 (35–51IU/L)
Total bilirubin 14 (3–17μmol/L)
Conjugated bilirubin 3.4 (1.0–5.1μmol/L
A Alcohol abuse
B Gilbert’s syndrome
C Gallstones
D Dublin–Johnson syndrome
E Non-alcoholic fatty liver disease
F Crigler–Najjar syndrome
G Alcoholic liver disease
H Paracetamol poisoning
I Hepatocellular carcinoma
A

Paracetamol poisoning (H) is a common cause of acute liver failure. The clinical features of acute liver failure reflect the diminished synthetic and metabolic functioning of the liver. Characteristics include reduced blood sugar level, metabolic acidosis, increased tendency to bleed and hepatic encephalopathy. Biochemical tests will reveal AST and ALT levels greater than 1000IU/L. AST and ALT levels will be greater than GGT and ALP levels, reflecting the hepatic rather than obstructive picture of the pathology.

448
Q
AST 34 (3–35IU/L)
ALT 32 (3–35IU/L)
GGT 134 (11–51IU/L)
ALP 123 (35–51IU/L)
A Alcohol abuse
B Gilbert’s syndrome
C Gallstones
D Dublin–Johnson syndrome
E Non-alcoholic fatty liver disease
F Crigler–Najjar syndrome
G Alcoholic liver disease
H Paracetamol poisoning
I Hepatocellular carcinoma
A

Alcohol abuse (A) can lead to deranged liver function tests. In the absence of underlying liver disease, biochemical investigation may demonstrate an isolated rise in GGT. There may also be mild elevations in AST and ALT, reflecting mild hepatic damage. Haematology results will show a macrocytic picture due to toxic effects of alcohol on the bone marrow. Isolated raised GGT levels may also occur due to the consumption of enzyme-inducing drugs such as phenytoin, carbamazepine and phenobarbitone.

Dubin–Johnson syndrome (D) is an autosomal recessive disorder that results in a raised conjugated bilirubin level due to reduced secretion of conjugated bilirubin into the bile. AST and ALT levels are normal.

Crigler–Najjar syndrome (F) is a hereditary disease resulting in either complete (type 1) or partial (type 2) reduction in the conjugating enzyme UDP glucuronosyl transferase causing an unconjugated hyperbilirubinaemia.

Alcoholic liver disease (ALD; G) occurs in three stages: alcoholic steatosis, alcoholic hepatitis and eventually cirrhosis. GGT, AST and ALT will be markedly elevated (AST:ALT ratio >2).

Hepatocellular carcinoma (HCC; I) occurs as a result of underlying cirrhosis. Raised α-fetoprotein levels can be indicative of HCC. Deranged liver function tests will reflect the underlying pathology

449
Q

A 38-year-old woman is referred by her GP to the Endocrine Clinic for further tests after experiencing fatigue and orthostatic hypotension. After a positive short synACTHen test, a long synACTHen test reveals a cortisol of 750nmol/L after 24 hours.

A Prolactinoma
B Grave’s disease
C Addison’s disease
D Schmidst’s syndrome
E Acromegaly
F Conn’s syndrome
G Kallman’s syndrome
H Secondary hypoaldosteronism
I De Quervain’s thyroiditis
A

Addison’s disease (C) is caused by primary adrenal insufficiency resulting in a reduced production of cortisol and aldosterone. It is diagnosed using the synACTHen test. In the short synACTHen test, baseline plasma cortisol is measured at 0 minutes, the patient is given 250μg of synthetic ACTH at 30 minutes and plasma cortisol is rechecked at 60 minutes; if the final plasma cortisol is

450
Q

A 48-year-old man visits his GP complaining of muscle pain and weakness. He is found to have raised blood pressure. Blood tests reveal Na 149 (135–145mmol/L) and K 3.1 (3.5–5.0mmol/L).

A Prolactinoma
B Grave’s disease
C Addison’s disease
D Schmidst’s syndrome
E Acromegaly
F Conn’s syndrome
G Kallman’s syndrome
H Secondary hypoaldosteronism
I De Quervain’s thyroiditis
A

Conn’s syndrome (F) is defined as primary hyperaldosteronism secondary to an aldosterone-producing adrenal adenoma. As a result of the high aldosterone levels produced there will be an increased excretion of potassium and reabsorption of sodium, leading to hypokalaemia and hypernatraemia. The increased delivery of sodium to the juxtaglomerular apparatus causes renin levels to be reduced. Plasma aldosterone will either be raised or inappropriately normal (as ACTH is suppressed, aldosterone should physiologically be reduced).

451
Q

A 39-year-old woman sees an endocrinologist due to recent onset galactorrhoea. She denies recent child birth. Thyroid function tests are found to be normal

A Prolactinoma
B Grave’s disease
C Addison’s disease
D Schmidst’s syndrome
E Acromegaly
F Conn’s syndrome
G Kallman’s syndrome
H Secondary hypoaldosteronism
I De Quervain’s thyroiditis
A

A prolactinoma (A) is a prolactin-producing tumour and is the most prevalent pituitary tumour. Prolactinomas are classified according to size: microprolactinoma 10mm diameter. The clinical consequences of prolactinoma are divided into, first, those that occur as a result of increased prolactin production and, second, effects due to the mass effect of the tumour. Hormonal effects of prolactin include amenorrhoea, galactorrhoea and gynaecomastia in males. Mass effects of the tumour can lead to compression of pituitary cells producing other hormones such as thyroid stimulating hormone, growth hormone and ACTH.

452
Q

A 46-year-old man is seen by his GP after experiencing tremors, heat intolerance and weight loss. His wife complained that his eyes were bulging. Blood tests reveal T3 (1.2–3.0nmol/L), T4 raised (70–140nmol/L), TSH 0(0.5–5.7mIU/L).

A Prolactinoma
B Grave’s disease
C Addison’s disease
D Schmidst’s syndrome
E Acromegaly
F Conn’s syndrome
G Kallman’s syndrome
H Secondary hypoaldosteronism
I De Quervain’s thyroiditis
A

Grave’s disease (B) is an autoimmune condition resulting in the production of TSH-receptor antibodies, leading to elevated levels of T3 and T4. TSH levels will therefore be suppressed as a result of negative feedback.
Clinical features will include exophthalmos, pretibial myxoedema, diffuse thyroid enlargement as well as other systemic features of hyperthyroiditis (tremor, excess sweating, heat intolerance and unintentional weight loss). There is a strong association with other autoimmune conditions such as vitiligo and type 1 diabetes mellitus

453
Q

A 45-year-old woman is referred to an endocrinologist due to the appearance
of enlarged hands and feet as well as a protruding jaw. After conducting an oral
glucose tolerance test, growth hormone levels are found to be 5mU/L (

A

Acromegaly (E) is caused by the increased secretion of growth hormone as a result of a pituitary adenoma (rarely there may be ectopic production). Serum growth hormone levels are not a useful marker of acromegaly due to its pulsatile release from the pituitary. The diagnostic test for acromegaly is the oral glucose tolerance test with synchronous growth hormone measurement: 75mg of glucose is administered to the patient; if growth hormone levels are not suppressed to below 2mU/L, a diagnosis of acromegaly is made.

Schmidst’s syndrome (D), also known as autoimmune polyendocrine syndrome type 2, is associated with Addison’s disease, hypothyroidism and type 1 diabetes mellitus.

Kallman’s syndrome (G) is a genetic disorder that results in hypogonadotropic hypogonadism. As a consequence there is reduced production of LH and FSH in the pituitary. Anosmia is an associated feature.

454
Q

A 25 year old male with a history of tuberculosis presenting with a plasma osmolality of 205mmol/l, potassium of 6.3mmol/l and sodium of 115mmol/l.

A. Diarrhoea
B. Rhabdomyolysis
C. SIADH
D. Vomiting
E. Acute Renal Failure
F. Diabetic ketoacidosis
G. Alcohol abuse
H. Renal tubular acidosis
I. Artifactual
J. Diuretic use
K. Haemorrhage
L. Cushing’s syndrome
M. Addison's disease
A

SIADH - other names: Schwartz-Bartter syndrome, SIAD—syndrome of inappropriate antidiuresis) is characterized by excessive release of antidiuretic hormone from the posterior pituitary gland or another source. The result is often dilutional hyponatremia in which the plasma sodium levels are lowered and total body fluid is increased. EUVOLAEMIC HYPONATRAEMIA
ADH acts in the distal portion of the renal tubule (Distal Convoluted Tubule) as well as on the collecting duct and causes the retention of water, but not solute. Hence, ADH activity effectively dilutes the blood (decreasing the concentrations of solutes such as sodium), causing hyponatremia; this is compounded by the fact that the body responds to water retention by decreasing aldosterone, thus allowing even more sodium wasting. For this reason, a high urinary sodium excretion will be seen.

This causes dilutional hyponatremia and all the consequences associated with that condition: headache, nausea, vomiting, and confusion may ensue. Severe hyponatremia may ca
use convulsions or coma.

Causes: Meningitis, Head injury (subarachnoid haemorrhage), small cell lung cancer, brain abscess, pneumonia, lung abscess, Guillain-Barre syndrome, hypothyroidism, sarcoidosis, drugs (amytriptyline, morphine, vincristine, oxytocin, valproic acid, cyclophosphamide, carbemazepine, ciprofloxacin)

Normal ranges:
Serum osmolality is 275-295mmol/kg
Na: 135-145 mmol/L

Hyponatraemia: determine whether true hyponatraemia using serum osmolality

  • If high: glucose/mannitol, infusion, ethanol
  • If normal: Spurious, drip arm sample, pseudohyponatraemia (hyperlipidaemia, paraproteinaemia)
  • If low = true hyponatraemia

TURP syndrome causes hyponatraemia as increased water reabsorption through the damaged prostate.

Treating Hyponatraemia:
Clinical Assessment
- Hypovolaemic: diarrhoea, vomiting, diuretics, salt losing nephropathy - Tx: ?fluid restoring if necessary
- Euvolaemic: Hypothyroidism, adrenal insufficiency, SIADH. Ix: TFTs, short synACTHen test, urine osmolality
- Hypervolaemic: Cardiac failure, cirrhosis, nephrotic syndrome. Ix: overload?

455
Q

A 76 year woman with known congestive cardiac failure presenting with digoxin toxicity

A. Diarrhoea
B. Rhabdomyolysis
C. SIADH
D. Vomiting
E. Acute Renal Failure
F. Diabetic ketoacidosis
G. Alcohol abuse
H. Renal tubular acidosis
I. Artifactual
J. Diuretic use
K. Haemorrhage
L. Cushing’s syndrome
M. Addison's disease
A

Diuretic Use
The classic features of digoxin toxicity are nausea, vomiting, diarrhea, abdominal pain, headache, dizziness, confusion, delirium, vision disturbance (blurred or yellow vision). It is also associated with cardiac disturbances including irregular heartbeat, ventricular tachycardia, ventricular fibrillation, sinoatrial block and AV block.

People with heart failure who take digoxin are commonly given medications called diuretics, which remove excess fluid from the body. Many diuretics can cause potassium loss. Low levels of potassium in the body increase the risk of digitalis toxicity. Digitalis toxicity may also result in persons who take the drug and who have low levels of magnesium in the body.

456
Q

Following a severe car crash, a patient’s ECG shows a broad QRS complex with peaked T waves.

A. Diarrhoea
B. Rhabdomyolysis
C. SIADH
D. Vomiting
E. Acute Renal Failure
F. Diabetic ketoacidosis
G. Alcohol abuse
H. Renal tubular acidosis
I. Artifactual
J. Diuretic use
K. Haemorrhage
L. Cushing’s syndrome
M. Addison's disease
A

Rhabdomyolysis
When muscle is damaged, a protein called myoglobin is released into the bloodstream. It is then filtered out of the body by the kidneys. Myoglobin breaks down into substances that can damage kidney cells.

Rhabdomyolysis may be caused by injury or other any condition that damages skeletal muscle.

Problems that may lead to this disease include:

  • Trauma or crush injuries
  • Use of drugs such as cocaine, amphetamines, statins, heroin, or PCP
  • Genetic muscle diseases
  • Extremes of body temperature
  • Ischemia or death of muscle tissue
  • Low phosphate levels
  • Seizures or muscle tremors
  • Severe exertion, such as marathon running or calisthenics
  • Lengthy surgical procedures
  • Severe dehydration

Breakdown products of damaged muscle cells are released into the bloodstream; some of these, such as the protein myoglobin, are harmful to the kidneys and may lead to kidney failure. The severity of the symptoms, which may include muscle pains, vomiting, and confusion, depends on the extent of muscle damage and whether kidney failure develops. The muscle damage may be caused by physical factors (e.g., crush injury, strenuous exercise, medications, drug abuse, and infections). Some people have a hereditary muscle condition that increases the risk of rhabdomyolysis. The diagnosis is usually made with blood tests and urinalysis. The mainstay of treatment is generous quantities of intravenous fluids, but may include dialysis or hemofiltration in more severe cases

457
Q

A 53 year old man in hospital following a minor operation was observed to have a potassium of 7.0mmol/l on a routine blood test but clinically well. A repeat test 4 hours later was 4.0mmol/l.

A. Diarrhoea
B. Rhabdomyolysis
C. SIADH
D. Vomiting
E. Acute Renal Failure
F. Diabetic ketoacidosis
G. Alcohol abuse
H. Renal tubular acidosis
I. Artifactual
J. Diuretic use
K. Haemorrhage
L. Cushing’s syndrome
M. Addison's disease
A

Artifactual

458
Q

An 18 year old woman presents comatose, with a urinary pH of 3.5 and plasma potassium of 6.5mmol/l. 6 hours after treatment potassium drops to 3.1mmol/l.

A. Diarrhoea
B. Rhabdomyolysis
C. SIADH
D. Vomiting
E. Acute Renal Failure
F. Diabetic ketoacidosis
G. Alcohol abuse
H. Renal tubular acidosis
I. Artifactual
J. Diuretic use
K. Haemorrhage
L. Cushing’s syndrome
M. Addison's disease
A

Diabetic Ketoacidosis
Insulin overdose/treatment in a diabetic patient will cause a redistributive hypokalaemia and concurrent metabolic alkalosis. Insulin causes a shift of potassium ions from the extracellular space to the intracellular space, thereby lowering blood potassium levels. Metabolic alkalosis can also cause a redistributive hypokalaemia; a reduced hydrogen ion concentration in the blood causes increased intracellular hydrogen ion loss to increase extracellular levels via Na+/H+ATPase; potassium ions therefore diffuse intracellularly to maintain the electrochemical potential. Adrenaline and re-feeding syndrome also cause redistributive hypokalaemia

459
Q

A 68 year old woman, K+ = 3.0 mmol/L with a history of congestive cardiac failure complains of general discomfort.

A. Rhabdomyalysis
B. Artefactual
C. Insulin administration
D. Hypokalaemia
E. Renal tubular disease
F. Burns
G. Cushing's disease
H. Addison's disease
I. Diuretic use
J. Hyperkalaemia
K. Laxative abuse
A

Diuretic use
Normal K+ range: 3.5-5.5 mmol/L - predominant intracellular cation

Presentation of hypokalaemia:

  • Mild: asymptomatic
  • Severe: lethargy, weakness, muscle pain, constipation, paraesthesia, tetany, arrhythmia

Either due to depletion or shift into cells (very rarely due to decreased intake).

Causes of hypokalaemia:

  1. GI loss: diarrhoea, vomiting, intestinal fistulae, laxative abuse
  2. Renal loss:
    - Hyperaldosteronism, excess cortisol (e.g. Conn’s syndrome, renal artery stenosis, Cushing’s)
    - Gitelman/Barter syndromes: hypokalaemia metabolic alkalosis due to mutation in transporters in DCT/thick ascending limb LOH respectively
    - Increased sodium delivery to distal nephron
    - Osmotic diuresis (thiazide & loop diuretics)
  3. Redistribution into cells: Insulin, B-agonists, alkalosis
  4. Rare causes: Rare tubular acidosis type 1 & 2, hypomagnesia

Treatment:

  • K 3-3.5: oral KCl e.g. sandoK tds for 2 days
  • K
460
Q

An ECG of a 27 year old man in casualty shows peaked T waves in leads V2 and V3.

A. Rhabdomyalysis
B. Artefactual
C. Insulin administration
D. Hypokalaemia
E. Renal tubular disease
F. Burns
G. Cushing's disease
H. Addison's disease
I. Diuretic use
J. Hyperkalaemia
K. Laxative abuse
A

Hyperkalaemia >5.5mmol/L
Less common than hypokalaemia but more dangerous: malaise, palpitations, muscle weakness, ECG changes, tented T-waves, loss of P-waves, prolonged QRS

Causes:

  • Excessive intake: oral, parenteral, stored blood transfusion
  • Transcellular Movement (ICF>ECF): acidosis, insulin shortage, tissue damage/catabolic state e.g. burns, rhabdomyolysis
  • Decreased excretion: acute renal failure (oliguric phases), chronic renal failure (late), K-sparing diuretics (spironolactone), mineralocorticoid deficiency (Addison’s), NSAIDs, ACEi

Treatment: 10mL 10% calcium gluconate (protects cardiac membrane), 50mL 50% dextrose + 10 units insulin (drives K+ into cells), nebulised salbutamol (longer lasting shift of K+ into cells)

H+ & K+ are linked - as one moves into a cell the other moves out. For every drop in pH pf 0.1 there is an increase in K+ of 0.7

461
Q

A 2 month old infant vomits profusely, pH = 7.57, H+= 26 nmol/L, HCO3= 50 mmol/L

A. Rhabdomyalysis
B. Artefactual
C. Insulin administration
D. Hypokalaemia
E. Renal tubular disease
F. Burns
G. Cushing's disease
H. Addison's disease
I. Diuretic use
J. Hyperkalaemia
K. Laxative abuse
A

Hypokalaemia
Presentation of hypokalaemia:
- Mild: asymptomatic
- Severe: lethargy, weakness, muscle pain, constipation, paraesthesia, tetany, arrhythmia

Either due to depletion or shift into cells (very rarely due to decreased intake).

Causes of hypokalaemia:

  1. GI loss: diarrhoea, vomiting, intestinal fistulae, laxative abuse
  2. Renal loss:
    - Hyperaldosteronism, excess cortisol (e.g. Conn’s syndrome, renal artery stenosis, Cushing’s)
    - Gitelman/Barter syndromes: hypokalaemia metabolic alkalosis due to mutation in transporters in DCT/thick ascending limb LOH respectively
    - Increased sodium delivery to distal nephron
    - Osmotic diuresis (thiazide & loop diuretics)
  3. Redistribution into cells: Insulin, B-agonists, alkalosis
  4. Rare causes: Rare tubular acidosis type 1 & 2, hypomagnesia

Treatment:

  • K 3-3.5: oral KCl e.g. sandoK tds for 2 days
  • K
462
Q

A 47 year old woman complains of tiredness, muscle weakness, mood swings and loss of appetite over several months. K+= 5.9 mmol/L

A. Rhabdomyalysis
B. Artefactual
C. Insulin administration
D. Hypokalaemia
E. Renal tubular disease
F. Burns
G. Cushing's disease
H. Addison's disease
I. Diuretic use
J. Hyperkalaemia
K. Laxative abuse
A

Addison’s Disease: primary adrenal insufficiency - autoimmune destruction of adrenal cortex due to immune reaction to 21-hydroxylase) - (with hypothyroidism is Schmidt’s syndrome).

Leads to increased K & Ca, but decreased Na & glucose. Causes postural hypotension, skin pigmentation (increased POMC & MSH to increase ACTH - negative feedback mechanism). Addisonian Crisis!!!

Diagnosis: synACTHen test

  • Short: 250microgram synACTHen. If plasma cortisol >550nmol/L, and rises >170nmol/L Addison’s can be excluded.
  • Long: 1mg synACTHen - in primary adrenal insufficiency cortisol is reduced at all stages. In secondary adrenal insufficiency there is a delayed but normal increase in cortisol response seen (>900nmol/L)

Adrenal Insufficiency:

  • Primary: 80% addisons, CAH, adenoma, TB, haemorrhage (Waterhouse-Friedrichsen syndrome)
  • Secondary: exogenous steroid use, pituitary tumours/trauma, hypothalamic tumours, Sheehan’s syndrome (postpartum)
463
Q

A 15 year old girl with K+ = 3.2 mmol/L admits to taking Bisacodyl over several months to lose weight.

A. Rhabdomyalysis
B. Artefactual
C. Insulin administration
D. Hypokalaemia
E. Renal tubular disease
F. Burns
G. Cushing's disease
H. Addison's disease
I. Diuretic use
J. Hyperkalaemia
K. Laxative abuse
A
Laxative abuse 
Hypokalaemia
Presentation of hypokalaemia:
- Mild: asymptomatic
- Severe: lethargy, weakness, muscle pain, constipation, paraesthesia, tetany, arrhythmia

Either due to depletion or shift into cells (very rarely due to decreased intake).

Causes of hypokalaemia:

  1. GI loss: diarrhoea, vomiting, intestinal fistulae, laxative abuse
  2. Renal loss:
    - Hyperaldosteronism, excess cortisol (e.g. Conn’s syndrome, renal artery stenosis, Cushing’s)
    - Gitelman/Barter syndromes: hypokalaemia metabolic alkalosis due to mutation in transporters in DCT/thick ascending limb LOH respectively
    - Increased sodium delivery to distal nephron
    - Osmotic diuresis (thiazide & loop diuretics)
  3. Redistribution into cells: Insulin, B-agonists, alkalosis
  4. Rare causes: Rare tubular acidosis type 1 & 2, hypomagnesia

Treatment:

  • K 3-3.5: oral KCl e.g. sandoK tds for 2 days
  • K
464
Q

A 47 year old female presents to her GP with severe loin pain. On further questioning the patient complains of a 6 month history of recurrent fevers and vomiting with more recent generalised weakness and pain in some of her joints. A subsequent blood test shows hypokalaemia.

A. Addison's disease
B. Vomiting
C. Diuretics
D. Drip arm sample
E. Renal tubular acidosis
F. Fistula
G. Diarrhoea
H. Delayed separation
I. Renal failure
J. Haemolysis
K. Corticosteroid use
A

Renal Tubular Acidosis
Renal failure tends to cause HYPERkalaemia as a function of reduced GFR. Renal tubular acidosis describes tubular abnormalities that give rise to acidosis without affecting GFR.

Type 1 or distal RTA results from a failure in H+ excretion from the distal tubule and collecting ducts. Type 2 is caused by a failure to reabsorb HCO3 from the proximal tubule. You can see how both of these abnormalities would cause a metabolic acidosis. In both cases there is associated Na wasting –> secondary hyperaldosteronism –> K excretion and hypokalaemia.

RTA is typically characterised by:

  • acidosis with HYPOkalaemia
  • acidosis with alkaline urine and positive urine anion gap.
  • nephrocalcinosis

Type 1 RTA is commoner than type 2 (typically caused by Fanconi syndrome) and may be idiopathic or secondary. A common cause of type 1 RTA is autoimmune, including SLE, which could be causing this ladies joint pain. The
severe loin pain is probably due to a renal stone. Another potential cause in this case is chronic pyelonephritis (accounting for 6mo of fevers).

465
Q

A 20 year old medical student presents to his GP with bowel disturbances. He recently returned from holiday in Thailand. Blood tests reveal that he is hypokalaemic.

A. Addison's disease
B. Vomiting
C. Diuretics
D. Drip arm sample
E. Renal tubular acidosis
F. Fistula
G. Diarrhoea
H. Delayed separation
I. Renal failure
J. Haemolysis
K. Corticosteroid use
A

Diarrhoea typically results in electrolyte (including K and HCO3) loss and consequent HYPOkalaemia and alkalosis. If the diarrhoea was severe enough (e.g. cholera) to cause severe dehydration and reduced organ perfusion, a lactic acidosis could potentially result.

In general though: diarrhoea –> hypokalaemia + alkalosis

466
Q

A junior doctor received a blood report from the pathology lab for a 50 year old male who was recovering from an inguinal hernia repair. The report described the patient as being hyperkalaemic. Most of the porters at the hospital were on strike at the time.

A. Addison's disease
B. Vomiting
C. Diuretics
D. Drip arm sample
E. Renal tubular acidosis
F. Fistula
G. Diarrhoea
H. Delayed separation
I. Renal failure
J. Haemolysis
K. Corticosteroid use
A

Delayed separation: If you leave blood lying around for too long, K (major intracellular cation) leakes out of the red blood cells into the serum causing a falsly elevated K. This is called delayed separation.

467
Q

An 82 year old female caught a bad cold on a flight to Heathrow for a holiday from India, where she has lived all her life. Six days later she comes into A+E weak, confused with abdominal pain. Blood tests show a potassium of 6.2mmol/L.

A. Addison's disease
B. Vomiting
C. Diuretics
D. Drip arm sample
E. Renal tubular acidosis
F. Fistula
G. Diarrhoea
H. Delayed separation
I. Renal failure
J. Haemolysis
K. Corticosteroid use
A

Addison’s disease - primary adrenal insufficiency - autoimmune destruction of adrenal cortex due to immune reaction to 21-hydroxylase) - (with hypothyroidism is Schmidt’s syndrome). Worldwide the commonest cause of Addison’s disease is TB (more prevalent in India). Intercurrent illness increases steroid medication requirement: if it is not increased, a crisis may result.

Leads to increased K & Ca, but decreased Na & glucose. Causes postural hypotension, skin pigmentation (increased POMC & MSH to increase ACTH - negative feedback mechanism). Addisonian Crisis!!!

Diagnosis: synACTHen test

  • Short: 250microgram synACTHen. If plasma cortisol >550nmol/L, and rises >170nmol/L Addison’s can be excluded.
  • Long: 1mg synACTHen - in primary adrenal insufficiency cortisol is reduced at all stages. In secondary adrenal insufficiency there is a delayed but normal increase in cortisol response seen (>900nmol/L)

Adrenal Insufficiency:

  • Primary: 80% addisons, CAH, adenoma, TB, haemorrhage (Waterhouse-Friedrichsen syndrome)
  • Secondary: exogenous steroid use, pituitary tumours/trauma, hypothalamic tumours, Sheehan’s syndrome (postpartum)
468
Q

A 72 year old male is referred to cardiothoracic surgery outpatients following an episode of unconsciousness. The patient had an aortic valve replacement operation 5 years ago. Following investigation the valve is found to have malfunctioned. A blood test shows that the patient is hyperkalaemic.

A. Addison's disease
B. Vomiting
C. Diuretics
D. Drip arm sample
E. Renal tubular acidosis
F. Fistula
G. Diarrhoea
H. Delayed separation
I. Renal failure
J. Haemolysis
K. Corticosteroid use
A

Haemolysis: mechanical heart valves may be associated with haemolysis (which raises the potassium)

469
Q

A 65 year old lady presents to A & E with her son who describes decreasing mental function over the last week or so. On questioning you discover that she is a smoker with a 40 pack year history and that she has had a chronic, productive cough for several weeks. Bloods include Na=120, K=4.5, Cl=85, HC03=22, serum osmolality=260

A. Psychogenic polydipsia
B. Diuretic excess
C. Iatrogenic
D. Illicit drug abuse
E. Normal
F. SIADH
G. Alcohol abuse
H. Cranial Diabetes insipidus
I. Dehydration
J. Nephrogenic diabetes insipidus
A

SIADH - The commonest cause of “SIADH” in cancer patients is not brain involvement, it’s nausea from chemotherapy - just that with chemo, the ADH is a “physiologically appropriate” response to nausea!

SIADH - other names: Schwartz-Bartter syndrome, SIAD—syndrome of inappropriate antidiuresis) is characterized by excessive release of antidiuretic hormone from the posterior pituitary gland or another source. The result is often dilutional hyponatremia in which the plasma sodium levels are lowered and total body fluid is increased. EUVOLAEMIC HYPONATRAEMIA
ADH acts in the distal portion of the renal tubule (Distal Convoluted Tubule) as well as on the collecting duct and causes the retention of water, but not solute. Hence, ADH activity effectively dilutes the blood (decreasing the concentrations of solutes such as sodium), causing hyponatremia; this is compounded by the fact that the body responds to water retention by decreasing aldosterone, thus allowing even more sodium wasting. For this reason, a high urinary sodium excretion will be seen.

This causes dilutional hyponatremia and all the consequences associated with that condition: headache, nausea, vomiting, and confusion may ensue. Severe hyponatremia may cause convulsions or coma.

Causes: Meningitis, Head injury (subarachnoid haemorrhage), small cell lung cancer, brain abscess, pneumonia, lung abscess, Guillain-Barre syndrome, hypothyroidism, sarcoidosis, drugs (amytriptyline, morphine, vincristine, oxytocin, valproic acid, cyclophosphamide, carbemazepine, ciprofloxacin)

Normal ranges:
Serum osmolality is 275-295mmol/kg
Na: 135-145 mmol/L

Hyponatraemia: determine whether true hyponatraemia using serum osmolality

  • If high: glucose/mannitol, infusion, ethanol
  • If normal: Spurious, drip arm sample, pseudohyponatraemia (hyperlipidaemia, paraproteinaemia)
  • If low = true hyponatraemia

TURP syndrome causes hyponatraemia as increased water reabsorption through the damaged prostate.

Treating Hyponatraemia:
Clinical Assessment
- Hypovolaemic: diarrhoea, vomiting, diuretics, salt losing nephropathy - Tx: ?fluid restoring if necessary
- Euvolaemic: Hypothyroidism, adrenal insufficiency, SIADH. Ix: TFTs, short synACTHen test, urine osmolality
- Hypervolaemic: Cardiac failure, cirrhosis, nephrotic syndrome. Ix: overload?

470
Q

A 75 year old lady is recovering from a hip replacement after fracturing her neck of femur. The post-operative period has been uneventful but today you, the F1, notice that her blood results are slightly abnormal: Na=126, K=3.2, serum osmolality=262. You consult your registrar, who tells you not to worry and advises ‘watchful waiting’.

A. Psychogenic polydipsia
B. Diuretic excess
C. Iatrogenic
D. Illicit drug abuse
E. Normal
F. SIADH
G. Alcohol abuse
H. Cranial Diabetes insipidus
I. Dehydration
J. Nephrogenic diabetes insipidus
A

Iatrogenic: The most common precipitant of hyponatremia in patients after surgery is the iatrogenic infusion of hypotonic fluids. This is exacerbated by pain and nausea, both of which cause nonosmotic release of ADH.

471
Q

A young drama student attends clinic complaining of polyuria and sleep disturbance. Her past medical history includes an appendicectomy, a skull fracture, and hayfever. Her biochemistry reveals Na=148, K=3.6. She denies excessive fluid intake.

A. Psychogenic polydipsia
B. Diuretic excess
C. Iatrogenic
D. Illicit drug abuse
E. Normal
F. SIADH
G. Alcohol abuse
H. Cranial Diabetes insipidus
I. Dehydration
J. Nephrogenic diabetes insipidus
A

Cranial Diabetes insipidus: History of skull fracture points to cranial DI. Nephrogenic diabetes insipidus can be seen in chronic renal insufficiency, lithium toxicity, hypercalcemia, hypokalemia, and tubulointerstitial disease. (None of which this pt seems to have)

Diabetes Insipidus:
Clinical features: hypernatraemia (lethargy, thirst, irritability, confusion, coma, fits), clinically euvolaemic, polyuria & polydipsia
Urine:plasma osmolality 600mOsmol/kg
- Primary polydipsia: urine concentration 400-600mOsmol/kg
- Cranial DI: urine concentrates only after giving DDAVP
- Nephrogenic DI: urine doesn’t concentrate, even after DDAVP

472
Q

A male medical student consults you after a lecture on disorders of acid-base balance and ion handling. He is concerned that his high fluid intake (2-3litres/day) may be causing a ‘dilutional hyponatraemia’ as he finds it hard to concentrate in lectures. Initial investigations include biochemistry: Na=126, K=3.8, pH=7.39.

A. Psychogenic polydipsia
B. Diuretic excess
C. Iatrogenic
D. Illicit drug abuse
E. Normal
F. SIADH
G. Alcohol abuse
H. Cranial Diabetes insipidus
I. Dehydration
J. Nephrogenic diabetes insipidus
A

Psychogenic polydipsia: dilutional!!!

Sodium and potassium are both low! pH is normal! High intake!!!

473
Q

A 37 year old banker presents to A & E complaining of nausea and dizziness. He describes treating his hangover this morning with a ‘good run’ and tells you that he had a coffee to ‘steady himself’ before coming in. On examination you note a mild tachycardia and cannot confidently assess his JVP. Routine bloods reveal Na=152, K=4.1, urea=25, creatinine=190.

A. Psychogenic polydipsia
B. Diuretic excess
C. Iatrogenic
D. Illicit drug abuse
E. Normal
F. SIADH
G. Alcohol abuse
H. Cranial Diabetes insipidus
I. Dehydration
J. Nephrogenic diabetes insipidus
A

Dehydration: hangover, exercise, caffeine!

Clinically hypovolaemic!

474
Q

Normovolaemic and hyponatraemic

A. SIADH
B. Vomiting
C. Osteomalacia
D. Diuretic excess
E. Pernicious anaemia
F. Cardiac failure
G. Guillain-Barre syndrome
H. COPD
I. Chronic renal failure
J. Acromegaly
K. Achalasia
A

SIADH

Euvolaemia, urinary Na > 20 : SIADH, primary polydipsia, severe hypothyroidism, drugs

Laboratory criteria:

  • True hyponatraemia
  • Clinically euvolaemic
  • Inappropriately high urine osmolality
  • Increased renal sodium excretion ( >20mmol/L)
  • Normal renal, adrenal, thyroid & cardiac function function
  • Essentially a diagnosis of exclusion

Causes:

  • Malignancy: small cell lung cancer, pancreas, prostate, lymphoma (ectopic secretion)
  • CNS disorders: meningioencephalitis, haemorrhage, abscess
  • Chest Disease: TB, pneumonia, abscess
  • Drugs: opiates, SSRIs

Treatment: fluid restriction

475
Q

Hypovolaemic with urinary Na+

A

Vomiting
Hypovolaemia with urinary Na > 20 = renal, and 20): diuretics, Addison’s, salt losing nephropathies

Presentations: tachycardia, postural hypotension, dry mucous membranes, decreased skin turgor, decreased urine output

Treatment: 0.9% saline fluid replacement

476
Q

Raised JVP, peripheral oedema and urinary Na+

A

Cardiac failure

Clinically fluid overloaded: raised JVP, oedema

urinary Na 20 = renal causes: ARF, CRF

Treatment: fluid restriction

477
Q

Hypotension with urinary Na+>20 mmol/L

A. SIADH
B. Vomiting
C. Osteomalacia
D. Diuretic excess
E. Pernicious anaemia
F. Cardiac failure
G. Guillain-Barre syndrome
H. COPD
I. Chronic renal failure
J. Acromegaly
K. Achalasia
A

Diuretic excess

Hypovolaemia with urinary Na > 20 = renal, and 20): diuretics, Addison’s, salt losing nephropathies

Presentation hypovolaemia: tachycardia, postural hypotension, dry mucous membranes, decreased skin turgor & urine output

Treatment: 0.9% saline fluid replacement

478
Q

Hypervolaemic with urinary Na+>20 mmol/L

A. SIADH
B. Vomiting
C. Osteomalacia
D. Diuretic excess
E. Pernicious anaemia
F. Cardiac failure
G. Guillain-Barre syndrome
H. COPD
I. Chronic renal failure
J. Acromegaly
K. Achalasia
A

Chronic renal failure

Hypervolaemia: peripheral oedema, bibasal crackles, raised JVP

Treatment: fluid restriction

urinary Na 20 = renal causes: ARF, CRF

479
Q

A 23 year old male patient presents to GI outpatients clinic following disturbances since his return from a one month trip to India. Routine bloods reveal that he is mildly hypokalaemic. Results of rigid sigmoidoscopy were normal

A. Addisonian crisis
B. Vomiting
C. Renal tubular acidosis
D. Fistula
E. Haemolysis
F. Renal Failure
G. Diarrhoea
H. Drip arm sample
I. Use of diuretics
J. Severe tissue damage
K. Use of corticosteroids
A

Diarrhoea

N.A. In the absence of adequate
thyroxine (and also cortisol), the renal tubules do not clear free water
at a normal rate. Thus if you drink some water, the kidneys are slow to
clear the free water, and thus hyponatraemia ensues.

480
Q

A 46 year old female presents with confusion and complains of hallucinations. On further questioning she reveals that she has been feeling generally tired and weak for the last 8 weeks. During this period she has lost 8 kg in weight. Her past history revealed an episode of TB 22 years ago. Her electrolyte results revealed Sodium 105 mmol/l; potassium 5.5 mmol/l and osmolality 220 mmol/l.

A. Addisonian crisis
B. Vomiting
C. Renal tubular acidosis
D. Fistula
E. Haemolysis
F. Renal Failure
G. Diarrhoea
H. Drip arm sample
I. Use of diuretics
J. Severe tissue damage
K. Use of corticosteroids
A

Addisonian crisis
Common symptoms:
- Sudden penetrating pain in the legs, lower back or abdomen
- Severe vomiting and diarrhea, resulting in dehydration
- Low BP
- Syncope
- Hypoglycemia
- Confusion, psychosis, slurred speech
- Severe lethargy
- Hyponatremia, Hyperkalemia, Hypercalcemia
- Convulsions, Fever

Primary adrenal insufficiency -> lack of mineralocorticoids and glucocorticoids.

Addison’s disease is also known as chronic primary adrenocortical insufficiency, to distinguish it from acute primary adrenocortical insufficiency, most often caused by Waterhouse–Friderichsen syndrome (haemorrhage).

Addison’s Disease refers to Primary Adrenal Insufficiency. There are numerous causes for this, such as Adrenal dysgenesis or adrenal destruction through some autoimmune process or it can be secondARY to some iatrogenic cause (usually a sudden stop in GC treatment or surgical damage/removal) etc..

AD leads to a decrease in all hormones produced by the Adrenals, including Cortisol and Aldosterone.
Considering that the Pituitary is functioning perfectly well, levels of ACTH will be high to try and stimulate the Adrenals, however in the absence of functioning adrenals, this has no effect. The resultant low levels of Cortisol & Aldosterone can then go on to cause an Adisonian Crisis.

In Secondary Adrenal Insufficiency (one where the Pituitary is not functioning correctly e.g - Sheehans,
Pituitary Apoplexy, Non-Functoning Pituitary Tumour etc…) ACTH levels (amongst others) are low, however the Adrenals are working fine. Some of the main symptoms of an Adissonian Crisis such as dehydration, Low BP and hyperkalemia are not common presenting signs of secondary adrenal insuff. because production of aldosterone is NOT primarily under the influence of anterior pituitary hormones. As you mentioned, it is mainly a stress response.

Therefore, with secondary adrenal insufficiency, aldosterone levels are close to normal and able to compensate for the lack of circulating cortisol by maintaining fluid balance and BP. Of course, it is possible to go on to develop an AC if the body is stressed even further, but generally speaking, Secondary Adrenal Insuff. will not cause a crisis

481
Q

A 78 year old male presents with hypokalaemia. He has previously been diagnosed with congestive heart failure, which has been controlled with medication. Serum sodium was 126 mmol/l and bicarbonate was raised.

A. Addisonian crisis
B. Vomiting
C. Renal tubular acidosis
D. Fistula
E. Haemolysis
F. Renal Failure
G. Diarrhoea
H. Drip arm sample
I. Use of diuretics
J. Severe tissue damage
K. Use of corticosteroids
A

Use of diuretics

482
Q

A 35 year old male body builder presents to his GP surgery with genital atrophy. Routine blood tests reveal that the patient is hypokalamic.

A. Addisonian crisis
B. Vomiting
C. Renal tubular acidosis
D. Fistula
E. Haemolysis
F. Renal Failure
G. Diarrhoea
H. Drip arm sample
I. Use of diuretics
J. Severe tissue damage
K. Use of corticosteroids
A

Use of corticosteroids
Some of the illegal artificial anabolic steroids have some mineralocorticoid activity. The genital atrophy suggests that it is an anabolic steroid. Remember that aldosterone & testosterone are very similar, and the artificial ones have bizarre mixtures of effects.

483
Q

A 19 year old female patient presents to A&E with severe dehydration, and is rapidly infused. Blood samples obtained by a trainee nurse reveals gross hyponatraemia. Glucose levels was also raised markedly.

A. Addisonian crisis
B. Vomiting
C. Renal tubular acidosis
D. Fistula
E. Haemolysis
F. Renal Failure
G. Diarrhoea
H. Drip arm sample
I. Use of diuretics
J. Severe tissue damage
K. Use of corticosteroids
A

Drip arm sample

484
Q

A 45 year old female with long-term poorly controlled asthma presents to her GP complaining of weight gain and excessive sweating. A recent routine abdominal CT scan revealed atrophy of the adrenal glands.

A. Cushing’s Syndrome
B. Ectopic ACTH Secretion
C. Aldosterone Secreting Adrenal Adenoma
D. Cushing’s Disease
E. Congenital Adrenal Hyperplasia
F. Iatrogenic Cushing’s Syndrome
G. Phaeochromocytoma
H. Multiple Endocrine Neoplasia Syndrome
I. Addison’s Disease
J. Nelson’s Syndrome
K. Pseudo-Cushing’s Syndrome
L. Addisonian Crisis
M. Adrenal Carcinoma
N. Schmidt’s Syndrome
A

Iatrogenic Cushing’s Syndrome
Excess glucocorticoid/cortisol

Moon face, buffalo hump, striae, acne, HTN, DM, proximal myopathy, impotence, hirsuitism, oligo/amenorrhoea

485
Q

The next patient on the endocrine ward round has just received the results of a high dose dexamethasone suppression test. The consultant informs you that the cortisol levels have been suppressed and asks you the most likely cause of this patient’s cushingoid symptoms

A. Cushing’s Syndrome
B. Ectopic ACTH Secretion
C. Aldosterone Secreting Adrenal Adenoma
D. Cushing’s Disease
E. Congenital Adrenal Hyperplasia
F. Iatrogenic Cushing’s Syndrome
G. Phaeochromocytoma
H. Multiple Endocrine Neoplasia Syndrome
I. Addison’s Disease
J. Nelson’s Syndrome
K. Pseudo-Cushing’s Syndrome
L. Addisonian Crisis
M. Adrenal Carcinoma
N. Schmidt’s Syndrome
A

Cushing’s Disease: pituitary tumour 85% of cushing’s syndrome), adrenal tumour represents 10%, ectopic ACTH producing (small cell lung Ca) is 5%

the fact that there was significant suppression of cortisol by day 2 of the test means that this COULD NOT be caused by ectopic ACTH.

If there is not a normal response on the low-dose test, abnormal secretion of cortisol is likely (Cushing’s Syndrome). This could be a result of a cortisol-producing adrenal tumour, a pituitary tumour that produces ACTH, or a tumour in the body that inappropriately produces ACTH. THe high-dose test can help distinguish a pituitary cause (Cushing’s Disease) from the others:

Cushing’s Syndrome caused by an adrenal tumour:

  • low dose (0.5mg dexamethasone): no change
  • high dose (2mg dexamethasone): NO CHANGE

Cushing’s Syndrome caused related to ectopic ACTH producing tumour:

  • low dose: no change
  • high dose: NO CHANGE

Cushing’s Syndrome caused by pituitary tumour (Cushing’s Disease):

  • low dose: no change
  • high dose: NORMAL SUPPRESSION
486
Q

A 35 year old female arrives in A&E at 16:30 in a very distressed state. Examination reveals tachycardia and postural hypotension. She complains of ongoing weakness and confusion following a recent operation on her knee. Blood tests reveal hyperkalaemia, hyponatraemia. Further tests measure cortisol levels at 50 nmol/L.

A. Cushing’s Syndrome
B. Ectopic ACTH Secretion
C. Aldosterone Secreting Adrenal Adenoma
D. Cushing’s Disease
E. Congenital Adrenal Hyperplasia
F. Iatrogenic Cushing’s Syndrome
G. Phaeochromocytoma
H. Multiple Endocrine Neoplasia Syndrome
I. Addison’s Disease
J. Nelson’s Syndrome
K. Pseudo-Cushing’s Syndrome
L. Addisonian Crisis
M. Adrenal Carcinoma
N. Schmidt’s Syndrome
A

Addisonian Crisis: adrenal insufficiency - AI, TB, adrenal haemorrhage (Water-house Friedrichson), amyloidosis

Increased K and Ca, decreased Na and glucose, postural hypotension, skin pigmentation, lethargy, depression

Short synACTHen: 250micrograms synACTHen, if plasma cortisol is >550nmol/L and rises >170nmol/L then Addisons is excluded (adrenals are producing & responsive to ACTH).
Long synACTHen:
1mg synACTHen, in primary adrenal insufficiency, cortisol is reduced in all stages (Addison’s, CAH, adenoma, infection, Waterhouse-Friedrichson). Whereas in secondary adrenal insufficiency (exogenous steroid use, pituitary trauma/tumours, hypothalamic tumours, Sheehan’s syndrome) a delayed but normal response is seen: >900nmol/L)

487
Q

A 52 year old male complains of muscle cramps and headaches. Examination reveals hypertension. Blood tests are ordered and reveal a marked hypokalaemia. The renin-aldosterone ratio is noted at 0.02 and the House Officer orders an abdominal CT scan.

A. Cushing’s Syndrome
B. Ectopic ACTH Secretion
C. Aldosterone Secreting Adrenal Adenoma
D. Cushing’s Disease
E. Congenital Adrenal Hyperplasia
F. Iatrogenic Cushing’s Syndrome
G. Phaeochromocytoma
H. Multiple Endocrine Neoplasia Syndrome
I. Addison’s Disease
J. Nelson’s Syndrome
K. Pseudo-Cushing’s Syndrome
L. Addisonian Crisis
M. Adrenal Carcinoma
N. Schmidt’s Syndrome
A

Aldosterone Secreting Adrenal Adenoma:

Hyperaldosteronism may be due to:
Primary: Adrenal tumour - Conn’s syndrome (solitary aldosterone secreting adenoma), CAH, adrenal carcinoma (rare)
Secondary: Juxtaglomerular cell turnover, renal artery stenosis, Barrter/Gittleman Syndrome (decreased Na absorption)

Presentation: uncontrollable HTN, high Na, low K, headache, metabolic alkalosis, polyuria, polydipsia

Investigations: Aldosterone:Renin ratio - raised in primary causes, but normal/increased with a high renin in secondary causes

488
Q

A 65 year old female presents to her new GP 5 years after an operation on her abdomen. She cannot remember the details of the operation but does remember that she was suffering from severe Cushing’s Disease at the time. She now notes a progressive “tanning” of the skin

A. Cushing’s Syndrome
B. Ectopic ACTH Secretion
C. Aldosterone Secreting Adrenal Adenoma
D. Cushing’s Disease
E. Congenital Adrenal Hyperplasia
F. Iatrogenic Cushing’s Syndrome
G. Phaeochromocytoma
H. Multiple Endocrine Neoplasia Syndrome
I. Addison’s Disease
J. Nelson’s Syndrome
K. Pseudo-Cushing’s Syndrome
L. Addisonian Crisis
M. Adrenal Carcinoma
N. Schmidt’s Syndrome
A

Nelson’s Syndrome is a potentially life-threatening condition which occurs when an adrenocorticotrophic hormone (ACTH) secreting tumour develops following therapeutic total bilateral adrenalectomy (TBA) for Cushing’s disease.

History - early presentation
Hyperpigmentation occurs in up to 42% of people even when diagnosed early, due to the action of ACTH on melanocytes.
Visual field defects occur in 10-57% and should be enquired about. They may be too insidious to have been noticed and formal testing may be required.

History - late presentation
Headaches are common with pituitary tumours and are probably the result of stretching of the diaphragma sellae. Features of raised intracranial pressure occur late and are uncommon because they require a tumour large enough to obstruct the flow of cerebrospinal fluid (CSF).
Hypopituitarism occurs when the hypothalamic-pituitary portal system is disrupted or normal pituitary tissue is destroyed by the tumour:
It may be partial rather than total. The anterior pituitary is more often involved than the posterior pituitary.
Often hormone deficiency is incomplete.
In children and adolescents, note growth and age of puberty. Cushing’s syndrome often slows growth in children but operation should return it. If not, investigation is required.
In all patients, enquire for symptoms of polyuria and polydypsia (due to diabetes insipidus), hypothyroidism and presence of galactorrhoea.
In women, amenorrhoea may be the first sign of pituitary disease. Galactorrhoea is uncommon in men but hyperprolactinaemia is a cause of erectile dysfunction.

489
Q

A 57 year old Type 1 diabetic woman presents with weight loss, weakness and depression. Examination reveals postural hypotension, hyperpigmentation in the palmar creases and widespread patchy vitiligo. Full blood count is unremarkable but U&Es reveal Na+ 130 mmol/l, K+ 6.0 mmol/l, Urea 7.4 mmol/l and Ca 2+ 2.70 mmol/l.

A. Congenital adrenal hyperplasia
B. Adrenal adenoma
C. Phaeochromocytoma
D. Addison's disease
E. Ectopic ACTH secretion
F. Iatrogenic Cushing's syndrome
G. Conn's syndrome
H. Carney's syndrome
I. Pseudo-Cushing's syndrome
J. Cushing's disease
K. Iatrogenic Addison's disease
L. Adrenal carcinoma
A

Addison’s disease
Primary adrenal insufficiency -> lack of mineralocorticoids and glucocorticoids.

Addison’s disease is also known as chronic primary adrenocortical insufficiency, to distinguish it from acute primary adrenocortical insufficiency, most often caused by Waterhouse–Friderichsen syndrome (haemorrhage).

Addison’s Disease refers to Primary Adrenal Insufficiency. There are numerous causes for this, such as Adrenal dysgenesis or adrenal destruction through some autoimmune process or it can be secondARY to some iatrogenic cause (usually a sudden stop in GC treatment or surgical damage/removal) etc..

AD leads to a decrease in all hormones produced by the Adrenals, including Cortisol and Aldosterone.
Considering that the Pituitary is functioning perfectly well, levels of ACTH will be high to try and stimulate the Adrenals, however in the absence of functioning adrenals, this has no effect. The resultant low levels of Cortisol & Aldosterone can then go on to cause an Adisonian Crisis.

In Secondary Adrenal Insufficiency (one where the Pituitary is not functioning correctly e.g - Sheehans,
Pituitary Apoplexy, Non-Functoning Pituitary Tumour etc…) ACTH levels (amongst others) are low, however the Adrenals are working fine. Some of the main symptoms of an Adissonian Crisis such as dehydration, Low BP and hyperkalemia are not common presenting signs of secondary adrenal insuff. because production of aldosterone is NOT primarily under the influence of anterior pituitary hormones. As you mentioned, it is mainly a stress response.

Therefore, with secondary adrenal insufficiency, aldosterone levels are close to normal and able to compensate for the lack of circulating cortisol by maintaining fluid balance and BP. Of course, it is possible to go on to develop an AC if the body is stressed even further, but generally speaking, Secondary Adrenal Insuff. will not cause a crisis

490
Q

A 32 year old woman presents with a one year history of weight loss, fatigue and hirsutism. Examination reveals thin skin, easy bruising, purple abdominal striae and a supraclavicular fat pad. Plasma cortisol and ACTH levels are both raised but suppress after high dose dexamethasone suppression test.

A. Congenital adrenal hyperplasia
B. Adrenal adenoma
C. Phaeochromocytoma
D. Addison's disease
E. Ectopic ACTH secretion
F. Iatrogenic Cushing's syndrome
G. Conn's syndrome
H. Carney's syndrome
I. Pseudo-Cushing's syndrome
J. Cushing's disease
K. Iatrogenic Addison's disease
L. Adrenal carcinoma
A

Cushing’s disease: Cushing’s Disease: pituitary tumour 85% of cushing’s syndrome), adrenal tumour represents 10%, ectopic ACTH producing (small cell lung Ca) is 5%

the fact that there was significant suppression of cortisol by day 2 of the test means that this COULD NOT be caused by ectopic ACTH.

If there is not a normal response on the low-dose test, abnormal secretion of cortisol is likely (Cushing’s Syndrome). This could be a result of a cortisol-producing adrenal tumour, a pituitary tumour that produces ACTH, or a tumour in the body that inappropriately produces ACTH. THe high-dose test can help distinguish a pituitary cause (Cushing’s Disease) from the others:

Cushing’s Syndrome caused by an adrenal tumour:

  • low dose (0.5mg dexamethasone): no change
  • high dose (2mg dexamethasone): NO CHANGE

Cushing’s Syndrome caused related to ectopic ACTH producing tumour:

  • low dose: no change
  • high dose: NO CHANGE

Cushing’s Syndrome caused by pituitary tumour (Cushing’s Disease):

  • low dose: no change
  • high dose: NORMAL SUPPRESSION
491
Q

A 64 year old man, who is known to suffer from ulcerative colitis, presents with a long history of weight gain, fatigue and depression. Examination reveals a moon-shaped face, centripetal obesity, thin skin and easy bruising. Serum cortisol levels are elevated and fail to suppress after low dose dexamethasone suppression test.

A. Congenital adrenal hyperplasia
B. Adrenal adenoma
C. Phaeochromocytoma
D. Addison's disease
E. Ectopic ACTH secretion
F. Iatrogenic Cushing's syndrome
G. Conn's syndrome
H. Carney's syndrome
I. Pseudo-Cushing's syndrome
J. Cushing's disease
K. Iatrogenic Addison's disease
L. Adrenal carcinoma
A

Iatrogenic Cushing’s syndrome
Excess glucocorticoid/cortisol

Moon face, buffalo hump, striae, acne, HTN, DM, proximal myopathy, impotence, hirsuitism, oligo/amenorrhoea

492
Q

A 21 year old man presents with rapid palpitations associated with chest tightness, severe headache, tremor and sweating. History reveals that the man had just consumed a large amount of alcohol.

A. Congenital adrenal hyperplasia
B. Adrenal adenoma
C. Phaeochromocytoma
D. Addison's disease
E. Ectopic ACTH secretion
F. Iatrogenic Cushing's syndrome
G. Conn's syndrome
H. Carney's syndrome
I. Pseudo-Cushing's syndrome
J. Cushing's disease
K. Iatrogenic Addison's disease
L. Adrenal carcinoma
A

Phaeochromocytoma: a neoplasm composed of cells similar to the chromaffin cells of the adrenal medulla.
They occur in patients of all ages, and may be sporadic or associated with MEN types IIA & IIB, neurofibromatosis type 1, von Hippel-Lindau syndrome.
Only 10% are malignant. There is a tendency to produce large amounts of catecholamine hormones (A & NA) leading to headache, palpitations, anxiety attacks, sweating and tremor, as well as life threatening high blood pressure or cardiac arrhythmias. Diagnosis is confirmed through urinary measurements of metabolites - VMA and metanephrines
Treatment: alpha blockade first then beta blockade then surgery once the patient is medically stable

493
Q

A 27 year old woman presents with a three month history of weight gain, deepening voice and secondary amenorrhoea. Examination reveals clitoromegaly, acne, greasy skin and hirsutism. Serum cortisol is grossly elevated and ACTH levels are undetectable.

A. Congenital adrenal hyperplasia
B. Adrenal adenoma
C. Phaeochromocytoma
D. Addison's disease
E. Ectopic ACTH secretion
F. Iatrogenic Cushing's syndrome
G. Conn's syndrome
H. Carney's syndrome
I. Pseudo-Cushing's syndrome
J. Cushing's disease
K. Iatrogenic Addison's disease
L. Adrenal carcinoma
A

Adrenal carcinoma - something is producing cortisol, and suppressing ACTH (negative feedback)

27 years old and only 3 month history so unlikely to be congenital.

Carcinoma is a cancer of the cortical cells, they are rare and highly aggressive and may be functional, producing steroid hormones, leading to the endocrine dysfunction seen in this case.

Adenomas are benign tumours of the adrenal cortex, and are relatively common. About 15% are functional producing glucocorticoids, mineralocorticoids and/or sex steroids causing endocrine disorders (Cushing’s syndrome, Conn’s syndrome, virilization of females and feminization of males.

494
Q

The commonest enzyme deficiency seen in CAH

A. 17-Hydroxyprogesterone
B. Normal ACTH levels
C. Hyponatreamia with Hypokalaemia
D. Deoxycortisol
E. Reduced Cortisol
F. Aldosterone
G. Hypernatreamia with Hypokalaemia
H. Hyponatreamia with Hyperkalaemia
I. 17α-Hydroxylase deficiency
J. Raised ACTH
K. Pregnanetriol
L. 11β-Hydroxylase deficiency
M. Hypernatreamia with Hyperkalaemia
N. Chromosome 6
O. 21-Hydroxylase Deficiency
A

21-Hydroxylase Deficiency
Cortisol is an adrenal steroid hormone that is required for normal endocrine function. Production begins in the second month of fetal life. Poor cortisol production is a hallmark of most forms of CAH (autosomal recessive disorder). Inefficient cortisol production results in rising levels of ACTH, which in turn induces overgrowth (hyperplasia) and overactivity of the steroid-producing cells of the adrenal cortex. The defects causing adrenal hyperplasia are congenital (i.e. present at birth).
Cortisol deficiency in CAH is usually partial, and not the most serious problem for an affected person. Synthesis of cortisol shares steps with synthesis of mineralocorticoids such as aldosterone, androgens such as testosterone, and estrogens such as estradiol. The resulting excessive or deficient production of these three classes of hormones produce the most important problems for people with CAH. Specific enzyme inefficiencies are associated with characteristic patterns of over- or underproduction of mineralocorticoids or sex steroids.

495
Q

Levels of this steroid are raised in the serum of CAH patients

A. 17-Hydroxyprogesterone
B. Normal ACTH levels
C. Hyponatreamia with Hypokalaemia
D. Deoxycortisol
E. Reduced Cortisol
F. Aldosterone
G. Hypernatreamia with Hypokalaemia
H. Hyponatreamia with Hyperkalaemia
I. 17α-Hydroxylase deficiency
J. Raised ACTH
K. Pregnanetriol
L. 11β-Hydroxylase deficiency
M. Hypernatreamia with Hyperkalaemia
N. Chromosome 6
O. 21-Hydroxylase Deficiency
A

17-Hydroxyprogesterone
Defective conversion of 17-hydroxyprogesterone to 11-deoxycortisol account for more than 95% of cases of CAH - this conversion is mediated by 21-hydroxylase due to mutations in the CYP21A2 gene.
CAH is a common, AR inherited disorder

Diagnosis:
- High serum concentration of 17 hydroxyprogesterone - most affected neonates have a concentration >105nmol/L

Classic CAH due to 21 hydroxyls deficiency results in one of 2 clinical syndromes:

  • A salt losing form
  • A non-salt losing (simple virilizing) form

Girls with either form present as neonates with ambiguous genitalia, with clitoral enlargement and a common urethral-vaginal orifice (urogenital sinus). Partial or complete fusion of the labial folds may also occur. Internal female reproductive organs (uterus & ovaries) are normal. If the ambiguous genitalia are not identified, affected females may present with a salt losing adrenal crisis at 1-2 weeks of age.

Boys present as neonates with a salt losing adrenal crisis (hyponatraemia, hyperkalaemia and failure to thrive), or as toddlers with signs of puberty (non-salt losing form). Newborn males show no overt signs of CAH, although phallic enlargement & scrotal hyper pigmentation is sometimes present

496
Q

Increased levels are seen in the urine of CAH patients

A. 17-Hydroxyprogesterone
B. Normal ACTH levels
C. Hyponatreamia with Hypokalaemia
D. Deoxycortisol
E. Reduced Cortisol
F. Aldosterone
G. Hypernatreamia with Hypokalaemia
H. Hyponatreamia with Hyperkalaemia
I. 17α-Hydroxylase deficiency
J. Raised ACTH
K. Pregnanetriol
L. 11β-Hydroxylase deficiency
M. Hypernatreamia with Hyperkalaemia
N. Chromosome 6
O. 21-Hydroxylase Deficiency
A

Pregnanetriol

Other abnormalities that may be present in either form (but mostly in the classic form) include high serum concentrations of androstenedione, 3-alpha-androstanediol glucuronide, testosterone, 21-deoxycortisol, and progesterone, and increased urinary excretion of metabolites of cortisol precursors, particularly pregnanetriol, pregnanetriol glucuronide, and 17-ketosteroids. (Pregnanetriol and its glucuronide are the major metabolites of 17-hydroxyprogesterone, and 17-ketosteroids are metabolites of androgens.)

497
Q

The sodium and potassium pattern seen in CYP21 deficiency.

A. 17-Hydroxyprogesterone
B. Normal ACTH levels
C. Hyponatreamia with Hypokalaemia
D. Deoxycortisol
E. Reduced Cortisol
F. Aldosterone
G. Hypernatreamia with Hypokalaemia
H. Hyponatreamia with Hyperkalaemia
I. 17α-Hydroxylase deficiency
J. Raised ACTH
K. Pregnanetriol
L. 11β-Hydroxylase deficiency
M. Hypernatreamia with Hyperkalaemia
N. Chromosome 6
O. 21-Hydroxylase Deficiency
A

Hyponatreamia with Hyperkalaemia

Patients with classic 21-hydroxylase deficiency have mineralocorticoid deficiency and are at risk for volume depletion, hyponatremia, and hyperkalemia. Patients are also at risk for hypoglycemia during an adrenal crisis

498
Q

A doctor suspecting his patient is suffering from CAH has just received some results that proves otherwise

A. 17-Hydroxyprogesterone
B. Normal ACTH levels
C. Hyponatreamia with Hypokalaemia
D. Deoxycortisol
E. Reduced Cortisol
F. Aldosterone
G. Hypernatreamia with Hypokalaemia
H. Hyponatreamia with Hyperkalaemia
I. 17α-Hydroxylase deficiency
J. Raised ACTH
K. Pregnanetriol
L. 11β-Hydroxylase deficiency
M. Hypernatreamia with Hyperkalaemia
N. Chromosome 6
O. 21-Hydroxylase Deficiency
A

Normal ACTH levels
In CAH inefficient cortisol production results in rising levels of ACTH, and therefore normal levels of ACTH would indicate that

499
Q

An overweight 35-year old shop-assistant visits her GP complaining of debilitating tiredness. Her periods have also become infrequent in this time. Despite it being a warm day, she wears a coat and jumper inside. On examination, she has a symmetrical painless lump on her neck. The patient has a history of well-controlled SLE.

A. Hashimoto’s thyroiditis
B. De Quervain’s thyroiditis
C. Cushing’s disease
D. Steroid abuse
E. PCOS
F. Pregnancy
G. Menopause
H. Cushing’s syndrome
I. Alcohol excess
J. Type II diabetes
K. Type I diabetes
L. Long-term insulin use
M. Simple Obesity
N. Prader-Willi Syndrome
A

Hashimoto’s thyroiditis
Auto-immune hypothyroidism: plasma cell infiltration & goitre, often elderly females, there may be an initial ‘Hashitoxicosis’ and positive auto anti-body titres for anti-thyroid peroxidase and anti-thyroglobulin anti-bodies.

Reference ranges:

  • TSH: 0.33-4.5 mu/L
  • Free T4: 10.2-22.0 pmol/L
  • Free T3: 3.2-6.5 pmol/L

Hypothyroidism: high TSH, low T4

Other auto-immune causes of hypothyroidism: Primary atrophic hypothyroidism - diffuse lymphocytic infiltration & atrophy and no goitre

Non-immune causes of hypothyroidism: iodine deficiency (commonest worldwide), thyroid agenesis/dysgenesis, post radio iodine/thyroidectomy, post-thyroiditis, drug induced (anti-thyroid drugs, lithium, amiodarone)

Treatment: T4 (levothyroxine) 50-125-250microg/day, titrated to normal TSH

De Quervains thyroiditis is a self limiting low uptake hyperthyroidism post viral infection with a PAINFUL goitre.

500
Q

A 14-year old boy visits his GP with his mother, having been recommended to by the school nurse, following an inability to attend PE lessons, due to his weight. Both seem generally unconcerned, with his mother stating that she never makes him do exercise at home anyway. The boy describes McDonalds and Playstation as his favourite hobbies. As a precaution, the GP performs a blood sample, which showed no endocrine abnormalities, but a raised cholesterol.

A. Hashimoto’s thyroiditis
B. De Quervain’s thyroiditis
C. Cushing’s disease
D. Steroid abuse
E. PCOS
F. Pregnancy
G. Menopause
H. Cushing’s syndrome
I. Alcohol excess
J. Type II diabetes
K. Type I diabetes
L. Long-term insulin use
M. Simple Obesity
N. Prader-Willi Syndrome
A

Simple Obesity: lifestyle factors and no endocrine abnormality suggest no medical cause for his obesity

Prader Willi Syndrome: deletion of 15q11-13 P (paternal copies of this region). Deletion of the same region on the maternal chromosome causes Angelman syndrome (AS).
PWS is the most common syndromic form of obesity, commonly presenting in infants and toddlers with hypotonia, feeding problems, failure to thrive (risk of asphyxia).
Common clinical features in older children and adolescents are voracious appetite, obesity, decreased cognition, hypogonadism, short stature.
Common co-morbid conditions: sleep apnoea, DM and gastric distension & rupture.
Treatment: GH unless contraindicated

501
Q

A 29-year old, clinically obese accountant presents with a relapsing of her acne, which had disappeared in her teens. On further questioning, she admits to infrequent periods over the last year, and a greater than normal growth of hair on her face. Blood tests show an elevated serum testosterone.

A. Hashimoto’s thyroiditis
B. De Quervain’s thyroiditis
C. Cushing’s disease
D. Steroid abuse
E. PCOS
F. Pregnancy
G. Menopause
H. Cushing’s syndrome
I. Alcohol excess
J. Type II diabetes
K. Type I diabetes
L. Long-term insulin use
M. Simple Obesity
N. Prader-Willi Syndrome
A

PCOS
Symptoms of hyperandrogenism, ovulatory dysfunction - hirsutism, persistent acne, pattern alopecia, menstrual irregularity, oligo-/amenorhea, excessive bleeding, elevated serum testosterone

PCOS increases the risk of developing infertility, endometrial hyperplasia & carcinoma, T2DM, metabolic syndrome, and possibly cardiovascular disease

Rotterdam diagnostic criteria

502
Q

A 33-year old overweight man complains of headaches and visual disturbances which he blames for two car accidents he has been involved in the last month. He has a ruddy appearance, and the GP notes that his weight is mainly concentrated in a ‘pot belly’. The man’s blood pressure is 150/100, and following a 48hr low –dose dexamethasone test, the patient has a cortisol of 500nm/L, and after a 48hr high-dose dexamethasone test, the cortisol was 250nmol/L

A. Hashimoto’s thyroiditis
B. De Quervain’s thyroiditis
C. Cushing’s disease
D. Steroid abuse
E. PCOS
F. Pregnancy
G. Menopause
H. Cushing’s syndrome
I. Alcohol excess
J. Type II diabetes
K. Type I diabetes
L. Long-term insulin use
M. Simple Obesity
N. Prader-Willi Syndrome
A

Cushing’s disease - pituitary adenoma - most common cause of Cushing’s syndrome. The tumours are usually micro adenomas, although 10-15% may be macro-adenomas.

May be part of MEN1 syndrome

Moon face, buffalo hump, striae, acne, HTN, DM, muscle weakness, proximal myopathy, hirsuitism, impotence

Diagnosis: low and high dose dexamethasone testing
ACTH levels will be raised, and cortisol will not suppress following low dose, but will halve following high dose

503
Q

A 65-year old overweight Indian gentleman presents to his GP, complaining of problems with his eyesight, which has become blurred recently, despite 20/20 vision for the rest of his life. On further questioning, he admits to increased urinary frequency, which he put down to his age, though recalls that he has been drinking more recently. On dip-sticking the urine, glucose was +++, and following a blood test, the blood sugar was 18mmol/L

A. Hashimoto’s thyroiditis
B. De Quervain’s thyroiditis
C. Cushing’s disease
D. Steroid abuse
E. PCOS
F. Pregnancy
G. Menopause
H. Cushing’s syndrome
I. Alcohol excess
J. Type II diabetes
K. Type I diabetes
L. Long-term insulin use
M. Simple Obesity
N. Prader-Willi Syndrome
A

Type II diabetes - chronic hyperglycaemia due to lack of insulin. Lack of insulin to drive mobilisation of energy stores into muscle, fat and liver results in hyperglycaemia, glycosuria, and over time chronic hyperglycaemia damages capillaries and markedly increases atherosclerosis.

Diagnosis:
Diabetes = fasting plasma glucose > 7mmol/L or random plasma glucose >11.1 mmol/L on 2 occasions, or 1 plus symptoms, or >11.1 2 hours following OGTT

Impaired glucose tolerance = fasting glucose

504
Q

A 15 year old overweight schoolgirl presents to her GP complaining of oligomenorrhoea. She is very self conscious and concerned about acne and excessive facial hair. Tests reveal raised serum LH and androgen concentrations.

A. BMI quantification
B. Urine dipstick
C. Chest X-ray
D. Thyroid autoantibodies
E. Ovarian ultrasound
F. Cortisol measurement
G. Exercise tolerance test
H. Lipid profile
I. Genetic testing
A

Ovarian ultrasound: PCOS
Symptoms of hyperandrogenism, ovulatory dysfunction - hirsutism, persistent acne, pattern alopecia, menstrual irregularity, oligo-/amenorhea, excessive bleeding, elevated serum testosterone

PCOS increases the risk of developing infertility, endometrial hyperplasia & carcinoma, T2DM, metabolic syndrome, and possibly cardiovascular disease

Rotterdam diagnostic criteria

505
Q

A 54 year old businessman comes to you with a wound in his foot that has failed to heal over a few weeks. You notice various other minor lesions on both feet and he tells you that he has recently ‘lost feeling’ in his legs. His BMI is 31 and his blood pressure is 145/95.

A. BMI quantification
B. Urine dipstick
C. Chest X-ray
D. Thyroid autoantibodies
E. Ovarian ultrasound
F. Cortisol measurement
G. Exercise tolerance test
H. Lipid profile
I. Genetic testing
A

Urine dipstick
Type II diabetes - chronic hyperglycaemia due to lack of insulin. Lack of insulin to drive mobilisation of energy stores into muscle, fat and liver results in hyperglycaemia, glycosuria, and over time chronic hyperglycaemia damages capillaries and markedly increases atherosclerosis.

Diagnosis:
Diabetes = fasting plasma glucose > 7mmol/L or random plasma glucose >11.1 mmol/L on 2 occasions, or 1 plus symptoms, or >11.1 2 hours following OGTT

Impaired glucose tolerance = fasting glucose

506
Q

A 38 year old secretary presents with a 3 month history of increasing fatigue and lethargy. She says that her muscles ‘don’t seem to work any more’ and you note slow relaxing reflexes on neurological examination. She complains of weight gain and seems very depressed.

A. BMI quantification
B. Urine dipstick
C. Chest X-ray
D. Thyroid autoantibodies
E. Ovarian ultrasound
F. Cortisol measurement
G. Exercise tolerance test
H. Lipid profile
I. Genetic testing
A
Thyroid autoantibodies: 
Reference ranges: 
- TSH: 0.33-4.5 mu/L
- Free T4: 10.2-22.0 pmol/L
- Free T3: 3.2-6.5 pmol/L

Hypothyroidism: high TSH, low T4

Other auto-immune causes of hypothyroidism: Primary atrophic hypothyroidism - diffuse lymphocytic infiltration & atrophy and no goitre

Non-immune causes of hypothyroidism: iodine deficiency (commonest worldwide), thyroid agenesis/dysgenesis, post radio iodine/thyroidectomy, post-thyroiditis, drug induced (anti-thyroid drugs, lithium, amiodarone)

Treatment: T4 (levothyroxine) 50-125-250microg/day, titrated to normal TSH

De Quervains thyroiditis is a self limiting low uptake hyperthyroidism post viral infection with a PAINFUL goitre.

Hashimoto’s thyroiditis
Auto-immune hypothyroidism: plasma cell infiltration & goitre, often elderly females, there may be an initial ‘Hashitoxicosis’ and positive auto anti-body titres for anti-thyroid peroxidase and anti-thyroglobulin anti-bodies.

507
Q

A 3 year old child is brought in by his parents who are concerned about his development, both physically and behaviourally. Her mother complains that he is continually eating, despite some feeding difficulties present in early infancy. On examination, you note some mild mental retardation, distinctive facial features and hypogonadism.

A. BMI quantification
B. Urine dipstick
C. Chest X-ray
D. Thyroid autoantibodies
E. Ovarian ultrasound
F. Cortisol measurement
G. Exercise tolerance test
H. Lipid profile
I. Genetic testing
A

Genetic testing - suspecting Prader Willi Syndrome: deletion of 15q11-13 P (paternal copies of this region). Deletion of the same region on the maternal chromosome causes Angelman syndrome (AS).
PWS is the most common syndromic form of obesity, commonly presenting in infants and toddlers with hypotonia, feeding problems, failure to thrive (risk of asphyxia).
Common clinical features in older children and adolescents are voracious appetite, obesity, decreased cognition, hypogonadism, short stature.
Common co-morbid conditions: sleep apnoea, DM and gastric distension & rupture.
Treatment: GH unless contraindicated

508
Q

A 60 year old ex-RAF pilot presents with breathlessness on exercise. Questioning reveals that he has difficulty sleeping and requires 3/4 pillows. He is wheezy but puts that down to a lifetime of smoking (40 pack years). Further examination reveals ankle oedema, crepitations and a displaced apex beat.

A. BMI quantification
B. Urine dipstick
C. Chest X-ray
D. Thyroid autoantibodies
E. Ovarian ultrasound
F. Cortisol measurement
G. Exercise tolerance test
H. Lipid profile
I. Genetic testing
A

Chest X-ray - clinically this chap is showing signs & symptoms of heart failure.

Macroscopically in heart failure the LV is hypertrophied and dilated with myocardial fibrosis. There may also be pulmonary oedema (pink exudate) and nutmeg liver.

LV failure: damming of blood within the pulmonary circulation causing dyspnoea, orthopnoea, PND, wheeze, fatigue leading to decreased peripheral blood pressure and flow.
Causes: IHD, HTN, aortic & mitral valve disease, non-ischaemia myocardial disease

RV failure: often secondary to LVF, but can be primarily caused by chronic severe pulmonary HTN. There is minimal pulmonary congestion but engorgement of systemic and portal venous systems, clinically seen as peripheral oedema, ascites, facial engorgement.

Investigations: BNP, CXR, ECG, Echo

509
Q

A 62 year old Indian gentleman visits his GP, complaining of pins and needles in his feet, which is worse at night. He also mentions that he has been passing urine more often than he used to, and puts this down to the fact that he has been drinking more. Blood tests reveal a fasting plasma glucose of 11.4 mmol/l.

A. Cushing's syndrome
B. Familial hypercholesterolaemia
C. Cushing's disease
D. Hashimoto's thyroiditis
E. Graves' disease
F. Alcohol excess
G. Menopause
H. PCOS
I. Type 1 diabetes
J. Type 2 diabetes
A

Type 2 diabetes: chronic hyperglycaemia due to lack of insulin. Lack of insulin to drive mobilisation of energy stores into muscle, fat and liver results in hyperglycaemia, glycosuria, and over time chronic hyperglycaemia damages capillaries and markedly increases atherosclerosis.

Diagnosis:
Diabetes = fasting plasma glucose > 7mmol/L or random plasma glucose >11.1 mmol/L on 2 occasions, or 1 plus symptoms, or >11.1 2 hours following OGTT

Impaired glucose tolerance = fasting glucose

510
Q

A 21 year old obese student visits her GP, feeling very depressed about her hair loss, which has got progressively worse since the age of 16. She is also worried about the irregularity of her periods, which has been going on for about a year and a half. Serum testosterone is raised.

A. Cushing's syndrome
B. Familial hypercholesterolaemia
C. Cushing's disease
D. Hashimoto's thyroiditis
E. Graves' disease
F. Alcohol excess
G. Menopause
H. PCOS
I. Type 1 diabetes
J. Type 2 diabetes
A

PCOS
Symptoms of hyperandrogenism, ovulatory dysfunction - hirsutism, persistent acne, pattern alopecia, menstrual irregularity, oligo-/amenorhea, excessive bleeding, elevated serum testosterone

PCOS increases the risk of developing infertility, endometrial hyperplasia & carcinoma, T2DM, metabolic syndrome, and possibly cardiovascular disease

Rotterdam diagnostic criteria

511
Q

A 31 year old research assistant presents to her GP complaining of weight gain. On questioning, she thinks that most of this weight has been put on around her middle, and her face looks much rounder. The patient has a history of ulcerative colitis that is well controlled by medication.

A. Cushing's syndrome
B. Familial hypercholesterolaemia
C. Cushing's disease
D. Hashimoto's thyroiditis
E. Graves' disease
F. Alcohol excess
G. Menopause
H. PCOS
I. Type 1 diabetes
J. Type 2 diabetes
A

Cushing’s syndrome
Iatrogenic Cushing’s syndrome
Excess glucocorticoid/cortisol

Moon face, buffalo hump, striae, acne, HTN, DM, proximal myopathy, impotence, hirsuitism, oligo/amenorrhoea

512
Q

A 45 year old sales assistant presents with extreme tiredness. On examination, the GP notes a painless lump on the front of her neck that moves up with swallowing, and that her hands are cold and dry. The patient has a history of pernicious anaemia.

A. Cushing's syndrome
B. Familial hypercholesterolaemia
C. Cushing's disease
D. Hashimoto's thyroiditis
E. Graves' disease
F. Alcohol excess
G. Menopause
H. PCOS
I. Type 1 diabetes
J. Type 2 diabetes
A

Hashimoto’s thyroiditis
Auto-immune hypothyroidism: plasma cell infiltration & goitre, often elderly females, there may be an initial ‘Hashitoxicosis’ and positive auto anti-body titres for anti-thyroid peroxidase and anti-thyroglobulin anti-bodies.

Reference ranges:

  • TSH: 0.33-4.5 mu/L
  • Free T4: 10.2-22.0 pmol/L
  • Free T3: 3.2-6.5 pmol/L

Hypothyroidism: high TSH, low T4

Other auto-immune causes of hypothyroidism: Primary atrophic hypothyroidism - diffuse lymphocytic infiltration & atrophy and no goitre

Non-immune causes of hypothyroidism: iodine deficiency (commonest worldwide), thyroid agenesis/dysgenesis, post radio iodine/thyroidectomy, post-thyroiditis, drug induced (anti-thyroid drugs, lithium, amiodarone)

Treatment: T4 (levothyroxine) 50-125-250microg/day, titrated to normal TSH

De Quervains thyroiditis is a self limiting low uptake hyperthyroidism post viral infection with a PAINFUL goitre.

513
Q

A 36 year old woman visits her GP, worried about the chest pain brought on by her daily run in the park. On examination, the GP notes dark patches on the backs of her hands. She mentions that her younger sister has the same dark patches. Serum cholesterol is 9.4 mmol/l.

A. Cushing's syndrome
B. Familial hypercholesterolaemia
C. Cushing's disease
D. Hashimoto's thyroiditis
E. Graves' disease
F. Alcohol excess
G. Menopause
H. PCOS
I. Type 1 diabetes
J. Type 2 diabetes
A

Familial hypercholesterolaemia
Dark patches are referring to plantar xanthomas. May also see tendon xanthoma, Marcus senilis cornea.
Type II is the most common cause:
- Loss of function mutations of LDL receptor or apo-beta100 genes
- Gain of function mutation of PCSK9 gene causing a decrease in LDL receptor levels
- Rarely, AR inheritance of LDLRAP1 mutation

Other causes of Primary Hypercholestrolaemia:

  • Polygenic hypercholesterolaemia: e.g. NPC1L1, HMGCR, CYP7A1 polymorphisms
  • Familial hyper-alpha-lipoproteinaemia: sometimes CTEP deficiency
  • Phytosterolaemia: mutations of ABC G5 & ABC G8

Lipid lowering therapy:

  • Statins: HMG CoA Reductase inhibitors
  • Bile acid sequestrants e.g. cholestramine
  • HDL raising drugs e.g. CTEP inhibitors
514
Q

An 11 year old boy is taken to the GP by his parents after complaining that “his wee-wee is a funny colour”. The parents reveal that their son hasn’t been too well lately, “He’s been very tired, feeling sick and has had temperature the last few days. We thought he’s just picked up a virus because he had a sore throat about 10days ago, but now that his urine has gone this smoky colour and his eyes are puffy, we thought we’d bring him in…”

A. Wilms tumour
B. Acute diffuse proliferative glomerulonephritis
C. Diabetic nephropathy
D. Alport's disease
E. Henoch-Schonlein purpura
F. Goodpasture's
G. Hypertensive renal damage
H. Polycystic kidney disease
I. Clear cell renal carcinoma
J. Wegener's granulomatosis
K. Cannonball metastases
L. Bacterial endocarditis
M. SLE
A

Acute diffuse proliferative glomerulonephritis

Acute post-infectious glomerulonephritis, occurs 1-3 weeks after streptococcal throat infection or impetigo (usually Group A-alpha haemolytic strep = strep pyogenes). The glomerular damage is thought to be due to immune complex deposition, causes haematuria (red cell casts), proteinuria, oedema, HTN
Bloods will show a raised ASOT titre and decreased C3
Biopsy:
- Light microscopy: increased cellularity of glomeruli
- Fluorescence microscope: granular deposition of IgG and C3 in GBM
- Electron microscope: subendothelial humps

Renal Pathology can be classified according to the part of the nephron it affects:

  1. Glomerulus:
    - Nephrotic syndrome: breakdown of selectivity of glomerular filtration barrier, swelling, frothy urine, proteinuria (>3.5g/24hr), hypoalbuminaemia, oedema, hyperlipidaemia
    - Nephritic syndrome: coca cola urine, haematuria, dysmorphic rbcs and red cell casts in the urine
  2. Tubules and Interstitium:
    - Acute tubular necrosis
    - Tubulointerstital nephritis: acute pyelonephritis, chronic pyelonephritis & reflux nephropathy, interstitial nephritis
  3. Blood vessels
    - Thrombotic microangiopathies (haemolytic uraemic syndrome and thrombotic thrombocytopenia purpura)
515
Q

A gentleman who presented with haemoptysis and haematuria. Histology shows the accumulation of macrophages in Bowmans capsule. Immunology reveals the patient is HLA-DR2, and possesses Anti-glomerualr basement membrane antibody.

A. Wilms tumour
B. Acute diffuse proliferative glomerulonephritis
C. Diabetic nephropathy
D. Alport's disease
E. Henoch-Schonlein purpura
F. Goodpasture's
G. Hypertensive renal damage
H. Polycystic kidney disease
I. Clear cell renal carcinoma
J. Wegener's granulomatosis
K. Cannonball metastases
L. Bacterial endocarditis
M. SLE
A

Goodpasture’s Syndrome: anti-GBM antibody against collagen4-A3), associated with HLA DRB1.
Light microscopy: crescents
Fluorescence microscopy: linear deposition of IgG in GBM

Causes of nephritic syndrome:

  1. Acute post infectious GN
  2. Rapidly progressive (crescenteric) GN: type 1 (anti-GBM anti-body - Goodpastures), type 2 (immune complex mediated - SLE, IgA nephropathy, post-infectious GN), type 3 (Pauci-immune - i.e. lack of anti-GBM or immune complex: cANCA - Wegeners granulmatosis, pANCA - microscopic polyangitis). Presents as nephritic syndrome, but oliguria and renal failure are more pronounced. Regardless of causes, all are characterised by the presence of crescents in the glomeruli.
  3. IgA nephropathy (Berger Disease)
  4. Hereditary Nephritis (Alports Syndrome)
  5. Thin Basement Membrane Disease (Benign Familial Haematuria)
516
Q

A 50 year old male with a persistant runny nose and sinusitis that is worsening. Immunology reveals circulating auto-antibodies against neutrophil cytoplasmic antigents (C-ANCA). Histology also shows the accumulation of macrophages in Bowmans capsule.

A. Wilms tumour
B. Acute diffuse proliferative glomerulonephritis
C. Diabetic nephropathy
D. Alport's disease
E. Henoch-Schonlein purpura
F. Goodpasture's
G. Hypertensive renal damage
H. Polycystic kidney disease
I. Clear cell renal carcinoma
J. Wegener's granulomatosis
K. Cannonball metastases
L. Bacterial endocarditis
M. SLE
A

Wegener’s granulomatosis
Type II hypersensitivity reaction: small and medium vessel vasculitis.
Pathology: Causes sinus problems, lung cavitation and haemorrhages, crescenteric glomerulonephritis.
Diagnosis: c-ANCA (against proteinase 3) granulomas
Treatment: corticosteroids, cyclophosphamide, co-trimoxazole

Type II hypersensitivity reaction = IgG or IgM antibody reactive with cell or matrix associated self antigen. Results in tissue damage, receptor blockade or activation.

517
Q

A 63 year old Scandanavian male presents with painless haematuria, fatigue, weight loss and fever. On examination a mass is found unilaterally in the loin. Family History reveals his father had Von Hippel-Lindau disease.

A. Wilms tumour
B. Acute diffuse proliferative glomerulonephritis
C. Diabetic nephropathy
D. Alport's disease
E. Henoch-Schonlein purpura
F. Goodpasture's
G. Hypertensive renal damage
H. Polycystic kidney disease
I. Clear cell renal carcinoma
J. Wegener's granulomatosis
K. Cannonball metastases
L. Bacterial endocarditis
M. SLE
A

Clear cell renal carcinoma

Renal Cell Carcinoma:

  • Clear cell carcinoma: well differentiated
  • Papillary carcinoma: commonest in dialysis associated cystic disease
  • Chromophobe renal carcinoma: pale, eosinophilic cells

Risk Factors: smoking, obesity, HTN, unopposed oestrogen, heavy metals, CKD

Clinical Features: costovertebral pain, palpable mass, haematuria
Paraneoplastic Syndrome: polycythaemia, hypercalcaemia, HTN, Cushing’s syndrome, amyloidosis

518
Q

A 70year old man being investigated for haematuria and loin discomfort develops dyspnoea.

A. Wilms tumour
B. Acute diffuse proliferative glomerulonephritis
C. Diabetic nephropathy
D. Alport's disease
E. Henoch-Schonlein purpura
F. Goodpasture's
G. Hypertensive renal damage
H. Polycystic kidney disease
I. Clear cell renal carcinoma
J. Wegener's granulomatosis
K. Cannonball metastases
L. Bacterial endocarditis
M. SLE
A

Cannonball metastases

519
Q

A 35-year-old alcoholic presents to A&E with confusion and maleana. On examination, he has signs of chronic liver disease and is pale and clammy. BP is 90/50mmHg and he has a weak thready pulse of 130bpm. Investigations reveal FBC: Hb 6.3g/dl, MCV 108fl, WCC 3.8 x 109/l, Plt 23 x 109/l; U&Es: Na+ 123mmol/l, K+ 4.4mmol/l, urea 27mmol/l, Cr 123umol/l.

A. Benign prostatic hypertrophy
B. Haemorrhage
C. Diabetes mellitus
D. Multiple myeloma
E. Carcinoma of the prostate
F. Henoch-Schonlein purpura
G. Diclofenac
H. IgA nephropathy
I. Renal artery stenosis
J. Rhabdomyolysis
A

Haemorrhage

Normal Values: Hb – males: 13.5-17.5g/dl Hb – females: 11.5 – 15.5g/dl MCV: 76-98fl WCC: 4-11 x 109/l Plt: 150-400 x 109/l Na+: 135-145mmol/l K+: 3.5-5.0mmol/l Urea: 2.5-6.5mmol/l Cr: 50-120umol/l CK: 23-175iu/l

520
Q

A 74-year-old man presents to his GP with increasing malaise and back pain associated with hesitancy and poor urinary stream. Subsequent investigations reveal U&Es: Na+ 134mmol/l, K+ 6.4mmol/l, urea 31.2mmol/l, Cr 1023umol/l; PSA 123nmol/l; bilateral hydronephrotic kidneys on USS.

A. Benign prostatic hypertrophy
B. Haemorrhage
C. Diabetes mellitus
D. Multiple myeloma
E. Carcinoma of the prostate
F. Henoch-Schonlein purpura
G. Diclofenac
H. IgA nephropathy
I. Renal artery stenosis
J. Rhabdomyolysis
A

Carcinoma of the prostate

Adenocarcinoma is the commonest in men over 50 years

Risk factors: age, race, family history, hormonal and environmental influences

Classically arises in the peripheral zone of the gland, and neoplastic tissue is firm (& yellow)
Local spread to bladder, and haematogenous spread is to bone (osteoblastic vertebral mets) - 50-60% present with metastases

Grading: Gleason System - based on the degree of differentiation and glandular patterns. Predominant pattern and worst pattern to give a score of 2-10.
Grade 1: 8

Diagnosis: History, examination, PSA,
Familial Factors: cancer susceptibility gene mapped to 1q24-25

Treatment: radiotherapy and surgery, external beam radiation for extra-prostatic spread

NB prostatic intraepithelial neoplasia (PIN): common finding in young men, 33% risk of carcinoma, does not cause an increase in PSA. Usual patterns: micro papillary, cribiform, flat, tufted

521
Q

A 61-year-old woman with known peripheral vascular and ischaemic heart disease is started on an ACEi by her GP. 3 weeks later she is admitted to hospital with increasing confusion and pruritis. Investigations reveal FBC: Hb 12.3g/dl, MCV 85.2fl, WCC 6.8 x 109/l, Plt 403 x 109/l; U&Es: Na+ 130mmol/l, K+ 7.4 mmol/l, urea 37mmol/l, Cr 841umol/l; urinalysis – protein ++, ketones +, blood nil.

A. Benign prostatic hypertrophy
B. Haemorrhage
C. Diabetes mellitus
D. Multiple myeloma
E. Carcinoma of the prostate
F. Henoch-Schonlein purpura
G. Diclofenac
H. IgA nephropathy
I. Renal artery stenosis
J. Rhabdomyolysis
A

Renal artery stenosis
Acute Renal Failure: rapid loss of renal function, manifesting as increased serum creatinine and urea. Complications include metabolic acidosis, hyperkalaemia, fluid overload, HTN, hypocalcaemia and uraemia.

Classification of AKI = RIFLE classification:

  • Risk: 1.5 fold increase serum Cr, or GFR drops 25%, or urine output 4 weeks
  • End stage renal disease: complete loss of kidney for >3 months

Causes:

PRE-RENAL: Most common cause of acute renal failure, no structural abnormality - caused by renal hypo-perfusion (hypovolaemia, sepsis, burns, acute pancreatitis, renal artery stenosis)

RENAL: Acute Tubular Necrosis (commonest causes ARF) - does not respond to restoration of circulating volume, Acute Glomerulonephritis, Thrombotic Microangiopathy

POST-RENAL: Obstruction to urine outflow as a result of stones, tumours (primary and secondary), prostatic hypertrophy and retroperitoneal fibrosis. Immediate relief of obstruction restores GFR fully with no structural damage. Chronic obstruction can cause structural damage e.g. glomerular ischaemia, tubular damage and long term interstitial damage

522
Q

An 84-year-old woman is found collapsed in her flat by a neighbour. She had a fall 3 days prior to her rescue and had been unable to get up or raise the alarm. On admission to hospital investigations reveal FBC: Hb 15.3g/dl, MCV 91.2fl, WCC 23.1 x 09/l, Plt 403 x 109/l; U&Es: Na+ 145mmol/l, K+ 7.1mmol/l, urea 32.9mmol/l, Cr 649umol/l; CK 23,089iu/l.

A. Benign prostatic hypertrophy
B. Haemorrhage
C. Diabetes mellitus
D. Multiple myeloma
E. Carcinoma of the prostate
F. Henoch-Schonlein purpura
G. Diclofenac
H. IgA nephropathy
I. Renal artery stenosis
J. Rhabdomyolysis
A

Rhabdomyolysis

523
Q

A 24-year-old man presents to his GP with an increasing rash over his lower limbs and buttocks associated with arthralgia and haematuria. He is admitted to the local hospital where investigations reveal deranged renal function and a raised serum IgA.

A. Benign prostatic hypertrophy
B. Haemorrhage
C. Diabetes mellitus
D. Multiple myeloma
E. Carcinoma of the prostate
F. Henoch-Schonlein purpura
G. Diclofenac
H. IgA nephropathy
I. Renal artery stenosis
J. Rhabdomyolysis
A

Henoch-Schonlein purpura

524
Q

A 65 yr old lady with ischaemic heart disease and peripheral vascular disease presents at a&e with increasing confusion, hiccups and pruritus. She was started on ACE inhibitors a week ago.

A. Wegner’s granulomatous
B. Acute tubular necrosis
C. Renal artery stenosis
D. Myeloma associated ARF
E. Acute interstitial nephritis
F. Acute glomerulonephritis
G. Renal obstruction
A

Renal artery stenosis

525
Q

A 21 yr old man is admitted to hospital with multiple fractures after his motorcycle collided into a lorry on the motorway. There is myoglobin in his urine

A. Wegner’s granulomatous
B. Acute tubular necrosis
C. Renal artery stenosis
D. Myeloma associated ARF
E. Acute interstitial nephritis
F. Acute glomerulonephritis
G. Renal obstruction
A

Acute tubular necrosis

526
Q

A 50 yr old lady with A BMI of 24 who had intermittent pain in the loin, with nausea and vomiting now has a low urine output and urinalysis shows microscopic haematuria.

A. Wegner’s granulomatous
B. Acute tubular necrosis
C. Renal artery stenosis
D. Myeloma associated ARF
E. Acute interstitial nephritis
F. Acute glomerulonephritis
G. Renal obstruction
A

Renal obstruction

527
Q

A 45 yr old man with known renal problems has bilateral leg oedema. There is blood in his urine, and urine stix testing also confirms the presence of protein. Microscopy also reveals red cell casts.

A. Wegner’s granulomatous
B. Acute tubular necrosis
C. Renal artery stenosis
D. Myeloma associated ARF
E. Acute interstitial nephritis
F. Acute glomerulonephritis
G. Renal obstruction
A

Acute glomerulonephritis

528
Q

A 25 yr old man presents to his GP with a cough, nasal discharge and swollen legs. He is extremely dehydrated and is taken to hospital.He has a high cANCA titre

A. Wegener’s granulomatous
B. Acute tubular necrosis
C. Renal artery stenosis
D. Myeloma associated ARF
E. Acute interstitial nephritis
F. Acute glomerulonephritis
G. Renal obstruction
A

Wegener’s granulomatous

529
Q

A 62-year old man presents with lethargy and tiredness. He tells you that he is ‘on painkillers for back pain after a fall at work 6 weeks ago’. On examination he is pale. Blood tests reveal urea 39.2 mmol/L (normal 1.7-8.3) and creatinine 1158 μmol/L (normal 62-106). His records show that he had a creatinine of 90 μmol/L 3 months ago.

A. Urethral stones
B. Nephrotic syndrome
C. Ureteric stones
D. Hyperkalemia
E. Chronic kidney disease
F. Thin membrane nephropathy
G. Acute interstitial nephritis
H. Hypokalemia
I. Renal acidosis
J. IgA nephropathy
A

Acute interstitial nephritis

530
Q

A 40-year old man presents acutely unwell with back pain that radiates to his groin, and nausea and vomiting. He tells you he has seen blood in his urine. On examination he is febrile.

A. Urethral stones
B. Nephrotic syndrome
C. Ureteric stones
D. Hyperkalemia
E. Chronic kidney disease
F. Thin membrane nephropathy
G. Acute interstitial nephritis
H. Hypokalemia
I. Renal acidosis
J. IgA nephropathy
A

Ureteric stones

531
Q

A poorly controlled 48-year old diabetic lady presents with a swollen face and ankles. Blood tests show albumin 3g.

A. Urethral stones
B. Nephrotic syndrome
C. Ureteric stones
D. Hyperkalemia
E. Chronic kidney disease
F. Thin membrane nephropathy
G. Acute interstitial nephritis
H. Hypokalemia
I. Renal acidosis
J. IgA nephropathy
A

Nephrotic syndrome

532
Q

A 25 year old man tells you he had dark brown urine after a sore throat and has since had microscopic haematuria. Renal biopsy reveals proliferation of the mesangium.

A. Urethral stones
B. Nephrotic syndrome
C. Ureteric stones
D. Hyperkalemia
E. Chronic kidney disease
F. Thin membrane nephropathy
G. Acute interstitial nephritis
H. Hypokalemia
I. Renal acidosis
J. IgA nephropathy
A

IgA nephropathy

533
Q

One of the complications of chronic kidney disease which has ECG features of peaked T waves, loss of the P wave and broad QRS complex

A. Urethral stones
B. Nephrotic syndrome
C. Ureteric stones
D. Hyperkalemia
E. Chronic kidney disease
F. Thin membrane nephropathy
G. Acute interstitial nephritis
H. Hypokalemia
I. Renal acidosis
J. IgA nephropathy
A

Hyperkalemia

534
Q

The gold standard for measuring glomerular filtration rate (GFR)

A. Bowman's capsule
B. Inulin
C. Serial creatinine readings
D. 20 mls/24 hrs
E. Serum creatinine
F. 30 mls/min
G. Injected radio-isotopes
H. Cystatin C
I. Glucose
J. 35 mls/min
K. Potassium exccretion
L. Iohexol
M. 40 mls/min
N. 20 mls/min
O. Phosphate excretion
P. Serum urea
A

Inulin

535
Q

Calculate the creatinine clearance for the following renal patient, following a 24 hour urine collection: urine volume 2litres; urine creatinine concentration 3mmol/l and plasma creatinine concentration 208 micro mol/l.

A. Bowman's capsule
B. Inulin
C. Serial creatinine readings
D. 20 mls/24 hrs
E. Serum creatinine
F. 30 mls/min
G. Injected radio-isotopes
H. Cystatin C
I. Glucose
J. 35 mls/min
K. Potassium exccretion
L. Iohexol
M. 40 mls/min
N. 20 mls/min
O. Phosphate excretion
P. Serum urea
A

20 mls/min

Creatine clearance = (creatinine’s urine concentration)* (Vol) / (plasma creatinine concentration) (note: the units have to match).
so, in this case:
Creatinine urine conc: 3mmol/l
Vol: 2l in 24hours: 1.39ml/minute
Plasma creatinine conc: 208micromol/l = 0.208mmol/l
so:putting this into the formula:
creat clearance = [(3mmol/l)*(1.39ml/min)] / 0.208mmol/l
creatinine clearance = 20ml/min
Hope that was clear.

536
Q

Calculate the GFR for the following renal patient, following a 24 hour urine collection: urine volume 2.7litres; urine creatinine concentration 2mmol/l and plasma creatinine concentration 107 micro mol/l.

A. Bowman's capsule
B. Inulin
C. Serial creatinine readings
D. 20 mls/24 hrs
E. Serum creatinine
F. 30 mls/min
G. Injected radio-isotopes
H. Cystatin C
I. Glucose
J. 35 mls/min
K. Potassium exccretion
L. Iohexol
M. 40 mls/min
N. 20 mls/min
O. Phosphate excretion
P. Serum urea
A

35 mls/min

537
Q

A good indicator of renal function

A. Bowman's capsule
B. Inulin
C. Serial creatinine readings
D. 20 mls/24 hrs
E. Serum creatinine
F. 30 mls/min
G. Injected radio-isotopes
H. Cystatin C
I. Glucose
J. 35 mls/min
K. Potassium exccretion
L. Iohexol
M. 40 mls/min
N. 20 mls/min
O. Phosphate excretion
P. Serum urea
A

Serial creatinine readings

538
Q

Reflects the muscle mass of a person

A. Bowman's capsule
B. Inulin
C. Serial creatinine readings
D. 20 mls/24 hrs
E. Serum creatinine
F. 30 mls/min
G. Injected radio-isotopes
H. Cystatin C
I. Glucose
J. 35 mls/min
K. Potassium exccretion
L. Iohexol
M. 40 mls/min
N. 20 mls/min
O. Phosphate excretion
P. Serum urea
A

Serum creatinine

539
Q

A 25-year-old man with a history of recurrent chest infections presents to an infectious disease specialist. A subsequent chest X-ray demonstrates widespread pulmonary infiltrates. A sputum stain using Gomori’s methenamine silver reveals characteristic cysts

A Streptococcus pneumoniae
B Moraxella catarrhalis
C Haemophilus influenzae
D Legionella pneumophila
E Mycoplasma pneumonia
F Chlamydia pneumoniae 
G Mycobacterium tuberculosis
H Pneumocystis jirovecii
I Staphylococcus aureus
A

Pneumocystis jirovecii (H) is a yeast-like fungus that primarily affects immunocompromised patients such as those with HIV. Pneumocystis pneumonia may be the presenting feature of HIV and patients with a CD4 count less than 200cells/μL are particularly susceptible. Clinically, Pneumocystis jirovecii infection presents with fever, non-productive cough, weight loss and night sweats. Chest X-ray may show signs of diffuse bilateral pulmonary infiltrates. Definitive diagnosis involves histological examination of sputum or bronchio-alveolar lavage fluid. Gomori’s methenamine silver stain reveals ‘flying saucer’ shaped cysts on microscopy

540
Q

A 54-year-old woman admitted to the respiratory ward is found to have right sided consolidation on chest X-ray. Histological examination reveals Gram-positive cocci arranged in pairs

A Streptococcus pneumoniae
B Moraxella catarrhalis
C Haemophilus influenzae
D Legionella pneumophila
E Mycoplasma pneumonia
F Chlamydia pneumoniae 
G Mycobacterium tuberculosis
H Pneumocystis jirovecii
I Staphylococcus aureus
A

Streptococcus pneumoniae (pneumococci; A) are α-haemolytic Gram-positive cocci arranged in pairs (diploccoci). As Streptococcus pneumoniae are capsulated bacteria, the Quelling reaction in which pneumo-cocci are mixed with anti-serum and methylene blue causes the capsule to swell can be visualized under the microscope. Optochin-sensitivity also differentiates pneumococcus from Streptococcus viridans (also α-haemolytic), which is optochin-insensitve. Clinically, lobar consolidation is visible on X-ray, which represents a collection of pus, bacteria and exudate in the alveoli

541
Q

A 65-year-old woman is brought into accident and emergency with severe respiratory distress. The patient’s history revealed that she had been seen by her GP due to a viral infection 2 weeks previously. Histological examination reveals Gram-positive cocci arranged in clusters

A Streptococcus pneumoniae
B Moraxella catarrhalis
C Haemophilus influenzae
D Legionella pneumophila
E Mycoplasma pneumonia
F Chlamydia pneumoniae 
G Mycobacterium tuberculosis
H Pneumocystis jirovecii
I Staphylococcus aureus
A

Staphylococcus aureus (I) are β-haemolytic Gram-positive cocci arranged in grape-like clusters. All staphylococci are also catalase posi-tive, whereas streptococci are catalase negative. Clinically, S. aureus can cause consolidation, cavitations of the lungs and empyema (pus in the pleural space). Staphylococcus aureus has a number of virulence factors including anti-immune proteins (haemolysins, leukocidins and penicil-linase) as well as tissue break-down proteins (hyaluronidase, staphylo-kinase and protease).

542
Q

A 40-year-old HIV positive man is seen by his GP. The patient admits a 4-week history of cough. The GP requests acid-fast staining of the patient’s sputum

A Streptococcus pneumoniae
B Moraxella catarrhalis
C Haemophilus influenzae
D Legionella pneumophila
E Mycoplasma pneumonia
F Chlamydia pneumoniae 
G Mycobacterium tuberculosis
H Pneumocystis jirovecii
I Staphylococcus aureus
A

Mycobacterium tuberculosis (G) is an acid-fast bacillus which is transmitted via aerosol droplets. Clinical manifestations include fever, cough (with possible haemoptysis), weight loss and night sweats. Tuberculosis is highly prevalent in HIV patients due to impaired cell-mediated immunity. Chest X-ray reveals bihilar lymphadenopathy. Most commonly, Ziehl–Neelson staining is performed on a sputum sample demonstrating acid-fast bacilli, but auramine–rhodamine staining can also be used. Mycobacterium tuberculosis, however, take approximately 6 weeks to culture, and hence faster polymerase chain reaction diagnostic tests are being developed.

543
Q

A 36-year-old engineer presents to his GP with a 1-week history of headache, myalgia and cough. Blood tests reveal hyponatraemia. A urinary antigen test is found to be positive

A Streptococcus pneumoniae
B Moraxella catarrhalis
C Haemophilus influenzae
D Legionella pneumophila
E Mycoplasma pneumonia
F Chlamydia pneumoniae 
G Mycobacterium tuberculosis
H Pneumocystis jirovecii
I Staphylococcus aureus
A

Legionella pneumophila (D) is an aerobic Gram-negative rod which causes an atypical pneumonia. It primarily affects those who work with air-conditioning units and can lead to milder Pontiac fever or more severe Legionnaire’s disease. Clinical features of legionellosis are non-specific and may include headache, myalgia, confusion, rhabdomyolysis and abdominal pain. Blood chemistry may reveal hyponatraemia, hypophosphataemia and/or deranged liver enzymes. Diagnosis involves culture of respiratory secretions on buffered charcoal yeast extract agar, although a rapid urinary antigen test can also be used.

Moraxella catarrhalis (B) are aerobic Gram-negative diploccoci. This bacterium is particularly problematic in patients with chronic lung disease and causes exacerbations of chronic obstructive pulmonary disorder (COPD). Other targets of infection include ears, eyes and central nervous system.

Haemophilus influenzae (C) are Gram-negative bacilli that cause influenza (flu) outbreaks annually. Chocolate agar is used as a culture medium. Further oxidase and catalase tests are positive.

Mycoplasma pneumoniae (E) are obligate intracellular bacteria which cause an atypical pneumonia or a mild bronchitis. A cold-agglutinin test can be used for the diagnosis. In rare cases, infection may lead to Stevenson–Johnson syndrome.

Chlamydia pneumoniae (F) are obligate intracellular bacteria which cause an atypical pneumonia. Less commonly, this infection can cause meningoencephalitis, arthritis, myocarditis and/or Guillain–Barré syndrome.

544
Q

A 34-year-old HIV-positive woman is seen in the GP clinic due to 3 days of diarrhoea, headaches and fever. History reveals the patient had recently drunk unpasteurized milk. The causative organism is found to be β-haemolytic with tumbling motility.

A Vibrio cholerae 
B Staphylococcus aureus
C Enterobacteriaecae
D Listeria monocytogenes
E Salmonella enteritidis
F Shigellae
G Campylobacter jejuni
H Giardia lamblia
I Entamoeba histolytica
A

Listeria monocytogenes (D) is a β-haemolytic anaerobic Gram-positive rod that may cause outbreaks of non-invasive gastroenteritis. Sources include refrigerated food and unpasteurized dairy products. Clinical features of listeria infection include watery diarrhoea, abdominal cramps, headaches and fever, but minimal vomiting. Listeria demonstrates ‘tumbling motility’ as a result of flagellar-driven movements. Neonates and immunocompromised patients are particularly susceptible. Invasive infection can cause more serious problems in these groups including septicaemia, meningitis and encephalitis.

545
Q

A 10-year-old girl has just returned from a summer swimming camp at Lake Windermere. She presents to accident and emergency with bloody diarrhoea and abdominal pain. Blood tests reveal anaemia and thrombocytopenia.

A Vibrio cholerae 
B Staphylococcus aureus
C Enterobacteriaecae
D Listeria monocytogenes
E Salmonella enteritidis
F Shigellae
G Campylobacter jejuni
H Giardia lamblia
I Entamoeba histolytica
A

Escherichia coli (C) is a Gram-negative rod-shaped bacterium that is a common cause of traveller’s diarrhoea in those returning from abroad. Transmission occurs via food and water that become contaminated with human faeces, as can swimming in contaminated lakes. Enterohaemorrhagic E. coli infection (serotype O157:H7) can lead to haemolytic uraemic syndrome (HUS), characterized by haemolytic anaemia, acute renal failure (uraemia) and a low platelet count (thrombocytopenia). Other diarrhoea-causing strains of E. coli include entero-toxigenic, enteropathogenic and enteroinvasive forms

546
Q

An 18-year-old on his gap year in India suddenly develops severe watery diarrhoea. Microscopy of his stool reveals no leukocytes but rods with fast movements.

A Vibrio cholerae 
B Staphylococcus aureus
C Enterobacteriaecae
D Listeria monocytogenes
E Salmonella enteritidis
F Shigellae
G Campylobacter jejuni
H Giardia lamblia
I Entamoeba histolytica
A

Vibrio cholerae (A) are comma-shaped oxidase positive bacteria, causing profuse watery diarrhoea containing no inflammatory cells on microscopy. Transmission occurs via the faecal-oral route. Vibrio cholerae colonizes the small intestinal section of the gut and secretes enterotoxin containing subunits A (active) and B (binding). B subunit binds to GM1 ganglioside on the intestinal epithelial cells. Intracellularly, there is activation of cAMP by A subunit, which causes active secretion of sodium and chloride ions; as a consequence water is lost due to the osmotic pull of NaCl

547
Q

A 25-year-old homosexual man presents to his GP with a 3-day history of foul smelling, non-bloody diarrhoea, with abdominal cramps and flatulence. Stool microscopy reveals pear-shaped organisms.

A Vibrio cholerae 
B Staphylococcus aureus
C Enterobacteriaecae
D Listeria monocytogenes
E Salmonella enteritidis
F Shigellae
G Campylobacter jejuni
H Giardia lamblia
I Entamoeba histolytica
A

Giardia lamblia (H) is a pear-shaped trophozite containing two nuclei, four flagellae and a suction disc. Transmission occurs via ingestion of a cyst from faecally contaminated water and food. Trophozites attach to the duodenum but do not invade. Instead, protein absorption is inhibited, drawing water into the lumen of the gastrointestinal tract. G. lamblia must be considered in travellers, hikers and homosexual men. Clinically, foul smelling non-bloody steatorrhoea is produced, with stool containing cysts visible on microscopy

548
Q

A 35-year-old woman presents to accident and emergency with fever, diarrhoea and signs of shock. Her husband mentions that she had attended a work colleague’s barbeque the previous day. The consultant believes superantigens are responsible for the patient’s condition.

A Vibrio cholerae 
B Staphylococcus aureus
C Enterobacteriaecae
D Listeria monocytogenes
E Salmonella enteritidis
F Shigellae
G Campylobacter jejuni
H Giardia lamblia
I Entamoeba histolytica
A

Staphylococcus aureus (B) are β-haemolytic Gram-positive cocci arranged in grape-like clusters. In the gastrointestinal tract, S. aureus produces the exotoxin TSST-1, which acts as a superantigen causing non-specific activation of T cells and subsequent release of IL-1, IL-2 and TNF-α. A massive non-specific immune response follows causing shock and multiple organ failure. Enterotoxin produced by bacteria causes vomiting and diarrhoea 12–24 hours after the culprit food has been consumed.

Salmonella typhi (E) infection, also known as enteric fever, multiplies in the Peyer’s patches of the small intestine. Clinical features include slow onset fever, constipation and splenomegaly. Rose spots are pathognomonic.

Shigellae (F) are non-motile, non-hydrogen sulphide producers. The bacteria cause dysentery via invasion of mucosal cells of distal ileum and colon as well as the production of an enterotoxin, known as Shiga toxin.

Campylobacter jejuni (G) are oxidase positive, non-motile bacteria. Transmission occurs via the faecal–oral route, generally due to contamination by dog faecal matter, causing a watery, foul smelling diarrhoea. Complications include Guillain–Barré syndrome and Reiter’s syndrome.

Entamoeba histolytica (I) is a motile trophozite. Ingestion of the cysts leads to colonization of caecum and colon, which may cause a ‘flask-shaped’ ulcer to develop. Clinical features involve dysentery, chronic weight loss and liver abscess formation

549
Q

A 45-year-old man presents to his GP with a 2-month history of headache. After a CT scan demonstrates an opacity, a lumbar puncture is performed and cerebrospinal fluid (CSF) analysis reveals a protein level of 4.5g/L (0.15–0.4), lymphocyte count 345 (1–5) and glucose 4.0mmol/L (2.2–3.3).

A Neisseria meningitides
B Herpes simplex virus-2
C Leptospira interrogans
D Listeria monocytogenes
E Cryptococcus neoformans
F Escherichia coli
G Streptococcus pneumoniae
H Borrelia burgdorferi
I Mycobacterium tuberculosis
A

Mycobacterium tuberculosis (I) may lead to a subacute or chronic meningitis. Symptoms are non-specific, including fever, headache and confusion. Focal signs may be present as a result of a cerebral granuloma. A tuberculous granuloma that occurs in the cortex of the brain, subsequently rupturing into the subarachnoid space, is termed a Rich focus. Diagnosis of tuberculous meningitis involves a lumbar puncture; the CSF appears colourless and characteristically has high protein, low glucose and raised lymphocyte levels. Nucleic acid amplification tests as well as imaging studies (CT and MRI) can be useful in the diagnostic work-up.

550
Q

A 26-year-old man has recently returned to the UK from a year of working in Africa where he was taking part in a charity farming project. He presents to accident and emergency with signs of meningism. A serological microscopic agglutination test is positive.

A Neisseria meningitides
B Herpes simplex virus-2
C Leptospira interrogans
D Listeria monocytogenes
E Cryptococcus neoformans
F Escherichia coli
G Streptococcus pneumoniae
H Borrelia burgdorferi
I Mycobacterium tuberculosis
A

Leptospira interrogans (C) causes leptospirosis (also known as Weil’s syndrome). Transmission occurs via contact with animals. Leptospira are thin aerobic spirochaetes that are tightly coiled. The first stage of infection is known as the leptospiramic phase, during which the patient suffers non-specific symptoms such as fever, headache, malaise and photophobia. In the second immune phase, IgM antibodies have formed and meningitis, liver damage (causing jaundice) and renal failure may develop. CSF examination will reveal a raised white cell count. The microscopic agglutination test is considered the gold standard for diagnosing leptospirosis.

551
Q

A 19-year-old woman who has recently started university is brought to accident and emergency with a headache and a spreading non-blanching rash. Gram-stain of a blood sample reveals the presence of Gram-negative diplococci

A Neisseria meningitides
B Herpes simplex virus-2
C Leptospira interrogans
D Listeria monocytogenes
E Cryptococcus neoformans
F Escherichia coli
G Streptococcus pneumoniae
H Borrelia burgdorferi
I Mycobacterium tuberculosis
A

Neisseria meningitides (meningococcus; A) is a Gram-negative diplococcus. Infants aged 6 months to 2 years are most at risk as well as large numbers of adults living in close quarters. Virulence factors include its capsule (antiphagocytic), endotoxin (lipopolysaccharide causes haemorrhage from blood vessels resulting in characteristic petechiae in meningococcaemia) and IgA1 protease (destroys IgA). Neisseria meningitides can lead to meningitis (headache, photophobia and neck stiffness) and meningococcaemia (signs of sepsis with spreading petechial rash). Neisseria meningitides is grown best on Thayer–Martin VCN media (only allows Neisseria species to grow)

552
Q

A 46-year-old man with a history of HIV presents to accident and emergency with neck stiffness, fever and severe photophobia. Examination of the CSF with India ink reveals yeast cells surrounded by halos

A Neisseria meningitides
B Herpes simplex virus-2
C Leptospira interrogans
D Listeria monocytogenes
E Cryptococcus neoformans
F Escherichia coli
G Streptococcus pneumoniae
H Borrelia burgdorferi
I Mycobacterium tuberculosis
A

Cryptococcus neoformans (E) is a polysaccharide encapsulated yeast that causes a subacute or chronic meningoencephalitis. It is transmitted by inhalation (the source of which is pigeon droppings). Cryptococcus neoformans is usually asyptomatic, but can be pathogenic in immuno-compromised patients such as those with HIV. As well as meningitis, C. neoformans can also cause pneumonia, skin ulcers and bone lesions. Diagnosis is made by examination of CSF; India ink staining reveals yeast cells with a surrounding halo. Cryptococcal antigen test is, however, a more sensitive test.

553
Q

A 35-year-old woman presents to her infectious disease specialist due to recurrent episodes of meningitis. During her last presentation CSF analysis reveals a protein level of 0.8g/L (0.15–0.4), lymphocyte count 290 (0–5) and glucose 2.2mmol/L (2.2–3.3)

A Neisseria meningitides
B Herpes simplex virus-2
C Leptospira interrogans
D Listeria monocytogenes
E Cryptococcus neoformans
F Escherichia coli
G Streptococcus pneumoniae
H Borrelia burgdorferi
I Mycobacterium tuberculosis
A

Herpes simplex virus 2 (HSV-2; B) is the most common cause of viral meningitis of all the herpes family. HSV-2 is transmitted via sexual contact or via the mother during birth. The virus infects mucosal epithelial cells or lymphocytes; retrograde transport occurs from peripheral nerves to ganglion. Viral causes of meningitis can be diagnosed on examination of CSF; it appears colourless, with a raised lymphocyte level, moderately raised protein and normal glucose concentration. Recurrent aseptic meningitis (Mollaret’s meningitis) can be caused by both HSV-1 and HSV-2

Listeria monocytogenes (D) is a Gram-positive bacillus. Infection usually occurs in neonates, the immunocompromised and elderly. Manifestations include meningitis, encephalitits, pneumonia and septicaemia

Escherichia coli (F) is a Gram-negative bacillus. The K1 antigen of the bacterium as well as a lack of circulating IgM are responsible for severe meningitis in neonates.

Streptococcus pneumoniae (G) is a Gram-positive α-haemolytic diplo-coccus. It is the most common cause of meningitis in adults together with N. meningitides.

Borrelia burgdorferi (H) is a Gram-negative zoonotic spirochaete that causes Lyme disease. In the late stages of disease the patient will experience arthritis, peripheral neuropathy and/or encephalopathy.

554
Q

A 28-year-old woman sees her GP complaining of fever, lower abdominal pain and painful intercourse. Vaginal swabs are sent for a nucleic acid amplification test which reveal sexually transmitted bacteria that can also cause lymphogranuloma venereum

A Treponema pallidum
B Klebsiella granulomatis
C Neiserria gonorrhoeae
D Trichomonas vaginalis
E Candidia albicans
F Chlamydia trachomatis
G Bacterial vaginosis
H Haemophilus ducreyi
I Herpes simplex virus 2
A

Chlamydia trachomatis (F) is a small Gram-negative obligate intracellular bacterium, causing the sexually transmitted infection chlamydiosis. It has an affinity towards columnar epithelia that line mucous membranes. Serovars D–K cause genital chlamydiosis (as well as opthalmia neonatorum) resulting in dyspareunia, dysuria and vaginal/penile discharge. Serovars L1, L2 and L3 cause lymphogranuloma venereum, defined by a painless papule or ulcer on the genitals which heals spontaneously; the bacteria migrate along regional lymph nodes leading to lymphadenopathy.

555
Q

A 68-year-old man presents to his GP with a gumma on his nose. On examination, the patient is found to have pupils that accommodate to light but do not react. The man admits to unprotected sexual intercourse during his youth

A Treponema pallidum
B Klebsiella granulomatis
C Neiserria gonorrhoeae
D Trichomonas vaginalis
E Candidia albicans
F Chlamydia trachomatis
G Bacterial vaginosis
H Haemophilus ducreyi
I Herpes simplex virus 2
A

Treponema pallidum (A) causes syphilis. Syphilis has three clinical stages: primary, secondary and tertiary. Primary syphilis is defined by a firm painless chancre that appears approximately 1 month after sexual contact and resolves within a few weeks. Secondary syphilis is a bacteriaemic stage during which a widespread rash forms with lymphadenopathy. Tertiary syphilis occurs decades after the primary infection and involves multiple organs: gummatous lesions on skin and bone, aneurysm of the aortic arch, peripheral neuropathy, tabes dorsalis and Argyll–Robertson pupils.

556
Q

A 35-year-old man presents to an infectious disease specialist with a painful penile ulcer and associated unilateral lymphadenopathy of the inguinal nodes. A swab of the ulcer is cultured on chocolate agar

A Treponema pallidum
B Klebsiella granulomatis
C Neiserria gonorrhoeae
D Trichomonas vaginalis
E Candidia albicans
F Chlamydia trachomatis
G Bacterial vaginosis
H Haemophilus ducreyi
I Herpes simplex virus 2
A

Haemophilus ducreyi (H) is a Gram-negative coccobacillus that causes a tropical ulcer disease (chancroid) and is contracted by sexual transmission. Chancroid is characterized by a painful genital ulcer that leads to unilateral painful swollen inguinal lymph nodes. Infected lymph nodes may rupture releasing pus. The differential diagnosis for genital ulcers includes syphilis (painless ulcer with bilateral painless lymphadenopathy), herpes simplex virus 1 and 2 (vesicles that eventually break down) and lymphogranuloma venereum (slowly developing painless inguinal lymph nodes). Haemophilus ducreyi can be cultured on chocolate agar.

557
Q

A 28-year-old woman sees her GP complaining of fever, lower abdominal pain and painful intercourse. A vaginal swab is taken and subsequent Gram-staining reveals Gram-negative diplococci.

A Treponema pallidum
B Klebsiella granulomatis
C Neiserria gonorrhoeae
D Trichomonas vaginalis
E Candidia albicans
F Chlamydia trachomatis
G Bacterial vaginosis
H Haemophilus ducreyi
I Herpes simplex virus 2
A

Neiserria gonorrhoeae (gonococcus; C) is an intracellular Gram-negative diplococcus that causes gonorrhoea. Virulence factors allow gonococci to evade phagocytosis and adhere to the non-ciliated epithelium of the fallopian tubes. In both men and women N. gonorrhoeae causes urethritis which presents with dysuria and purulent discharge (with associated dyspareunia in women). Long-term complications include pelvic inflammatory disease in women and epididymitis, prostititis as well as urethral stricture in men. Systemic invasion of bacteria causes pericarditis, endocarditis, meningitis and/or septic arthritis. Diagnosis involves Gram stain and culture on Thayer–Martin VCN medium, or PCR.

558
Q

A 35-year-old woman presents to her GP with a 2-week history of a fishy odorous vaginal discharge, which occurs especially after sexual intercourse. Microscopy of the discharge reveals clue cells.

A Treponema pallidum
B Klebsiella granulomatis
C Neiserria gonorrhoeae
D Trichomonas vaginalis
E Candidia albicans
F Chlamydia trachomatis
G Bacterial vaginosis
H Haemophilus ducreyi
I Herpes simplex virus 2
A

Bacterial vaginosis (BV; G) is caused by an imbalance in the naturally occurring bacterial flora of the vagina and is a condition associated with sexual activity (not transmitted). A ‘fishy’ smelling white–cream vaginal discharge is characteristically produced. Diagnosis involves obtaining vaginal swabs. A litmus test will indicate loss of acidity with a pH greater than 4.5 (normal vaginal pH = 3.8–4.2). If a sample of the discharge is visualized under a microscope with sodium chloride, clue cells will be seen.Klebsiella granulomatis (B) is a Gram-positive rod that causes the ulcer-ating sexually transmitted infection donovanosis. It is diagnosed using giemsa stain of biopsy, which reveals Donovan bodies.Trichomonas vaginalis (D) is a flagellated protozoan that causes vaginal discharge and urethritis in humans. It is otherwise asymptomatic and can be diagnosed by wet preparation microscopy, culture or PCR.Candida albicans (E) is a fungal infection that causes candidiasis (thrush). Superficially, infection causes redness, itching and discharge from the vagina. In immunocompromised patients, infection can involve the oesophagus as well as causing candidaemia.Herpes simplex virus 2 (HSV-2; I) causes genital herpes. Infection causes fluid-filled blisters to form over the genital area.

559
Q

A 54-year-old man presents to his GP with a 1-week history of fever, cough and fatigue. On examination his respiratory rate is 20 breaths per minute and he is normotensive. Subsequent chest X-ray reveals right lower lobe consolidation.

A Amoxicillin 
B Doxycycline
C Co-amoxiclav IV
D Meropenam
E Chloramphenicol
F Cefotaxime
G Vancomycin
H Trimethoprim
I Flucloxacillin
A

Amoxicillin (A) is a β-lactam antibiotic which inhibits enzymes responsible for cell wall synthesis, leading to osmotic lysis of the bacteria. As a result, β-lactams are ineffective against bacteria that lack cell walls such as Mycoplasma spp. and Chlamydia spp. In this case, amoxicillin is the best choice antibiotic to treat mild community acquired pneu-monia. It is also useful in the treatment of urinary tract infection (UTI), Listeria meningitis, endocarditis prophylaxis and protection against Streptococcus pneumoniae in asplenic patients. Major side effects can be divided into allergic (anaphylaxis) and non-allergic (Steven–Johnson syndrome) consequences

560
Q

A 38-year-old man presents to accident and emergency with an inflamed and swollen right leg. He mentions that he had cut the same leg 2 days previously playing football. A swab of the area isolates Staphylococcus aureus

A Amoxicillin 
B Doxycycline
C Co-amoxiclav IV
D Meropenam
E Chloramphenicol
F Cefotaxime
G Vancomycin
H Trimethoprim
I Flucloxacillin
A

Flucloxacillin (I) is a β-lactam antibiotic that is especially effective against Gram-positive bacteria that produce β-lactamase, for example S. aureus. Just like amoxicillin, flucloxacillin inhibits cell wall synthesis. Indications for its use include staphylococcal skin infections such as cellulitis (in this case), folliculitis and mastitis as well as pneumonia (adjunct), osteomyelitis, septic arthritis, endocarditis and prophylaxis in surgery. A rare side effect of flucloxacillin is cholestatic jaundice which may develop weeks after treatment is stopped

561
Q

A 34-year-old woman presents to her GP with lower abdominal pain and dysuria. A dipstick of her urine reveals the presence of protein, white cells and nitrites.

A Amoxicillin 
B Doxycycline
C Co-amoxiclav IV
D Meropenam
E Chloramphenicol
F Cefotaxime
G Vancomycin
H Trimethoprim
I Flucloxacillin
A

Trimethoprim (H) is an inhibitor of folate metabolism; it impairs synthesis of DNA by interfering with folic acid metabolism. It is used in the treatment of uncomplicated UTIs. Trimethoprim should be used with caution in patients with megaloblastic anaemia due to its interaction with folate. Side effects of trimethoprim include thrombocytopenia, megaloblastic anaemia and hyperkalaemia (via antagonism of sodium channels in the distal convoluted tubule of nephrons). Trimethoprim combined with another folate inhibitor, sulphamethoxazole, forms co-trimoxazole, which is used in the treatment of Pneumocystis jirovecii infection

562
Q

A 56-year-old man is being cared for on the surgical ward after excision of a segment of his bowel after being diagnosed with colorectal carcinoma. The following day the surgical wound site is found to be inflamed. The patient has a fever and his blood pressure is slowly declining. Blood cultures reveal Gram-positive cocci arranged in clusters that are resistant to β-lactam antibiotics.

A Amoxicillin 
B Doxycycline
C Co-amoxiclav IV
D Meropenam
E Chloramphenicol
F Cefotaxime
G Vancomycin
H Trimethoprim
I Flucloxacillin
A

Vancomycin (G) is the drug of choice in cases of methicillin-resistant Staphylococcus aureus infections (MRSA). Vancomycin is a glycopeptide antibiotic that inhibits cell wall synthesis. It is too large to traverse the cell wall of Gram-negative bacteria and hence is primarily targeted to Gram-positive bacteria. Side effects include renal failure, ototoxicity, blood disorders, rash and anaphylaxis. Due to the potential side effects, serum drug levels must be monitored. Vancomycin is also a second-line antibiotic in the treatment of Clostridium difficile infection.

563
Q

An 18-year-old woman student presents to accident and emergency with head-ache, neck stiffness and photophobia. CT scan reveals no raised intracranial pres-sure. Gram-negative diploccoci are visualized on Gram-staining of the patient’s CSF.

A Amoxicillin 
B Doxycycline
C Co-amoxiclav IV
D Meropenam
E Chloramphenicol
F Cefotaxime
G Vancomycin
H Trimethoprim
I Flucloxacillin
A

Cefotaxime (F) is a third generation cephalosporin and is the drug of choice in treating Neisseria meningitidis, which is the most common cause of meningitis in the UK. Cefotaxime is a β-lactam antibiotic and therefore inhibits cell wall synthesis. If meningitis is suspected in the community, the patient should be started on benzyl-penicillin until they are transferred to a secondary care unit. Cefotaxime is also useful in the treatment of pyelonephritis, sepsis secondary to hospital acquired pneumonia and soft tissue infections.

Doxycycline (B) is a tetracycline antibiotic that interferes with protein synthesis by binding to the 30S ribosomal subunit. It is used in COPD exacerbations, sexually transmitted infections (gonorrhoea and chlamydia) and acne.

Co-amoxiclav IV (augmentin; C) is the combination of amoxicillin and clavulinic acid (β-lactamase inhibitor). It is usually prescribed when β-lactamase-producing strains are suspected and other treatment has failed.

Meropenem (D) is a broad-spectrum carbapenem antibiotic which is used in the management of severely sick patients, usually in inten-sive care. It is resistant to β-lactamase, including extended spectrum β-lactamase producing bacteria.

Chloramphenicol (E) acts on 50S ribosomes to inhibit protein synthesis. It is used in cases of Rocky Mountain spotted fever & conjunctivitis. Side effects include aplastic anaemia

564
Q

A 38-year-old man presents to his GP with vomiting, mild fever and loss of appetite. He admits to travelling to sub-Saharan Africa 2 months previously. On examination the patient is evidently jaundiced.

A Human immunodeficiency virus (HIV)
B Epstein–Barr virus (EBV)
C Hepatitis B virus
D Cytomegalovirus (CMV)
E Hepatitis D virus
F Varicella zoster virus
G Hepatitis C virus
H Human herpes virus 8
I Influenza virus
A

Hepatitis B virus (HBV; C) is a double-stranded DNA virus that is prevalent in sub-Saharan Africa. It is transmitted via sexual contact, contaminated blood products, intravenous drug use as well as vertical transfer from mother to child during child birth. The virus has an incubation period of 2–6 months with 80 per cent of infections remaining acute and 20 per cent becoming chronic with risk of cirrhosis and hepatocellular carcinoma. HBV antigens include HBsAg (surface antigen), HBcAg (core antigen) and HBeAg (soluble antigen)

565
Q

A 39-year-old homosexual man is referred to the gastroenterology department for an oesophogastroduodenoscopy (OGD) due to recent onset odynophagia. The OGD reveals multiple raised white plaques that can be removed by endo-scopic scraping.

A Human immunodeficiency virus (HIV)
B Epstein–Barr virus (EBV)
C Hepatitis B virus
D Cytomegalovirus (CMV)
E Hepatitis D virus
F Varicella zoster virus
G Hepatitis C virus
H Human herpes virus 8
I Influenza virus
A

Human immunodeficiency virus (HIV; A) possesses single-stranded RNA as well as enzymes (reverse transcriptase, integrase and protease) in its core. HIV is transmitted via sexual intercourse, blood products, intra-venous drug use and vertically from mother to child. HIV infects CD4+ Tcells; within the cell the RNA undergoes reverse transcription to make DNA which is integrated into the host DNA; the virus then becomes latent or buds to infect further. AIDS (CD4+ count

566
Q

A 15-year-old girl presents to her GP complaining of a sore throat, fever, fatigue and loss of appetite. A blood film demonstrates atypical lymphocytes and monospot test is positive.

A Human immunodeficiency virus (HIV)
B Epstein–Barr virus (EBV)
C Hepatitis B virus
D Cytomegalovirus (CMV)
E Hepatitis D virus
F Varicella zoster virus
G Hepatitis C virus
H Human herpes virus 8
I Influenza virus
A

Epstein–Barr virus (EBV; B) primarily infects B lymphocytes, binding via a complement receptor. Transmission involves person-to-person transfer through close contact. EBV is associated with glandular fever (infectious mononucleosis) which causes pharyngitis, lymphadenopathy, fever, splenomegaly and hepatomegaly. Rare sequelae include thrombocytopenia and erythema multiforme. EBV can also cause Hodgkin’s lymphoma (latent reactivation of EBV), Burkitt’s lymphoma and nasopharyngeal cancers. It is diagnosed on blood film (atypical lymphocytes), monos-pot test (positive heterophil antibody test) and/or EBV antibodies in the blood

567
Q

A 68-year-old woman presents to her GP after a 3-day history of fever, cough, headache and nasal congestion. The doctor believes her symptoms are due to a virus that binds to sialic acid receptors.

A Human immunodeficiency virus (HIV)
B Epstein–Barr virus (EBV)
C Hepatitis B virus
D Cytomegalovirus (CMV)
E Hepatitis D virus
F Varicella zoster virus
G Hepatitis C virus
H Human herpes virus 8
I Influenza virus
A

Influenza virus (I) is part of the orthomyxoviridae group of viruses and causes epidemics of influenza annually. The influenza virus causes primary pneumonia as well as delayed secondary bacterial pneumonia and otitis media in immunocompromised patients. It is a spherical virion with haemagglutinin (HA) and neuraminidase (NA) glycoproteins on the surface. HA binds to sialic acid receptors present in the upper respiratory tract; viral RNA is subsequently inserted into the host cell and HA is cleaved by clara cell tryptase. NA cleaves neuraminic acid, a component of protective mucin; as a result the protective barrier is disrupted exposing sialic acid receptor sites beneath. NA also has a role facilitating the release of newly formed influenza virions.

568
Q

A 55-year-old man who is being treated for lung cancer with chemotherapeutic agents sees his oncologist for a routine check-up. There is a rash in a dermatomal pattern on the patient’s forehead; the patient complains that there is a burning sensation in the distribution of the rash.

A Human immunodeficiency virus (HIV)
B Epstein–Barr virus (EBV)
C Hepatitis B virus
D Cytomegalovirus (CMV)
E Hepatitis D virus
F Varicella zoster virus
G Hepatitis C virus
H Human herpes virus 8
I Influenza virus
A

Varicella zoster virus (VZV; F) is a droplet-spread herpes virus that causes chickenpox in children and shingles in adults. Chickenpox is characterized by fever, malaise and a rash (erythematous base with fluid top) that spreads over the body. Complications include secondary bacterial infection and encephalitis. VZV remains dormant in the dorsal root ganglia and may reactivate in states of immunosuppression. The most common symptom is neuralgia which occurs in a dermatomal distribution; other manifestations include encephalitis, Guillain–Barré syndrome, facial palsy and progressive outer retinal necrosis

Cytomegalovirus (CMV; D) can be transmitted vertically from contaminated blood products and transplant organs (50% risk of infection in seropositive donor and seronegative recipient). It is especially prevalent in the immunocompromised.

Hepatitis D virus (HDV; E) is a helical single-stranded RNA virus that requires hepatitis B co-infection. HDV produces an acute-on-chronic picture in HBV patients and puts them at higher risk of cirrhosis and subsequent liver failure.

Hepatitis C virus (HCV; G) is a single-stranded RNA virus that is trans-ferred via contaminated blood products and vertically from mother to child during birth. Twenty per cent remain acute, while 80 per cent become chronic.

Human herpes virus 8 (HHV-8; H) is transmitted primarily via saliva as well as semen to a lesser extent. It causes Kaposi’s sarcoma, primary effusion lymphoma and multicentric Castleman’s disease

569
Q

A 40-year-old man presents to an infectious disease specialist with a 4-month history of weight loss, fever and malaise. On examination the patient has lymphadenopathy. His CD4 count is found to be 289 copies/μL. The patient is started on lamivudine, ritonavir and one other drug

A Acyclovir
B Oseltamivir
C Interferon-α
D Zidovudine
E Gancylcovir
F Lamivudine
G Efivarenz
H Ritonavir
I Adamantadine
A

Zidovudine (D) is a nucleoside reverse transcriptase inhibitor (NRTI) used in the treatment of HIV/AIDS (as well as prevention of vertical transmission from infected mothers). Treatment is commenced once the CD4 count falls below 350 copies/μL. Zidovudine works by inhibiting the action of the enzyme reverse transcriptase, preventing the conversion of HIV RNA to DNA, which consequently cannot be incorporated into the host DNA. Side effects include anaemia, neutropenia, hepatic and cardiac dysfunction as well as myopathy. The standard treatment regimen involves the use of two nucleoside reverse transcriptase inhibitors (NRTIs) and a non-nucleoside reverse transcriptase inhibitor (NNRTI; Efivarenz) or a protease inhibitor (PI; Ritonavir).

570
Q

A 38-year-old intravenous drug user presents to an infectious disease special-ist with a 1-week history of fever and malaise; on examination hepatomegaly is noted. The patient is found to be HBeAg positive and is subsequently commenced on lamivudine and one other drug

A Acyclovir
B Oseltamivir
C Interferon-α
D Zidovudine
E Gancylcovir
F Lamivudine
G Efivarenz
H Ritonavir
I Adamantadine
A

Interferon-α (IFN-α; C) is a protein that is used in the treatment of hepatitis B; it potentiates the immune system to fight active viral infection. IFN-α acts on the JAK-STAT pathway; IFN-α binds to the IFN-α receptor, causing phosphorylation of STAT1 and STAT2, which subse-quently form a complex with IRF9 (a transcription factor), leading to the synthesis of anti-viral proteins. A NRTI and IFN-α is the standard treatment for hepatitis B infection. Pegylated-IFN-α is used in the treat-ment of hepatitis C; similar to IFN-α, the addition of polyethylene glycol increases the half life of the drug.

571
Q

A 25-year-old man presents to his GP with a 3-day history of fever, cough, body aches and severe headaches. The patient is told to rest and drink plenty of fluids. However, he returns the following week stating his symptoms have not improved and is started on a drug that acts on viral neuraminidase.

A Acyclovir
B Oseltamivir
C Interferon-α
D Zidovudine
E Gancylcovir
F Lamivudine
G Efivarenz
H Ritonavir
I Adamantadine
A

Oseltamivir (B) is a viral neuraminidase inhibitor used in the treatment of influenza. Osteltamivir is in fact a pro-drug; once metabolized in the liver the active form GS4071 is produced. Once a newly formed influenza virion is produced, the surface viral protein haemagglutinin is bound to sialic acid receptors along the upper respiratory tract. Neuraminidase is normally responsible for cleaving the haemagglutinin–sialic acid receptor bond, hence facilitating the release of newly formed virions. Therefore, inhibiting neuraminidase activity prevents further viral replication

572
Q

A 3-year-old girl diagnosed with severe combined immunodeficiency is due to undergo a bone marrow transplant. She is given a drug as prophylaxis against cytomegalovirus infection

A Acyclovir
B Oseltamivir
C Interferon-α
D Zidovudine
E Gancylcovir
F Lamivudine
G Efivarenz
H Ritonavir
I Adamantadine
A

Gancyclovir (E) is a 2 ́-deoxyguanosine analogue used in the treat-ment of cytomegalovirus (CMV) infection. It is the first line drug for the prophylaxis of CMV in bone marrow transplant patients. 2 ́-deoxy-guanosine is phosphorylated to the triphosphate form, which prevents viral DNA polymerase from elongating viral DNA and therefore inhibits CMV replication. Gancyclovir can cause bone marrow toxicity; it may therefore be prescribed together with granulocyte-colony stimulating factor (G-CSF). Gancyclovir is also used in the treatment of human herpes virus 6 (HHV-6) and Epstein–Barr virus infection.

573
Q

A 28-year-old woman presents to her GP with cold sores dotted across her lower lip. She is started on a medication that inhibits DNA polymerase function to speed the healing processes

A Acyclovir
B Oseltamivir
C Interferon-α
D Zidovudine
E Gancylcovir
F Lamivudine
G Efivarenz
H Ritonavir
I Adamantadine
A

Acyclovir (A) is a guanosine analogue anti-viral drug used primarily in the treatment of herpes simplex virus infections (HSV-1 and HSV-2). It is converted to acyclo-guanosine monophosphate (acyclo-GMP) by viral thymidine kinase. Acyclo-GMP is further phosphorylated to acyclo-guanosine triphosphate (acyclo-GTP). Acyclo-GTP is incorporated into the viral DNA strand, terminating the chain and stopping DNA polymerase from functioning. Aciclovir is also indicated for the treatment of varicella zoster, Epstein–Barr virus and cytomegalovirus infections (with decreasing efficacy)

Lamivudine (F) is an NRTI (analogue of cytidine). It leads to the inhibition of reverse transcriptase and is therefore effective for the treatment of hepatitis B and HIV.

Efavirenz (G) is an NNRTI used in the treatment of HIV. The drug causes inhibition of the reverse transcription enzyme.

Ritonavir (H) is a protease inhibitor used in the management of HIV. Ritonavir inhibits viral assembly by preventing the cleavage of proteins that belong to newly formed virions.

Amantidine (I) is an M2 ion channel inhibitor preventing the uncoating of influenza virions and therefore inhibiting entry into susceptible cells.

574
Q

A 38-year-old man with known HIV presents to his GP with a 1-week history of white coloured creamy deposits inside his mouth. The patient is prescribed an oral nystatin wash.

A Cryptoccus neoformans
B Pityriasis versicolour
C Aspergillus flavus
D Histoplasma capsulatum
E Phialophora verrucosa
F Tinea capitis
G Sporothrix schenckii
H Tinea corporis
I Candida albicans
A

Candida albicans (I) can affect both immunocompetent and immuno-compromised hosts. In the immunocompetent host, clinical features range from oral thrush (creamy-white patches with red base over mucous membranes of mouth; treated with nystatin) to vaginitis (vaginal inflammation, pruritis and discharge; speculum examination reveals patches of cottage cheese-like clumps fixed to vaginal wall). In immuno-compromised patients, C. albicans infection leads to oesophagitis, characterized by odynophagia. Candidaemia can lead to severe flu-like symptoms and can be diagnosed by testing for blood β-D-glucan (a component of fungal cell walls)

575
Q

A 45-year-old man with known HIV presents to accident and emergency with headache, nausea, confusion and fever. Investigation of the patient’s CSF with India ink stain reveals yeast cells surrounded by a halo.

A Cryptoccus neoformans
B Pityriasis versicolour
C Aspergillus flavus
D Histoplasma capsulatum
E Phialophora verrucosa
F Tinea capitis
G Sporothrix schenckii
H Tinea corporis
I Candida albicans
A

Cryptococcus neoformans (A) is an encapsulated yeast that is transmitted via inhaled spores from pigeon droppings. It is usually asymptomatic in most cases. 75% of cases occur in immunocompromised patients, characterized by the development of sub-acute or chronic meningitis. Cryptococcal meningitis is fatal without treatment due to the associated cerebral oedema and brainstem compression. Diagnosis is made by CSF analysis with India ink stain which reveals yeast cells surrounded by a halo (polysaccharide capsule). A cryptococcal antigen test can also be used which offers higher sensitivity

576
Q

A 35-year-old woman presents to her GP with hyperpigmented spots on her back. Scrapings of the affected areas reveal a ‘spaghetti with meatballs’ appearance under the microscope.

A Cryptoccus neoformans
B Pityriasis versicolour
C Aspergillus flavus
D Histoplasma capsulatum
E Phialophora verrucosa
F Tinea capitis
G Sporothrix schenckii
H Tinea corporis
I Candida albicans
A

Pityriasis versicolor (B) is a chronic fungal infection caused by Malassezia furfur, characterized by hypopigmentation (in patients with dark skin tones) and hyperpigmentation (in patients with pale skin tones). Spots affect the back, underarm, arms, legs, chest, neck and rarely the face. Microscopic investigation of the M. furfur with potassium hydroxide reveals a ‘spaghetti with meatballs’ appearance. Wood’s light may also reveal an orange fluorescence in some cases

577
Q

A 48-year-old HIV positive man who has recently migrated from sub-Saharan Africa presents to accident and emergency with chest pain, shortness of breath, fever and cough. A chest X-ray demonstrates a spherical opacity in the upper left lung field

A Cryptoccus neoformans
B Pityriasis versicolour
C Aspergillus flavus
D Histoplasma capsulatum
E Phialophora verrucosa
F Tinea capitis
G Sporothrix schenckii
H Tinea corporis
I Candida albicans
A

Aspergillus flavus (C) is a fungus that commonly grows on stored grains and can cause a spectrum of disease. Allergic reaction in the airways may cause allergic broncho-pulmonary aspergillosis (ABPA) which occurs due to an IgE mediated type I hypersensitivity reaction leading to bronchospasm and eosinophilia. Infection in pre-formed lung cavities (for example in TB patients) may lead to a fungal ball visible on chest X-ray (aspergilloma). Invasive aspergillosis is a chronic necrotizing infection that may occur in neutropenic patients (chemotherapy) or patients with end stage AIDS (CD4 count

578
Q

A 32-year-old gardener presents to his GP with small raised lesions on his left arm. He remembers working in a garden a few days previously which had been swamped with rose-thorns

A Cryptoccus neoformans
B Pityriasis versicolour
C Aspergillus flavus
D Histoplasma capsulatum
E Phialophora verrucosa
F Tinea capitis
G Sporothrix schenckii
H Tinea corporis
I Candida albicans
A

Sporothrix schenckii (Rose garderner’s disease; G) is a fungus found in soil and plants that causes sporotrichosis. A prick by thorns causes nodular lesions to appear on the surface of the skin. Initially the lesions will be small and painless; left untreated they become ulcerated. Infection may also spread to joints, bone and muscle by this route. Inhalation of spores may lead to pulmonary disease and systemic infection may lead to central nervous system involvement. Treatment options include itraconazole, fluconazole and oral potassium iodide.

Histoplasma capsulatum (D) is a fungus transmitted by inhaled spores; it is highly prevalent in the Mississippi River region. Although mostly subclinical, a minority of infections will proceed to a chronic progressive lung disease.

Phialophora verrucosa (E) is a copper coloured soil saprophyte found on rotting wood that causes chromoblastomycosis. Infection is characterized by a warty lesion resembling a cauliflower.

Tinea capitis (F) is a cutaneous dermatophyte fungal infection of the scalp leading to scaly red lesions with loss of hair. It primarily affects children. Infection is characterized by an expanding ring on the scalp.

Tinea corporis (H) is also known as ringworm. It is a cutaneous dermatophyte fungal infection affecting the trunk, arms and legs. It is identified by raised red rings

579
Q

A 45-year-old man has returned to the UK from a holiday to France. A week later he presents with flu-like symptoms, drenching sweats, a recurring fever and is beginning to complain of a lower back pain. He admits to have brought back some local cheeses on visits to regional farms

A Psittacosis
B Rabies
C Brucellosis
D Q fever
E Leptospirosis
F Mycobacterium marinium
G Lyme disease
H Cat scratch disease
I Rocky mountain spotted fever
A

Brucellosis (C) is a Gram-negative rod-shaped bacterium that is harboured by cattle (Brucella abortus), goats (B. melitensis), pigs (B. suis) and dogs (B. canis). Brucella spp. are transmitted by inhalation, unpasteurized dairy produce and direct contact with animals. Symptoms include fever, myalgia, arthralgia, tiredness and in chronic cases may be associated with depression. Diagnosis is made by blood culture on Castaneda medium. Complications include granulomatous hepatitis (histology of liver biopsy demonstrates granulomata), endocarditis, oseteomyelitis and thrombocytopenia.

580
Q

A 36-year-old man presents to his GP with a painful right knee. He states that he visited the Prairie regions of Canada a month previous to this episode and states that his wife had mentioned there was a red rash on his back; on examination a target shaped rash is observed

A Psittacosis
B Rabies
C Brucellosis
D Q fever
E Leptospirosis
F Mycobacterium marinium
G Lyme disease
H Cat scratch disease
I Rocky mountain spotted fever
A

Lyme disease (G) is caused by the spirochaete Borrelia burgdorferi which is transmitted by the Ixodes ticks harboured by certain species of mice and deer. Initial symptoms include erythema migrans (a spreading annular skin lesion with a characteristic target-shaped appearance), malaise, fever and musculoskeletal pain. Several weeks after the primary infection, the patient may experience neurological (headache, meningitis and Bell’s palsy) and cardiac (arrhythmias, myocarditis and pericarditis) effects. Late features include arthralgia and arthritis.

581
Q

A 38-year-old sewage worker presents to his GP with 1-week history of flu-like symptoms with diarrhoea. A microscopic agglutination test reveals the diagnosis.

A Psittacosis
B Rabies
C Brucellosis
D Q fever
E Leptospirosis
F Mycobacterium marinium
G Lyme disease
H Cat scratch disease
I Rocky mountain spotted fever
A

Leptospirosis (Weil’s disease; E) is a zoonotic disease caused by Leptospira interrogans which is harboured by both wild and domestic animals. It is transmitted via drinking water that has become contaminated with the urine of infected animals; as a result those involved in water-sports and sewage workers are at particular risk. Leptospirosis is characterized by an influenza-like disease with/without gastrointestinal symptoms. Diagnosis can be made by ELISA, PCR or microscopic agglutination test (MAT). Long-term complications include hepatitis and renal failure.

582
Q

A 48-year-old man presents to his GP with flu-like symptoms. On examination the patient has a maculopapular rash on his trunk. The patient also shows an area where a vague bite mark is visible.

A Psittacosis
B Rabies
C Brucellosis
D Q fever
E Leptospirosis
F Mycobacterium marinium
G Lyme disease
H Cat scratch disease
I Rocky mountain spotted fever
A

Rocky Mountain spotted fever (I) is caused by Rickettsia spp. infection, a Gram-negative bacteria, most prevalent in North and South America. It is harboured in small wild rodents and domestic animals (transmitted to humans by ticks). Rickettsia bacteria invade the endothelial lining of capillaries causing a vasculitis. Clinical features include headache, fever, myalgia, vomiting and confusion. Late signs include a rash that is maculopapular and/or petechial on the distal parts of the limbs which then spreads to the trunk and face. Rocky Mountain spotted fever may lead to thrombocytopenia, hyponatraemia and/or elevated liver enzymes

583
Q

A 34-year-old bird handler presents to his GP with a few days’ history of fever, mild cough and myalgia. The patient states that his shop had recently taken a new shipment of parrots from Central America. Giemsa staining of the patient’s sputum reveals cytoplasmic inclusions.

A Psittacosis
B Rabies
C Brucellosis
D Q fever
E Leptospirosis
F Mycobacterium marinium
G Lyme disease
H Cat scratch disease
I Rocky mountain spotted fever
A

Psittacosis (A) is a zoonotic infectious disease caused by Chlamydia psittaci which is contracted from a wide variety of birds (parrots, pigeons and cockatiels to name a few). Human symptoms mainly involve a severe pneumonia (with or without hepatitis). Although the patient may report mild symptoms, the X-ray will generally appear to show severe pathology. Diagnosis is made by visualizing cytoplasmic inclusions on Giemsa or fluorescent antibody stained sputum or biopsy sample.

Rabies (B) is a viral zoonotic infectious disease caused by a bite or scratch, usually from an infected dog or bat. Infection leads to progressive and incurable encephalitis, hydrophobia and muscle spasm. Cerebral Negri bodies (inclusion bodies) are pathognomonic.

Q fever (D) is caused by Coxiella burnetti. Transmission occurs by inhalation of aerosols of urine, faeces or amniotic fluid from infected livestock.

Mycobacterium marinium (F) is harboured by fish and is transmitted by a bite or injury from the fin. Infection causes nodules to appear on the elbows, knees and feet.

Cat scratch disease (H) is caused by Bartonella spp. bacteria transmitted by bites from cats. Classically, infection results in tender and swollen lymph nodes with headache and backache. Atypically, infection may result in Parinaud’s oculoglandular syndrome.

584
Q

A 10-year-old boy is brought to see the GP by his mother as he has recently developed parotid swelling associated with a fever. Blood tests reveal a raised amylase level. The boy’s mother reveals that his immunization schedule is not complete as they were living in Tunisia at the time.

A Rubella
B Syphilis
C Measles
D Hepatitis B
E Mumps
F Listeria monocytogenes
G Cytomegalovirus
H Haemophilus influenzae
I HIV
A

Mumps (E) is spread by droplets in the air which travel via the lungs to parotid tissue and subsequently to distant sites. Clinical features of infection consist of fever, malaise and transient hearing loss. Parotitis is characteristic of mumps infection with unilateral or bilateral swelling and pain on chewing. Plasma amylase levels may be elevated as a result of inflammation of the salivary glands. Complications such as viral meningitis, orchitis/oophoritis, mastitis and arthritis may result from long-standing infection. The MMR vaccine given at 12–18 months has drastically reduced the incidence of mumps

585
Q

A 3-week-old baby develops vomiting and is feeding poorly. On examination he has a reduced level of consciousness and an arched back. Analysis of the CSF reveals the presence of Gram-positive rods

A Rubella
B Syphilis
C Measles
D Hepatitis B
E Mumps
F Listeria monocytogenes
G Cytomegalovirus
H Haemophilus influenzae
I HIV
A

Listeria monocytogenes (F) is a β-haemolytic anaerobic Gram-positive rod that can cause meningitis in the neonate to 3 months age group. Listeria monocytogenes may be transmitted vertically from mother to baby in utero (due to the ingestion of infected food by the mother) or during birth (transvaginal transfer). Early signs of meningitis are non-specific in the age group affected (fever, poor feeding, vomiting, seizures and reduced consciousness) whereas late signs include a bulging fontanelle, neck stiffness, opisthotonos (arched back), Brudzinski and Kernig signs positive as well as meningococcaemia.

586
Q

A 3-year-old girl presents to the GP with a cough, fever and runny nose. On examination, the child has white spots scattered on the buccal mucosa. Her mother admits that she denied her child a certain vaccine due to scares presented by the media.

A Rubella
B Syphilis
C Measles
D Hepatitis B
E Mumps
F Listeria monocytogenes
G Cytomegalovirus
H Haemophilus influenzae
I HIV
A

Measles (C) is a viral respiratory system infection caused by the genus Morbillivirus. Infection presents with cough, coryza, conjunctivitis and/or a discrete maculopapular rash. White spots on the buccal mucosa (Koplik spots) are pathognomonic for measles. Complications of measles infection may involve the respiratory (pneumonia and tracheitis) and neurological (febrile convulsions and encephalitis) systems. Subacute sclerosing panencephalitis (SSPE) may occur several years after the primary infection; infection persists in the central nervous system leading to loss of neurological function, dementia and eventually death.

587
Q

A 4-year-old boy presents to accident and emergency with a reduced level of consciousness, headache and neck stiffness. Analysis of the CSF reveals the presence of Gram-negative rods. The child’s mother reveals that his immunization record is not complete as they have only migrated from Ethiopia recently

A Rubella
B Syphilis
C Measles
D Hepatitis B
E Mumps
F Listeria monocytogenes
G Cytomegalovirus
H Haemophilus influenzae
I HIV
A

Haemophilus influenzae (H) is a Gram-negative rod shaped bacterium that causes meningitis in children older than 3 months who have not been vaccinated. Other organisms that cause meningitis in older children include Streptococcus pneumoniae and Neisseria meningitidis. Diagnosis involves culture of the bacteria using chocolate agar, with subsequent Gram-stain and microscopy. Latex particle agglutination and PCR are more sensitive and specific investigative tests. The Haemophilus influenzae type B (Hib) vaccine has dramatically reduced Hib-related meningitis; the first dose is given when the child is 8 weeks old.

588
Q

An 8-month old girl is seen by a paediatrician due to concerns about developmental delay. On examination cataracts are noted in both eyes. Echocardiography reveals a patent ductus arteriosus.

A Rubella
B Syphilis
C Measles
D Hepatitis B
E Mumps
F Listeria monocytogenes
G Cytomegalovirus
H Haemophilus influenzae
I HIV
A

Rubella (German measles; A) is a viral infection which can be congenital or acquired. Congenital rubella syndrome (CRS) occurs in a develop-ing fetus if the mother has contracted rubella in her first trimester. CRS is characterized by sensorineural deafness, eye abnormalities (cataracts, glaucoma, retinopathy) and congenital heart disease (patent ductus arteriosus). Other associations include microcephaly and developmen-tal delay. Acquired rubella is transmitted via the respiratory route. Characteristically, a rash appears on the face which spreads to the trunk and disappears after a few days.

Hepatitis B (D) may be vertically transmitted from mother to child during childbirth. Mothers who are HBeAg positive are especially at risk of transmitting the virus; infection may become chronic in 20 per cent of cases.

Syphilis (B) can be congenitally transmitted. Symptoms that may develop in the first few years of life include hepatosplenomegaly, rash, fever and neurosyphilis. Long-term complications include saddle-nose deformity, Higoumenakis’ sign (unilateral enlargement of the clavicle) and Clutton’s joints (symmetrical joint swelling).

Cytomegalovirus (G) may be transmitted in the perinatal period from infected mothers. Presentation may include low birth weight, microcephaly, seizures and/or petechial rash.HIV (I) transmission may occur in utero or during birth. Infected mothers are advised to take Zidovudine during pregnancy; the infant is required to take Zidovudine for 6 weeks following birth

589
Q

A 24 year-old Asian man presents with a persistent cough. A sputum sample is taken and cultured on Lowenstein–Jensen medium, appearing as brown, granular colonies after several weeks. The organism implicated is:

A Coxiella burnetti
B Streptococcus pneumoniae
C Mycobacterium tuberculosis
D Legionella pneumophilia
E Mycobacterium leprae
A

Mycobacterium tuberculosis

This gentleman is most likely suffering from Mycobacterium tuberculosiswhich characteristically presents with a persistent cough, haemoptysis, fever, night sweats and weight loss. Lowenstein–Jensen medium is a growth medium used to culture Mycobacterium species at 37°C. The most common indication for its use is to culture Mycobacterium tuber-culosis (C), where it appears as brown coffee-coloured (buff), granular bread crumb-like colonies (rough) which often stick to the bottom of the growth plate and are hard to remove (tough). This is often remembered as ‘buff, rough and tough’. It usually takes approximately 4–6 weeks to obtain these visible colonies, an important fact to remember when treat-ing patients. Another characteristic feature is the formation of serpen-tine rods from chains of cells in smears. There are a few other important points to remember about staining results for Mycobacterium tuberculosis. They are classified as acid-fast bacteria, because they are resistant to losing their colour during staining procedures. The Ziehl–Neelson stain is the most common method used to stain this type of bacterium, and they appear bright red against a blue background. The stain con-tains carbofuchsin, a pink dye which binds to the unique mycolic acids found in the mycobacterium cell wall. Another stain that can be used for acid-fast bacilli is the auramine stain, which also binds to mycolic acids to give a yellow fluorescence.

Mycobacterium leprae (E) is another acid-fast bacillus, responsible for causing leprosy. It can be detected using a skin biopsy or nasal smear, using Fite stain. It has proven difficult to culture on artificial cell media, but instead has been grown on mouse foot pads and nine-banded arma-dillos. Symptoms of leprosy include hypopigmented skin lesions, nod-ules and loss of sensation.

Coxiella burnetii (A) causes Q fever, which was first described in abbatoir workers. It is an obligate intracellular Gram-negative bacte-rium found in farm animals and pets, and is transmitted by aerosol or contact with animal products like milk or faeces. It manifests as flu-like symptoms, but can progress to an atypical pneumonia or less often a granulomatous hepatitis. Typical chest X-ray features include a ground glass appearance. It does not grow on Lowenstein–Jensen medium.

Streptococcus pneumoniae (B) is a Gram-positive coccus causing a lobar pneumonia, and can be differentiated from Streptococcus viridans using an optochin test. Streptococcus pneumoniae and viridans are alpha haemolytic, but Strep. pneumoniae are optochin sensitive whilst Strep. viridans are optochin resistant. It also does not grow on Lowenstein–Jensen medium.

Legionella pneumophilia (D) is a Gram-negative bacterium that causes Legionnaire’s disease. It typically presents initially with flu-like symp-toms, progressing to a productive cough and sometimes diarrhoea and confusion due to hyponatraemia. It can be detected using a urinary antigen test, or by culture on buffered charcoal yeast extract, but not Lowenstein–Jensen medium

590
Q

A 24-year-old HIV-positive Asian man presents with a cough. A Mantoux test is performed. After 72 hours, the wheal diameter is measured at 5.8mm. This indicates:

A He has never been exposed to TB
B He has been exposed to TB
C He has had a BCG vaccination in the past
D He has latent TB which is now reactivated
E It is not possible to say

A

B - He has been exposed to TB
The Mantoux test is a diagnostic test for tuberculosis. It consists of an intradermal injection of 0.1mL of purified protein derivative (PPD) tuberculin, which is a glycerol extract of the bacillus. The diameter of the induration that subsequently forms is read 48–72 hours later, but one also needs to take into account the patient’s risk of being infected with TB and of progression to disease if they were infected in interpret-ing the result. The Centers for Disease Control and Prevention provide the following classification for the skin test:

1 An induration of 5mm or more is considered positive in:
•Patients with HIV
•A recent contact of a person with TB disease
•People with fibrotic changes on chest radiograph consistent with prior TB
•Patients with organ transplants
•People who are immunosuppressed for other reasons (for exam-ple taking the equivalent of >15mg/day of prednisone for 1 month or longer)

2 An induration of 10mm or more is considered positive in:
•Recent immigrants (

591
Q

An 18-year-old university student develops a lower lobe pneumonia, with a raised white cell count and CRP. A sputum culture reveals a Gram-positive optochin-sensitive diplococcus. The most likely causative agent is:

A Staphylococcus aureus
B Streptococcus viridans
C Mycoplasma pneumoniae
D Streptococcus pneumoniae
E Haemophilus influenzae
A

Streptococcus pneumoniae
It is useful to remember that streptococci can essentially be divided into alpha haemolytic, beta haemolytic and non-haemolytic groups. Alpha haemolytic streptococci can be further divided into Strep. pneumo-niae (D) and Strep. viridans (B) according to their optochin sensitivity (amongst other factors). The beta haemolytic streptococci are further classified according to Lancefield groups A, B, C, F and G. Finally the non-haemolytic streptococci include the enterococci. Optochin is an antibiotic used to differentiate Strep. pneumoniaefrom other alpha haemolytic streptococci such as Strep. viridans. The pneumococcus will typically produce a zone of inhibition around an optochin disc, indicating that it is sensitive to the antibiotic, whereas Strep. viridans is resistant to it so its growth will not be affected. This can be remembered using the mnemonic ‘OVeR PS’ (Optochin – Viridans Resistant, Pneumococci Sensitive). As the organism in the question is optochin sensitive, the answer is (D).

Staphylococcus aureus would not be optochin sensitive, so (A) is not the correct answer. It is a Gram-positive bacterium that obtained its name because of the golden yellow colonies that form when grown on blood agar plates (aurum is Latin for gold).

Mycoplasma pneumoniae(C) generally causes an atypical pneumonia in children and young adults. It is called the ‘walking pneumonia’ because patients can some-times continue walking around despite suffering from it, and many are asymptomatic. The clinical features of this pneumonia can on occasion be relatively insignificant compared to the radiological findings. It too is not optochin sensitive so is not the correct answer here.

Haemophilus influenzae (E) is a Gram-negative bacillus so can be easily eliminated as a potentially correct answer here. Clinically the pneumonia caused by Haemophilus is not easily distinguished from that caused by Strep. pneumoniae.

592
Q

A 58-year-old Caucasian alcoholic man presents to his GP with a history of sudden onset high fever, flu-like symptoms and, thick, blood stained sputum. A chest X-ray is arranged which shows marked upper lobe cavitation. The most likely causative agent is:

A Klebsiella pneumoniae
B Mycobacterium tuberculosis
C Staphylococcus aureus
D Moraxella catarrhalis
E Pnemocystis jirovecii
A

Klebsiella pneumoniae (A) is a Gram-negative rod-shaped bacillus that can cause an atypical pneumonia, most frequently in alcoholics. It can result in sudden, severe systemic upset in these patients, and the pro-duction of thick, purulent and sometimes blood-stained sputum said to resemble ‘red-currant jelly’. Haemoptysis occurs more frequently with K. pneumoniae than with pneumonia caused by other bacteria. Radiological features can include upper lobe consolidation, with marked cavitation as described in the question. It is more likely to lead to complications such as lung abscesses and empyemas than pneumonias caused by Strep. pneumoniae.

Mycobacterium tuberculosis (B) can cause haemoptysis and upper lobe cav-itation. Whilst a plausible answer, the indication that the patient is alco-holic, coupled with the characteristic description of thick, blood-stained sputum, is more characteristic of Klebsiella. Also note the absence of other typical indicators of tuberculosis such as night sweats, weight loss and Asian ethnicity.

Pneumonia caused by Staphyloccous aureus (C) can follow an influenza virus infection, and may result in the formation of abscesses. The radiological findings can include extensive cavitation, and thin walled abscesses may break down to give a cystic appearance. Whilst S. aureuscould potentially lead to the above clinical picture, Klebsiella is again more likely to give blood-stained sputum in an alcoholic.

Moraxella catarrhalis (D) is a Gram-negative diplococcus that may cause pneumonia in patients with underlying lung disease such as chronic obstructive pulmonary disease (COPD). It can be implicated in an infective exacerbation of their condition in these patients. It can also lead to laryngitis, otitis media and sinusitis. Given the presumed absence of an underlying lung condition in this patient, it is less likely to be the causative agent than Klebsiella.

Pneumocystis jirovecii (E) tends to affect immunocompromised patients, and used to be called pneumocystis pneumonia (PCP). Typical clinical features include severe shortness of breath, dry cough and the pres-ence of bilateral crackles. If you get an HIV patient whose saturations drop on exertion in a question, think about this organism! It does not normally give bilateral cavitation on a chest X-ray, but instead would characteristically show peri-hilar interstitial infiltrates, giving a ‘bat’swing’ appearance. Histology may reveal classic boat-shaped organisms, and the diagnostic stain used is the silver stain.

593
Q

A 27-year-old intravenous drug user presents with a 2-week history of fevers, weight loss and a systolic murmur. The most likely causative agent is:

A Streptococcus viridans
B Candida albicans
C Staphylococcus aureus
D Streptococcus bovis
E Kingella
A

Staphylococcus aureus

Infective endocarditis can be classified into two broad categories: acute and sub-acute. Acute infective endocarditis is less common, and the most likely causative agent is Staphylococcus aureus (C). It can affect both normal and abnormal valves, and can typically be found in intravenous drug users, such as the patient described. The tricuspid valve is most commonly affected in these cases, which can easily be remembered as this is the first valve that the bacteria will encounter following injection into a vein. Therefore, (C) is the correct answer in this case.

The other category of infective endocarditis is the sub-acute form, which is more common. It is most often caused by Streptococcus viridans (A), and usually occurs on damaged valves. Patients typically present with an insidious onset of fevers, night sweats, and weight loss. Other clinical features can result from emboli, such as cerebral emboli causing a stroke, or less commonly recurrent pulmonary embo-li in right sided endocarditis. If asked about the signs of endocarditis, steer away from mentioning the rare eponymous signs first! You can remember the signs as rules of two: two signs in the hands include clubbing and splinter haemorrhages, two signs in the abdomen are splenomegaly and microscopic haematuria, and two signs elsewhere can include new or changing heart murmurs and embolic phenomena. Remember that the most common valves to be affected are the aortic and mitral valves.

Fungi such as Candida albicans (B) are a much less common cause of endocarditis. They can also be found in intravenous drug users, but this is much less likely than Staphylococcus aureus. They can include Aspergillus and Candida species, and usually cause a sub-acute picture.

Strep. bovis (D) has also been implicated as a rarer cause of infective endocarditis, and is part of the natural flora of the bowel. If found in a patient with endocarditis, a colonoscopy may be important as its presence is associated with colonic malignancies.

The HACEK organisms consist of a Gram-negative group which includes Haemophilus parainfluenzae, Aggregatibacter, Cardiobacterium hominis, Eikenella corrodens and Kingella (E). They typically result in a culture negative endocarditis.

Whilst all of the above answers are possible, the single best answer is Staphylococcus aureus because the patient is an intravenous drug user and has developed an acute form of the disease

594
Q

A patient with shingles is treated with an anti-viral. The drug used is a guano-sine analogue and acts as a substrate for viral thymidine kinase. The most likely drug she has been given is:

A Foscarnet
B Lamivudine
C Cidofovir
D Acyclovir
E Ganciclovir
A

Acyclovir (D) is a guanosine analogue that causes obligate chain ter-mination when it attaches to DNA. It is phosphorylated by the enzyme thymidine kinase found in viruses, which is far more effective than the cellular thymidine kinase for this process. This means that normal cells which are not infected by the virus are not affected as much by acyclovir, as there is no viral thymidine kinase present. The acyclovir monophosphate which then forms is further phosphorylated to a diphos-phate and then to a triphosphate by the cellular thymidine kinase. This triphosphate potently inhibits viral DNA polymerase, leading to chain termination. It is effective against the herpes viruses, for example herpes simplex and herpes zoster which causes shingles.

Foscarnet (A) is a pyrophosphate analogue used for the treatment of CMV, and works by inhibiting viral DNA polymerase via a different mechanism to acyclovir. It can also be used for herpes zoster, but is usually a second line if the infection is resistant to acyclovir. It does not require phosphorylation by viral thymidine kinase.

Lamivudine (B) is used for the treatment of hepatitis B and HIV, not shingles. It is a cyti-dine analogue, and acts as a potent nucleoside analogue reverse tran-scriptase inhibitor.

Cidofovir (C) works by inhibiting viral DNA poly-merase, and is used for cytomegalovirus retinitis. It is not dependent on phosphorylation by viral enzymes in the way acyclovir is.

Ganciclovir (E) is thought to be the drug of choice for treating cytomegalovirus infections. Unlike acyclovir it is also phosphorylated by uninfected cells, so it is more toxic in comparison.

595
Q

According to the UK immunization schedule, which vaccine should be given to a 2-month-old baby who has already received DTaP (diptheria, tetanus, pertussis), IPV (polio) and Hib (Haemophilus influenzae type B) vaccines?

A Pneumococcus
B MMR
C Meningitis C
D BCG
E Hepatitis B
A

Pneumococcus

The current UK immunization schedule is as follows:

  • Two months: Hib/IPV/DTaP/PCV•Three months: Hib/IPV/DTaP/Men C
  • Four months: Hib/IPV/DTaP/PCV/Men C
  • Twelve months: Hib/Men C
  • Thirteen months: MMR/PCV
  • Three years four months old or soon after: MMR/DTaP/IPV
  • 13–18 years: Booster Diptheria and tetanus/IPVDTap stands for diphtheria, tetanus and acellular pertussis, and IPV is the inactivated poliovirus vaccine.

This can be challenging to remember but is often asked about in exams. One way of remembering it is this: H = Hib, I = IPV, D = DTaP, P = PCV, Men = Men C, M = MMR:
•Two months: H.I.De your little baby Please.
•Three months: the first three as above, but meningitis C instead of PCV
•Four months: all the above
•Twelve months: now we want to H.I.De them from Men. C?!
•Thirteen months: MMR/Please!
•Three years four months old or soon after: things are starting to D.I.M now so we need to booster it!
•13–18 years: I’D like my teenager to go back for a jab!

596
Q

A 24-year-old sexually active woman presents to her GP with dysuria. A urinary tract infection is diagnosed. Which of the following is the most likely causative agent?

A Enterobacter
B Escherichia coli
C Klebsiella pneumoniae
D Staphylococcus saphrophyticus
E Proteus mirabili
A

The most common cause of a urinary tract infection in all groups of patients is Escherichia coli (B). Do not be misled by the fact that the patient is a young, sexually active woman. The E. coli bacterium is a lactose-fermenting Gram-negative rod. It has various properties that aid its pathogenesis: a flagellum to enable it to move upstream, fim-brae so that it can adhere to the urothelium, and haemolysin to form pores in white blood cells. It also has a protective capsule called the K-antigen.

The other lactose fermenting organisms are Klebsiella and Enterobacter, whilst non-lactose fermenting organisms include Proteus and Pseudomonas. Lactose fermenting organisms turn MacConkey agar pink, whereas non-lactose fermenters do not. Useful investigations for urinary tract infections can include a urine dipstick to look for nitrites and leukocytes, and urine cultures looking for a bactiuria of greater than 105 colony forming units.

Enterobacter (A) is less commonly implicated in urinary tract infections. They are Gram-negative, lactose-fermenting, rod-shaped bacteria. They tend to cause opportunistic infections in immunocompromised hosts, so it is unlikely to be the causative agent here.

Klebsiella (C) is also a Gram-negative, non-motile, lactose-fermenting organism. Patients that are immunocompromised or have indwelling catheters are at increased risk of Klebsiella infections.

Staphylococcus saphrophyticus (D) is a Gram-positive cause of urinary tract infections. It is the second most common cause of urinary tract infections in young, sexually active women after E. coli. It is coagulase positive like other staphylococcal species, but catalase negative unlike S. aureus.

Proteus mirabilis (E) is an example of a non-lactose fermenting bacterium. It is not a likely cause of this patient’s UTI, but is more common in young boys and hospitalized patients. It has the ability to split urea into ammonia, which raises the pH of the urine. This predisposes to the formation of phos-phate stones, particularly staghorn calculi

597
Q

A 44-year-old woman patient returns from her holiday in India with a 2-day history of watery, offensive diarrhoea, bloating, excessive flatulence and abdomi-nal pain. The GP obtains a stool sample. Microscopy reveals a flagellate pear-shaped protozoan. The most likely organism implicated is:

A Bacillus cereus
B Salmonella enteritidis
C Giardia lamblia
D Entamoeba histolytica
E Cryptosporidium parvum
A

Giardia lamblia (C) is a flagellated protozoan parasite which causes giardiasis. It attaches to the small bowel wall, but does not invade it. If you can remember this fact, you will find it easier to remember that it interferes with absorption, and so leads to the classic symptoms of weight loss, flatulence, chronic diarrhoea and bloating, as in the patient in this question. Because it does not invade the small bowel wall, the diarrhoea is not bloody but it is watery. Microscopy of a stool sample may show a pear-shaped protozoan. If you imagine a pear making you feel very bloated, you will remember this fact which often crops up in questions! Very rarely, a string test may be done if other methods to detect the parasites fail but there is still a high index of clinical suspi-cion. A gelatine capsule attached to a long string is swallowed, with the end of the string remaining outside the mouth and taped to the patient’s cheek. It remains in place for about 4–6 hours, before the end is exam-ined under the microscope. Treatment of giardiasis is typically oral metronidazole.

Bacillus cereus (A) causes food poisoning with vomiting and sometimes diarrhoea, usually from re-heated food such as rice. The symptoms appear quite quickly, within a couple of hours of consumption. The bacteria produce spores, which are activated by changes in temperature (such as re-heating or refrigerating food) to produce heat-labile and heat-stabile toxins. The patient in question has no vomiting, no history of eating re-heated food and has unusual symptoms of flatulence and bloating which would not normally be associated with this infection.

Salmonella enteritidis (B) is an important cause of food poisoning, usually from contaminated meat and eggs. Do not confuse this with enteric fevers which are systemic illnesses caused by Salmonella typhi and paratyphi. The incubation period is 12–48 hours, after which the patient may present with nausea, vomiting and malaise, after which follows abdominal pain and diarrhoea. This can occasionally become bloody. Most cases are self limiting within approximately 3 days. Again the patient in the question does not fit this picture.

Entamoeba histolytica (D) causes amoebic dysentery. Trophozites invade the bowel wall and lead to classical ‘flask shaped ulcers’, causing bloody diarrhoea. Trophozites can enter the portal vein and lead to liver abscesses too. The entamoeba cysts classically have four nuclei. One way to remember these somewhat obscure facts that appear in exam questions is to picture four runners ‘ent-ering’ a race, and the winner gets a silver flask!

Cryptosporidium parvum (E) is another parasite that causes acute, watery diarrhoea which is not usually bloody. This is of particular concern in immunocompromised patients, such as those with AIDS, where the severe dehydration can be fatal

598
Q

A 21-year-old medical student returns from her elective in India with a history of abdominal cramps, vomiting, fevers and profuse, watery stools which she describes as resembling ‘rice-water’. The GP obtains a stool sample. Analysis reveals curved, comma shaped organisms that were shown to be oxidase positive. The most likely organism implicated is:

A Hepatitis A
B Clostridium difficile
C Yersinia enterocolitica
D Campylobacter jejuni
E Vibrio cholerae
A

Vibrio cholerae (E) causes profuse watery diarrhoea and vomiting. It can in fact be one of the most rapidly fatal infectious illnesses if not treated, because of the severe dehydration causing circulatory shock. The bacteria produce a toxin which has an A and a B subunit. It is the A subunit which activates a G protein and results in the production of cAMP, which initiates the secretion of Na+, K+, Cl-, and HCO3- into the small intestine lumen. Most people only have a mild illness which simply resembles other diarrhoeal illnesses. Sometimes, as in this case, the diar-rhoea is profuse and is known colloquially as ‘rice-water’ stools because of its appearance. The diagnosis is predominantly clinical, but if stool culture is performed the classical appearance will be of curved shaped, oxidase-positive organisms. You can remember this as the Cholera Comma! Rehydration therapy forms the mainstay of treatment.

Hepatitis A (A) can cause diarrhoea in travellers to developing countries, but the stools are not ‘rice-water’, and the organisms would not be demonstrated on microscopy as above. The more common symptoms are flu-like, and for this reason it can be mistaken for influenza. Jaundice can also develop with tender hepatomegaly. Treatment is mainly supportive – there is no specific anti-viral for hepatitis A.

Clostridium difficile (B) is an anaerobic, Gram-positive rod that produces enterotoxins and cytotoxins. It can cause antibiotic-associated diarrhoea. A severe form is known as pseudomembranous colitis, of which a complication is toxic megacolon. Diagnosis is usually by demonstrating the presence of the Clostridium difficile toxin in faeces using cell-culture assay or immunoassay.

Enterocolitis caused by Yersinia enterocolitica (C) is also characterized by fever, abdominal pain and diarrhoea. It is a Gram-negative rod, so would not result in the comma-shaped organisms found here. It is normally self-limiting.

Campylobacter jejuni (D) is a common cause of gastroenteritis. Whilst vomiting is not a prominent feature, the diarrhoea that occurs can be bloody. Sources can include unpasteurized milk and meat – remember this as the food and drink you might take with you camping! It is a Gram-negative bacterium that is also oxidase positive, but has a cork-screw rather than a comma appearance. Diagnosis is made by stool culture. It is usually self-limiting, but sometimes oral erythromycin is used for treatment.

The oxidase test mentioned in the question is used to determine if bacteria produce a certain type of oxidase enzyme. Important oxidase positive organisms that sometimes appear in questions are: Pseudomonas, Neisseria, Campylobacter, Helicobacter, Moraxella, Vibrio, and Legionella. This can be remembered using the mnemonic: ‘Pu.N.C.H. Me Very Lightly!’

599
Q

A 35-year-old HIV-positive man presents to his GP complaining of a general feeling of tiredness, weight loss and night sweats. On examination there is hepato-splenomegaly and hyperpigmentation of the skin. The most likely diagnosis is:

A Visceral leishmaniasis
B Cutaneous leishmaniasis
C Mucocutaneous leishmaniasis
D Malaria
E Schistosomiasis
A

Visceral leishmaniasis
Leishmaniasis is transmitted by phlebotomine sandflies and occurs in Africa, America and the Middle East. Visceral leishmaniasis (A) is also known as ‘Kala-azar’, and the most common clinical features include fever and splenomegaly. Hepatomegaly, skin hyperpigmentation and dry warty skin occur less frequently, and bone marrow invasion can result in pancytopenia. It can be mistaken for malaria, which is dangerous as it can be fatal if left untreated.

L. donovani and L. infantum are thought to cause the disease in Africa, Asia and Europe, whilst L. chagasi is implicated in South America.

The most common form of leishmaniasis is called cutaneous leishmaniasis (B), where an itchy papule develops at the bite site and develops into an ulcer with raised edges. Local lymphadenopathy can also occur, but the lesion usually heals within 8 months leaving a depigmented scar. The organisms implicated are L. major and L. tropica.

Mucocutaneous leishmaniasis (C) can produce destructive and disfiguring facial lesions, and so is the most feared form of cutaneous leishmaniasis. It may begin in the same way as the cutaneous form, but years later ulceration can appear in mucous membranes leading to mutilation of those areas. It is most often caused by L. braziliensis. A single ulcer caused by L. majoror L. tropica may be left to heal spontaneously, but otherwise the first-line drug for leishmaniasis is a pentavalent antimonial such as sodium stibogluconate.

Malaria (D) can present with non-specific flu-like symptoms, but hyper-pigmentation of the skin is not a feature. Hepatosplenomegaly can occur however, and other clinical features might include malaise, head-ache, vomiting or diarrhoea. Malaria should be considered as the most likely diagnosis in a patient with a fever returning from an endemic area.

Schistosomiasis (E), also known as bilharzia, is transmitted by blood flukes. An itchy rash, known as ‘swimmer’s itch’, may develop at the site where the vectors penetrate the skin. They may then migrate to the liver, causing ‘Katayama fever’ with clinical features such as fever, rash, myalgia and sometimes hepatosplenomegaly. Following maturation in the liver, the flukes migrate to either mesenteric veins causing intestinal schistosomiasis, or to the urinary tract leading to urinary schistosomiasis. Hepatosplenomegaly can occur, but again the dry warty skin lesions described are not usually a feature

600
Q

A 22-year-old student presents to accident and emergency with a raised, erythematous, scaly ulcer on his forearm which has not been healing. On examination he is also found to have lymphadenopathy. He gives a history of recently returning from a 2-month trek in the rainforests of South America. Tissue is aspirated from the margin of the ulcer, and the organism is cultured in Novy–MacNeal–Nicolle medium. The organism implicated is:

A Toxoplasma gondii
B Treponema pallidum
C Leishmania dovani
D Leishmania major
E Leishmania braziliensi
A

Leishmania major

The picture described is consistent with cutaneous leishmaniasis, the most common form of leishmaniasis. An itchy, scaly papule develops at the bite site and develops into a crusty ulcer with raised edges. Local lymphadenopathy can also occur, but the lesion usually heals within 8 months leaving a depigmented scar called an oriental sore. The organisms implicated are Leishmania major (D) and L. tropica. You can remember this if you picture lots of skin lesions cropping up in travellers from the ‘major tropics’! It is found in many countries, ranging from South America to the Middle East. Diagnosis can be by Giemsa staining of slit skin smears, or from tissue aspirated from the ulcer,

The organism can be cultured on Novy–Macneal–Nicolle medium as described in the question.The other forms of leishmaniasis are visceral and mucocutane-ous. Visceral leishmaniasis is also known as ‘Kala-azar’, and the most common clinical features include fever and splenomegaly. Hepatomegaly, skin hyperpigmentation and dry warty skin occur less frequently.

Leishmania donovani (C) and L. infantum are thought to cause the disease in Africa, Asia and Europe, whilst L. chagasi is implicated in South America.

Mucocutaneous leishmaniasis can produce destructive and disfiguring facial lesions, and so is the most feared form of cutaneous leishmaniasis. It may begin in the same way as the cutaneous form, but years later ulceration can appear in mucous membranes leading to mutilation of those areas. It is most often caused by L. braziliensis (E).

Toxoplasma gondii (A) is a protozoal disease, for which the cat family is the definitive host. It can infect humans by eating undercooked meat or from contact with cat faeces. Like other protozoa they become trophozites in the gut and spread to the brain, eyes and lungs. It would not normally cause an ulcer, and cannot be cultured in Novy–Macneal–Nicolle medium. Most infections are asymptompatic in immunocompetent hosts, but in the immunocompromised or in fetuses affected via pregnant mothers the consequences can be fatal. In AIDS patients it can have neurological manifestations such as cranial nerve palsies, meningo-encephalitis and focal neurological deficits secondary to a space-occupying lesion. In the eyes it can cause chorioretinitis. Characteristically a CT scan may show ring enhancing lesions with sur-rounding oedema. If you can picture the ‘O’s in tOxO-plasmosis jump-ing out at you like rings on a CT scan, you should be able to recall this important diagnostic fact!

Treponema pallidum (B) is the organism responsible for syphilis. The first stage of syphilis presents with a papule which ulcerates to become a painless chancre. This may be associated with regional lymphadenop-athy that is also painless. This fact often crops up in exam questions and can be remembered by thinking of the word ‘syphilis’ correspond-ing to the 2 Ss found in painless! This organism also cannot be grown as described, but diagnosis would be by dark ground microscopy or serology of anti-treponemal antibodies

601
Q

A 35 year-old male clothing merchant has returned to the UK 2 weeks ago from a visit home to Syria. A week later he presents with flu-like symptoms, drenching sweats and a recurring fever and is beginning to complain of lower back pain. After further questioning, he mentioned that he worked on a farm during his trip. He is successfully treated with oral doxycycline and gentamicin. What is the most likely diagnosis?

A Malaria
B Tuberculosis
C Influenza
D Brucellosis
E Typhoid
A

Brucellosis

The Brucella species are Gram-negative, rod shaped, intracellular bacteria that cause a highly contagious zoonosis known as brucellosis (D). The causative agent in cattle is B. abortis, but in dogs it is B. canis. Infection in cattle can lead to miscarriages, hence the name ‘abortis’. Infection is usually contracted from unsterilized milk, cheese or meat.
Clinical features of brucellosis can include a long history of undulating fevers, arthralgia and myalgia, weight loss, fatigue, lymphadenopathy, sacroilitis and depression. Many cases present as pyrexia of unknown origin. Hepatomegaly and/or splenomegaly can sometimes be found on examination.You can remember these by picturing an old man called Bruce, walking with a stick due to his back and muscle pain (arthralgia, myalgia, sacroilitis), feeling down (depression), looking thin (anorexia), sweating (fevers), and with lots of protruding lumps (hepatosplenomegaly and lymphadenopathy)! Draw this picture, label it with these features and it will be easier to remember! The most common diagnostic method is serum agglutination for antibodies. Antibiotics that can be used for treatment include doxycylcine and gentamicin for approximately 6 weeks, though streptomycin and rifampicin are other agents that are used. Because the bacteria are intracellular, usually more than one antibiotic is needed and relapse is common.

Whilst tuberculosis (B) can cause drenching sweats, it would not typically cause lower back pain and certainly would not be treated with doxycycline. Do not forget the treatment for TB can be remembered as RIPE: Rifampicin, Isoniazid, Pyrazinamide and Ethambutol. The last two drugs, pyrazinamide and ethambutol, are usually only used for the first 2 months. The features of brucellosis and influenza (C) are not dis-similar, but influenza would usually have an incubation period of 1–4 days. Note also the history of working on a farm: any unusual facts like this are not red herrings, but are usually put in to help guide you to the right answer! Treatment of influenza would not be with doxycycline, but is usually symptomatic.

Typhoid (E) is caused by Salmonella typhi, and again can present with non-specific features like brucellosis. However, there are a few unusual clinical features of typhoid that are worth remembering using the mnemonic A.B.C.C.D.E: Abdominal distension, Bradycardia, Cough, Constipation, Diarrhoea and Erythematous rose spots. Antibiotics of choice in the treatment of typhoid are the quinolones such as ciprofloxacin for 2 weeks.

602
Q

A 50-year-old man has returned from hiking a segment of the Appalachian Trail on the Eastern coast of the USA during the summer months. Ten days later he presents to casualty with flu-like illness and a rash showing some central fading. What is the most likely organism implicated?

A Herpes simplex
B Epstein–Barr virus
C Streptococcus pyogenes
D Treponema pallidum
E Borrelia burgdorferi
A
Borrelia burgdorferi (E) is a Gram-negative bacterium that causes Lyme disease. It is a spirochaete, which is the name for a group of bacteria that are helically coiled in shape. Lyme disease is actually thought to be the most common vector borne disease in England and Wales. It is named after a town called Lyme in Connecticut, where the disease was first seen. The vector is a tick called the Ixodes tick, which can be found on deer and rodents. 
Lyme disease is a multisystemic disorder which has three main stages: the local stage, disseminated stage and a late stage. The local stage involves a characteristic skin lesion called erythema chronicum migrans, usually appearing 7–10 days after the initial infection. It usually starts off as a red macule or papule, and approximately 1 week later expands to leave a target appearance with an area of central fading. Other symptoms at this stage are usually constitutional, such as a fever and headache. The somewhat unusual features of the next stage can be remembered using the word PEACH: Peripheral neuropathy, Erythema chronicum migrans (persists in this stage), Arthritis, Cranial nerve palsies and Heart block. Finally, the late stage can include persistent arthritis and chronic encephalitis. Treatment is with oral antibiotics, usually doxycycline.

The rash would not be in keeping with any of the other organisms listed. Herpes simplex (A) causes an acute viral disease. Type 1 is primarily responsible for oral–facial lesions, whereas Type 2 is responsible for genital disease – remember this as Type 2 requires 2 people to lead to the disease! Several anti-virals can be effective in treating the disease, particularly acyclovir.

The Epstein–Barr virus (B) is also one of the herpes viruses, and is responsible for causing infectious mononucleosis. A rash is not typically a feature of this disease, but sometimes the virus can cause erythema multiforme. This rash usually consists of itchy papules, which may evolve into target lesions.Streptococcus pyogenes (C) is a Gram-positive bacterium responsible for many conditions, including impetigo and rheumatic fever. The rash in rheumatic fever is known as erythema marginatum, which can be easily confused with erythema chronicum migrans of Lyme disease. A good way to remember which is which is to picture a person sucking a Lyme and getting a ‘chronic migraine’ from doing so! In terms of appearance, erythema marginatum consists of pink ring lesions which usually occur on the trunk, arms and legs but with facial sparing.

Treponema pallidum (D) is another spirochaete, like Borrelia burgdorferi, but is the organism responsible for causing syphilis. The skin lesion with syphilis is called a chancre, and is classically a painless ulcer with sharp borders

603
Q

A 26-year-old squash player is admitted with a red, swollen left knee. He reports no history of trauma. On examination he has a temperature of 38°C. A joint aspirate is taken. What is the most likely causative organism?

A Neisseria gonorrhoeae
B Staphyloccocus aureus
C Haemophilus influenzae
D Streptococcus viridans
E Chlamydia trachomatis
A

Neisseria gonorrhoeae

The patient in this question is presenting with septic arthritis. Other differentials might include gout or pseudogout, but it is paramount to consider septic arthritis as it is a rheumatalogical emergency! Typical features are like those described in the question: the patient is often pyrexial, the joint is swollen and painful with limited range of move-ment, and the skin overlying the joint is warm and erythematous.

Classically the patient will refuse to move the joint at all. The most important investigation is an aspirate of the joint: the fluid aspirated may appear purulent and have a high neutrophil count. The most common cause of septic arthritis in young, sexually active adults is Neisseria gonorrhoeae (A). A Gram-stain of this aspirate would reveal Gram-negative diplococci. It is less likely for this organism to lead to joint destruction than a staphylococcal arthritis. The two forms of disseminated gonoccocal infection are the septic arthritis form (as described in this case), and the bacteraemic form. Other clinical features of the bacteraemic form might include a migratory polyarthralgia and a vesicular or papular rash.

Staphyloccocus aureus (B) is the most common causative organism in older patients. A Gram-stain of the joint aspirate would reveal Gram-positive cocci. It is thought that a septic arthritis infected with this organism is more likely to be destructive than a gonoccocal arthritis, and a joint can be destroyed within 24 hours if left untreated.

Haemophilus influenzae (C) is a Gram-negative bacterium that used to be the most common cause of septic arthritis in children, but is not typically found in adults.

Streptococcus viridans (D) is an alpha haemolytic, optochin resistant streptococcus that does not typically cause septic arthritis. It is, however, the most common cause of sub-acute bacterial endocarditis

604
Q

A 26-year-old squash player is admitted with a red, swollen left knee. He reports no history of trauma. On examination he has a temperature of 38°C. A joint aspirate is taken which grows Gram-negative diplococci. What is the antibiotic treatment regimen of choice for this patient?

A Oral flucloxacillin for 4–6 weeks
B IV flucloxacillin for 4–6 weeks
C IV flucloxacillin for 2–4 weeks
D IV flucloxacillin and vancomycin for 6–8 weeks
E IV cefotaxime for 4–6 weeks
A

IV cefotaxime for 4–6 weeks
The patient in this question is presenting with septic arthritis, and the most likely cause given the joint aspiration findings of Gram-negative diplococci is Neisseria gonorrhoeae. The British National Formulary (BNF) advises the use of intravenous cefotaxime for 4–6 weeks (E) if gonococcal arthritis or a Gram-negative infection is suspected. The BNF is a good source of information for looking up the latest guidelines regarding antibiotic treatment regimens for common types of infection.

Cefotaxime is a third generation cephalosporin. Cephalosporins are part of the beta-lactam group of antibiotics which work by inhibiting cell wall synthesis. The penicillins are also part of this group. There are different generations of cephalosporins, with those of later generations having increasing Gram-negative but decreasing Gram-positive cover. Cefotaxime is also used to treat meningitis and gonorrhoea. Some of the other commonly used third generation cephalosporins are ceftizoxime and ceftriaxone – you can remember these because they all have a ‘t’ in their names, just like in ‘third’ generation.

If a staphylococcal cause of septic arthritis is suspected, IV flucloxacillin for 4–6 weeks (B) would be the preferred regimen.

Flucloxacillin is a narrow spectrum beta-lactam antibiotic, used to treat infections caused by Gram-positive organisms. Some bacteria, such as Staphylococcus aureus, produce an enzyme called beta-lactamase which renders this class of antibiotics ineffective. However, flucloxacillin is beta-lactamase stable, and so is used to treat staphylococcal infections. It is ineffective against MRSA, in which case vancomycin is preferred.

Oral flucloxacillin (A) or IV therapy of only 2–4 weeks’ treatment (C) would not be sufficient to clear the infection in this case. IV flucloxacillin and vancomycin (D) together would not be needed.

605
Q

You order hepatitis B serology tests for one of your patients, a 24-year-old man who is an intravenous drug user. The results that come back from the laboratory are as follows:

  • HBsAg = positive
  • Anti-HBs = negative
  • HBeAg = positive
  • Anti-HBe = negative
  • Anti-HBc IgM = negative
  • Anti-HBc IgG = positive

What is the most likely diagnosis based on these results?

A The patient has chronic hepatitis B infection which is currently highly infectious
B The patient has chronic hepatitis B infection which is not currently infectious
C The patient has acute hepatitis B infection which is not currently infectious
D The patient is immune due to hepatitis B vaccination
E The patient is immune due to natural infection

A

The patient has chronic hepatitis B infection which is currently highly infectious.

The HBsAg positive indicate that the patient has hepatitis B, and the HBeAg indicates that it is highly infectious (A). The anti-HBc IgG is also a marker that it is a chronic infection.The different hepatitis B surface antigens and antibodies can become quite confusing, but are often asked about in exam questions.

Here is a summary of what you should know:
• HBsAg – The ‘s’ stands for surface, and refers to a protein on the surface of the virus. It is the first detectable antigen to appear after someone has been infected, and can be positive in acute or chronic disease. Patients who still carry this antigen after 6 months are termed hepatitis carriers. It is this antigen that is used to make the hepatitis B vaccine
• Anti-HBs – This is an IgG antibody that appears after the host has cleared the infection, and indicates recovery. It is also found in a person who has been vaccinated against hepatitis B (D)
• HBeAg – the ‘e’ antigen is often used as a marker of infectivity, as it is only found in the blood when the virus is actively replicating. If you find this hard to remember, think of the ‘e’ standing for ‘eek! I’m infectious!’ If the patient was not infectious (B), this would not be present
• Anti-HBc IgM – this indicates that the patient has recently been infected with hepatitis B, and is a marker of acute infection (C)
• Anti-HBc IgG – this is produced in response to the core antigen, and often persists for life. You can remember this as the ‘c’ standing for ‘chronicity’, as it is the difference between IgM and IgG antibodies which can tell you whether the infection is acute or chronic. And to remember which way round it is, think of ‘My Gosh, he’s chronic!’ If the patient was immune from natural infection (E), HBsAg would not be positive, but anti-HBc IgG would be

606
Q

You order hepatitis B serology tests for one of your patients, a 24-year-old man who is an intravenous drug user. The results that come back from the laboratory are as follows:

  • HBsAg = negative
  • Anti-HBs = positive
  • HBeAg = negative
  • Anti-HBe = negative
  • Anti-HBc IgM = negative
  • Anti- HBc IgG = negative

What is the most likely diagnosis based on these results?
A The patient has chronic hepatitis B infection which is currently highly infectious
B The anti-HBs is a false positive result
C The patient has a resolved hepatitis B infection
D The patient is immune due to hepatitis B vaccination
E The patient is immune due to natural infection

A

The patient is immune due to hepatitis B vaccination.

Remember from the previous question that the anti-HBs antibody appears after the host has cleared the infection, and indicates recovery.

It is also found in a person who has been vaccinated against hepatitis B (D). If you get an exam question which only has the anti-HBs positive, think of vaccination! Levels of this antibody are measured to see if the patient has responded adequately to the vaccine.The patient has not got hepatitis (E) nor is he highly infectious (A) given that HBsAg and HbeAg are negative. HBsAg can be negative in the case of a resolved infection (C), but you would expect to see some marker of previous infection such as anti-HBc IgG, which usually persists for life.It is much more likely that the patient has been vaccinated than it is to be a false positive result (B)!

607
Q

A 79-year old woman is admitted to the hospital for treatment of pneumonia and is commenced on intravenous antibiotic therapy. Her respiratory symptoms begin to improve, but 5 days later she develops profuse diarrhoea. The most appropriate treatment is:
A Oral metronidazole for 7 days
B Oral metronidazole for 14 days
C Isolation and treatment with intravenous fluids
D IV metronidazole for 7 days
E Oral co-amoxiclav for 7 days

A

Oral metronidazole for 14 days

Broad spectrum antibiotics, such as those used for pneumonia, can eradicate a patient’s normal gut flora and therefore increase their sus-ceptibility to Clostridium difficile infection. This is particularly true of penicillin derivatives (as was most likely used to treat her pneumonia), clindamycin, and third generation cephalosporins. It classically presents with profuse watery diarrhoea, usually of acute onset. The most common time for it to occur is 4–9 days after the antibiotics are started, but it can occur up to 2 months after discontinuing treatment. Clostridium difficile is a Gram-positive, anaerobic rod-shaped bacterium. The gold standard for diagnosis is detection of the C. difficile toxin in a stool sample.

The two most feared complications are pseudomembranous colitis and toxic megacolon. Pseudomembranous colitis is essentially an acute, severe colitis, which is named as such because of the formation of ‘pseudomembranes’ on the mucosa of the colon. These are thought to be composed of exudative material produced from the bacterium. Toxic megacolon can also be a complication of inflammatory bowel disease. The colon becomes severely distended, and clinical features include pyrexia, severe abdominal pain and bloating.

First line treatment for infection with C. difficile is oral metronidazole, with a suggested duration of treatment of 10–14 days (B). Metronidazole is classified as a nitroimidazole antibiotic, and is particularly useful for the treatment of anaerobic organisms and protozoa. You can remember three of the key organisms it is used to treat by remembering that ‘Met is out to G.E.T you difficult bugs!’ (Giardia, Entamoeba, Trichomonas and C. difficile). Patients are usually advised to avoid consuming alcohol whilst taking this antibiotic because of the potential reaction that can occur characterized by nausea, shortness of breath, flushing and vomiting.The course of treatment should be at least 10–14 days, so 7 would not suffice (A). Whilst the patient must be isolated and rehydrate adequately, this alone would not be enough (C).

IV metronidazole is not necessary in the first instance (D), unless the patient had a life-threatening infection.

Co-amoxiclav (E), otherwise known as augmentin, is not used to treat C. difficile infections.

608
Q

A 79-year old woman is admitted to hospital for treatment of pneumonia and is commenced on intravenous antibiotic therapy. Her respiratory symptoms begin to improve, but 5 days later she develops profuse diarrhoea. After treatment with oral metronidazole she shows gradual improvement, but the profuse diarrhoea returns 2 weeks later. The same organism is found to be responsible. The most appropriate course of action is:

A Oral metronidazole for 7 days
B Oral metronidazole for 14 days
C Isolation and treatment with intravenous fluids
D IV metronidazole for 7 days
E Oral vancomycin for 14 day
A

Oral metronidazole for 14 days

This patient’s repeated diarrhoea may be caused by persistent infection with Clostridium difficile (spore germination), new infection or resistant bacteria. Current guidelines recommend the use of a repeat course of metronidazole for the treatment of recurrent C. difficile infection (B).

As explained previously, a 7-day course of metronidazole (A) is not considered a sufficient duration of treatment to eradicate the bacterium.

Again, isolation and IV fluid resuscitation (C) is necessary but not adequate as a single measure in the management of this woman.

Intravenous metronidazole (D) is only needed if a patient is not responding to vancomycin, the infection is life-threatening, or for patients with ileus.

Oral vancomycin for 10–14 days (E) is given for:
•Third or subsequent episodes
•Severe infection
•Infection not responding to metronidazole
•Patients who cannot tolerate metronidazole

Vancomycin is classified as a glycopeptide antibiotic. Its mechanism of action is to inhibit cell wall synthesis in Gram-positive bacteria, and it is not effective for Gram-negative bacteria. You can remember this if you picture an ambulance ‘Van’ with a big red cross (for Gram-positive) on the side! Because it cannot pass through the lining of the intestine, it is usually given intravenously. However, in the case of Clostridium difficile infection we give it orally, as this stays in the gut where it is needed. One rare but important side effect you might hear about is ‘red man syndrome’, a reaction to the drug which consists of a sudden onset erythematous, pruritic rash over the face, neck and upper torso. To remember this fact, picture the driver of the red cross van emerging very angry, with a bright red face!

609
Q

A 65-year old retired mechanic is brought by his family to his GP due to their concern over his recent increase in confusion. This has occurred rapidly over the past 4 months, and he now struggles to recognize members of his family. His daughter also reports occasionally seeing intermittent, jerky movements of both his arms. The GP organizes a CT scan and dementia screen, which are both found to be normal. Which is the next most useful diagnostic test for the GP to order?

A MRI brain
B Electroencephalogram
C Electrocardiogram
D Ultrasound scan of both carotids
E Tonsillar biopsy
A

Electroencephalogram

The key here is the rapidly progressive nature of the condition in a relatively young patient. He shows the characteristic sudden decline in cog-nitive function, combined with the presence of myoclonic jerks and the lack of positive investigation results so far. This is highly suggestive of sporadic Creutzfeldt–Jakob disease (CJD), the name given to a common group of prion diseases. The word prion is derived from the words ‘protein’ and ‘infection’, and it so follows that a prion is a highly infectious agent composed of protein.

There are essentially three different forms of CJD which you should be aware of:
1 Sporadic Creutzfeldt–Jakob Disease (80 per cent)
2 Acquired (

610
Q

A 61-year-old patient has recently been diagnosed with sporadic CJD. His GP is keen to do a lumbar puncture. Which of the following statements is true regard-ing this investigation in this situation?

A The lumbar puncture is used to look for the levels of protein, glucose and polymorphs
B The lumbar puncture is used to look for the levels of a protein called 14-3-3
C A lumbar puncture is the most specific test for variant CJD
D The lumbar puncture is not useful in sporadic CJD, but is an important test in variant CJD
E A tonsillar biopsy would be a more useful test than a lumbar puncture for sporadic CJD

A

The lumbar puncture is used to look for the levels of a protein called 14-3-3

The lumbar puncture in CJD is used to analyze the CSF for a protein named ‘14-3-3’ (B). Note that routine analysis of the cerebrospinal fluid (CSF) is normal in CJD, therefore looking at levels of protein, glucose and polymorphs (A) would not be useful to distinguish between possible causative agents of the clinical features as it is in meningitis

‘14-3-3’ is a term for a large group of proteins which have different functions in eukaryotic cells, such as in cell signalling. However, its measurement in CJD is a time consuming process, and as it is a normal neuronal protein it can be released into the CSF as a result of many other normal neuronal insults. It is therefore not a specific finding (C), and the test can be positive in other conditions such as a recent stroke, viral encephalitis or a subarachnoid haemorrhage. The 14-3-3 protein is present in both variant and sporadic CJD, therefore (D) is incorrect.

Variant CJD has several important differences from sporadic CJD:
1 It typically occurs in younger patients (median age of onset 26 years) than sporadic CJD
2 The median survival time is approximately 14 months, compared to 4 months for sporadic CJD
3 Psychiatric features may dominate in the initial stages, before neurological features such as ataxia, myoclonus, chorea, dementia and peripheral sensory symptoms appear
4 The MRI in variant CJD shows the ‘positive pulvinar sign’ (enhanced signal of nuclei in the thalamus)
5 The classical EEG findings are often absent in the variant form
6 A tonsillar biopsy is sensitive and specific in the variant form, but is not a useful test in the sporadic form (E)

611
Q

A 16-year-old student complains of a headache of recent onset at school. He is taken to accident and emergency and on examination has a temperature of 37.6°C. A lumbar puncture is performed, and the results are as follows:

  • Appearance: Clear fluid
  • Protein: 0.82g/L
  • WCC: 90.5 × 107 (>95 per cent lymphocytes)
What is the most likely diagnosis?
A Subarachnoid haemorrhage
B Tension headache
C Bacterial meningitis
D Viral meningitis
E Tuberculous meningitis
A

Viral meningitis

In this context, the two most immediately worrying diagnoses for the onset of an acute headache are a subarachnoid haemorrhage and bacterial meningitis as both of these may be fatal if rapid intervention does not occur.

  • Bacterial meningitis typically has a very high neutrophil count, with a high protein and low glucose
  • Viral meningitis typically has a very high lymphocyte count
  • Tuberculous meningitis has similar features to viral meningitis in terms of the cell types found in the CSF, but has a particularly high protein

Using this, the easiest way to tell what the cause of the patient’s headache is in this scenario is the high lymphocyte count found on the lumbar puncture. This, combined with its clear appearance and slightly high protein content, indicates that the most likely cause is viral meningitis (D).

A bacterial meningitis (C) is more likely to look turbid in appearance and would not have a predominant lymphocytosis, whilst a menin-gitis caused by TB (E) would usually have a higher protein content.

A lumbar puncture in a subarachnoid haemorrhage (A) may be normal, or may have an increased number of red blood cells. The CSF is also examined for the presence of xanthochromia – this is the term for the yellowish appearance seen when red blood cells enter the CSF and are broken down. The presence of pyrexia combined with lumbar puncture results clearly points towards excluding a simple tension headache (B).

612
Q

A 42-year-old alcoholic is admitted with abdominal distension. The shifting dullness test is positive and he is found to have diffuse abdominal tenderness. His observations are as follows: pulse 115, blood pressure 116/83, temperature 37.9°C. The next best course of action is:

A Begin therapeutic paracentesis
B Observe, administer analgesia and closely monitor his vital signs
C Commence intravenous spironolactone
D Commence intravenous amoxicillin
E Commence intravenous cefotaxime
A

Commence intravenous cefotaxime

This patient is presenting with features suggestive of spontaneous bacterial peritonitis (SBP), which is a form of peritonitis in the absence of a contiguous source of infection. This usually results from the develop-ment of portal hypertension in patients with chronic liver disease. This group of patients are particularly susceptible as they are often immuno-compromised.

The pyrexia and tachycardia, in conjunction with the clinical features of abdominal tenderness and ascites, make this the most likely diagnosis in this patient. Other typical clinical features might include nausea, vomiting, confusion, general malaise or features of hepatic encephalopathy. In approximately 15 per cent of patients SPB can be asymptomatic.

A prompt diagnostic paracentesis is needed to make the diagnosis, and SPB is confirmed by the presence of:

1 Ascitic fluid WCC of 500 cells/mm3
2 or Neutrophil count of >250 cells/mm3

Do not confuse a diagnostic paracentesis with a therapeutic paracentesis (A): in the latter the purpose is to remove the fluid, for example to relieve abdominal pressure or in the case of respiratory compromise. This may be appropriate later, but only once SBP has been excluded from the results of a diagnostic paracentesis or treated.

The most common organisms isolated in patients with SBP include E. coli, Gram-positive cocci and enterococci. Although local antibiotic guidelines may differ, of the options listed cefotaxime (E) is one of the most extensively studied and has been proven to be effective. It is usually given for at least 5 days. Other third generation cephalosporins such as ceftriaxone can also be used.

Amoxicillin (D) would not provide sufficient cover against Gram-negative organisms.

Whilst analgesia and close observation are also important measures (B), the high risk of mortality in SBP necessitates prompt antibiotic treatment.

Spironolactone (C) is used for the treatment of uncomplicated ascites, but initial antibiotic treatment would take precedence in the case of SBP.

613
Q

A 63-year-old asymptomatic housewife is referred to a gastroenterologist after her GP found that she had abnormal liver function tests on a routine blood test. A thorough history reveals that she received a blood transfusion during her pregnancy in 1979. Further tests confirm that she has contracted hepatitis C. She is commenced on a course of anti-viral treatment. Which of the following factors is most significant in influencing her chance of clearing the virus?

A The length of time between contracting the disease and being diagnosed
B The route by which she contracted the disease
C Her liver function test results
D The virus genotype
E The level of alpha-feto-protein

A

The virus genotype

Hepatitis C is a single stranded RNA virus that is similar in structure to the ‘flaviviruses’. It can cause a slowly progressive disease of the liver that is frequently asymptomatic and which cannot be vaccinated against.

Routes of transmission include:
•blood products (before 1991, when screening of blood donors for the disease was introduced)
•intravenous drug use
•sexual transmission
•vertical transmission
•less commonly: needle-stick injuries, tattoos

Acute hepatitis occurs in approximately 20 per cent of patients following exposure to the virus. The symptoms are usually mild, such as malaise, arthralgia, jaundice and lethargy. Others may remain asymptomatic. Up to 85 per cent of patients this may persist and cause chronic infection, and approximately 20 per cent of these will develop liver cirrhosis in 20 years.

There are six different genotypes of the virus. The most common in the UK is genotype 1 (40–50 per cent), genotypes 2 and 3 are responsible for another 40–50 per cent, and the remainder is due to genotypes 4, 5 and 6. It has been found that genotype 1 is associated with a poorer response to anti-viral treatment (with interferon and ribavarin) than the other genotypes, so the answer here is (D). In fact, the recommended duration for combination therapy is 24 weeks for people with the 2nd and 3rd genotypes, but 48 weeks for genotypes 1, 4, 5, and 6.

Patients with hepatitis C who develop cirrhosis are estimated to have approximately a 1–2 per cent annual risk of developing hepatocellular carcinoma. Alpha-feto protein is the main tumour marker for this can-cer, but its levels do not influence response to treatment (E). Similarly, the length of time between contracting the disease and being diagnosed (A), the route by which she contracted the disease (B), or her liver function test results (C) do not impact her response as significantly as the viral genotype does. A liver biopsy is also often performed and is thought to be the most accurate way to obtain information about the severity of the liver disease. It is thought that the severity of the fibrosis can also help to predict which patients are more likely to respond to antiviral treatment.

614
Q
A 63-year-old asymptomatic housewife is referred to a gastroenterologist after her GP found that she had abnormal liver function tests on a routine blood test. A thor-ough history reveals that she received a blood transfusion during her pregnancy in 1979. The best test to confirm whether the patient has hepatitis C would be:
A Liver biopsy
B Anti-hepatitis C antibodies
C Alanine aminotransferase levels
D Hepatitis C RNA PCR
E Viral genotyping
A

Hepatitis C RNA PCR

There are several different tests which are helpful in investigating the disease:

1 Hepatitis C RNA PCR (D) – This can be used to differentiate between a current and past infection. A quantitative test to detect the number of hepatitis C RNA particles (called the ‘viral load’) can also be performed. This can be very useful to detect a patient’s response to the anti-viral treatment. Therefore, this is the best diagnostic test for hepatitis C
2 Anti-hepatitis C antibodies (B) – a positive test would indicate exposure to the disease, but results should be interpreted with caution because it cannot distinguish between current or past infection. In addition, it can take up to 3 months for these antibodies to appear after exposure, so an initial negative test can be misleading. It has also been suggested that a weakly positive test might actually be a false positive, so this is not the best diagnostic test. However, it may be performed initially, and if the patient has two positive results a hepatitis C RNA PCR is used to confirm the diagnosis
3 Viral genotyping (E) – this is used to determine the genotype of virus present. The most common, genotype 1, is less likely to respond to treatment than genotypes 2 or 3 and requires longer therapy
4 Liver biopsy (A) – this would be the most accurate means of determining the stage and severity of liver damage caused by the virus, and may be useful to assess the patient’s likelihood to respond to treatment. However, it would be performed after the suspected diagnosis has been confirmed
5 Alanine aminotransferase levels (ALT) (C) – this is not a diagnostic test, but can be useful aid in the initial stages of confirming the diagnosis. The ALT to AST (aspartate aminotransferase) ratio is typically 2 this is more suggestive of alcoholic liver disease. You can remember this because AST is indicative of Smirnoff drinking, whereas ALT is indicative of viraL aetiology!

615
Q

A 33-year-old backpacker visits his GP complaining of feeling weak, lethargic and feverish since he returned from his trip to South Africa 3 months previously. He is accompanied by his wife, who reports a change in his behaviour and disturbed sleeping pattern since his return. On examination, his GP discovers that he has enlarged cervical lymph nodes, and there is a small chancre on his forearm that is approximately 2cm in diameter. The most likely causative organism is:

A Plasmodium falciparum
B Trypanosoma brucei gambiense
C Trypanosoma brucei rhodesiense
D Trypanosoma cruzi
E Leishmania infantum
A

Trypanosoma brucei rhodesiense

Human African trypanosomiasis is also known as sleeping sickness, and is an infection transmitted by the tsetse fly in sub-Saharan Africa.

There are two main types:
1 Trypanosoma brucei gambiense (B) is found in west and central Africa, is responsible for over 95 per cent of cases, and causes a chronic infection. It can take months or even years for symptoms to appear. You can remember this as gambiense causes a gradual infection
2 Trypanosoma brucei rhodesiense (C) is found in south and eastern Africa, accounts for under 5 per cent of cases, and causes an acute infection with symptoms appearing over a few weeks or months. You can remember this as rhodesiense causes a rapid infection. As this patient’s symptoms appeared 3 months after returning from his travels, this is more likely to be the causative agent here

A subcutaneous chancre can develop at the site where the tsetse fly bites, and symptoms such as fevers, weakness, arthralgia and headache can then appear. Posterior cervical lymphadenopathy can also occur, especially with T. brucei gambiense. This is known as Winterbottom’s sign. Later the parasite can cross the blood–brain barrier resulting in neurological features such as disturbance of the sleep cycle, ataxia, behavioural changes and psychiatric disturbance. Treatment is with drugs such as pentamidine and suramin in the early stages.

Plasmodium falciparum (A) is an organism responsible for causing malaria. Whilst it should be considered in all patients with a fever returning from an endemic area, the changes in behaviour and sleep disturbance described in this patient make this a less likely cause.

Trypanosoma cruzi (D) causes Chagas disease which is carried by the reduviid bug. Chronic infection can appear weeks to years after the initial infection, affecting the cardiac and gastrointestinal systems.

Leishmania infantum (E) is responsible for leishmaniasis, features of which can include fever, hepatosplenomegaly and lymphadenopathy. Again, it is less likely to cause behavioural changes and sleep is unlikely to be affected.

616
Q

A 20-year-old student seeks medical attention due to recent difficulty in swallowing, and severe weight loss. A thorough travel history reveals that he returned several months ago from a gap year in Brazil. During his trip he remembers becoming unwell at one point with a fever, diarrhoea, vomiting and swollen eyelids, but this resolved in approximately 3 weeks with no treatment. A chest x-ray is ordered as one of his investigations, and this reveals marked dilatation of his oesophagus. The vector responsible for transmitting this disease is:

A Tsetse fly
B Reduviid bug
C Sandfly
D Aedes mosquito
E Ixodes tick
A

Reduviid bug

Trypanosoma cruzi is responsible for causing Chagas disease, a potentially life-threatening disease which is spread by reduviid bugs (B) in Brazil. These are also known as ‘kissing bugs’. A red nodule, called a chagoma, can appear at the site of the bite.There are two forms of the disease: acute and chronic. In the acute phase, patients may experience non-specific symptoms such as fever, lethargy, diarrhoea, and vomiting. A characteristic feature, but one which occurs in less than 50 per cent of cases, is a purplish swelling of the eyelids (called Romana’s sign). To put this all together, picture Tom Cruise (Trypanosoma cruzi) starring in a gladiator film as a Roman (Romana’s sign) wearing purple sunglasses (swollen eyelids) and being kissed (kissing bugs) by lots of fans ‘ready with their video cameras’ (reduviid!)The chronic phase can occur even years after the initial bite, and typi-cally affects the heart and gastrointestinal tract. You can remember its effects by thinking of it causing both dilatation and dysfunction in three organs: in the heart (dilatation = dilated cardiomyopathy, dys-function = arrhythmias), in the colon (dilatation = megacolon, dysfunction = constipation) and in the oesophagus (dilatation = mega oesophagus, dysfunction = dysphagia). Bennzimidazole or nifurtimox are effective medications used to treat this disease.

The tsetse fly (A) is responsible for causing human African trypanosomiasis, also known as sleeping sickness, in sub-Saharan Africa. The clinical features of this disease can include changes to the sleep–wake cycle and psychiatric disturbance.

The sandfly (C) transmits Leishmaniaspecies in Africa, America and the Middle East.

The Aedes mosquito (D) is a type of mosquito that causes Dengue fever.

The Ixodes tick (E), also known as the ‘deer tick’, transmits the organism responsible for causing Lyme disease. None of these vectors would cause the spectrum of clini-cal features described in this patient

617
Q

A 46-year-old Somalian woman presents to her GP with a dry cough and weight loss of 5kg over 3 weeks. She is sent to the hospital, and a chest X-ray reveals cavitating lung lesions. The most appropriate therapy is:

A Rifampicin and isoniazid for 6 months, ethambutol and pyrazinamide for 2 months
B Rifampicin and isoniazid for 2 months, ethambutol and pyrazinamide for 6 months
C Rifampicin and pyrazinamide for 4 months, ethambutol and isoniazid and for 2 months
D Rifampicin and streptomycin for 4 months, pyrazinamide and ethambutol for 2 months
E Rifampicin, isoniazid, ethambutol and pyrazinamide for 6 months

A

A Rifampicin and isoniazid for 6 months, ethambutol and pyrazinamide for 2 months

Current guidelines in the UK recommend the following antibiotic treat-ment for pulmonary tuberculosis:
•Isoniazid and rifampicin for 6 months
•Pyrazinamide and ethambutol for the first 2 months

The purpose of the initial phase where all four drugs are taken together is to rapidly reduce the number of bacteria and prevent drug resistant bacteria emerging. Culturing TB can take several weeks, so treatment is started without waiting for the culture results if clinical features or histopathology results are highly suggestive.

Streptomycin (D) is an aminoglycoside that is rarely used in the UK, but may be used in the initial phase if it has been shown that the organism is resistant to isoniazid.

It is not usually necessary to continue all four drugs after 2 months (E). The continuation phase consists of rifampicin and isoniazid alone for 2 months. A good way to remember this is to think of the word R.I…P.E (rifampicin, isonizaid, pyrazinamide, ethambutol), with the first two letters given for 6 months, and the last two letters on the end of the word drop-ping off after 2 months! Answers (B) and (C) are therefore incorrect.

In the case of tuberculous meningitis, direct spinal cord involvement, and for resistant organisms a longer course of treatment may be necessary. A corticosteroid such as dexamethasone should be started at the same time as anti-tuberculosis therapy in meningeal or pericardial tuberculosis

618
Q

A 46-year-old Somalian woman presents to her GP with a dry cough and weight loss of 5kg over 3 weeks. She is sent to the hospital, and a chest X-ray reveals cavitating lung lesions. She is started on a course of anti-tuberculous medication. Which of the following statements about this regimen is true?

A Liver function tests only need to be checked in those with pre-existing liver disease
B Ethambutol can cause a peripheral neuropathy
C Pyridoxine should always be given with isoniazid treatment
D Rifampicin can cause optic neuritis
E Ethambutol should be avoided in renal failure

A

Ethambutol should be avoided in renal failure

Remember that treatment for pulmonary TB usually consists of two phases – an initial phase with rifampicin, isoniazid, pyrazinamide and ethambutol for 2 months, and then a continuation phase with rifampicin and isoniazid only for 4 months.

Streptomycin and ethambutol are two anti-tuberculous drugs which should preferably be avoided in patients with renal impairment (E). If they have to be used the dosage should be reduced and the plasma drug concentration closely monitored. A patient’s renal function should be checked routinely before anti-tuberculous medication is started.The side effect that is particularly worrying with the use of ethambutol is its ocular toxicity, and this is more likely in renal impairment as it is renally excreted. This can present with changes in visual acuity, colour blindness and restriction of visual fields. Therefore a patient’s visual acuity should be assessed with a Snellen chart prior to starting treat-ment, and they should be strongly advised to stop the medication and seek advice if they become aware of any change in their vision. This side effect does not occur with rifampicin (D).

Liver function should be tested in everyone before starting antituberculous therapy, as isoniazid, rifampicin and pyrazinamide are all hepatotoxic (A). Further checks are not needed unless the patient has pre-existing liver disease, is alcohol dependent or develops symptoms of liver disease.

Rifampicin can commonly cause a transient disturbance to liver function tests in the first 2 months, but this does not usually necessitate any changes to the treatment regimen.

The only common side effect of isoniazid is a peripheral neuropathy. This can be remembered by ‘isoniazid causes a sensory neuropathy’. Pyridoxine (vitamin B6) is not given routinely as a prophylactic measure in patients using isoniazid, but may be given in those with pre-existing risk factors such as diabetes, alcohol dependence and HIV (C).

Ethambutol does not cause a peripheral neuropathy (B)

619
Q

A 35-year-old banker develops a fever, vomiting and diarrhoea after a barbeque. This resolves within 2 weeks, but he then suddenly develops unilateral facial weakness. This is followed by severe muscle weakness which rapidly spreads over the next 5 days from his feet and legs to his trunk. The most likely diagnosis is:

A Polio
B Lyme disease
C Guillan–Barré syndrome
D Haemolytic uraemic syndrome
E Influenza
A

Guillan–Barré syndrome

This scenario is characteristic of Guillan–Barrè syndrome. If you remember that this disease is also known as AIDP – acute inflammatory demyelinating polyradiculopathy – you can remember the underlying pathology more easily. It is usually triggered by an infection, and it is thought that a suppressed T-cell response results in an immunological reaction that targets the peripheral nerves.The triggering infection is most commonly Campylobacter jejuni (as alluded to here), but other common causes can include Mycoplasma pneumoniae and viruses such as cytomegalovirus and influenza.

The key clinical features that guide you to this diagnosis are:
1 An antecedent infection
2 Sudden progressive muscle weakness within approximately 3 weeks of the onset of the original infection
3 The muscle weakness evolving rapidly over 1–3 weeks
4 The weakness is often symmetrical, and usually begins distally and ascends over time
5 Cranial nerves can also be affected, most commonly presenting as a unilateral facial weakness
6 Reflexes are usually absent
7 Sensory abnormalities are not a common feature

The key worrying features of this disease are:
1 Autonomic involvement – with tachycardia, fluctuating blood pressure and arrhythmias
2 Respiratory involvement – which can lead to type 2 respiratory failure

Treatment is usually supportive, but intravenous immunoglobulin therapy for 5 days or plasma exchange has been shown to be effective.

Polio (A) would not characteristically have an antecedent diarrhoeal infection, and facial weakness is less likely. The paralytic phase would not typically have an ascending pattern of weakness, and fasciculations prior to paralysis are an important feature.

Lyme disease (B) can cause facial weakness, but you would normally expect a question pointing you to this diagnosis to mention the characteristic target rash of erythema chronicum migrans. The somewhat unusual features of the second stage of Lyme disease can be remembered using the word PEACH: Peripheral neuropathy, Erythema chronicum migrans (per-sists in this stage), Arthritis, Cranial nerve palsies and Heart block.

Haemolytic uraemic syndrome (D) is a triad remembered by the phrase ‘He’s (haemolytic) got your (ur-aemic) MAT (the triad of microangio-pathic haemolytic anaemia, acute renal failure and thrombocytopenia). Cases that feature diarrhoea often occur in children, and are usually caused by E. coli O157. The neurological features described here would not normally be associated with this syndrome.

Influenza (E) is again not likely to manifest with this characteristic neurological pattern of symptoms.

620
Q

A young girl returns from visiting her relatives in India, feeling feverish and with flu-like symptoms. A diagnosis of malaria is suspected. Her fevers started on Monday, regressed for a few days and then returned on Thursday. She was well again over the weekend, and was then brought to the GP the following Monday when her fever had again returned. The most likely causative agent in this case is:

A Plasmodium falciparum
B Plasmodium vivax
C Plasmodium ovale
D Plasmodium malariae
E Plasmodium knowlesi
A

Plasmodium malariae

Malaria should always be considered as a diagnosis in a patient presenting with a fever from an endemic country – mainly Africa, South and Central America, Asia and the Middle East. It is transmitted by the female Anopheles mosquito. There are five different types. They can be differentiated clinically according to the pattern of the fever they cause. Each of the different members of the Plasmodium genus results in a different periodicity of fever. ‘Tertian’ malaria means that the fever occurs every 3 days (i.e. days 1, 3 and 5 and so on) and ‘quartan’ malaria means it occurs every fourth day (i.e. days 1, 4, 7 and so on), as in this case.

P. malariae (D) causes quartan malaria, so is the correct answer here. This type of malaria is said to be benign, and is the least common form found in the UK.

It is worth remembering that malaria does not usually cause a rash or lymphadenopathy, so think again if these are featured in a question on a fever in a returning traveller! Features of malaria on examination might include anaemia, jaundice and hepatosplenomegaly.

621
Q

A young girl returns from visiting her relatives in India, feeling feverish and with flu-like symptoms. A diagnosis of malaria is suspected. The form of the malaria parasite which invades erythrocytes is known as a:

A Sporozite
B Schizont
C Merozite
D Hypnozoite
E Gametocyte
A

Merozite

Malaria has a complex life cycle, with two phases. The ‘erthrocytic’ phase involves red blood cells, whereas the ‘exoerthryocytic phase’ involves the liver.
The basic stages are:
1 An infected mosquito injects sporozites (A) from its saliva into a person’s blood stream when it bites
2 These enter the blood stream and are taken to the liver where they infect hepatocytes
3 Here they multiply for a varying period of time, and then differentiate to form haploid merozites (C). These have a ‘signet ring’ appearance. Schizonts (B) are oval-shaped inclusions that contain the merozoites. Note that P. vivax and P. ovale sporozoites may not develop into merozites immediately, but can form hypnozoites (D) that remain dormant in the liver
4 The merozites escape from the liver into the blood stream and infect red blood cells – the erythrocytic phase
5 They multiply further in the erythrocytes, and will be released from them at intervals. The waves of fever the patient experiences correspond to when the merozites are released from the erythrocytes
6 Some of the merozoites develop into sexual forms of the parasite, called male and female gametocytes (E). When a mosquito bites an infected human, it ingests the gametocytes which form gametes inside the mosquito
7 These then fuse to form oocytes and then sporozites – ready to inject into a person.

622
Q

A 55-year-old housewife returns from visiting her relatives in India, with a high fever and with flu-like symptoms. A diagnosis of uncomplicated falciparum malaria is confirmed. The most appropriate management plan is:

A Discharge with oral quinine and doxycycline
B Discharge with oral mefloquine and chloroquine
C Admit, give IV paracetemol and observe
D Admit and give IV quinine
E Admit and give oral quinine and doxycycline

A

Admit and give oral quinine and doxycycline

All patients with falciparum malaria should be admitted to hospital initially, so answers (A) and (B) are automatically excluded. Children should be kept in for at least 24 hours, and infants, pregnant women and the elderly need to be closely monitored because they can deterio-rate rapidly. The treatment options then depend on whether the malaria is uncomplicated or complicated.

Uncomplicated malaria can be treated with one of the following:
1 Oral quinine plus doxycycline for 5–7 days (E)
2 Co-artem (artemetherelumefantrine) for 3 days
3 Atovaquone–proguanil (Malarone) for 3 days

Therefore the correct answer here is (E). Giving paracetamol without anti-malarials would not be adequate, so clearly (C) is not suitable. Chloroquine and mefloquine (B) are not recommended for the treatment of falciparum malaria in the UK.

Oral treatment would suffice in an uncomplicated case of falciparum malaria such as this, but in a severe case the first line anti-malarial used in the UK is IV quinine (D). IV artesunate may also be considered in the case of very severe disease instead of or in addition to quinine, but this is not always widely available. The treatment for non-falciparum malaria is quite different. In uncomplicated infection, chloroquine is used initially followed by a 2-week course of primaquine. The choloroquine treats the parasites in the eryth-rocytes only, thus primaquine is still needed to kill the hypnozoites that remain latent in the liver.

Glucose-6-phosphate dehydrogenase deficiency is an X-linked recessive hereditary disease, and anti-malarial drugs can cause acute haemolysis in these patients. The drugs thought to be particularly troublesome are primaquine and choloroquine, but others may be dangerous at high doses. For this reason glucose-6-phosphate dehydrogenase levels are checked in patients before starting anti-malarial treatment.

623
Q

A 55-year-old housewife returns from visiting her relatives in India, with a high fever and with flu-like symptoms. Thick and thin films are requested, and Maurer’s clefts are seen under the microscope. The diagnosis is:

A Plasmodium falciparum
B Plasmodium vivax
C Plasmodium ovale
D Plasmodium malariae
E Plasmodium knowlesi
A

Plasmodium falciparum
The most reliable way to diagnose malaria is via a blood film, and traditionally a thick and thin blood film are requested. Most people remember this fact, but not the reason behind it! Thick films are better than thin films at picking up lower levels of infection, but thin films allow the specific species to be identified. Both types of films are used together to make the diagnosis.

In the erythrocytic life cycle of the malarial parasite, disc-like granulations can be seen at the edge of the cell using an electron microscope. These are known as Maurer’s clefts, and are found in falciparum malaria (A). They are thought to be used by the parasite for protein sorting and export. They are larger and coarser than the Schuffner’s dots seen with P. vivax (B) and P. ovale (C). These are punctuate granulations again seen under the microscope in erythrocytes invaded by the tertian malaria parasite. These two structures are worth remembering for exam questions!

P. malariae (D) causes ‘quartan’ malaria, meaning the fever occurs every fourth day (i.e. days 1, 4, 7 and so on). P. knowleski (E) is much less common, and mainly occurs in southeast Asia (such as in Borneo). Maurer’s clefts and Schuffner’s dots would not typically be found in infection with these species

624
Q
Ca 2.4 (2.2–2.6 mmol/L)
PTH 4.2 (0.8–8.5pmol/L)
ALP 250 (30–150u/L)
PO4 1.1 (0.8–1.2mmol/L)
Vitamin D 76 (60–105nmol/L) 
A Primary hyperparathyroidism
B Secondary hyperparathyroidism
C Tertiary hyperparathyroidism
D Pseudohypoparathyroidism
E Primary hypoparathyroidism
F Osteoporosis
G Osteomalacia
H Paget’s disease
I Familial benign hypercalcaemia
A

Paget’s disease (H) is a condition associated with impaired bone remodelling. New bone is larger but weak and prone to fracture. The pathogenesis has been postulated to be linked to paramyxovirus. All calcium blood studies will be normal apart from ALP, which will be raised. Paget’s disease is associated with extreme bone pain, bowing and chalk-stick fractures. Bossing of the skull may lead to an eighth cranial nerve palsy and hence hearing loss. X-ray findings include lytic and sclerotic lesions

625
Q
Ca 3.1 (2.2–2.6mmol/L)
PTH 10.5 (0.8–8.5pmol/L)
ALP 165 (30–150u/L)
PO4 0.6  (0.8–1.2mmol/L)
Vitamin D 78 (60–105nmol/L) 
A Primary hyperparathyroidism
B Secondary hyperparathyroidism
C Tertiary hyperparathyroidism
D Pseudohypoparathyroidism
E Primary hypoparathyroidism
F Osteoporosis
G Osteomalacia
H Paget’s disease
I Familial benign hypercalcaemia
A

Primary hyperparathyroidism (A) is caused by a parathyroid adenoma or parathyroid chief cell hyperplasia that leads to increased PTH production. Primary hyperparathyroidism leads to hypercalcaemia due to a raised PTH level. PTH achieves this by activating osteoclastic bone resorption (increasing blood ALP), stimulating calcium reabsorption in the kidney (with concurrent excretion of phosphate) and potentiating the action of the enzyme 1α hydroxylase in the kidney. 1αHydroxylase acts on 25-hydroxyvitamin D3 to produce 1,25-dihydroxy-vitamin D3 (calcitriol), which increases gut absorption of calcium

626
Q
Ca 2.1 (2.2–2.6mmol/L)
PTH 10.4 (0.8–8.5pmol/L)
ALP 190 (30–150u/L)
PO4 0.69  (0.8–1.2mmol/L)
Vitamin D 41 (60–105nmol/L) 
A Primary hyperparathyroidism
B Secondary hyperparathyroidism
C Tertiary hyperparathyroidism
D Pseudohypoparathyroidism
E Primary hypoparathyroidism
F Osteoporosis
G Osteomalacia
H Paget’s disease
I Familial benign hypercalcaemia
A

Osteomalacia (G; rickets in children) results from insufficient bone mineralization, secondary to vitamin D or phosphate deficiency. Low vitamin D causes hypocalcaemia, due to reduced 1,25-dihydoxyvitamin D3 production, and hence reduced reabsorption of calcium from the gut. Low blood calcium levels cause an increase in production of PTH in an attempt to normalize calcium. Therefore, calcium levels will either be low or inappropriately normal. Increased bone resorption will cause ALP levels to rise.

627
Q
Ca 3.8 (2.2–2.6mmol/L)
PTH 9.6 (0.8–8.5pmol/L)
ALP 50 (30–150u/L)
PO4 1.9  (0.8–1.2mmol/L)
Vitamin D 82 (60–105nmol/L) 
A Primary hyperparathyroidism
B Secondary hyperparathyroidism
C Tertiary hyperparathyroidism
D Pseudohypoparathyroidism
E Primary hypoparathyroidism
F Osteoporosis
G Osteomalacia
H Paget’s disease
I Familial benign hypercalcaemia
A

Familial benign hypercalcaemia (I) is a genetic condition leading to raised blood calcium levels. The disease results from a mutation in the calcium receptor located on the parathyroid glands and kidneys. This receptor defect therefore leads to underestimation of calcium, causing an increased production of PTH, despite the raised calcium levels. It is important to distinguish these patients from hyperparathyroid patients as the management of these conditions differs. Receptor failure in the kidneys reduces calcium excretion, leading to a hypocalcuric state

628
Q
Ca 1.8 (2.2–2.6mmol/L)
PTH 0.69 (0.8–8.5pmol/L)
ALP 89 (30–150u/L)
PO4 1.5 (0.8–1.2mmol/L) 
Vitamin D 76 (60–105nmol/L) 
A Primary hyperparathyroidism
B Secondary hyperparathyroidism
C Tertiary hyperparathyroidism
D Pseudohypoparathyroidism
E Primary hypoparathyroidism
F Osteoporosis
G Osteomalacia
H Paget’s disease
I Familial benign hypercalcaemia
A

Primary hypoparathyroidism (E) is defined as dysfunction of the parathyroid glands leading to reduced production of PTH. As a result, the actions of PTH are blunted leading to reduced bone resorption as well as renal and gut calcium reabsorption. As a consequence there is hypocalcaemia and hyperphosphataemia. Other causes of hypocalcaemia include pseudoparathyroidism, vitamin D deficiency, renal disease (unable to make 1,25-dihydroxyvitamin D3), magnesium deficiency (magnesium required for PTH rise) and post-surgical (neck surgery may damage parathyroid glands).

Secondary hyperparathyroidism (B) is defined as the release of PTH as a consequence of hypocalcaemia that arises due to non-parathyroid pathology. The most common cause is chronic renal failure.

Tertiary hyperparathyroidism (C) results from hyperplasia of the parathyroid glands after a long period of secondary hyperparathyroidism. Autonomous production of PTH causes hypercalcaemia.

Pseudohypoparathyroidism (D) is a genetic condition in which there is resistance to PTH. As a result patients have high PTH and phosphate levels but are hypocalcaemic.

Osteoporosis (F) results in reduced bone density and all calcium studies are normal. Menopause, alcohol and drugs such as goserelin and steroids are risk factors

629
Q

A 13-year-old boy presents to his GP with parotitis with pain in his testes. His previous history reveals an incomplete childhood vaccination record.

A Bence–Jones protein
B Carcino-embryonic antigen
C Caeruloplasmin
D Fibrinogen
E Amylase
F Ferritin
G α-Fetoprotein
H Albumin
I CA125
A

Amylase (E) is an enzyme that breaks down starch into maltose. Serum amylase levels are often elevated during inflammation involving the parotid glands (parotitis) as occurs in mumps. Amylase is produced in the salivary glands, the parotid gland being the largest producer of the enzyme. Inflammation of the parotid glands cause a release of amylase into the blood stream, hence elevating levels. Raised serum amylase levels are also used in the diagnosis of pancreatitis; the pancreas is another amylase producing site.

630
Q

A 50-year-old patient who has a 4-week history of tiredness undergoes a colonoscopy. Bleeding is noted in the large intestine.

A Bence–Jones protein
B Carcino-embryonic antigen
C Caeruloplasmin
D Fibrinogen
E Amylase
F Ferritin
G α-Fetoprotein
H Albumin
I CA125
A

Ferritin (F) is an intracellular protein responsible for the safe stor-age of iron, as free iron can be toxic to cells. Gastrointestinal bleeding may cause iron deficiency anaemia (microcytic anaemia), characterized haematologically by a reduced serum iron, raised total iron binding capacity and reduced ferritin. Ferritin levels will distinguish between other causes of microcytic anaemia: anaemia of chronic disease (raisedferritin) and thalassaemia (normal ferritin). As ferritin is an acute-phase protein, it will also be raised secondary to inflammation.

631
Q

A 62-year-old smoker with a history of ulcerative colitis presents to his GP with weight loss and tiredness. The patient admits noticing fresh blood mixed in with the stool.

A Bence–Jones protein
B Carcino-embryonic antigen
C Caeruloplasmin
D Fibrinogen
E Amylase
F Ferritin
G α-Fetoprotein
H Albumin
I CA125
A

Bence–Jones proteins (A) are monoclonal globular proteins that are a diagnostic feature of multiple myeloma. Multiple myeloma is defined as the proliferation of plasma cells in the bone marrow and is commonly associated with the elderly population. Malignant plasma cells produce monoclonal antibodies and/or κor λ light chains (paraproteins). The light chains appear in the urine and can be detected by electrophoresis of a urine sample as a monoclonal band. Bence–Jones proteins are also a feature of Waldenstrom’s macroglobulinaemia and amyloid light chain amyloidosis.

632
Q

A 42-year-old woman presents to her GP with weight loss and abdominal pain. Bimanual examination reveals a mass in the left adnexa.

A Bence–Jones protein
B Carcino-embryonic antigen
C Caeruloplasmin
D Fibrinogen
E Amylase
F Ferritin
G α-Fetoprotein
H Albumin
I CA125
A

CA-125 (cancer antigen 125; I) is a protein encoded by the MUC16 gene that may suggest the presence of ovarian cancer. Its low sensitivity and specificity prevents it from being a diagnostic marker but it is useful when used in conjunction with imaging modalities for the diagnosis of ovarian cancer. Many ovarian cancers are coelomic epithelial carcinomas and hence will express CA-125, which is a coelomic epithelium-related glycoprotein. CA-125 may be associated with endometrial, pancreatic and breast carcinomas but plasma levels are most elevated in ovarian cancer.

633
Q

A 15-year-old boy is brought in by his mother who has noted a change in his behaviour as well as a tremor. On slit lamp examination, Keiser–Fleischer rings are noted around the iris.

A Bence–Jones protein
B Carcino-embryonic antigen
C Caeruloplasmin
D Fibrinogen
E Amylase
F Ferritin
G α-Fetoprotein
H Albumin
I CA125
A

Caeruloplasmin (C) is a copper carrying protein encoded by the CP gene. Low plasma caeruloplasmin levels are associated with Wilson’s disease, an autosomal recessive condition in which there is an accumulation of copper within organs due to a defect in the copper trans-porter ATP7B (linking copper to caeruloplasmin). As a result caeruloplasmin is degraded in the blood stream. Clinical manifestations include neurological and psychiatric symptoms, and copper accumulation within the iris of the eyes leading to Keiser–Fleischer rings is pathognomonic

Carcino-embryonic antigen (CEA; B) is a glycoprotein that is raised primarily in gastrointestinal cancers such as colorectal carcinoma, gastric carcinoma and pancreatic carcinoma.

Fibrinogen (D) is a glycoprotein synthesized in the liver. It has an essential role in the coagulation cascade, being converted to fibrin in the presence of thrombin, an essential process during clot formation.

α-Fetoprotein (G) is a tumour marker especially raised in hepatocellular carcinoma and germ cell tumours. α-Fetoprotein is also used antenatally to screen for neural tube defects and Down syndrome.

Albumin (H) is synthesized in the liver. Low plasma albumin levels result in oedema (liver disease, nephrotic syndrome and malabsorption). Raised plasma albumin levels are associated with dehydration.

634
Q

A 40-year-old patient with a history of Graves’ disease presents with bilateral weakness of her legs. On examination she is Babinski sign positive and blood tests reveal a megaloblastic anaemia.

A Vitamin A
B Vitamin B1
C Vitamin B2
D Vitamin B6
E Vitamin B12
F Vitamin C
G Vitamin D
H Vitamin E
I Vitamin K
A

Vitamin B12 (cobalamin; E) deficiency may result from pathologies affecting the stomach or ileum, as well as pernicious anaemia. In pernicious anaemia, autoantibodies exist against intrinsic factor. Pernicious anaemia is also commonly associated with other autoimmune conditions, such as Graves’ disease. Anaemia is a common manifestation of vitamin B12 deficiency, with raised mean cell volume and hypersegmented neutrophils evident. Subacute combined degeneration of the cord can also result, causing ataxia and progressive weakness in limbs and trunk; Babinski sign may be positive.

635
Q

A 26-year-old man presents to his GP with a 5-month history of bleeding gums. Petechiae are also observed on the patient’s feet. The man admits he has had to visit his dentist recently due to poor dentition

A Vitamin A
B Vitamin B1
C Vitamin B2
D Vitamin B6
E Vitamin B12
F Vitamin C
G Vitamin D
H Vitamin E
I Vitamin K
A

Vitamin C (F) is a water soluble vitamin, essential for the hydroxylation of collagen. When deficiency of vitamin C is present, collagen is unable to form a helical structure and hence cannot produce cross-links. As a consequence, damaged vessels and wounds are slow to heal. Vitamin C deficiency results in scurvy, which describes both bleeding (gums, skin and joints) and bone weakness (microfractures and brittle bones) tendencies. Gum disease is also a characteristic feature.

636
Q

A 5-year-old girl who is a known cystic fibrosis sufferer is noted by her mother to have developed poor coordination of her hands and on examination her reflexes are absent. Blood tests also reveal anaemia.

A Vitamin A
B Vitamin B1
C Vitamin B2
D Vitamin B6
E Vitamin B12
F Vitamin C
G Vitamin D
H Vitamin E
I Vitamin K
A

Vitamin E (tocopherol; H) is an important anti-oxidant which acts to scavenge free radicals in the blood stream. Deficiency leads to haemolytic anaemia as red blood cells encounter oxidative damage and are consequently broken down in the spleen. Spino-cerebellar neuropathy is also a manifestation, which is characterized by ataxia and areflexia. Vitamin E deficiency has also been suggested to increase the risk of ischaemic heart disease in later life, as low-density lipoproteins become oxidized perpetuating the atherosclerotic process.

637
Q

A 35-year-old man who is being treated for tuberculosis develops a rash on his trunk. Blood tests also reveal anaemia.

A Vitamin A
B Vitamin B1
C Vitamin B2
D Vitamin B6
E Vitamin B12
F Vitamin C
G Vitamin D
H Vitamin E
I Vitamin K
A

Vitamin B6 (pyridoxine; D) is an essential co-factor in a number of metabolic pathways including the synthesis of amino acids and neuro-transmitters. Common causes of deficiency include reduced dietary intake and isoniazid use for the treatment of tuberculosis. Vitamin B6 deficiency causes blood and skin abnormalities. Haematologically, vitamin B6 deficiency causes sideroblastic anaemia; dermatologically seborrhoeic dermatitis can occur. Diagnosis is made by determining erythrocyte levels of aspartate aminotransferase

638
Q

A 40-year-old known alcoholic develops confusion and an unsteady gait. On examination bilateral lateral rectus palsy is noted

A Vitamin A
B Vitamin B1
C Vitamin B2
D Vitamin B6
E Vitamin B12
F Vitamin C
G Vitamin D
H Vitamin E
I Vitamin K
A

Vitamin B1 (thiamine; B) deficiency most commonly occurs in cases of alcoholism. The acute presentation of vitamin B1 deficiency is Wernicke’s encephalopathy, characterized by the triad of confusion, ophthalmoplegia and ataxia. Chronic alcoholism can lead to Korsakoff’s syndrome (amnesia and confabulation) and peripheral neuropathy. Beriberi can also occur, classified into wet and dry beriberi. Wet beri-beri presents in a similar manner to heart failure, with cardiomegaly, oedema and dyspnoea. Dry beriberi involves an ascending impairment of nervous function involving both sensory (paraesthesia) and motor (foot drop, wrist drop and paralysis) components.

Vitamin A (A) deficiency primarily impairs the production of rods and hence causes night blindness; ocular epithelial changes also cause con-junctival Bitot’s spots. Deficiency may cause predisposition to measles and diarrhoeal illnesses.

Vitamin B2 (riboflavin; C) deficiency leads to mucosal damage and hence presents with angular stomatitis, glossitis and/or corneal ulceration.

Vitamin D (G) deficiency results from reduced dietary intake as well as inadequate sunlight exposure. Deficiency leads to bone pathology, including rickets in children and osteomalacia in adults.

Vitamin K (I) deficiency may result from reduced intestinal uptake or dietary deficiency. Presenting features may include ecchymosis, petechiae, haematomas and slow healing at wound sites.

639
Q

An 18-month-old girl is seen by the GP. Her mother is concerned by the child’s brittle hair and inability to walk. The mother reports her daughter has had two previous convulsions.

A Phenylketonuria (PKU)
B Peroxisomal disorders
C Maple syrup urine disease
D Short-chain acyl-coenzyme A dehydrogenase (SCAD) deficiency
E Von Gierke’s disease
F Fabry’s disease
G Urea cycle disorder
H Homocystinuria 
I Galactosaemia
A

Homocystinuria (H) is an amino acid disorder in which there is a deficiency in the enzyme cystathionine synthetase. This metabolic disorder presents in childhood with characteristic features such as very fair skin and brittle hair. The condition will usually lead to developmental delay or progressive learning difficulties. Convulsions, skeletal abnormalities and thrombotic episodes have also been reported. Management options include supplementing with vitamin B6 (pyridoxine) or maintaining the child on a low-methionine die

640
Q

A fair haired 8-month-old baby, born in Syria, is seen together with his mother in the paediatric outpatient clinic. He is found to have developmental delay and a musty smell is being given off by the baby.

A Phenylketonuria (PKU)
B Peroxisomal disorders
C Maple syrup urine disease
D Short-chain acyl-coenzyme A dehydrogenase (SCAD) deficiency
E Von Gierke’s disease
F Fabry’s disease
G Urea cycle disorder
H Homocystinuria 
I Galactosaemia
A

Phenylketonuria (PKU; A) is also an amino acid disorder. Children classically lack the enzyme phenylalanine hydroxylase, but other co-factors may be aberrant. Since the 1960s PKU has been diagnosed at birth using the Guthrie test but in some countries the test may not be available. The child will be fair-haired and present with developmental delay between 6 and 12 months of age. Later in life, the child’s IQ will be severely impaired. Eczema and seizures have also been implicated in the disease process

641
Q

A 9-month-old baby is seen in accident and emergency as her mother has reported that she has become ‘floppy’. The baby is found to be hypoglycaemic and on examination an enlarged liver and kidneys are noted.

A Phenylketonuria (PKU)
B Peroxisomal disorders
C Maple syrup urine disease
D Short-chain acyl-coenzyme A dehydrogenase (SCAD) deficiency
E Von Gierke’s disease
F Fabry’s disease
G Urea cycle disorder
H Homocystinuria 
I Galactosaemia
A

Von Gierke’s disease (E) is one of nine glycogen storage disorders, in which a defect in the enzyme glucose-6-phosphate results in a failure of mobilization of glucose from glycogen. The metabolic disease presents in infancy with hypoglycaemia. The liver is usually significantly enlarged and kidney enlargement can also occur. Other glycogen storage disorders (and enzyme defects) include Pompe’s (lysosomal α-glucosidase), Cori’s (amylo-1,6-glucosidase) and McArdle’s (phosphorylase); each disorder presents with varying degrees of liver and muscle dysfunction

642
Q

A 14-day-old girl of Jewish descent presents with lethargy, poor feeding and hypotonia. The paediatrician examining the child also notices excessively sweaty feet.

A Phenylketonuria (PKU)
B Peroxisomal disorders
C Maple syrup urine disease
D Short-chain acyl-coenzyme A dehydrogenase (SCAD) deficiency
E Von Gierke’s disease
F Fabry’s disease
G Urea cycle disorder
H Homocystinuria 
I Galactosaemia
A

Maple syrup urine disease (C) is an organic aciduria, a group of disorders that represent impaired metabolism of leucine, isoleucine and valine. As a result, toxic compounds accumulate causing toxic encephalopathy which manifests as lethargy, poor feeding, hypotonia and/or seizures. Characteristic of maple syrup urine disease are a sweet odour and sweaty feet. The gold standard diagnostic test is gas chromatography with mass spectrometry. Management involves the avoidance of the causative amino acids

643
Q

A 5-month-old boy is seen by the community paediatrician due to concerns of developmental delay. On examination dysmorphic features are noted, as well as a ‘cherry-red spot’ on the baby’s trunk

A Phenylketonuria (PKU)
B Peroxisomal disorders
C Maple syrup urine disease
D Short-chain acyl-coenzyme A dehydrogenase (SCAD) deficiency
E Von Gierke’s disease
F Fabry’s disease
G Urea cycle disorder
H Homocystinuria 
I Galactosaemia
A

Fabry’s disease (F) is a lysosomal storage disorder in which there is deficiency in α-galactosidase. Presentation is almost always a child with developmental delay together with dysmorphia. Other findings may involve movement abnormalities, seizures, deafness and/or blindness. On examination, hepatosplenomegaly, pulmonary and cardiac problems may be noted. The pathognomonic feature of lysosomal stor-age disorders is the presence of a ‘cherry-red spot.’

Peroxisomal disorders (A) result in disordered β-oxidation of very-long-chain fatty acids (VLCFA); these accumulate in the blood stream. In neonates, such disorders lead to seizures, dysmorphic features, severe muscular hypotonia and jaundice.

Short-chain acyl-coenzyme A dehydrogenase (SCAD) deficiency (D) is one of the four fatty acid oxidation disorders, which is unique in its neonatal presentation with failure to thrive, hypotonia, metabolic acidosis and hyperglycaemia.

Urea cycle disorders (G) arise due to deficiency in one of the six enzymes in the urea cycle, resulting in hyperammonaemia. Enzyme deficiency occurs in an autosomal recessive fashion. Symptoms depend on age of presentation, but overall encephalopathy ensues with primarily neurological features.

Galactosaemia (I) results from the deficiency in the enzyme galactose-1-phosphate uridyl transferase (Gal-1-PUT). Symptoms occur in the infant after milk ingestion, usually poor feeding, vomiting, jaundice and hepatomegaly. A galactose-free diet is the primary management option

644
Q

A 35-year-old man presents to accident and emergency with feelings of lightheadedness and slurred speech. His wife mentions that the patient has been walking around ‘like a drunk’. The man’s blood pressure is found to be low.

A Procainamide
B Lithium
C Methotrexate
D Theophylline
E Gentamicin
F Carbamazepine
G Cyclosporine
H Phenytoin
I Digoxin
A

Phenytoin (H) is a commonly used anti-epileptic agent. Serum levels of phenytoin must be monitored due to its narrow therapeutic range (10–20μg/mL). Phenytoin also exhibits saturation kinetics; a small rise in dose may lead to saturation of metabolism by CYP enzymes in the liver, hence producing a large increase in drug concentration in the blood as well as associated toxic effects. Phenytoin toxicity can lead to hypotension, heart block, ventricular arrhythmias and ataxia.

645
Q

A 45-year-old woman is told she may be demonstrating signs of toxicity, 12 hours after being given an initial dose of medication. She has a coarse tremor and complains of feeling nauseous.

A Procainamide
B Lithium
C Methotrexate
D Theophylline
E Gentamicin
F Carbamazepine
G Cyclosporine
H Phenytoin
I Digoxin
A

Lithium (B) is a therapeutic agent used in the treatment of bipolar dis-order. Drug monitoring is essential (12 hours post dose) due to its low therapeutic index as well as the potential life-threatening effects of toxicity. Lithium is excreted via the kidneys and therefore serum drug levels may increase (with potential toxicity) in states of low glomerular filtration rate, sodium depletion and diuretic use. Features of lithium toxicity include diarrhoea, vomiting, dysarthria and coarse tremor. Severe toxicity may cause convulsions, renal failure and possibly death.

646
Q

A 65-year-old man being treated as an inpatient develops sudden onset ‘ringing in his ears’ as well as difficulty hearing.

A Procainamide
B Lithium
C Methotrexate
D Theophyllinein 
E Gentamicin
F Carbamazepine
G Cyclosporine
H Phenytoin
I Digoxin
A

Gentamicin (E) is an aminoglycoside antibiotic, particularly useful against Gram-negative bacteria. It exhibits a low therapeutic index. Factors that may potentiate toxicity include dosage, kidney function (gentamicin is excreted through the kidneys) and other medications such as vancomycin. Gentamicin is an ototoxic and nephrotoxic agent and hence toxicity can lead to deafness and renal failure. Toxic effects on the ear are not limited to hearing, as the vestibular system is also affected, which may cause problems with balance and vision

647
Q

A 45-year-old woman is seen by her GP for a routine medications review. The patient complains of recent onset abdominal pain and tiredness. An electrocardiogram (ECG) reveals prolonged PR interval.

A Procainamide
B Lithium
C Methotrexate
D Theophylline
E Gentamicin
F Carbamazepine
G Cyclosporine
H Phenytoin
I Digoxin
A

Digoxin (I) is an anti-arrhythmic agent used in the treatment of atrial fibrillation and atrial flutter. Symptoms of under-treatment and toxic-ity are similar. Toxicity commonly arises due to the narrow therapeutic index of the agent. Non-specific symptoms of toxicity include tiredness, blurred vision, nausea, abdominal pain and confusion. ECG changes may include a prolonged PR interval and bradycardia. As digoxin is excreted via the kidneys, renal failure may cause accumulation of digoxin.

648
Q

A 45-year-old man presents to his GP for a routine medications review. The patient complains of recent diarrhoea and headaches. The GP notes the patient was treated with erythromycin for a community acquired pneumonia 1 week previous to the consultation

A Procainamide
B Lithium
C Methotrexate
D Theophylline
E Gentamicin
F Carbamazepine
G Cyclosporine
H Phenytoin
I Digoxin
A

Theophylline (D) is a drug used in the treatment of asthma and COPD. A low therapeutic index and wide variation in metabolism between patients lead to requirement for drug monitoring. Toxicity may manifest in a number of ways including nausea, diarrhoea, tachycardia, arrhythmias and headaches. Severe toxicity may lead to seizures. The toxic effects of theophylline are potentiated by erythromycin and cipro-floxacin. Without monitoring, many patients would be under-treated.

Procainamide (A) is an anti-arrhythmic agent. Toxicity may lead to rash, fever and agranulocytosis. Drug induced lupus erythematosus may result from toxic levels.

Methotrexate (C) is an anti-folate drug used in the treatment of cancers and autoimmune conditions. Toxicity may lead to ulcerative stomatitis, leukocytopenia and rarely pulmonary fibrosis.

Carbamazepine (F) is an anti-convulsant medication. Toxic levels may commonly result in headaches, ataxia and abdominal pain. Toxicity may also cause SIADH and, rarely, aplastic anaemia.

Cyclosporine (G) is an immunosuppressant. Toxicity is associated with acute renal failure. Calcium channel antagonists and certain antibiotics such as erythromycin predispose to nephrotoxicity, whereas anti-convulsants such as phenytoin reduce blood levels of the drug

649
Q

A 67-year-old woman presents to accident and emergency after having a fall. She is diagnosed with a fractured neck of femur which is fixed with a hemi-arthroplasty. She also suffers from metastatic breast cancer. Four days postoperatively, she develops shortness of breath with an increased respiratory rate of 24 breaths per minute. The doctor on call takes an arterial blood gas sample which shows the following results:

pH 7.48
PaO2 15.4kPa on 2L of oxygen
pCO2  2.6kPa
Base excess +1
Saturations 99 per cent

What does the blood gas show?
A Metabolic alkalosis with respiratory compensation
B Metabolic alkalosis
C Respiratory alkalosis with metabolic compensation
D Respiratory alkalosis
E None of the above

A

This lady has most likely suffered a pulmonary embolism manifesting as an acute onset of shortness of breath. Acid–base questions are best approached in three steps: first, decide if the pH shows an alkalosis or an acidosis. Next look at the PaCO2 and decide if it is high or low. Carbon dioxide dissolves in water to form carbonic acid, a weak acid. Therefore, if the concentration of carbon dioxide is high, it will lower the pH. You must then decide if the PaCO2 is compounding or helping the patient’s pH – in other words, is it worsening an acidotic patient or compensating for an alkalotic patient? Finally, look at the base excess. A greater positive base excess implies a higher concentration of bicarbonate, which is a base. Unlike carbon dioxide, therefore, high levels of bicarbonate will raise the pH.

In this scenario, the pH is 7.48 meaning the patient is alkalotic with a low PaCO2, implying a respiratory cause. There is no compensation as the base excess of +1 is within normal limits. Unlike respiratory compensation, metabolic compensation takes several days.

650
Q

A 19-year-old female student presents to the GP with low mood, lethargy and muscle weakness. She is anxious that she is putting on weight and admits to purging after meals to keep her weight under control for several months. She has a past history of depression and is taking citalopram. On examination, her body mass index is 18, she is clinically dehydrated with signs of anaemia including conjunctival pallor. She has bilateral parotidomegaly and the GP also notices erosions of the incisors. He orders some blood tests which reveal the following:

Hb 9.5
White cells 7.8
Platelets 345
Na 143
K 3.1
Urea 8.5
Creatinine 64
Arterial pH 7.49 
Urinalysis is normal except for acidic urine. The cause of this patient’s acidic urine is:
A Acute renal failure
B Renal tubular acidosis
C Citalopram
D Anaemia
E Physiological
A

Physiological

This is a difficult question but the answer can be deduced with a basic knowledge of electrolyte physiology. This patient suffers from bulimia nervosa as characterized by the use of characteristic purging after meals to keep her weight under control. The main abnormalities in the investigations reveal a hypokalaemia with arterial alkalosis and paradoxical aciduria. The alkalosis is likely to be due to excessive purging leading to a loss of hydrogen ions. The hypokalaemia is secondary to the metabolic alkalosis as potassium and hydrogen are transported across cell membranes by the same transporter. The reduction of plasma hydrogen ions leads to increased potassium uptake leading to hypokalaemia. As part of a normal homeostatic mechanism, potassium is exchanged for hydrogen ions in the distal convoluted tubule of the nephron, resulting in an apparent paradoxical aciduria.

Acute renal failure (A) tends to give hyperkalaemia and metabolic acidosis. This is due to the failure of homeostatic mechanism, the causes of which are classically defined as pre-renal, renal or post-renal.

Pre-renal failure is caused by a reduction in glomerular filtration rate. This may be due to reduced blood flow or reduced perfusion pressure. Common causes include hypovolaemia or hypotension from shock.

Intrinsic renal failure has a wide aetiology including drugs, inflammation and infection.

Post-renal failure is caused by obstruction anywhere from the collecting ducts distally. This classically presents in elderly men with prostatic disease with urinary retention relieved by catheterization.

Citalopram (C) is a selective serotonin reuptake inhibitor (SSRI) used in the treatment of depression. Some SSRIs cause hyponatraemia, but not usually hypokalaemia.

Renal tubular acidosis (B) generally causes a lack of ability to acidify urine and hyperkalaemia. The exception is type II renal tubular acidosis with a bicarbonate leak in the proximal convoluted tubule where hypokalaemia is common but the urine is only acidified during systemic metabolic acidosis. This is not the case in this patient.

Finally anaemia (D) does not usually cause electrolyte abnormalities.

651
Q

A 55-year-old man with severe learning difficulties presents with shortness of breath on exertion, fever and a productive cough of rusty red sputum. On examination, there is increased bronchial breathing in the lower right zone with inspiratory crackles. The patient is clinically euvolaemic, and urine dipstick is normal. A chest X-ray demonstrates right lower zone consolidation with the presence of air bronchograms. He is on carbemezepine for epilepsy and risperidone. Blood tests reveal the following:

Hb 13.4
White cell count 12.8
C reactive protein 23
Na 123
K 4.7
Urea 6
Creatinine 62

What is the most likely cause of hyponatraemia?

A Pneumonia
B Carbamezepine
C Risperidone
D Syndrome of inappropriate antidiuretic hormone (SIADH)
E Cerebral salt wasting syndrome
A

Carbamezapine
This patient’s hyponatraemia is most likely secondary to Carbamezepine therapy (B), a well documented side effect of this anti-epileptic medication. Carbamezepine stimulates the production of vaso-pressin, the mechanism of action of which will be discussed shortly. It is also one of the ‘terrible 3 Cs’ which cause aplastic anaemia, the other two being carbimazole and chloramphenicol. Any patient with signs of infection or bleeding must be taken very seriously as fulmi-nant sepsis may ensue without prompt treatment. This patient, how-ever, has mounted a white cell response with a normal platelet count therefore making aplastic anaemia unlikely.Pneumonia (A) does not normally cause a sodium abnormality on its own. Less commonly, Legionnaire’s disease caused by the bacterium Legionella pneumophilia can have extrapulmonary features including hyponatraemia, deranged liver function tests and lymphopenia. This is unlikely to be the case as this organism often colonizes water tanks in places with air conditioning and has a prodromal phase of dry cough with flu-like symptoms.

The alternative indirect pulmonary cause of hyponatraemia is lung cancer producing a SIADH; the tumour pre-disposes the patient to pneumonia by obstructing the normal ciliary clearance of the bronchi. It is unlikely in this patient given the lack of smoking history or cachexia.

Risperidone (C) is an atypical antipsychotic and only very rarely causes hyponatraemia. More common side effects include gastrointestinal disturbance and dry mouth.

SIADH (D) is the excessive production of anti-diuretic hormone (also called vasopressin) from the posterior pituitary. Its release is stimulated physiologically by osmoreceptors responding to an increased plasma osmolality, as well as baroreceptors responding to decreased intravascular volume. Vasopressin activates vasopressin2 receptors in the renal collecting duct principal cells, which in turn activate adenylate cyclase to increase intracellular cyclic AMP levels. This is turn increases aquaporin 2 gene transcription and the protein inserts into the apical membrane of the cells allowing free water influx to normalize increased plasma osmolality. SIADH occurs when there is excessive production of vasopressin leading to a euvolaemic hyponatraemia. It is a diagnosis of exclusion and requires two criteria in the blood, two criteria in the urine and three exclusion criteria and can be remembered as ‘two low in the blood, two high in the urine, three exclusions every-where else’.

1 Two low in the blood – hyponatraemia and hypo-osmolality
2 Two high in the urine – high urinary sodium >20mmol/L and high urinary osmolality
3 Three exclusions – NO renal/adrenal/thyroid/cardiac disease, NO hypovolaemia, NO contributing drugs.

Cerebral salt wasting (CSW) syndrome (E) occurs after head injury or neurosurgical procedures where a natriuretic substance produced in the brain leads to sodium and chloride loss in the kidneys, reducing intra-vascular volume and leading to water retention. There is therefore a baroreceptor-mediated stimulus to vasopressin production. It resembles SIADH in that both are hyponatraemic disorders seen after head injury with high urinary sodium, urinary osmalility and vasopressin levels. The difference is the primary event in CSW is high renal sodium chloride loss, not high vasopressin release.

652
Q

A patient with end stage renal failure presents with depression. He is on haemodialysis three times a week but feels it is not working anymore and is getting more tired lately. He says he has lost his appetite and consequently feels rather constipated too. He feels his mind is deteriorating and there is little worth in attending dialysis anymore. His doctor wants to exclude a reversible cause of his depression and orders some blood tests. The doctor finds the patient has a raised corrected calcium, normal phosphate levels and high parathyroid hormone levels. What is the diagnosis?

A Primary hyperparathyroidism
B Secondary hyperparathyroidism
C Tertiary hyperparathyroidism
D Pseudohypoparathyroidism
E Pseudopseudohypoparathyroidism
A

Tertiary Hyperparathyroidism

This patient has tertiary hyperparathyroidism (C) given the presence of elevated calcium levels with high parathyroid levels in the presence of chronic renal failure. Plasma calcium levels are controlled via parathyroid hormone (PTH) which is produced in the parathyroid glands situated within the thyroid gland. Reduced ionized calcium concentration is detected by the parathyroid glands leading to a release of PTH which circulates in the blood stream. PTH increases calcium resorption from the kidneys whilst increasing phosphate excretion. PTH also stimulates 1-alpha hydroxylation of 25-vitamin D to make 1,25-vitamin D. Finally, PTH increases bone resorption of calcium via osteoclast activation. The sum effects of increased PTH levels are to increase plasma calcium concentration and to reduce phosphate concentration. PTH has an indirect, but very important, mechanism via 1,25-vitamin D which acts to increase gut absorption of calcium.

Tertiary hyperparathyroidism (C) is seen in the setting of chronic renal failure and chronic secondary hyperparathyroidism leads to hyperplastic or adenomatous change in the parathyroid glands resulting in autonomous PTH secretion. The causes of calcium homeostasis dysregulation are multifactorial including tubular dysfunction leading to calcium leak, inability to excrete phosphate leading to increased PTH levels and parenchymal loss resulting in lower activated vitamin D levels. As a result tertiary hyperparathyroidism gives a raised calcium with a very raised PTH, with normal or low phosphate. Serum alkaline phosphatase is also raised due to the osteoblast and osteoclast activity (note, osteoblasts produce alkaline phosphatase. This is why there is a normal alkaline phosphatase in myeloma, as it directly stimulates the osteoclasts). Treatment of tertiary hyperparathyroidism is subtotal parathyroidectomy. Tertiary hyperparathyroidism is differentiated from primary hyperparathyroidism (A) by the presence of chronic renal failure but is otherwise difficult to distinguish biochemically.

Primary hyperparathyroidism is most commonly caused by a solitary adenoma in the parathyroid gland. Surgeons sometimes use sestamibi technetium scintigraphy to locate the offending adenoma prior to surgical removal.

Secondary hyperparathyroidism (B) occurs where there is an appropriately increased PTH level responding to low calcium levels. This is commonly due to chronic renal failure or vitamin D deficiency but can be seen in any pathology resulting in reduced calcium or vitamin D absorption or hyperphosphataemia.

Pseudohypoparathyroidism (also known as Albright’s osteodystrophy) results from a PTH receptor insensitivity in the proximal convoluted tubule of the nephron. As a result, calcium resorption and phosphate excretion fail despite high PTH levels. Furthermore, other physical signs associated with this condition include short height, short 4th and 5th metacarpals, reduced intelligence, basal ganglia calcification, and endocrinopathies including diabetes mellitus, obesity, hypo-gonadism and hypothyroidism. Type 1 pseudohypoparathyroidism is inherited in an autosomal dominant manner where the renal adenylate cyclase G protein S alpha subunit is deficient, thus halting the intracellular messaging system activated by PTH. Patients with pseudopseudohypoparathyroidism (E) have similar physical features to pseudohypoparathyroidism but with no biochemical abnormalities of calcium present. This condition is a result of genetic imprinting where the phenotype expressed is dependent on not just what mutation is inherited but also from whom. In other words, inheriting the pseudo-hypoparathyroidism mutation from one’s mother leads to pseudohypo-parathyroidism, but inheriting it from one’s father leads to pseudo-pseudohypoparathyroidism. At the molecular level, this is signalled by differential methylation of genes thus providing a molecular off switch controlling its expression.

Another example of genetic imprinting occurs in Prade–Willi syndrome and Angelman’s syndrome, caused by a microdeletion on chromosome 15